Parkin Test Bank 3

March 17, 2017 | Author: eralgi | Category: N/A
Share Embed Donate


Short Description

Download Parkin Test Bank 3...

Description

US T co hi in py s w st ri or ru gh k ct t is or la p s’ ws ro us a te e nd cte on i d ly s f by . or

Reproduced by Pearson Addison-Wesley from electronic files supplied by author. Copyright © 2005 Pearson Education, Inc. Publishing as Pearson Addison-Wesley, 75 Arlington Street, Boston, MA 02116 All rights reserved. This manual may be reproduced for classroom use only. Printed in the United States of America. ISBN

0-321-23359-X

1 2 3 4 5 6 OPM 07 06 05 04

P r e f a c e

Introduction

Seventh Edition Revisions We have made substantial revisions in preparing the test banks for the Seventh Edition:

This book is one of six test banks, each carefully crafted to be part of the most complete package of test banks ever offered to support a beginning economics textbook. Three of the test banks are designed to accompany Michael Parkin’s Microeconomics, Seventh Edition and three accompany Michael Parkin’s Macroeconomics, Seventh Edition. The complete set of six books comprises Microeconomics Test Bank, Volumes I, II, and III and Macroeconomics Test Bank, Volumes I, II, and III. These books have undergone major revisions for this edition. Both the Microeconomics Test Bank Volumes I and II and the Macroeconomics Test Bank Volumes I and II now contain only multiple choice questions. The multiple choice questions that in previous editions were in Volume III of the test banks have now been melded into Volumes I and II. Volume III for both the Microeconomics Test Bank and Macroeconomics Test Bank now contains large numbers of essay questions, numerical questions, graphing questions, and extended problems. All of the different types of questions in Volume III have answers suitable for distribution to your students.



All the questions have been reviewed, to ensure consistency with the text and clarity for the students. Questions dealing with material eliminated from the seventh edition of the textbook were deleted and a large number of new questions were added to cover both the new as well as the old topics in the seventh edition. The new questions are identified by an asterisk (*) following the style of question.



The artwork was reviewed and changed to remain consistent with the seventh edition of the textbook.



To the greatest extent possible, the questions have been ordered so that they follow the order the material is presented in the corresponding textbook chapter. You generally will find all questions on the same topic clustered together so you can easily select the one you want. In addition, within each chapter the multiple choice questions are separated by each major section of the text chapter so that if you assign to the students only part of a chapter, it will be easy to select questions from that specific part.



Many new essay, numeric, and graphing questions have been added, all with suggested answers. You can use these answers to show students the type of answer that you were expecting. Volume III is now specialized to include these types of questions.



The test banks are available in both a user-friendly computerized test bank and in Word doc files. For some instructors, the ease of simply cutting and pasting from the Word files will exceed the utility of the powerful computerized test bank. With this edition, we accommodate both preferences.

Test Bank Principles Three principles guided the writing and revising of the questions: ♦ The questions should be “fail” insofar as the topic of the questions has been explained in the textbook. ♦

A question should not be a guessing game forcing the students to puzzle out what the question asks.



An instructor must be absolutely secure in the knowledge that each question contains material covered in the textbook.

I endeavored to insure that all questions meet all the criteria so that they are, as Donald Dutkowsky put it, “bullet proof.” III

Edited by Foxit Reader Copyright(C) by Foxit Software Company,2005-2007 PREFACE For Evaluation Only.

IV

Volumes I and II Taken together, there are nearly 15,000 multiple choice questions in Volumes I and II of the test bank. These questions have been written by many contributors. I have edited the questions to ensure that each conformed with the writing style established in the book because this style has been carefully crafted for maximum student comprehension. I also arranged the questions so that they are in the same order that the topics are presented within the textbook. In addition, among questions dealing with a similar topic, the questions are ordered with non-numerical questions first, questions dealing with a numerical table second, and questions dealing with a figure third. In addition to questions from the text, at the end of each chapter Volume I also contains multiple choice questions that are either the same as or else closely related to the questions in the Study Guide and on the MyEconLab student website for the book. Each of these questions is identified as to its source. So, if you have assigned these student supplements to your class, you have the means of “rewarding” students who are using them by asking questions drawing on what they have been studying. The questions in each chapter of Volume I and Volume II are drawn from material covered only in that chapter. A new feature of Volume II are “Part Review” questions. These sections feature questions that are drawn from material covered in more than one chapter in each Part. These questions are all new. You can use these questions if you want more integrative questions that lead students to think about broader issues. I have been teaching principles classes for over two decades, have written hundreds of exams, and won several teaching awards. Nonetheless, I was stunned by the quality of the questions in the test banks. I lost count of the number of times that I marveled at a question and wished that I had thought of it. While creating this test bank took significantly more effort than preparing an ordinary test bank, I am sure that the quality you will find made the task worthwhile. Volume III Volume III has been completely redone. It now consists of essay questions, numeric questions, graphing questions, true or false questions, and extended problems. Within each book, you will find each type of

question grouped together—that is, the essay questions for each chapter are first, followed by the numeric/graphing questions for each chapter, and so on. In this fashion, if you want to write a strictly essay question exam, it will be easy to do so. Of course, within each chapter, the questions are arranged in the same order that the topics appear in the textbook. The extended problems need new to this edition of the test banks. For each chapter there are 3 to 5 extended problems. These questions, are, as the name implies, longer questions. Sometimes the questions for a chapter build upon each other; other times the questions are independent. These questions can be used on exam. However, they may also be given to the students and used as homework. I think the best way to view these questions is as a valuable supplement to the outstanding questions already in the textbook. How to Assemble a Test Because I have been teaching while working on these test banks, it has been natural to use them to create tests for my class. Having thousands of excellent questions immediately at hand made writing examinations an easy and (almost) pleasant task. Because my class consists of between 1,500 and 2,100 students a semester, I use exclusively multiple choice questions. For each test, I decide a priori approximately how many questions I want from each chapter. Clearly my sense of the relative importance of the material plays a role here. I think this reflection is crucial because it ensures that my examinations cover the material that I deem most important for my students to know. Because I write my tests using Word, I start the actual writing process by taking an old test and eliminating all but one question. Then, using the Word test bank files, I locate a question that I want to use, copy the question from the test bank file to my test, and immediately change the question number so that it is correct. As it happens, I prefer a slightly different paragraph style for my questions and answers than what is used in the test bank. Using the Word “Format Painter” (the paint brush symbol) I copy the format I prefer from the old question that I saved to the newly pasted question. Once I have a new question formatted with my preferred paragraph style, I delete the old question. I change the style of the question and answers because I prefer a slightly different font and because I format the

PREFACE

question and the first three answers using the “keep with next” command from Word’s Format-ParagraphLine and Page Breaks menu. The last answer for the question does not have this command. By using the “keep with next” command for the question and first three answers, I ensure that I do not have a question or answer break across a page. As I copy and paste each question, I keep track of how many questions I have from each chapter. I do not slavishly adhere to my initial decision of how many questions I want from each chapter, but I also do not stray too far. Finally, I print an initial copy of my examination and proofread it to be sure that I have not inadvertently given away the answer to one question with another question. I make any necessary corrections and am done. Given the quality of the test banks, I have that found I can easily write a high-quality 35-40 question examination within a little more than an hour. Final Comments Just as Sir Isaac Newton stood on the shoulders of the giants who came before him, so, too do these test banks reflect the superb work of the authors who initially wrote and compiled them. So it is entirely fitting to thank them: ♦ Peter von Allmen of Moravian College ♦ Sue Bartlett of the University of South Florida ♦ Jill Boylston Herndon of the University of Florida ♦ Kevin Carey of American University ♦ Leo Chan of the University of Kansas ♦ Carol Dole of the State University of West Georgia ♦ Donald H. Dutkowsky of Syracuse University ♦ Andrew Foshee of McNeese State University ♦ James Giordano of Villanova University ♦ John Graham of Rutgers University ♦ Veronica Z. Kalich of Baldwin-Wallace College ♦ Sang Lee of Southeastern Louisiana University ♦ Melinda Nish of Salt Lake Community College

V

♦ ♦ ♦ ♦ ♦ ♦

Terry Olson of Truman State University Ed Price of Oklahoma State University Rochelle L. Ruffer of Youngstown State University Virginia Shingleton of Valparaiso University Della Sue of Marist College Nora Underwood of the University of California, Davis ♦ Robert Whaples of Wake Forest University ♦ Peter A. Zaleski of Villanova University Contributing questions for the Seventh edition microeconomic chapters is: ♦ Constantin Oglobin of Georgia Southern University Constantin also created all the Extended Problems for the entire book! Contributing questions for the Seventh edition macroeconomic chapters is: ♦ William Mosher of Clark University I have tried to make these Test Banks as helpful and useful as possible. Undoubtedly I have made some mistakes; mistakes that you may see. I have a standing offer in the Study Guide asking students who find any errors to notify me and promising that I will acknowledge them in all future editions of the Study Guide. I will make the same offer here: If you find any errors or have any comments or questions, please let me know and, if you want, I will note your help in all future editions of the test banks. And so keeping this promise: ♦

Dr. B. N. Ghosh of Eastern Mediterranean University in North Cyprus. Dr. Ghosh found several errors and I greatly appreciate his efforts!

My address is below, or you can reach me via E-mail at [email protected]. Mark Rush Economics Department University of Florida Gainesville, Florida 32611 December 2003

Table of Contents

Preface

iii

ESSAY QUESTIONS Chapter 1

What is Economics?

1

Appendix

Graphs in Economics

7

Chapter 2

The Economic Problem

9

Chapter 3

Demand and Supply

15

Chapter 4

Elasticity

21

Chapter 5

Efficiency and Equity

29

Chapter 6

Markets in Action

37

Chapter 7

Utility and Demand

47

Chapter 8

Possibilities, Preferences, and Choices

53

Chapter 9

Organizing Production

59

Chapter 10

Output and Costs

63

Chapter 11

Perfect Competition

69

Chapter 12

Monopoly

77

Chapter 13

Monopolistic Competition and Oligopoly

85

Chapter 14

Regulation and Antitrust Law

93

Chapter 15

Externalities

101

Chapter 16

Public Goods and Common Resources

109

Chapter 17

Demand and Supply in Factor Markets

115

Appendix

Market Power in the Labor Market

121

Chapter 18

Economic Inequality

123

Chapter 19

Uncertainty and Information

129

Chapter 20

Trading With the World

133

VII

VIII

NUMERIC AND GRAPHING QUESTIONS Chapter 1

What is Economics?

143

Appendix

Graphs in Economics

145

Chapter 2

The Economic Problem

153

Chapter 3

Demand and Supply

163

Chapter 4

Elasticity

169

Chapter 5

Efficiency and Equity

175

Chapter 6

Markets in Action

183

Chapter 7

Utility and Demand

191

Chapter 8

Possibilities, Preferences, and Choices

197

Chapter 9

Organizing Production

203

Chapter 10

Output and Costs

207

Chapter 11

Perfect Competition

215

Chapter 12

Monopoly

223

Chapter 13

Monopolistic Competition and Oligopoly

231

Chapter 14

Regulation and Antitrust Law

235

Chapter 15

Externalities

241

Chapter 16

Public Goods and Common Resources

247

Chapter 17

Demand and Supply in Factor Markets

249

Chapter 18

Economic Inequality

253

Chapter 19

Uncertainty and Information

257

Chapter 20

Trading With the World

259

IX

TRUE OR FALSE QUESTIONS Chapter 1

What is Economics?

263

Appendix

Graphs in Economics

265

Chapter 2

The Economic Problem

267

Chapter 3

Demand and Supply

269

Chapter 4

Elasticity

271

Chapter 5

Efficiency and Equity

273

Chapter 6

Markets in Action

275

Chapter 7

Utility and Demand

277

Chapter 8

Possibilities, Preferences, and Choices

279

Chapter 9

Organizing Production

281

Chapter 10

Output and Costs

283

Chapter 11

Perfect Competition

285

Chapter 12

Monopoly

287

Chapter 13

Monopolistic Competition and Oligopoly

289

Chapter 14

Regulation and Antitrust Law

291

Chapter 15

Externalities

293

Chapter 16

Public Goods and Common Resources

295

Chapter 17

Demand and Supply in Factor Markets

297

Appendix

Market Power in the Labor Market

299

Chapter 18

Economic Inequality

301

Chapter 19

Uncertainty and Information

303

Chapter 20

Trading With the World

305

X

EXTENDED PROBLEMS Chapter 1

What is Economics?

309

Chapter 2

The Economic Problem

313

Chapter 3

Demand and Supply

319

Chapter 4

Elasticity

325

Chapter 5

Efficiency and Equity

329

Chapter 6

Markets in Action

337

Chapter 7

Utility and Demand

343

Chapter 8

Possibilities, Preferences, and Choices

347

Chapter 9

Organizing Production

353

Chapter 10

Output and Costs

357

Chapter 11

Perfect Competition

361

Chapter 12

Monopoly

367

Chapter 13

Monopolistic Competition and Oligopoly

373

Chapter 14

Regulation and Antitrust Law

379

Chapter 15

Externalities

383

Chapter 16

Public Goods and Common Resources

387

Chapter 17

Demand and Supply in Factor Markets

391

Chapter 18

Economic Inequality

395

Chapter 19

Uncertainty and Information

399

Chapter 20

Trading With the World

405

CHAPTER 1

WHAT IS ECONOMICS?

(yet) provide this service. Alternatively, another rich person might enjoy life so much that he or she wants 25 hours in a day in order to have more time for more enjoyment. But, such a want is impossible. By way of another, perhaps more realistic example, Malcom Forbes was the founder of Forbes magazine and was very rich. However, he did not win every piece of art that he bid upon at auctions. Even though Mr. Forbes was very rich, he still passed on some art when the price got so high that he thought given his resources, the price exceeded what he was willing to pay. Mr. Forbes wanted the art, but he was not willing to bid higher in order to win it. Mr. Forbes faced scarcity.

Essay Questions Topic: Scarcity Skill: Recognition

1)

What do economists mean when they discuss “scarcity”? Answer: Scarcity occurs whenever people’s wants exceed the ability of the available resources to meet these wants. Because people’s wants are effectively infinite—it is always possible to imagine more good things to want to have—wants will always exceed what can be produced with the available resources, and so scarcity will always be present. Topic: Scarcity Skill: Conceptual

Topic: Scarcity Skill: Conceptual

2)

What is the relationship between wants, factors of production, scarcity, and choices? Discuss the relationship for an individual and for a society. Answer: A person faces scarcity whenever his or her wants exceed what he or she can obtain using his or her resources. Because the person cannot fulfill all of his or her wants, the person is forced to choose which wants will be satisfied and which wants will remain unsatisfied. The same results hold true for a society. All societies face scarcity because people’s wants are essentially infinite, so that the factors of production available are not sufficient to fulfill everyone’s wants. Because of this fact, societies must make choices about which (and whose) wants will be satisfied and which (and whose) wants will remain unsatisfied.

4)

Explain why both rich and poor people experience scarcity. Answer: Scarcity exists when people’s wants exceed their ability to satisfy the wants. People’s wants are literally infinite, so just as a poor person can want more, so too can a richer person. Therefore both rich and poor experience scarcity. Topic: Scarcity Skill: Conceptual

5)

What is the difference between scarcity and poverty? Answer: Scarcity exists when availability is less than people want. Poverty exists when availability is less than people need. Everyone suffers scarcity; only an unfortunate minority suffers poverty.

Topic: Scarcity Skill: Conceptual

Topic: Definition of Economics Skill: Recognition

3)

Why do economists say that even very rich people face scarcity? Answer: A person faces scarcity whenever his or her wants exceed what he or she can obtain using his or her resources. Even very rich people want things that they cannot have. An older rich person, for instance, might want to have all of his or her youthful energy, but medical science cannot

6)

Define economics and describe its branches of study. Answer: Economics is the social science that studies the choices made by individuals, businesses, government, and entire societies as they cope with scarcity. It has two branches, microeconomics and macroeconomics. Microeconomics is the study of the choices made by individuals and businesses, 1

2

CHAPTER 1

the way they interact, and the influence that governments exert on these choices. Macroeconomics is the study of the aggregate (total) effects on the national economy and the global economy of the choices that individuals, businesses, and governments make. Topic: Microeconomics and Macroeconomics Skill: Recognition

7)

What is the difference between microeconomics and macroeconomics? Answer: Microeconomics studies the decisions of smaller economic actors, such as individual consumers or individual firms, and how the government can affect these decisions, say through how it regulates an industry. Macroeconomics studies the aggregate, or economy-wide, consequences of the decisions made by individuals and firms. Macroeconomics also studies the aggregate effects of government policies, such as the Federal Reserve’s decisions to raise or lower interest rates. Topic: Microeconomics and Macroeconomics Skill: Conceptual

8)

What is the difference between microeconomics and macroeconomics? Give an example of an issue each studies. Answer: Essentially microeconomics studies individual units within the economy, such as the choices made by individual consumers or individual firms. Macroeconomics studies the overall or aggregate economy. Microeconomics examines the factors that affect employment at an individual firm. Macroeconomics examines the factors that affect economy-wide unemployment. Topic: Microeconomics and Macroeconomics Skill: Conceptual

9)

Below is a student’s answer to the question “What is microeconomics?” If you were the instructor, how would you correct the student’s answer? “Microeconomics is the study of how government influences the choices made by individuals and businesses and of the performance of the whole national economy.” Answer: The answer is partially correct. Microeconomics is the study of the choices that individuals and businesses make, the way these choices interact in markets, and the influence of the government. But the performance of the national econ-

omy is the subject of macroeconomics, not microeconomics. Topic: Human Capital Skill: Conceptual

10) China’s population is about 1.2 billion, while the population of the United States is about 280 million. This fact means that China has much more human capital than the U.S. does. True or false? Explain your answer. Answer: False. Population can measure the quantity of a nation’s labor resource, but the population numbers don’t tell us anything about skills that this labor force obtained from education, on-thejob training, and work experience, which are called human capital. Thus, the population numbers in the statement only tell us that China is likely to have more labor than the United States, but it does not necessarily mean that it also has more human capital. Topic: Entrepreneurship Skill: Recognition

11) Explain what entrepreneurship is and why it is considered a factor of production. Answer: Entrepreneurship is the resource (the people) that runs businesses. Entrepreneurs organize the other resources, land, labor, and capital. It is a factor of production because people with the desire and talent to successfully organize a business are needed to run businesses. Topic: Self-Interest and Social Interest Skill: Conceptual

12) An analyst on a local news channel argues that the recent corporate scandals “demonstrated very clearly that self interest always contradicts social interest.” Do you agree or disagree? Substantiate your answer. Answer: You should disagree. The recent corporate scandals only show that self interest might contradict social interest. But they don’t prove that this is necessarily the case as we can find many realworld examples of how people guided by selfinterest promote society’s well-being. In fact, under the market system the whole economy operates through the decisions made by self-interested individuals. And countries such as the United States have proven to be more successful in promoting social interest than were centrally planned, or communist, economies where peo-

WHAT IS ECONOMICS?

ple’s self interest was suppressed and all important economic decisions were made by government. Topic: Tradeoffs Skill: Conceptual

13) What is a tradeoff? Give an example. Answer: A tradeoff occurs when one thing must be given up to get another. Tradeoffs are pervasive; at the personal level, students tradeoff time spent studying for time they otherwise could have spent socializing. Topic: Opportunity Cost Skill: Recognition

14) What is opportunity cost? Answer: Opportunity cost is the highest-valued alternative given up when selecting an action. For instance, the opportunity cost of studying an hour is whatever the highest-valued alternative would have been for the hour spent studying. Topic: Opportunity Cost Skill: Conceptual

15) What is an opportunity cost? Give an example of an opportunity cost that is paid in money and an opportunity cost that is not explicitly paid. For each example, explain why you think this is an opportunity cost. Answer: An opportunity cost of something is the highest-valued alternative you give up to get it. An example of an opportunity cost paid in money is the cost of tuition that a student pays to get his or her college degree. This expenditure is an opportunity cost because to get a college degree, the student gives up goods and services that he or she would have bought for the money spent on tuition. If this student quits a job to go to college, the student also gives up the money he or she could have earned working. This opportunity cost is an example of an opportunity cost that is not explicitly paid in money. Topic: Opportunity Cost Skill: Conceptual

16) Your friend is preparing for this exam and in your practice session makes the following statement: “Instead of attending microeconomics class for two hours, Kiki could have played tennis or watched a movie. Therefore, the opportunity cost of attending class is the tennis and the movie she

3

had to give up.” Is your friend’s analysis correct or not? Explain your answer. Answer: Your friend’s analysis is incorrect. The opportunity cost of an action is the highest-valued alternative forgone, not all alternatives forgone. Kiki’s opportunity cost of studying for her exam is either the tennis or the movie, whichever she would have done had she not studied. Topic: Opportunity Cost Skill: Conceptual

17) Rather than go out to eat by yourself, you decide to stay at home and fix dinner for yourself and your two roommates. Your roommates applaud your decision. Your first roommate tells you that your decision to eat at home has no opportunity cost because you already have all the dinner ingredients in your pantry. Is this roommate’s comment correct? Answer: Your first roommate’s comment is incorrect. The opportunity cost of preparing dinner at home is whatever is the highest-valued alternative forgone, which, given your choice boiled down to staying home or going out, is going out to eat. Hence the opportunity cost of fixing dinner at home is going out to eat. Topic: Opportunity Cost Skill: Conceptual

18) A student can spend the next hour studying for a finance test, hiking along the Oregon coast, watching a rerun of Buffy the Vampire Slayer on television, or napping. If the student decides to study, what is the opportunity cost of her choice: hiking, watching television, or napping? Answer: With the information given, it is impossible to determine the opportunity cost. The opportunity cost is the highest-valued alternative forgone and the problem does not give the student’s ranking of the options. For instance, if the student thinks that if she had not studied she would have watched Buffy, then watching Buffy is the opportunity cost. However, if the student thinks that if she were not studying, she would be strolling along the beach, then the beach walk is the opportunity cost.

4

CHAPTER 1

Topic: Marginal Benefit and Marginal Cost Skill: Recognition

Topic: Positive and Normative Skill: Recognition

19) Define marginal cost and marginal benefit. Answer: Marginal cost is the opportunity cost of a small (one-unit) increase in an activity. Marginal benefit is the benefit of a small (one-unit) increase in an activity.

23) What is the difference between positive and normative statements? Answer: Positive statements tell what is and normative statements tell what ought to be. Positive statements can be tested to determine if they are correct or not, while normative statements use value judgments and so cannot be tested. For example, two economists might agree on the positive assertion that if the government spent its funds purchasing pharmaceutical drugs for poor older Americans rather than poor children, then poor older Americans would use more drugs and poor children would use fewer. But they might disagree on the normative conclusion of whether the government should pursue this policy. One economist might argue “It is not fair to have senior citizens suffer because they cannot afford medicine” and the other economist might argue “It is not fair to have children suffer because their parents cannot afford medicine.”

Topic: Marginal Benefit Skill: Recognition

20) What is the difference between a total benefit and a marginal benefit? Answer: The total benefit is all the benefit from all of an activity. The marginal benefit is the additional benefit from an additional amount of an activity. Topic: Incentives, Marginal Cost and Marginal Benefit Skill: Conceptual

21) In New State, the bottling law requires that people get a refund of five cents when they return an empty bottle or can. Why does the state pay people to return bottles? In your answer, be sure to mention the role played by incentives. Answer: Policy makers know that people making choices respond to incentives. Instead of throwing away bottles and cans, people will now bring the used bottles and cans to the designated areas for recycling in order to receive their payment. Thus policy makers have taken advantage of people’s decision making by increasing the marginal benefit of returning bottles in order to reduce litter and clean the environment. Topic: Incentives, Marginal Cost and Marginal Benefit Skill: Conceptual

22) If the government raises the tax on cigarettes, what is the effect on people’s incentives and choices? Answer: The government raises the tax on cigarettes to discourage smoking. With a higher tax the price of cigarettes rises. The opportunity cost of smoking increases, which gives people incentive to cut their consumption of cigarettes.

Topic: Positive and Normative Skill: Recognition

24) “The difference between positive and normative statements is that a positive statement is always true while a normative statement might or might not be true.” True or false? Explain. Answer: False. The difference between positive and normative statements is that a positive statement is about what is, while a normative statement is about what ought to be. A positive statement can be tested against the facts and may be proved to be right or wrong, whereas a normative statement depends on values and cannot be tested. Topic: Positive and Normative Skill: Conceptual

25) Two economists can agree that raising the minimum wage creates unemployment yet one might argue that raising the minimum wage is a good policy and the other that it is a bad policy. Why can this difference exist? Be sure to use the terms positive and normative in your answer. Answer: Positive statements are statements that describe how the world is. Positive statements can be tested and so, ultimately, any disagreements about positive statements should be resolved. The statement that “Raising the minimum wage cre-

WHAT IS ECONOMICS?

ates unemployment” is a positive statement and, on the basis of repeated testing, most economists agree that it is a correct positive statement. Normative statements, however, are statements that describe how the world ought to be. Normative statements depend on people’s values and cannot be tested. So one economist might argue that raising the minimum wage is a good policy because this economist thinks that, although it is unfortunate that some people lose their jobs, the fact that others retain their jobs and their wages rise more than outweighs the harm created by the unemployment. Another economist might strongly differ because the second economist thinks that the harm inflicted on people who lose their jobs more than outweighs any good from some workers being paid more. This difference of opinion can last indefinitely because there is no way to test the two economists’ beliefs to determine which is correct. Topic: Positive and Normative Skill: Conceptual

26) What is a positive statement? Give an example. Answer: A positive statement addresses “what is” and can be tested. An example of a positive statement is “An increase in the price of gas decreases the quantity of gas demanded.” Topic: Positive and Normative Skill: Conceptual

27) Explain whether the statement, “There is life on Mars,” is a normative or positive statement. Answer: The statement is a positive statement because it does not depend on a value judgment. Instead, it is a statement that tries to describe “what is” and hence is testable. Of course, in order to test the assertion, it would be necessary to go to Mars to ascertain if there is life present. While it is difficult (!) at present to actually carry out the test, nonetheless the statement is testable and hence is a positive statement. Topic: Positive and Normative Skill: Conceptual

28) Explain whether the statement, “Hillary Clinton was elected President of the United States in 2000,” is a normative or positive statement. Answer: The statement is a positive statement because it does not depend on a value judgment. Instead, it is a statement that tries to describe “what is” and hence is testable. Now, it is indeed the

5

case that Hillary Clinton was not elected president in 2000, so when we test the statement we discover that it is incorrect. But, whether the statement is correct or not has no bearing on whether the statement is positive or normative. Thus, the statement “Hillary Clinton was elected President in 2000” is a positive, albeit incorrect, statement. Topic: Positive and Normative Skill: Conceptual

29) What is a normative statement? Give an example. Answer: A normative statement is a statement about what ought to be. It is a value judgment or opinion and so cannot be proven true or false. An example of a normative statement is “Students should attend school year round to receive a better education.” Topic: Positive and Normative Skill: Conceptual

30) Explain whether the statement “The government should increase tariffs on Japanese cars to protect the American car industry from competition,” is a normative or positive statement. Answer: The statement is normative. The statement is a normative statement because it depends on a value judgment, namely that the government should protect the American car industry from competition. Topic: Economic Science Skill: Recognition

31) List the three tasks of economic science. Briefly explain each. Answer: The three tasks are observing and measuring, model building, and testing. These tasks involve: ♦ Observing and measuring – economists must collect data on the different topics they study. These data measure a variety of activities including the number of hours worked, the level of interest rates, output of specific firms or an entire economy, and so on. ♦ Model building – economists simplify reality by describing the economic world using models. Models can be verbal, use mathematics, or be graphical in nature. ♦ Testing – economists test the models they formulate to see how closely their descriptions of reality mimic actually match it. If the models

6

CHAPTER 1

perform well repeatedly, they form the basis for an economic theory. Topic: Models Skill: Conceptual

32) Why is it necessary for models to simplify reality? Answer: The real world is incredibly complex. However, most of the complexity concerns relatively trivial details that do not affect how events function. For instance, an economic model of driving an automobile might focus on the engine, the transmission, the tires, the brakes, and the steering mechanism while assuming the absence of a radio, an air conditioner, a heater, the back seats, the doors, and so forth. The point is that an automobile will function without the unessential components that are ignored and, if they were included, the key factors that actually make the car run might well be lost to our vision in a morass of detail. In order to better understand the factors that actually enable an automobile to run, a model of an automobile simplifies the real automobile by ignoring nonessential details. Topic: Economics: A Social Science Skill: Conceptual

33) What does ceteris paribus mean and why do economists use it? Answer: Ceteris paribus is Latin for “other things being equal” or “if all other relevant things remain the same.” Economists use the ceteris paribus assumption because in the real world most economic variables are influenced by more than one other variable. By invoking the ceteris paribus assumption, economists can focus on the effect of any one variable by itself and thereby gain a better understanding of the impact of each variable.

Topic: Economics: A Social Science Skill: Conceptual

34) What is the importance of the ceteris paribus assumption? Answer: As a social science, economics is concerned with how societies deal with the issue of scarcity. In order to simplify the world and focus on a particular aspect, economists try to isolate the ramifications of making a small change in only the factor of interest. Topic: Post Hoc Fallacy Skill: Recognition

35) What is the post hoc fallacy? Give an example of the post hoc fallacy. Answer: The post hoc fallacy is an error in reasoning that confuses correlation with causation. The post hoc fallacy occurs when someone reasons that because one event occurred before a second event, the first event caused the second. While it is often the case that an event occurs first and thereby causes a second event, this reasoning is not always correct. For instance, on the way to take this exam, you might have found and picked a fourleaf clover or your path might have been crossed by a black cat. But, if you earn a high score on this exam, very few people would make the post hoc fallacy of attributing the high score to the (alleged) good luck from the four-leaf clover even though picking the clover occurred before you took the test. Similarly, if you receive a poor score on this exam, few would attribute the low score to the (alleged) bad luck from the black cat, even though the cat crossed your path before you took the exam.

APPENDIX 1 GRAPHS IN ECONOMICS Topic: Graphing Data Skill: Recognition

1)

Why do economists use graphs? Answer: Graphs help economists, and others, to visualize the relationships between economic variables. Graphs that plot variables together help economists understand if the variables are related and how they are related. Graphs also help provide a visual picture of economic models that link different variables. Indeed, many other disciplines use such visual models. For example, architects work with blueprints (their model) and the blueprints represent every detail of a building. Economists’ models do not reflect of every detail of the real world, but the graphs that they use nonetheless are valuable because they help clarify the linkages between the variables. Topic: Graphs Skill: Recognition

2)

What are three main kinds of graphs? Answer: The three main types of graphs are scatter diagram, time-series graph, and cross-section diagram. Topic: Time Series Skill: Recognition

3)

What kind of information is conveyed in a timeseries graph? Answer: A time series graph reveals four types of information. First, it shows the actual value of the variable(s) at each point in time. Second, it shows whether the variable(s) is rising or falling as time passes. Third, it shows the speed with which the variable(s) is changing. Finally, it shows the presence—or absence—of a trend. Topic: Relationships Skill: Recognition

4)

What are the two different types of relationships that variables can have? Explain each. What do

these relationships look like when they are graphed? Answer: Variables can have two relationships: positive (or direct) and negative (or inverse). A positive relationship occurs when the variables move in the same direction, so that when one increases, the other also increases. A negative relationship occurs when the variables move in the opposite direction, so that when one increases, the other decreases. When a positive relationship is graphed, the line slopes upward to the right. When a negative relationship is graphed, the line slopes downward to the right. Topic: Relationships Skill: Recognition

5)

What is the difference between a positive and a negative relationship? Answer: Two variables are positively related when an increase (decrease) in one is associated with an increase (decrease) in the other. In this case, the variables move together, in the same direction. Two variables are negatively related when an increase (decrease) in one is associated with a decrease (increase) in the other. In this case, the variables move in the opposite direction. Topic: Unrelated Variables Skill: Recognition

6)

A graph of two variables is a vertical line. What is the interpretation of this result? Answer: When the graph of two variables is a vertical line, the variables are not related because, with this graph, whenever the variable measured along the vertical axis changes, the variable measured along the horizontal axis does not change.

8

Topic: Slope Skill: Recognition

7)

What does the slope of a straight line equal? How is the slope of a curved line calculated? Answer: The slope of a straight line is calculated between two points on the line. Between the two points on the line, the slope equals the change in the value of the variable measured on the vertical axis (the y-axis) divided by the change in the value of the variable measured on the horizontal axis (the x-axis). The slope of a curved line is calculated at a point on the line. At that point on the curved line, draw a straight line that touches the curved line at only that point. Then, calculate the slope of the straight line. The slope of the curved line at that point equals the slope of the straight line.

APPENDIX 1

Topic: Relationships Among More Than Two Variables Skill: Recognition

8)

“It is impossible to represent a three variable relationship in a two-dimensional graph.” Is this statement true or false? Explain your answer. Answer: The statement is false because it is possible to represent a three variable relationship in a two dimensional graph. To do so, start by focusing on two of the variables. Assume that the third variable does not change (the ceteris paribus assumption) and then graph the relationship between the two variables. The graph shows how these two variables are related when the third variable does not change. When the third variable does change, then the entire relationship between the two graphed variables changes. In other words, the line showing the relationship between the two graphed variables shifts so that it becomes an entirely new line. The shift in the line shows how the third variable influences the other two.

CHAPTER 2

THE ECONOMIC PROBLEM

Topic: Production Possibilities Frontier Skill: Recognition

Topic: Production Possibilities Frontier Skill: Recognition

1)

A production point beyond the production possibilities frontier represents what? Answer: A production point beyond the production possibilities frontier is an unattainable combination of goods and services.

5)

Topic: Production Possibilities Frontier Skill: Recognition

Topic: Production Possibilities Frontier Skill: Conceptual

2)

6)

Explain how the production possibilities frontier illustrates scarcity. Answer: The PPF illustrates scarcity because we cannot attain the points outside the frontier.

Topic: Production Possibilities Frontier Skill: Recognition

3)

“If Mexico is currently operating at a point beyond its production possibilities frontier, then there are unemployed resources in Mexico.” Is this statement true or false? Briefly explain your answer. Answer: The statement is false. It is false on two counts. First, production points beyond the production possibilities frontier are unattainable, so it is not possible for Mexico to be producing at such a point. Second, points within not beyond the production possibilities frontier have unemployed resources. Topic: Production Possibilities Frontier Skill: Recognition

4)

“If Mexico is currently operating at a point inside its production possibilities frontier, then there are unemployed resources in Mexico.” Is this statement true or false? Briefly explain your answer. Answer: The statement is true. Points within the production possibilities frontier are attainable, so it is possible for Mexico to be producing at a point within its frontier. At points within the production possibilities frontier, there are unemployed resources.

Are all points inside the production possibilities frontier unattainable? Answer: No, all points within the production possibilities frontier are attainable, though there are unemployed resources at these points.

A Russian economist says: “There was no unemployment in the former Soviet Union, but I believe the Soviet economy produced below its PPF.” How could this statement be correct? Answer: An economy can produce at a point within its PPF for two reasons: (1) unemployment; (2) misallocated resources. So even if there was no unemployment in the Soviet economy, it still could be below its PPF because if resources are not assigned to tasks for which they would be the best match. Topic: Production Possibilities and Tradeoffs Skill: Conceptual

7)

In the movie Cast Away, Tom Hanks plays a FedEx efficiency expert stranded on a deserted island. While on the island, he divides his time between catching fish, gathering coconuts, painting, and building a raft. Suppose that these were Mr. Hanks’ only activities. Did he face an opportunity cost from pursuing any of these activities? Why or why not? Answer: Yes, Mr. Hanks faces an opportunity cost from all of these endeavors. If he decides to use his time catching fish, he cannot gather coconuts, paint, or build a raft. Whatever he would have been doing, not opting to catch fish is his opportunity cost of catching fish. Similarly, time spent on building his raft means less time painting, or fewer coconuts for breakfast, or fewer fish for dinner.

10

Topic: Production Possibilities and Opportunity Costs Skill: Conceptual

8)

Explain the connection between opportunity cost and the PPF. Answer: When moving along the production possibilities frontier, more of one good or service can be obtained only by giving up another good or service. The good or service given up is the opportunity cost of the good or service obtained. Topic: Production Possibilities Frontier Skill: Conceptual

9)

Explain why the production possibilities frontier bows outward. Answer: The bowed outward PPF reflects increasing opportunity costs. The opportunity cost increases as we produce more of a good because resources are not equally productive in all activities. For example, people with several years of experience working for Sony are good at producing CDs, but not very good at making pizza. So if we want more pizza and move some of these workers from Sony to Domino’s, we get a relatively large decrease in the quantity of CDs per one additional pizza produced. Of course, first we try to move those workers who have less experience with Sony and who may have some skills to produce pizza. But if we want to produce even more pizza, we eventually will have to move some more experienced Sony workers to Domino’s, where they might be very unproductive and therefore the quantity of CDs that we will give up to produce an additional pizza, which is the opportunity cost of producing pizza, will increase. Topic: Production Possibilities Frontier Skill: Conceptual

10) How can a combination of goods be unattainable? Answer: A combination of goods can be unattainable if producing that combination requires more resources and technology than are available. These combinations of goods lie beyond the production possibilities frontier.

CHAPTER 2

Topic: Production Possibilities Frontier Skill: Conceptual

11) What economic concepts are represented in the production possibilities model? Answer: There are a large number of economic concepts illustrated by the production possibilities frontier: ♦ Scarcity of resources: The production possibilities frontier is a frontier between attainable and unattainable combinations. ♦ Opportunity cost: The negative slope of the production possibilities frontier indicates that in order to get more of one good, you must produce less of the other (a tradeoff). ♦ Increasing opportunity cost: The bowed out production possibilities frontier represents the increasing opportunity cost when more of a good or service is produced. ♦ Efficiency: Points on the production possibilities frontier use all resources while points below the production possibilities frontier represent unemployment. Topic: Increasing Opportunity Cost Skill: Conceptual

12) Moving on a bowed out PPF, what happens to the opportunity cost of a good as more of it is produced? Answer: The bowed out PPF indicates that as the amount of the good produced increases, the good’s opportunity cost increases. Topic: Increasing Opportunity Cost Skill: Conceptual

13) Why is the production possibilities frontier bowed out? Answer: The production possibilities frontier is bowed out because resources are not equally productive in all uses. The resources used to produce robots are different from the resources used to produce pizzas. Thus, as more of one good is produced, say robots, less productive resources must be used to increase the number of robots produced. Hence the opportunity cost of the additional robots increases, which gives the production possibilities frontier a bowed out shape.

THE ECONOMIC PROBLEM

Topic: Increasing Opportunity Cost Skill: Conceptual

14) When does the production possibilities frontier have a bowed out shape rather than be a straight line? Answer: If as the production of one good or service increases, its opportunity cost increases means that the production possibilities frontier will be bowed out. Only if the opportunity cost remains constant as the production of a good increases is the production possibilities frontier a straight line. Topic: Increasing Opportunity Cost Skill: Conceptual

15) When economists state that the opportunity cost of a product increases as more of it is produced, what do they mean? What is the opportunity cost? Answer: In general, the opportunity cost of increasing the production of one good or service is the forgone production of another good or service. The statement that the opportunity cost of a product increases as more of it is produced applies to production points on the production possibilities frontier. On the production possibilities frontier, resources are fully employed. Hence to increase the production of one good or service, resources must be switched away from the production of another good or service and hence the production of that good or service decreases. And, as more of the first good or service is produced, the opportunity cost of an additional unit becomes larger, so that the opportunity cost increases. Topic: Increasing Opportunity Cost Skill: Conceptual

16) What is the relationship between the bowed out shape of the production possibilities frontier and the increasing opportunity cost of a good as more of it is produced? Answer: The production possibilities frontier is bowed out because the opportunity cost of a good increases as more of it is produced. As the first unit of the good measured along the horizontal axis is produced, resources that are extremely well suited for its production can be used. Because of the suitability, not many resources need to be devoted to its production, so the opportunity cost—the decrease in the production of the good measured along the vertical axis—is not large. At this loca-

11

tion along the production possibilities frontier, the slope of the production possibilities frontier is shallow. But, as more of the product is produced, resources that are not as well suited must be devoted to its production. Consider one of the last units of this good, just before the production possibilities frontier intersects the horizontal axis. By the time the nation produces this much of the product, to produce one more unit means that resources that are extremely poorly suited in its manufacture must be used. Because these resources are not well suited, a lot of them must be used and, because a lot of them must be used, the opportunity cost in terms of the forgone other good is large. With the large decrease in the production of the good along the vertical axis, the slope of the production possibilities frontier at this location is steep. So, the production possibilities frontier goes from having a shallow slope to a steep one, that is, the production possibilities frontier is bowed outward. Topic: Marginal Benefit Skill: Recognition

17) Why does the marginal benefit curve have a negative slope? Answer: Each successive increase in the consumption of any good or service provides a lower level of satisfaction, or benefit, than the preceding unit of consumption. For a specific example, think of drinking water on a hot day. What is the first glass worth? How about the second and third? The marginal benefit is the benefit from each additional glass of water and, as the example indicates, the marginal benefit decreases as the amount of the good increases. Topic: Marginal Cost and Marginal Benefit Skill: Conceptual

18) Explain the difference between marginal cost and marginal benefit. Answer: Marginal benefit is the benefit someone in society obtains when another unit of a good or service is produced. Marginal cost is the cost to someone in society of producing another unit of a good or service.

12

Topic: Efficient Use of Resources Skill: Conceptual

19) Compare and contrast production and allocative efficiency. Answer: Production efficiency means that we are operating at a point on the production possibilities frontier and so we cannot produce more of a good or service without producing less of some other good or service. Production efficiency occurs at all points on the PPF. At any point inside the frontier, production is inefficient because we have unemployed resources. Allocative efficiency means that we are producing the goods and services that society values most highly and so allocative efficiency implies that we are operating on the frontier. But not every point on the production possibilities frontier is the combination of goods and services valued most highly by society. Allocative efficiency only occurs at a single point on the PPF. To insure that allocative efficiency exists, we must compare the marginal benefit of a good with its marginal cost. When production is such that the marginal benefit equals the marginal cost, then we are producing the allocatively efficient level of output. Topic: Allocative Efficiency Skill: Conceptual

20) “Allocative efficiency in the production of cherries means that consumers can eat all of the cherries they desire.” Is this statement true or false? Answer: Allocative efficiency means that we are producing the goods and services society values most highly. It does not mean that consumers can afford all of the cherries that they desire. The allocatively efficient quantity of cherries is the level of production such that the marginal benefit of a pound of cherries equals the marginal cost of a pound of cherries. The marginal cost of any product will be positive, so the marginal cost of a pound of cherries at the allocatively efficient quantity will be positive. Hence for the allocatively efficient quantity of cherries, the marginal benefit of cherries also must be positive. In order for consumers to have all the cherries they desire, the marginal benefit of a pound of cherries must be zero. (If the marginal benefit is positive, consumers desire more cherries.) Therefore the allocatively efficient quantity of cherries is not the

CHAPTER 2

quantity at which consumers are able to eat all they desire. Topic: Allocative Efficiency Skill: Conceptual

21) “If an economy is producing at a point on its PPF, it has achieved allocative efficiency.” True or false? Explain. Answer: If an economy is producing at a point on its PPF, it has achieved production efficiency, but not necessarily allocative efficiency. We have achieved allocative efficiency if we are producing at the point on the PPF that we prefer to all other points because at this point we cannot produce more of any good without giving up some other good that we value more highly. Topic: Allocative Efficiency Skill: Conceptual

22) “Allocative efficiency requires that the maximum number of people have access to all of the goods and services that our economy produces.” Is this statement true or false? Explain your answer. Answer: The statement is false. Allocative efficiency requires that production be such that the marginal benefit equals the marginal cost. Allocative efficiency has nothing to do with requiring that the maximum number of people have access to all the goods and services produced. Topic: Economic Growth Skill: Recognition

23) What factors generate economic growth? Answer: Two key factors create economic growth: Technological change and capital accumulation, including the accumulation of additional human capital. Both technological change and capital accumulation shift the nation’s PPF outward. Topic: Economic Growth Skill: Conceptual

24) Country X and Country Y are both efficient in production. Country X devotes one third of its resources to accumulating capital and the other three fourth to consumption. Country Y devotes one fourth of its resources to accumulating capital and two thirds to consumption. Which country will grow faster? In which country the opportunity cost of economic growth is higher? Explain. Answer: Country X will grow faster because it devotes a greater fraction of its resources to accumulating

THE ECONOMIC PROBLEM

capital. As a result, Country X will have more productive resources in the future, which will allow it to expand its production possibilities more quickly, so its economy will grow faster. But the opportunity cost of economic growth is also higher in Country X. This country sacrifices a greater fraction of current consumption to produce more capital. Topic: Comparative Advantage Skill: Conceptual

25) What is comparative advantage? Give an example. Answer: Comparative advantage is the ability of a person to produce a good at a lower opportunity cost compared to another person. A lower opportunity cost means that the person gives up less to produce the good compared to another person. For example, one person may need to give up one hour of typing to get dinner made while another person must give up two hours of typing to produce the same dinner. Topic: Comparative Advantage and Absolute Advantage Skill: Conceptual

26) “When a person has an absolute advantage in producing a good, the person necessarily has a lower opportunity cost of producing it.” Is this assertion true or false? Answer: The assertion is incorrect. An absolute advantage is when a person can produce more of the good than someone else. A comparative advantage relies on a comparison of opportunity costs, so a person has a comparative advantage in producing a good if the person can produce the good at a lower opportunity cost. Topic: Comparative Advantage and Absolute Advantage Skill: Recognition

27) “When a person can produce more of a good or service than another person, the first person has the comparative advantage in producing the good.” Is this assertion correct or incorrect? Explain your answer. Answer: The assertion is incorrect. The statement describes an absolute advantage, that is, a person has an absolute advantage in the production of a good if the person can produce more of it than someone else. Comparative advantage, however, relies on a comparison of opportunity costs. A person

13

has a comparative advantage in producing a good if the person can produce the good at a lower opportunity cost than another person. Topic: Comparative Advantage and Absolute Advantage Skill: Conceptual

28) Why is it likely that the United States has an absolute advantage in goods and yet it still ends up importing them from other countries? Answer: The United States might have an absolute advantage in producing a good but not a comparative advantage. In this case, the opportunity cost of producing the good in the United States is higher than in another country. Thus the United States will import the product from the other country. Topic: Comparative Advantage and Absolute Advantage Skill: Conceptual

29) The United States has an absolute advantage in producing sugar over all of the other sugar producing countries. Does this fact mean that we should not import any sugar from the other countries? Answer: Having an absolute advantage doesn’t mean that the United States should engage in the production of sugar. If the opportunity cost of sugar in the United States is higher than in the other countries, then the other countries will have the comparative advantage. The countries with the comparative advantage are the ones that should do the producing. Quite likely these other nations have the comparative advantage and so it would be good policy for the United States to import sugar from these nations. Topic: Comparative Advantage and Absolute Advantage Skill: Conceptual

30) “If the United States has an absolute advantage in the production of wheat, it should specialize in wheat and export wheat.” True or false? Explain. Answer: Whether or not the United States should specialize in the production of wheat and export wheat depends on whether or not it has a comparative advantage in producing wheat. The fact that the United States has an absolute advantage in wheat does not necessarily mean that it also has a comparative advantage in this product.

14

Topic: Dynamic Comparative Advantage Skill: Recognition

31) What is a dynamic comparative advantage? What generates it? What are some examples of nations that have used this type of comparative advantage? Answer: A dynamic comparative advantage is generated by learning-by-doing. As a nation trains its labor force to produce a good or service, initially the nation might not have a comparative advantage in producing the good or service. But if there is learning-by-doing, as more of the good or service is produced, the workers learn how to produce it at a lower opportunity cost. Eventually the opportunity cost falls enough so that the nation winds up possessing a comparative advantage in the production of the good or service. Hong Kong, South Korea, and Taiwan are countries that have pursued and gained dynamic comparative advantages, particularly in electronics and biotechnology. Topic: Gains from Trade Skill: Recognition

32) What is the difference between comparative advantage and absolute advantage? Answer: A person has a comparative advantage if he or she can produce a good or service at lower opportunity cost than anyone else. In other words, for this person to produce a good or service, fewer other goods or services must be given up. A person has an absolute advantage if he or she can produce more of a good or service in a given time period than anyone else. Absolute advantage does not indicate the (opportunity) cost of producing the particular good or service. Topic: Gains from Trade Skill: Conceptual

33) Why does it make sense for economies to specialize according to comparative advantage and trade? Answer: A person has a comparative advantage in an activity that they can perform at a lower opportunity cost than other people. By participating in the activity in which they have a comparative advantage, less is given up. Because resources are scarce, more can be produced with available resources when less is given up. And, by trading

CHAPTER 2

people can consume more than just what they produce. Topic: Gains from Trade Skill: Conceptual

34) “The United States is more productive in most activities than are most of other countries because it has an absolute advantage in the production of most goods and services. Therefore we should restrict international trade as it only benefits other countries at the expense of the United States.” Comment on this statement. Answer: The United States may be more productive than other countries in producing most goods and services so that it has an absolute advantage in most products, but it sill has a comparative disadvantage in many goods and services. Thus the United States and can gain from buying these goods and services from other countries and selling to these other countries the goods and services in which the United States has a comparative advantage to them. For example, the United States can have an absolute advantage over China in producing both cars and grain, but if China has a comparative advantage in grain, it can produce grain with a lower opportunity cost, that is, fewer cars given up to get a thousand tons of grain, than can the United States. In this case, the United States can benefit by importing Chinese grain and paying for it with fewer cars then the United States would have to give up if the same amount of grain was produced domestically. Topic: The Market Economy Skill: Conceptual

35) How do property rights help organize production and trade? Answer: Property rights are necessary in order for people to specialize. If people specialize in production, they will want to consume more than just what they produce. Without property rights, people would worry that someone else would take their production, leaving them with little or nothing to trade for the other goods and services they want to consume.

CHAPTER 3

DEMAND AND SUPPLY

Topic: Price and Opportunity Cost Skill: Conceptual

Topic: Law of Demand Skill: Recognition

1)

4)

Explain why a relative price is an opportunity cost. Answer: A relative price is the ratio of the price of one good or service to the price of another good or service. It tells us how much of one good or service must be given up in order to obtain more of the other good. Topic: Price and Opportunity Cost Skill: Conceptual

2)

What is the difference between a money price and a relative price? When the demand and supply model predicts that the price of coffee will rise, is the model predicting that the money price rises or the relative price rises? Answer: The money price of a good is the number of dollars that must be given up in exchange for it. A relative price is the opportunity cost of a good in terms of another good. A relative price of good X is the quantity good Y that we forgo to get a unit of good X. When the supply-and-demand model predicts that the price of coffee will rise, it is the relative price, that is the model predicts that the price of coffee will rise relative to the average price of other goods. The money price might or might not rise. Topic: Law of Demand Skill: Recognition

3) What is the law of demand? Answer: The law of demand states that other things remaining the same, if the price of a good rises, the quantity demanded of that good decreases, and if the price of a good falls, the quantity demanded of that good increases.

What leads to a decrease in the quantity demanded of a good or service? Answer: The quantity demanded of a good or service decreases when the price of the product increases. Topic: Demand Curve Skill: Conceptual

5)

An economist says: “The demand curve has two interpretations.” What does the economist mean? Answer: The first interpretation is that the demand curve shows the quantities of a good or service that consumers are willing and able to buy at each price, other things being equal. The second interpretation is that the demand curve is a willingness-and-ability-to-pay curve, so that for each quantity it shows the highest price that someone is willing and able to pay for one more unit. Topic: Change in Demand Skill: Recognition

6)

List the factors change demand and shift the demand curve. Tell what happens to demand and the demand curve when there is an increase in the factor. Answer: One factor that changes demand is a change in income. An increase in income increases demand and shifts the demand curve rightward for a normal good. An increase in income decreases demand and shifts the demand curve leftward for an inferior good. A change in the price of a substitute or complement also changes demand. An increase in the price of a substitute increase demand and shifts the demand curve rightward while an increase in the price of a complement decreases demand and shifts the demand curve leftward. Expectations, the population, and preferences also change demand. If people expect their income to increase, or if they expect the price of the good to be higher in the future, or if the population in-

16

CHAPTER 3

creases (so that the number of buyers increases), or if people’s preferences for the good increase, demand increases and the demand curve shifts rightward. Topic: Change in Demand, Prices of Related Goods Skill: Analytical

7)

Computers are a complement to computer software. Suppose the price of a computer falls. How does this fall in price affect the demand for computer software and the demand curve for computer software? Answer: The fall in the price of a complement increases the demand for a product. Hence the fall in the price of a computer increases the demand for computer software and shifts the demand curve for computer software rightward. Topic: Change in Demand, Income Skill: Recognition

8)

What is the difference between a normal good and an inferior good? Give an example of each. Answer: A good is a normal good if an increase in incomes leads to an increase in demand for a good. Most goods are normal goods. An example of a normal good is new clothes. A good is an inferior good if an increase in income leads to a decrease in demand for the good. Second-hand clothing that can be purchased at thrift stores is an inferior good. Topic: Change in Demand, Income Skill: Recognition

9)

Consumers’ income declines and, as a result, the demand for margarine increases. Is margarine a normal or an inferior good? Explain. Answer: Margarine is an inferior good. An inferior good is one for which demand increases as income decreases, which describes the situation outlined for margarine in the question. Topic: A Change in the Quantity Demanded Versus a Change in Demand Skill: Recognition

10) Explain the difference between a change in demand and a change in quantity demanded. What leads to each of these changes? Answer: A change in demand occurs when consumers will buy more or less of a product at every price; a change in the quantity demanded occurs when the price changes and consumers buy more or

less. A change in demand is reflected by a shift of the entire demand curve, while a change in the quantity demanded is reflected by a movement along one demand curve. Only a change in the price of the good brings about a change in the quantity demanded. A change in demand is brought about by a change in any of the other influences on demand, namely, the prices of related goods, income, expectations, the number of buyers, and preferences. Topic: A Change in the Quantity Demanded Versus a Change in Demand Skill: Conceptual

11) Your friend Tony opened a pizzeria. You helped him to advertise his pizza, which is in fact the best pizza in town. As a result, the demand for Tony’s pizza increases and your friend, noticing lines of customers, raises the price of his pizza. But then he fears that the higher price will cause demand to decline, which will cause the price to drop. Is Tony right in his analysis of the situation? Explain. Answer: Tony is confusing a change in demand (a shift of the demand curve) with a change in quantity demanded (a movement along the demand curve). An increase in the price of his pizza cannot cause the demand for his pizza to decline, that is, it cannot shift the demand curve for his pizza leftward.. The rise in the price results in a decrease in the quantity of pizza demanded. So Tony need not fear that the demand for his pizza will decrease as a result of a higher price. Topic: Demand Curve Skill: Conceptual

12) An economist says: “The supply curve has two interpretations.” What does the economist mean? Answer: The first interpretation is that the supply curve shows the quantities of a good or service that producers are willing and able to sell at each price, other things being equal. The second interpretation is that the supply curve is a minimumsupply-price curve, so that for each quantity it shows the lowest price at which someone is willing to sell another unit.

DEMAND AND SUPPLY

Topic: Change in Supply Skill: Recognition

13) List the factors that change supply and shift the supply curve. Tell what happens to supply and the supply curve when there is an increase in the factor. Answer: The factors that change supply are technology, the number of sellers, expected future prices, prices of resources and prices of related goods. An advance in technology, an increase in the price of a complement in production, an increase in expected prices, and an increase in the number of sellers all lead to an increase in supply and a rightward shift in the supply curve. An increase in the price of a substitute in production or an increase in the prices of resources leads to a decrease in supply and a leftward shift in the supply curve. Topic: Change in Supply, Prices of Related Goods Produced Skill: Recognition

14) What are substitutes in production? Answer: Goods are substitutes in production when one good can be produced in place of the other, that is, when the goods are produced using the same resources. Topic: Change in Supply, Number of Suppliers Skill: Conceptual

15) Suppose that the number of companies selling computer software decreases. How does this change affect the supply of computer software and the supply curve of computer software? Answer: A decrease in the number of sellers decreases the supply. Hence the decrease in the number of companies selling computer software decreases the supply of computer software and shifts the supply curve of computer software leftward. Topic: Change in Supply, Technology Skill: Conceptual

16) Suppose that the technology used to produce computers advances. How does this change affect the supply of computers and the supply curve of computers? Answer: An advance in technology increases the supply of computers. Hence increases in technology shift the supply curve of computers rightward.

17

Topic: A Change in the Quantity Supplied Versus a Change in Supply Skill: Recognition

17) What leads to a decrease in the quantity supplied of a good or service? Answer: The quantity supplied of a good or service decreases when the price of the product decreases. Topic: A Change in the Quantity Supplied Versus a Change in Supply Skill: Recognition

18) What is the difference between quantity supplied and supply? Answer: Quantity supplied is the amount that people are willing to sell during a specific period for a specific price. It deals with one quantity at one price. Supply is the relationship between the quantity supplied and the price of the good. Supply applies to various prices and various quantities. Topic: Price Adjustment; Surplus Skill: Conceptual

19) Explain how price can be a regulator, that is, how it can coordinate the plans of buyers and sellers. Answer: If the price is too high, the quantity supplied exceeds the quantity demanded so there is a surplus. The surplus forces the price lower. The lower price increases the quantity demanded and decreases the quantity supplied bringing the market to equilibrium, where the quantity demanded equals the quantity supplied and where the plans of buyers and sellers are coordinated. If the price is too low, the quantity demanded exceeds the quantity supplied so there is a shortage. The shortage forces the price higher. The higher price decreases the quantity demanded and increases the quantity supplied bringing the market to equilibrium, again coordinating the plans of buyers and sellers. Topic: Price Adjustment; Shortage Skill: Recognition

20) When does a shortage occur? Answer: A shortage occurs when the price is below the equilibrium price. When the price is less than the equilibrium price, the quantity demanded is greater than the quantity supplied.

18

Topic: Price Adjustment; Surplus Skill: Recognition

21) When does a surplus occur? Answer: A surplus occurs when the price is above the equilibrium price. When the price exceeds the equilibrium price, the quantity supplied is greater than the quantity demanded. Topic: Price Adjustment; Shortage Skill: Recognition

22) At prices above the equilibrium price, what occurs? Answer: If the price exceeds the equilibrium price, there is a surplus because the quantity supplied exceeds the quantity demanded. With a surplus, the law of markets points out that the price will fall. As the price falls, the quantity supplied decreases and the quantity demanded increases, thus decreasing the size of the surplus. The price will continue to fall as long as there is a surplus, that is, as long as the price exceeds the equilibrium price. Ultimately the price will fall to equal the equilibrium price, at which time the surplus will be eliminated and the price will no longer change. Topic: Predicting Changes in Price and Quantity; Demand Changes Skill: Analytical

CHAPTER 3

fall, which, when the price falls enough, eliminates the surplus. Topic: Predicting Changes in Price and Quantity; Supply Changes Skill: Analytical

25) “If the price of crude oil falls, the demand for gasoline will increase, so people will by more gas and the price of gas will go up.” Is this statement true or false? Explain. Answer: The analysis is false. If the price of crude oil falls, the supply of gasoline increases, because crude oil is a resource used to produce gasoline. The prices of resources used to produce the good influence its supply, not demand. So, if the price of oil falls, the supply of gasoline increases and the supply curve shifts rightward. The equilibrium price of gasoline falls. It is true that people will buy more gasoline, but this happens not because the demand increases, but because a lower price results in a movement down the demand curve so that the quantity demanded increases. Topic: Predicting Changes in Price and Quantity; Supply Changes Skill: Analytical

23) When the demand for blue jeans increases, what happens next? Answer: If the demand for blue jeans increases, then at all prices buyers are more willing and more able to buy blue jeans. The demand curve for blue jeans shifts rightward. With the curve shifts, at the initial price a shortage of jeans will emerge. The law of supply and demand will force the price higher. Hence an increase in demand for blue jeans leads to a rise in the price of a pair of blue jeans and an increase in the quantity of blue jeans.

26) Personal computers are becoming less expensive as new technology reduces the cost of production. In a supply and demand model, explain the effects of the technological innovations and their effect on the quantity of computers. Answer: Advances in technology increase the supply of computers and the supply curve of computers shifts rightward. The price of a computer thus falls. The demand curve does not shift. Rather, on the demand side there is an increase in quantity demanded, or movement along the curve, in response to the falling price. The equilibrium quantity of computers increases.

Topic: Predicting Changes in Price and Quantity; Supply Changes Skill: Analytical

Topic: Predicting Changes in Price and Quantity; Supply Changes Skill: Analytical

24) Why does an increase in the supply of computers lead to a lower price for a computer? Answer: When the supply of a good, such as computers, increases, the supply curve shifts rightward. This shift means sellers are more willing and more able to sell computers at all prices than they were before. With this change, a surplus of computers results. The surplus forces the price to

27) In the market for bicycles, explain what happens to the supply and demand curves when there is an increase in the price of steel used to make bikes. Answer: An increase in the price of steel is an increase in the price of a resource used to make the good. As a result, the supply of bicycles decreases and the supply curve shifts leftward. There is no change to the demand, so the demand curve does

DEMAND AND SUPPLY

not shift. The equilibrium price of a bicycle rises and the equilibrium quantity decreases. Topic: Predicting Changes in Price and Quantity; Demand/Supply Increase Skill: Analytical

28) What is the effect on the price and quantity of a product if both the demand and supply simultaneously increase? Answer: The equilibrium quantity unambiguously increases. The effect on the equilibrium price is ambiguous. The equilibrium price rises if the increase in demand exceeds the increase in supply. The equilibrium price falls if the increase in supply exceeds the increase in demand. The equilibrium price is unchanged if the increase in demand equals the increase in supply. Topic: Predicting Changes; Demand Decreases and Supply Increases Skill: Analytical

19

tity and the decrease in supply decreases the quantity. Hence the net effect on the quantity is ambiguous. Topic: Predicting Changes; Demand Increases and Supply Decreases Skill: Analytical

31) In early 2003 the price of computer memory chips rose. In a demand and supply model, shifts in what curve(s) could have brought about the higher price? Answer: The higher price could have been brought about by an increase in demand, a decrease in supply, or the combination of an increase in demand combined with a decrease in supply. Hence the higher price could have been the result of a rightward shift in the demand curve, a leftward shift in the supply curve, or a combined rightward shift of the demand curve and leftward shift of the supply curve.

29) What is the effect on the price and quantity of a product if the demand decreases and the supply simultaneously increases? Answer: The equilibrium price unambiguously falls. The effect on the equilibrium quantity is ambiguous. The equilibrium quantity decreases if the decrease in demand exceeds the increase in supply. The equilibrium quantity increases if the increase in supply exceeds the decrease in demand. The equilibrium quantity is unchanged if the decrease in demand equals the increase in supply.

Topic: Predicting Changes in Price and Quantity Skill: Analytical

Topic: Predicting Changes; Demand Increases and Supply Decreases Skill: Analytical

Topic: Market Equilibrium Skill: Analytical

30) Computers are a complement to computer software. Suppose the price of a computer falls. Simultaneously, suppose that the number of companies selling computer software decreases. How do these changes affect the price and quantity of computer software? Answer: The fall in the price of a computer increases the demand for computer software and the demand curve for computer software shifts rightward. A decrease in the number of sellers decreases the supply of computer software and the shifts the supply curve of computer software leftward. The increase in demand and decrease in supply both raise the price, so the price definitely rises. The increase in demand increases the quan-

32) During the real estate boom of the mid-1990s, the prices of new and existing homes rose year after year yet people purchased more homes year after year. Can this outcome be explained as an exception to the law of demand? Answer: No; it is explained by a rightward shift of the demand curves for new and existing homes. The rightward shift was caused by rising consumer incomes and lower mortgage interest rates.

33) If the equilibrium relative price for a two-liter bottle of Coca-Cola is $1.50 today, just like it was ten years ago, can we safely say that all supply and demand conditions in the market for Coke have remained very stable all these years? Answer: Not necessarily. The demand curve might have shifted rightward continuously due to population growth in the United States and growing demand for Coke in other countries world-wide. Although that alone would have driven up the price, there could have been other factors shifting the supply curve rightward, such as improved technology for producing and transporting Coke, or declining sugar prices because of some great sugar harvests. Regardless of the reason, if the supply increased, so that the supply curve shifted

20

CHAPTER 3

rightward, then the increase in supply, which leads to a fall in the equilibrium price, can offset any increase in the demand. So, even if the price has remained constant, the only accurate statement is that any change in demand was accompa-

nied by an equal sized change in the supply in the same direction.

CHAPTER 4

ELASTICITY

Topic: The Price Elasticity of Demand Skill: Recognition

Topic: The Price Elasticity of Demand Skill: Conceptual

1)

“The price elasticity of demand is a measure of how sensitive demanders are to changes in the price of a product.” Is this statement true or false? Answer: The assertion is true. All elasticities measure the sensitivity, or responsiveness, of some variable to a change in an influence. The price elasticity of demand measures how strongly demanders respond to a change in the price of the good or service.

4)

Topic: The Price Elasticity of Demand Skill: Recognition

Topic: Inelastic and Elastic Demand Skill: Recognition

2)

5)

What is the price elasticity of demand and how is it measured? Answer: The price elasticity of demand is a unit-free measure of responsiveness of the quantity demanded of a good to a change in its price when all other influences on buyers plans remain the same. To calculate the price elasticity of demand we divide the percentage change in quantity demanded by the percentage change in price. Topic: The Price Elasticity of Demand Skill: Recognition

3)

What is the price elasticity of demand? In terms of percentage changes, what is its formula? Answer: The price elasticity of demand is a measure of the responsiveness of quantity demanded to a change in price. The formula for the price elasticity of demand is the magnitude of the percentage change in quantity demanded divided by the percentage change in price.

When does a decrease in supply raise the price more: When demand is elastic or when demand is inelastic? When OPEC decreases the supply of oil, the price of gasoline skyrockets. Hence is the demand for gasoline elastic or inelastic? Answer: A decrease in supply raises the price more when demand is inelastic. The skyrocketing price of gasoline indicates that the demand for gasoline is inelastic.

What are the three cases for the price elasticity of demand? Briefly define each. Answer: Demand can be elastic, inelastic, or unit elastic. Elastic demand occurs when the percentage change in quantity demanded exceeds the percentage change in price. Inelastic demand occurs when the percentage change in quantity demanded is less than the percentage change in price. Unit elasticity occurs when the percentage change in price equals the percentage change in demand. Topic: Inelastic and Elastic Demand Skill: Recognition

6)

In a study session, your friend says, “Demand is elastic if the percentage change in the price exceeds the percentage change in quantity demanded.” Is your friend correct? Answer: No, your friend is incorrect. Demand is elastic if the percentage change in the price is less than the percentage change in the quantity demanded.

22

Topic: Inelastic and Elastic Demand Skill: Recognition

7)

If the percentage change in quantity demanded is greater than the percentage change in price, can you determine if the demand is elastic, unit elastic, or inelastic? Explain your answer. Answer: The demand is elastic. The formula for the price elasticity of demand is the percentage change in the quantity demanded divided by the percentage change in the price. And, if the elasticity of demand exceeds one, then the demand is elastic. In this case, the percentage change in the quantity demanded is larger than the percentage change in the price, so the elasticity of demand will be greater than one. Hence the demand is elastic. Topic: Inelastic and Elastic Demand Skill: Conceptual

8)

Does the fact that a product’s price elasticity of demand is inelastic violate the law of demand? Answer: No, if the demand for a product is inelastic, when the price falls, the quantity demanded increases, but by a smaller percentage than the percentage fall in price. Thus the negative relationship between the price and the quantity demanded remains, and the demand curve still slopes downward, which is the law of demand. Topic: Perfectly Elastic Demand Skill: Recognition

9)

What does a horizontal demand curve indicate about the price elasticity of demand? Answer: If the demand curve for a good is horizontal, the demand for the good is perfectly elastic. Hence the price elasticity of demand is infinite. Topic: Elasticity Along a Straight-Line Demand Curve Skill: Recognition

10) What happens to the price elasticity of demand moving down along a downward-sloping, linear demand curve? Answer: Moving down along a downward-sloping, linear demand curve, the elasticity of demand falls in value. Demand changes from elastic to inelastic at the midpoint of the demand curve as the quantity demanded increases moving along demand curve.

CHAPTER 4

Topic: Elasticity Along a Straight-Line Demand Curve Skill: Conceptual

11) “The price elasticity of demand is constant along a straight-line demand curve.” Is this statement true or false? Answer: False. The price elasticity of demand decreases when moving downward along a straightline demand curve. Topic: Total Revenue and Elasticity Skill: Recognition

12) What effect does a price increase have on the total revenue of the producers? Answer: The effect of a price increase in total revenue depends on the elasticity of demand. If the demand is elastic, then total revenue will decrease because the decrease in the quantity demanded will outweigh the effect of the higher price. If the demand is inelastic, then total revenue will increase. In this case, the decrease in the quantity demanded is proportionally less than the increase in price and so the higher price leads to increased total revenue. Finally, if the demand is unit elastic, then the higher price does not change the total revenue. The percentage decrease in the quantity demanded just equals the percentage increase in the price and so the two effects just offset each other. The total revenue does not change. Topic: Total Revenue and Elasticity Skill: Recognition

13) Explain the total revenue test. Answer: The total revenue test estimates whether demand for the good is elastic, unit elastic or inelastic based on what happens to total revenue when the price of a good changes. If price and total revenue move in opposite directions, demand is elastic. If price and revenue move in the same direction, demand is inelastic. If a price change does not change total revenue, demand is unit elastic.

ELASTICITY

Topic: Total Revenue and Elasticity Skill: Conceptual

14) A local pizzeria charges $10 for a medium pizza. The owner of the pizzeria wants to increase his total revenue. A recent market research shows that the price elasticity of demand for his pizza is about 1.5. Should the pizzeria lower or raise the price? Explain your answer. Answer: The pizzeria should lower the price. Since the demand is price elastic, the percentage decrease in price will be smaller than the percentage increase in the quantity of pizza demanded caused by this price decrease. Therefore, the total revenue will increase. Topic: Total Revenue and Elasticity Skill: Conceptual

15) If a decrease in price increases total revenue, what can you determine about the elasticity of demand for the good? Answer: If a decrease in price increases total revenue, the demand for the good is elastic, that is, the elasticity of demand exceeds 1.00. Topic: Total Revenue and Elasticity Skill: Conceptual

16) The demand for oil is inelastic. So, does an increase in the price of oil mean an increase in total revenue or a decrease in total revenue for oil producers? Answer: Because the demand is inelastic, an increase in price increases the total revenue of the oil producers. Topic: Total Revenue and Elasticity Skill: Conceptual

17) “If the price falls and, as a result, the total revenue decreases, demand is elastic.” Is the previous assertion correct? Answer: No, the assertion is incorrect. If the demand is elastic, a fall in price increases the quantity demanded enough so that total revenue increases. Topic: Total Revenue and Elasticity Skill: Conceptual

18) If the owner of a local movie theater wanted to increase his total revenue from movie admissions, what should the owner do with ticket prices? Answer: If movie goers’ demand is elastic, the owner should lower prices. If the demand is inelastic, the owner should raise prices. If the demand is unit

23

elastic, price changes have no effect on total revenue. Topic: Total Revenue and Elasticity Skill: Conceptual

19) If a college wanted to increase its revenues from tuition payments, should it increase the tuition of day and evening students alike? Answer: If the demand from each class of students is inelastic, then a tuition increase for each will increase the school’s revenue. However, if either class had an elastic demand, then a tuition decrease would be needed to increase revenue. Topic: Total Revenue and Elasticity Skill: Conceptual

20) Can electric utility companies can always raise their total revenue by raising their rates? Answer: No. As electric companies continue to raise rates, customers move up along their demand curves for electricity, obeying the law of demand. As they do, they eventually enter an elastic portion of their demand curves, along which total revenue will decrease when rates increase further. Topic: Total Revenue and Elasticity Skill: Analytical

21) The price of chocolate chip cookies is $5.50 per pound. The manufacturer needs to decide whether to raise the price to $5.95 per pound or lower the price to $4.95 per pound. Recent test market results indicate that the demand for the cookie is elastic. If the manufacturer wants to increase the total revenue from sales of cookies, what price would you recommend for the cookie? Answer: You should recommend that the price be lowered to $4.95. Because the demand for the cookie is elastic, the lower price will generate a large enough increase in sales so that total revenue increases.

24

Topic: Total Revenue and Elasticity Skill: Conceptual

22) Anna owns the Sweet Alps Chocolate store. She charges $10 per pound for her hand made chocolate. You, the economist, have calculated the elasticity of demand for chocolate in her town to be 2.5. If she wants to increase her total revenue, what advice will you give her? Answer: You should tell her to lower her price. Because demand is elastic, lowering the price will increase the total revenue. Topic: Total Revenue and Elasticity Skill: Conceptual

23) You are the brand manager of Crest toothpaste and you observe that when you increase the price of Crest, your total revenue increases. How is that possible? Answer: The demand for Crest toothpaste must be inelastic, so that the percentage increase in price is greater than the percentage decrease in quantity demanded, thereby increasing the total revenue. Topic: Factors That Influence the Price Elasticity of Demand Skill: Recognition

24) What factors determine the magnitude of the price elasticity of demand? Answer: There factors determine the magnitude of the elasticity of demand: the closeness of substitutes, the time elapsed since a price change, and the proportion of income spent on the good. The more substitutes for a good, the more elastic its demand. For instance, luxuries have more substitutes than necessities, and so the elasticity of demand for luxuries exceeds that for necessities; and, narrowly defined goods have more substitutes than broadly defined goods, and so the elasticity of demand for narrowly defined goods exceeds that for broadly defined goods. The more time that has elapsed since a price change, the more substitutes consumers can find, and so the elasticity of demand is larger the more time passes. Finally, the larger the fraction of consumers’ income spent on a good, the larger is its elasticity of demand.

CHAPTER 4

Topic: Factors That Influence the Price Elasticity of Demand Skill: Recognition

25) “The fewer the number of substitutes for a product, the more elastic the demand for that product.” Is the previous statement true or false? Answer: The statement is false. The greater the number of substitutes, the more elastic the demand for that product. Conversely, the fewer the number of substitutes, the less elastic (the more inelastic) the demand for that product. Topic: Factors That Influence the Price Elasticity of Demand Skill: Recognition

26) If a good has only a few, poor substitutes, is its demand be elastic or inelastic? Answer: The demand is inelastic. The fewer substitutes for a good, the more inelastic (less elastic) its demand. Topic: Factors That Influence the Price Elasticity of Demand Skill: Conceptual

27) Explain why the number of substitutes influences the price elasticity of demand. Answer: The price elasticity of demand is a measure of how responsive the quantity demanded is to a change in price. If a good has many substitutes, it is easy to switch away from it when its price rises. In this case, a rise in price substantially decreases the quantity demanded. Similarly, when the price falls, consumers can switch into the good and away from many other (substitute) products. So a fall in price substantially increases the quantity demanded. Because changes in price have major effects on the quantity demanded, the demand is elastic. Topic: Factors That Influence the Price Elasticity of Demand Skill: Conceptual

28) Which demand is more price elastic, for all personal computers or for Dell computers? Explain your answer. Answer: The demand for Dell computers is more elastic than the demand for all personal computers. Dell computers have very close substitutes (Gateway or Hewlett Packard computers), but it is difficult to find a good substitute for personal computers as a group. Plus, anything that substi-

ELASTICITY

tutes for personal computers also substitutes for Dell computers. So Dell computers have more very close substitutes than do personal computers as a group. Topic: Factors That Influence the Price Elasticity of Demand Skill: Conceptual

29) Which is larger: The price elasticity of demand for food or the price elasticity of demand for oranges? Why? Answer: The price elasticity of demand for oranges is larger than the price elasticity of demand for food. The elasticity of demand for oranges is larger because there are many more substitutes for oranges (apples, grapefruit, lemons, and so forth) than there are substitutes for food. Topic: Factors That Influence the Price Elasticity of Demand Skill: Conceptual

30) Water is considered a necessity. So, is the demand for water elastic or inelastic? Answer: The demand for necessities is inelastic, so the demand for water is inelastic. Topic: Factors That Influence the Price Elasticity of Demand Skill: Conceptual

31) Studies have shown that the price elasticity of demand for necessities, such as food, are higher in developing countries and lower in developed countries. What is the reason for this difference in elasticity? Answer: One of the determinants of elasticity is the proportion of income spent on a good or service. The higher the proportion of income spent on a good, the larger the price elasticity of demand for that good. People in developing countries have low incomes and therefore spend a large part of it on food. For example, in Tanzania 62 percent of income is spent on food. On the other hand, in United States only 12 percent of income is spent on food. The price elasticity of demand for food therefore will be larger in developing countries as compared to developed countries. Topic: Cross Elasticity of Demand Skill: Recognition

32) The price elasticity of demand is always positive, as is the price elasticity of supply. Is the cross elas-

25

ticity of demand always positive? Explain your answer. Answer: No, the cross elasticity of demand is not always positive. The cross elasticity of demand is positive for goods that are substitutes and negative for goods that are complements. Hence the sign of the cross elasticity of demand indicates whether the goods are substitutes or complements. Topic: Cross Elasticity of Demand Skill: Recognition

33) Explain why the cross elasticity of demand for substitute goods is positive and the cross elasticity of demand for complements is negative. Answer: The equation for cross elasticity of demand is the percentage change in quantity demanded of product A divided by the percentage change in price of another product, B. Consider the case in which A and B are substitutes. In this case, when the price of B falls, the quantity of A demanded decreases and when the price of B rises, the quantity of A demanded increases. Because the quantity demanded of product A is in the numerator, then the decrease in the quantity of A demanded when the price of the other product falls means, that the cross elasticity will be positive. (There is a negative number in the numerator and also a negative number in the denominator, so the fraction, the cross elasticity, is positive.) For a price increase, there are positive numbers in both the numerator and denominator, so again the fraction is positive. In either case, the result will be a positive number, that is, the cross elasticity of demand is positive. Next, consider the case in which A and B are complements. In this case, an increase in the price of B decreases the quantity demanded of A and a decrease in the price of B increases the quantity demanded of A. Here, the change in quantity in the numerator always has the opposite sign from the change in the price in the denominator, so the resulting cross elasticity of demand will be negative.

26

CHAPTER 4

Topic: Cross Elasticity of Demand Skill: Recognition

Topic: Cross Elasticity of Demand Skill: Analytical

34) If the cross elasticity of demand between two goods is negative, are the goods substitutes or complements? Answer: If the cross elasticity of demand is negative, then the goods are complements.

38) “The number of substitutes available affects the price elasticity of demand for a good. Thus one way to know if apples and oranges are substitutes for each other is to look at the price elasticity of demand for each.” Comment on this assertion. Answer: Their separate elasticities would not indicate whether apples and oranges are substitutes for each other. If apples and oranges are not substitutes but each had many other substitutes, then their separate price elasticities will be high. To conclude that apples and oranges are substitutes for each other, the cross elasticity between them must be positive.

Topic: Cross Elasticity of Demand Skill: Recognition

35) What kind of elasticity is relevant when you are trying to figure out how a price cut by the burger shop next door will affect the demand for your pizza? Explain. Answer: The cross elasticity of demand is the relevant elasticity. It is a measure of the responsiveness of the demand for a good to a change in the price of a related good. In the case in question, the required elasticity is the cross elasticity of demand for pizza with respect to the price of a burger. Topic: Cross Elasticity of Demand Skill: Conceptual

36) If the cross elasticity of demand between peanut butter and milk is −1.11, then are peanut butter and milk substitutes or complements? Answer: Because the cross elasticity of demand is negative, peanut butter and milk are complements. Topic: Cross Elasticity of Demand Skill: Analytical

37) If cable TV subscriptions and movie rentals are substitutes for each other, what is the effect in each of these markets of an increase in wages for people who work for the cable TV company? Answer: The supply curve of cable TV will shift leftward because wages are a cost of production, thus raising the price of cable TV. Cable TV subscribers decrease the quantity of cable TV they demand (they move leftward along their demand curve for cable TV). Cable TV subscribers increase their demand for movie rentals (the demand curve for movie rentals shifts rightward) so that the equilibrium price and quantity of movie rentals both increase. Hence, when the price of cable TV goes up then the quantity of movie rentals increases, so their cross elasticity of demand is positive.

Topic: Income and Cross Elasticities of Demand Skill: Recognition

39) How are the cross elasticity of demand and income elasticity of demand similar and how are they different from the price elasticity of demand? Answer: The cross and income elasticities are similar to the price elasticity of demand because all examine how strongly demanders respond to a change in a relevant factor. The price elasticity of demand examines how strongly demanders respond to a change in the price of the product. The cross elasticity of demand studies how strongly demanders respond to a change in the price of a related product. And the income elasticity of demand examines how strongly demanders respond to a change in income. The formulas for all three elasticities also are similar. All three use percentage changes and all three divide the percentage change in the quantity demanded of the good by the percentage change in the relevant factor. The elasticities also differ. For instance, as outlined above, all three concentrate on a different factor: the good’s price (for the price elasticity of demand); the price of a related good (for the cross elasticity of demand); and income (for the income elasticity of demand.) The price elasticity of demand is always positive (because we use the magnitudes of the percentage changes or, equivalently, we take the absolute value of the percentage changes) whereas the cross and income elasticities of demand can be either positive or negative.

ELASTICITY

27

Topic: Income Elasticity of Demand Skill: Conceptual

Topic: Elasticity of Supply Skill: Recognition

40) During a recession when people’s incomes drop, grocery chains experience much smaller declines in sales than do upscale department stores. Explain this phenomenon using the concept of elasticity. Answer: During a recession, people’s income decreases leading to a decrease in the quantity of normal goods they buy. The demand for necessities, however, is less income elastic, that is, less responsive to a decrease in income, than the demand for luxuries. So grocery stores, which sell mainly necessities, lose fewer sales than do upscale department stores, which sell luxury items.

44) What is the price elasticity of supply? List and briefly define three cases of the price elasticity of supply. Answer: The price elasticity of supply measures how responsive quantity supplied is to a change in the price of the good. Supply can be price elastic, if the percentage change in the quantity supplied exceeds the percentage change in the price, price inelastic, if the percentage change in the quantity supplied is less than the percentage change in the price, or unit elastic, if the percentage change in the quantity supplied equals the percentage change in the price.

Topic: Income Elasticity of Demand Skill: Conceptual

Topic: Inelastic and Elastic Supply Skill: Recognition

41) Joe’s monthly income increases from $1,000 to $2,000. As a result, he decreases the number of his fast food meals from 20 to 5 per month. To Joe, are fast-food meals a normal or an inferior good? What kind of elasticity can tell the answer? Explain. Answer: Fast-food meals are an inferior good to Joe because his income elasticity of demand for fastfood meals is negative: as Joe’s income increases, the quantity of fast-food meals he demands decreases.

45) If the price elasticity of supply for corn is 3.12, then is the supply of corn elastic or inelastic? Answer: Because the price elasticity of supply exceeds one, the supply of corn is elastic.

Topic: Income Elasticity of Demand Skill: Conceptual

42) If the income elasticity of demand for a Miami Dolphin season ticket is 2.34, then are Dolphin season tickets a normal or an inferior good? Answer: Because the income elasticity of demand is positive, Dolphin season tickets are a normal good. Topic: Income Elasticity of Demand Skill: Conceptual

43) The income elasticity of demand for store brands of soda (that is, non-name brands) is negative. What does this fact indicate about consumers’ perceptions about the store brands? Answer: The negative income elasticity indicates that the store brands of soda are considered by many consumers to be an inferior good. Hence as income increases, consumption of these sodas decreases as consumers opt for name sodas, such as Pepsi and Coke.

Topic: Factors That Influence the Elasticity of Supply Skill: Conceptual

46) Is supply more elastic or less elastic as more time passes after a price change? Explain your answer. Answer: Supply becomes elastic as more time passes after a price change. Consider the case of an increase in price. As more time passes, producers have time to gather more resources, such as building additional factories, and to implement new production techniques. Hence as time passes, the quantity supplied after a price hike increases and the supply becomes more elastic. Topic: Factors That Influence the Elasticity of Supply Skill: Conceptual

47) Explain why the availability of resources affects the elasticity of supply. Answer: A good that can be produced using commonly available resources has costs that increase only a little when it increases its production. amount. Its supply will be more elastic. If instead, the production of a good requires very specialized resources, an increase in production will be very costly. Its supply will be more inelastic.

28

CHAPTER 4

Topic: Inelastic and Elastic Supply Skill: Analytical

Topic: Elasticity of Supply Skill: Conceptual

48) If addiction makes cigarettes such a necessity, is it correct to think that cigarettes are perfectly inelastic in both supply and demand. Answer: Cigarettes might be perfectly inelastic in demand over a small range of prices. However, at high enough prices there is some decrease in the quantity demanded. Some consumers would quit, others would cut down the quantity they smoke each day, and more kids would find it too costly to get started. On the supply side, a higher price will lead to more tobacco being planted, so that as time passes the quantity supplied would increase. So, as time passes the supply of tobacco is not perfectly inelastic.

49) Is there any set of relationships between price elasticity of supply and total revenue similar to the relationships between price elasticity of demand and total revenue? Answer: No. When the price increases along an upward sloping supply curve, total revenue will increase whether supply is elastic or inelastic. When price decreases along an upward sloping supply curve, total revenue will decrease whether supply is elastic or inelastic.

CHAPTER 5

EFFICIENCY AND EQUITY

tional benefit from each unit of a good consumed. The additional benefit is measured by the amount of other goods and services we are willing to forgo to get one more unit of the good in question. Hence the demand curve is the same as the marginal benefit curve because the demand curve shows precisely what the marginal benefit curve measures.

Topic: Marginal Benefit Skill: Conceptual

1)

Is the marginal benefit someone receives from a good or service the same as the price the person pays? Explain your answer. Answer: Generally the marginal benefit of a good or service is different than the price that is paid. The marginal benefit is the maximum amount a consumer is willing to pay for a good or service. Typically the consumer can buy the good or service for a price less than the maximum. (Indeed, the difference between the maximum price the consumer is willing to pay and the price actually paid is the consumer surplus.) However, it might be the case that the price precisely equals the maximum the consumer is willing to pay. In this case alone, the marginal benefit equals the price. (And in this case, the amount of the consumer surplus equals zero.) Topic: Marginal Benefit Curve Skill: Recognition

2)

“A demand curve is the same as a marginal cost curve.” Is this statement correct or incorrect? Explain your answer. Answer: The statement is incorrect. A demand curve is a marginal benefit curve not a marginal cost curve. A demand curve is a marginal benefit curve because it can be used to find the maximum price people are willing to pay for a given quantity, that is, the marginal benefit of the given quantity. Topic: Marginal Benefit Curve Skill: Conceptual

3)

Why is the demand curve the same as the marginal benefit curve? Answer: The demand curve for a good tells us for any quantity of the good, the dollar’s worth of other goods and services that we are willing to forgo in order to get one more unit of the good in question. The marginal benefit of a good is the addi-

Topic: Value, Willingness to Pay, and Demand Skill: Conceptual

4)

What is the relationship between the marginal benefit curve and the demand curve. Explain. Answer: The demand curve is also the marginal benefit curve. The demand curve shows the maximum price that consumers are willing to pay to buy another unit of the good. Marginal benefit is the value of one more unit of a good, which can be expressed as the maximum price that people are willing to pay to get it. So the demand curve is the marginal benefit curve. Topic: Marginal Benefit Curve Skill: Conceptual

5)

The demand curve is the same as another curve. Which curve is the same as the demand curve? Why are the curves the same? Answer: The demand curve is the same as the marginal benefit curve. For any quantity, the demand curve shows the dollar value of other goods and services the consumer is willing to forgo to get another unit of the good. (This amount is the maximum price the consumer is willing to pay and equals the price from the demand curve vertically above each quantity.) But the amount of other goods and services the consumer is willing to forgo is the marginal benefit of the good. Hence along the demand curve the price associated with each quantity of the good is the same as the marginal benefit of that quantity. (So that, for instance, the price associated with the 3rd quan-

30

CHAPTER 5

tity is the same as the marginal benefit of the 3rd unit.) Therefore the demand curve is the same as the marginal benefit curve.

producing another car yields a benefit that exceeds the cost of producing the car. So we are producing too few cars.

Topic: Marginal Benefit Curve Skill: Conceptual

Topic: Consumer Surplus Skill: Recognition

6)

11) What is consumer surplus? Answer: Consumer surplus equals the difference between the value of a unit of a good and the price paid for it. The total consumer surplus in a market equals the area under the demand curve and above the price.

What are the two ways demand curves can be interpreted? Answer: Demand curves can be used to find either: 1) the quantity demanded at a given price. In this view, the curve is a demand curve; 2) the maximum price people are willing to pay for a given quantity. In this view the curve is a marginal benefit curve. Topic: Marginal Cost Skill: Conceptual

7) What is the marginal cost of a good? Answer: The marginal cost of a good is the opportunity cost of producing one more unit of it. The key is that the marginal cost refers to the cost of one additional unit, not to the total cost of producing all the units. Topic: Allocative Efficiency Skill: Recognition

8) What is allocative efficiency? Answer: Allocative efficiency is a situation when it is not possible to produce more of one good without giving up the production of some other good that is valued more highly. Topic: Allocative Efficiency Skill: Conceptual

What is the relationship between the social marginal benefit of milk and social marginal cost of milk when the efficient quantity of milk is produced? Answer: The social marginal benefit of milk equals its social marginal cost.

Topic: Consumer Surplus Skill: Recognition

12) What must be true for a consumer to enjoy a consumer surplus from a good? Answer: The value to the consumer of the good must be greater than the price of the good. Topic: Consumer Surplus Skill: Analytical

13) A new car in the dealer’s showroom had a sticker price of $35,900. Sally liked the car but decided she would pay no more than $32,000 for it, otherwise she would do without it. After haggling with the dealer, she purchased the car for $31,500. Did she gain any consumers surplus? If so, how much? If not, why not? Answer: Yes. Consumer surplus is the difference between the highest price a consumer is willing to pay (that is, the value of the good) minus the actual price paid. Sally gained a consumer surplus of $500.00.

9)

Topic: Allocative Efficiency Skill: Conceptual

10) “If the social marginal benefit of a car exceeds the social marginal cost of a car, we are producing too many cars.” True or false? Explain. Answer: False. If the social marginal benefit of a car exceeds the social marginal cost of a car, we can gain value by producing another car and we are losing value by not producing it. If the marginal social benefit exceeds the marginal social cost,

Topic: Consumer Surplus Skill: Analytical

14) Bob gets thirsty driving on a highway on a hot day. He is willing to pay $3.00 for a bottle of spring water. If he stops at a gas station and buys a bottle of spring water for $1.00, will he gain a consumer surplus? If yes, how much? Explain. Answer: Yes, Bob gets a consumer surplus. Consumer surplus is the difference between the value of a good, measured by the consumer’s willingness to pay, and the actual price that he pays. So Bob’s consumer surplus is $3.00 − $1.00, which is $2.00.

EFFICIENCY AND EQUITY

31

Topic: Consumer Surplus Skill: Conceptual

Topic: Producer Surplus Skill: Recognition

15) If the demand for a good does not change, how will an increase in the price of that good affect the consumer surplus from it? Answer: The consumer surplus equals the difference between the value of the good and its price. If the demand for a good does not change, then the value of that good does not change. Hence an increase in the price decreases the consumer surplus from that good.

18) What is producer surplus? Answer: Producer surplus is the price of a good minus the opportunity cost of producing it, summed over the quantity sold.

Topic: Value, Price and Cost Skill: Conceptual

16) Explain the difference between the words “value,” “price,” and “cost.” Answer: Value refers to the marginal benefit of consuming one more unit of a good and is illustrated by the demand curve. Price is determined in the market by the forces of supply and demand. Price is what the supplier receives for selling a good and is what the buyer pays when purchasing a good. Cost refers to what must be given up to produce a good. The marginal cost of producing one more unit of a good is illustrated by the supply curve. Topic: Marginal Cost Curve Skill: Conceptual

17) The supply curve is the same as another curve. What other curve is the same as the supply curve? Why are the curves the same? Answer: The supply curve is the same as the marginal cost curve. The marginal cost is the cost of producing one more unit of a good. For any quantity, the supply curve shows the minimum price for which a producer is willing to produce another unit of the good. (This price is equal to the price on the supply curve vertically above each quantity.) A producer is willing to produce a unit of the good if the price covers all costs of the producing that unit, that is, if the price equals the cost of producing the unit. The cost of producing the unit is the marginal cost. Hence along the supply curve, the price associated with each quantity is equal to the marginal cost of each quantity. Therefore the supply curve is the same as the marginal cost curve.

Topic: Producer Surplus Skill: Recognition

19) What must be true for a producer to obtain a producer surplus from the sale of a good? Answer: The price must be greater than the marginal cost of producing the good. Topic: Producer Surplus Skill: Conceptual

20) “If the price of a ticket to Sea World exceeds the marginal cost of the ticket by $13, a producer surplus exists for Sea World.” Is this statement true or false? Answer: The statement is true. Anytime the price exceeds the marginal cost, there is a producer surplus. In this case, the producer surplus is $13 dollars. Topic: Consumer and Producer Surplus Skill: Conceptual

21) What is the significance of the concepts “consumer surplus” and “producer surplus”? Answer: Each is a measure of economic welfare or well-being. The more consumer surplus a buyer can gain from a transaction, the greater is consumer welfare. The more producer surplus a seller can gain from a transaction, the greater is producer welfare. Total surplus is equal to the sum of consumer and producer surplus. Topic: Consumer and Producer Surplus Skill: Analytical

22) Can both producer surplus and consumer surplus exist at the same time in a particular market? Answer: Yes. The actual price can be above the seller’s opportunity cost and below the buyer’s maximum purchase price. So, buyer and seller can both earn a surplus on the same transaction. Topic: Efficiency of Competitive Markets Skill: Conceptual

23) When less than the efficient amount of a good is produced, how does the marginal social benefit of

32

the last unit produced compare to its marginal social cost? Answer: When less than the efficient amount of a good is produced, the marginal social benefit of the last unit produced exceeds its marginal social cost. The fact that the marginal social benefit exceeds the marginal social cost indicates that producing additional units of the good will move the amount of production closer to the efficient quantity. Topic: Efficiency of Competitive Markets Skill: Conceptual

24) Why is a competitive market efficient? Answer: Efficiency is attained when production is such that the marginal social benefit equals the marginal social cost. When a competitive market is at equilibrium, the quantity demanded equals the quantity supplied, that is, the demand and supply curves cross. But the marginal social benefit curve is the same as the demand curve and the marginal social cost curve is the same as the supply curve. Thus equilibrium occurs at the point where the marginal social benefit curve crosses the marginal social cost curve. As a result, so the marginal social benefit equals the marginal social cost and hence the market is efficient. Topic: Invisible Hand Skill: Recognition

25) Briefly describe the concept of the “invisible hand.” Answer: The invisible hand suggests that a competitive market attains allocative efficiency simply by the forces of supply and demand, with no government involvement needed to organize the resources, or set the efficient price and quantity. Topic: Invisible Hand Skill: Recognition

26) What is the “invisible hand”? Answer: The invisible hand is a concept articulated by Adam Smith that suggests that competitive markets are efficient. Smith discussed how selfinterested buyers and sellers, without government involvement, interact with one another to bring about market efficiency. In Smith’s words, market participants are “led by an invisible hand to promote an end (efficiency) which was no part of his intention.”

CHAPTER 5

Topic: Invisible Hand Skill: Recognition

27) When economists refer to “the invisible hand,” what do they mean? Answer: In his book, Wealth of Nations, Adam Smith wrote that a participant in a competitive market is “led by an invisible hand to promote an end which was no part of his intention.” Market participants act in their own self-interest, attempting to maximize their own well-being, yet, in the process, the result is an efficient use of resources. This result occurs because the market forces of supply and demand invisibly guide resources to their highest valued uses. Topic: Invisible Hand Skill: Conceptual

28) Explain how the invisible hand delivers an efficient market outcome. Answer: The invisible hand concepts claims that people, acting in their own self interest, send resources to be used at their highest value. As a result, resources are efficiently used to produce the output of goods and services that people most highly value. With this outcome, producer and consumer surplus are maximized. Topic: Deadweight Loss Skill: Conceptual

29) “If there is an inefficient level of nursing care in South America, a deadweight loss exists.” Is this statement true or false? Answer: The statement is true. Anytime there is an inefficient level of production, a deadweight loss exists. Topic: Obstacles to Efficiency Skill: Conceptual

30) What are some of the potential obstacles that can prevent a market from reaching the efficient outcome? Briefly define each obstacle. Answer: The obstacles basically fall into two camps: Obstacles that occur because the government does not intervene in the market and obstacles that occur because the government does intervene in the market. In the first group are the issues of external costs and benefits, public goods, common resources, and monopoly. An external cost or benefit occurs when a cost or benefit from production falls upon someone other than the producer or when a cost or benefit from consumption falls

EFFICIENCY AND EQUITY

upon someone other than the demander. A public good is a good or service that can be consumed simultaneously by everyone even if they didn’t pay for the good or service. Public goods create the free rider problem, in which people consume the good without paying for it. A common resource is a resource that no one owns and that everyone can use. Finally, a monopoly occurs when a single producer controls the market by being the only producer. In the case of an externality, public good, or monopoly, government intervention has the possibility of increasing the market’s efficiency. The second set of obstacles occur when a market is otherwise efficient but nonetheless the government intervenes in the market. The second group is comprised of price ceilings and floors as well as taxes, subsidies, and quotas. A price ceiling makes prices higher than a certain price illegal; a price floor makes prices lower than a certain price illegal. Taxes increase the price paid by the buyer and decrease the price received by the seller. Subsidies have the opposite effect, decreasing the price paid by the buyer and increasing the price received by the seller. Finally, quotas are direct limits on the amount of a good that a firm can produce. Because this quantity will usually be less than the quantity the market would produce, the quota is inefficient. Topic: Obstacles to Efficiency Skill: Conceptual

31) Can price ceilings and price floors interfere with the efficient quantity of a good being produced? Answer: Yes. A price ceiling set below the market equilibrium price results in suppliers producing less than the efficient quantity. A price floor set above the equilibrium price results in demanders buying a smaller quantity than the efficient quantity. Both price ceilings and price floors decrease the actual quantity below the efficient quantity. Topic: Is the Competitive Market Fair? Skill: Conceptual

32) Explain the difference between the “fair rules” and the “fair results” guidelines for competitive market fairness. Answer: Economists do not agree on how to judge whether an outcome is fair. Some believe that an outcome is not fair if the results are not fair. Con-

33

sider how incomes are distributed by the labor market. Because income is not distributed equally, one group of economists believes that the labor market is not a fair market. To make the outcome (and results) fair, these economists believe that income should be redistributed by the government. Other economists are not concerned with fair results. They aren’t concerned with the unequal distribution of income as long as the rules to earn income are fair. As long as there is voluntary exchange and property rights are enforced, this group of economists believes the market outcome is fair. Topic: Is the Competitive Market Fair? Skill: Conceptual

33) What are the two views of fairness? How does each view redistribution of income from the rich to the poor? Answer: One view is that “it’s not fair if the result isn’t fair.” This view requires that income should be redistributed from the rich to the poor in order to create a fair result. Another view is that “it’s not fair if the rules aren’t fair.” This view requires that private property may be transferred only under voluntary exchange, so redistribution of income is not fair unless it is voluntary. Topic: Fairness Skill: Recognition

34) What are the two big approaches to thinking about fairness? Answer: (1) It’s not fair if the result is not fair. (2) It’s not fair if rules are not fair. Topic: Fair Results Skill: Conceptual

35) How can a person argue that health care services in America are provided efficiently, but not fairly? Answer: The assertion that the health market is efficient implies that competitive market forces determines the level and quality of health care. Where the demand for and supply of health care intersects, the marginal social benefit and marginal social cost of health care are equal. This equality means that the people who are demanding health care are willing and able to pay the price, which is equal to the marginal social cost of providing health care.

34

CHAPTER 5

However, at that market price there are many people who cannot afford health care. These people, probably the poor and uninsured, are left to go to clinics, crowd emergency rooms, or else do without basic health care needs because of their lack of ability to pay the market price. This outcome could easily violate the “it’s not fair if the result isn’t fair” view of fairness. Thus observers who assert that the U.S. health care system might be efficient but isn’t fair are using the “results” view of fairness. Topic: Fair Results Skill: Conceptual

36) Often politicians assert that a price, such as the price of gasoline or the rent for an apartment, is too high and that it is unfair for these prices to be so high. If these products are traded in competitive markets, what fairness rule are politicians using? Why? Answer: The fairness rule is one of “It’s not fair if the results aren’t fair.” The claim that the prices are too high to be fair is a claim that buyers are being unfairly harmed by having to pay such high prices. The assertion that people are harmed because the price is too high is looking at the results of the process because if the price had been lower, the assertion of unfairness would not have been made. Topic: Utilitarianism Skill: Conceptual

37) What approach to fairness argues in favor of government policies that redistribute income so that there is more equality of income? Answer: The general approach to fairness that argues in favor of government redistribution is a “fair results approach,” that is, an approach that argues “it isn’t fair unless the results are fair.” Utilitarianism is the principle that states that a society should strive for “the greatest happiness for the greatest number of people.” Utilitarians argued that complete equality of income was the only income distribution that met their requirement so Utilitarians asserted that government policies to redistribute income are necessary for fairness.

Topic: Utilitarianism Skill: Conceptual

38) Explain the utilitarianism principle. How is it deficient? Answer: Utilitarianism is a principle that states that we should strive to achieve the greatest happiness for the greatest number. In order to do this, income must be redistributed from the rich to the poor until there is complete equality between all people. The rationale is that everyone has the same basic wants and a similar capacity to enjoy life. Also, the greater a person’s income, the smaller is the marginal benefit of a dollar. By transferring money from the rich to the poor, more is gained than is lost and the two people added together are better off. The deficiency is that the principle ignores the costs of making income transfers. Income can be redistributed through taxation. However, taxing people’s income from employment makes them work less which results in the quantity of labor being less than the efficient quantity. Taxing people’s income from capital induces them to save less. It results in the quantity of capital being less than the efficient quantity. With smaller quantities of both labor and capital, the quantity of goods and services produced is less than the efficient quantity. In addition, not all of the increase in taxes collected would be transferred to the poor. Some of the tax collection would be used to cover administrative costs. Topic: The Big Tradeoff Skill: Recognition

39) What is the “big tradeoff”? Answer: The “big tradeoff” is the tradeoff between efficiency and fairness. The idea is that if the government redistributes income so that it is more equally shared, output decreases so that it is less than the efficient amount. Output shrinks because such redistribution blunts people’s incentives to work. Hence redistributing income so that everyone has the same amount of income might end up insuring that everyone’s incomes are smaller than if less redistribution is pursued.

EFFICIENCY AND EQUITY

Topic: The Big Tradeoff Skill: Conceptual

40) Why do societies face a tradeoff between the size of the economic pie and the degree of equality with which it is shared? Answer: A person’s share of the economic pie is determined by his or her income. To increase the degree of equality, income must be redistributed away from richer people and towards poorer people. Income is transferred from the rich to the poor by means of taxes. Taxes discourage work and saving. Because people work less, the nation’s output decreases When saving decreases, so does investment in capital, which also decreases the nation’s output. Thus taxes that redistribute income in order to make for a more equal income distribution decrease the nation’s output, that is, shrink the economic pie. Topic: The Big Tradeoff Skill: Conceptual

41) Why does the problem of the big tradeoff arise when the government engages in the process of redistributing income using taxes and transfers? Answer: There are two reasons why the big tradeoff problem arises. First, in the process of transferring income from the people who have to those who do not have, an administrative cost is incurred by society. The result is that $1 taxed is not $1 transferred. Hence the effort to make incomes more equal decreases the average income. Second, taxing people’s income is a disincentive to work, while taxing people’s savings is a disincentive to accumulate capital. As a result, people work less and save less, both of which decrease the amount of goods and services produced and decrease people’s income. Hence once again the effort to make incomes more equal decreases the average income. Topic: John Rawls Skill: Conceptual

42) Explain the modified version of utilitarianism proposed in the book entitled “A Theory of Justice,” by the philosopher John Rawls and its relationship to the “big tradeoff.” Answer: Rawls was aware that extensive redistribution of income could decrease the average income, which is the tradeoff captured by the “big tradeoff.” Thus perfect equality of income might result in everyone having lower income than would oth-

35

erwise be the case. To overcome this issue, according to John Rawls, the fairest distribution of the economic pie is the one that provides the poorest person the largest income possible. Hence less redistribution will take place than if perfect equality was the goal. As a result, average income will decrease less and the poorest person would be better off. Topic: John Rawls Skill: Conceptual

43) Explain the principle “Make the poorest as well off as possible.” Who proposed it? Answer: This principle was proposed by John Rawls. According to this principle, taking all the costs of income transfer into account, the fair distribution of the economic pie is one that makes the poorest person as well off as possible. The incomes of rich people should be taxed, and after paying the costs of administering the tax and transfer system, what is left should be transferred to the poor. But the taxes must not be so high that they make the economic pie shrink to the point at which the poorest person ends up with a smaller piece. The goal is to make the piece enjoyed by the poorest person as big as possible. Topic: Symmetry Principle Skill: Recognition

44) “The principle that people in similar situations be treated similarly is called Utilitarianism.” Is the preceding statement correct or incorrect? Answer: The statement is incorrect. The principle that people in similar situations be similarly treated is called the symmetry principle. Utilitarianism is the principle that we should strive to achieve “the greatest happiness for the greatest number of people.”

36

Topic: Fair Rule Skill: Conceptual

45) Bill Gates is a founder of Microsoft and the world’s richest individual. Suppose Microsoft sells more software and Mr. Gates acquires another billion dollars in wealth. Simultaneously, suppose a burglar whose income is well below average broke into Bill Gates’ house and stole a million dollars worth of antiques. Using the “it’s not fair if the rules aren’t fair” approach to fairness, is Mr. Gates’ acquisition of additional wealth fair? Is the (poor) thief’s acquisition fair? Answer: In order for Mr. Gates to become richer, Microsoft had to convince consumers to buy their products. The consumers’ choices were voluntary. That is, the consumer engaged in a voluntary transaction with Microsoft and, as a result, Mr. Gates gained wealth. (And the consumers gained the software.) Because the exchange was voluntary, it is a fair exchange according to the “it’s not fair if the rules aren’t fair” approach. The burglar,

CHAPTER 5

however, did not engage in a voluntary transaction with Mr. Gates. Mr. Gates suffered an involuntary transaction with the burglar. Involuntary transactions violate the symmetry principle and hence the thievery is not fair according to the “rules” approach. Notice that the fairness has nothing to do with the incomes of Mr. Gates and the burglar; instead, fairness hinges on whether the transaction was voluntary. Topic: Fair Rule Skill: Conceptual

46) According to the “fair rules” view of fairness, are taxes fair? Explain. Answer: Taxes are unfair according to the fair rules view because they are an involuntary transfer of private property. While most economists and most people support fair taxation, there is little agreement on what constitutes a fair tax.

CHAPTER 6

MARKETS IN ACTION be rented at this price, with or without a law, so the ceiling has no effect.

Topic: Housing Markets and Rent Ceilings Skill: Conceptual

1)

What are the effects of a rent ceiling set below the equilibrium rent? Answer: A rent ceiling is the maximum legal rent. If the rent ceiling is set below the equilibrium rent, the rent ceiling creates a shortage of apartments. Consumer and producer surplus are both reduced. Hopeful renters face must engage in increased search activity. In order to rent an apartment, renters and landlords might engage in black market activity that effectively raises the rent. Topic: Housing Markets and Rent Ceilings Skill: Conceptual

2)

In the housing market, if a rent ceiling of $600.00 is imposed when the equilibrium rent is $500.00, why will nothing change? Answer: A rent ceiling is the maximum legal rent. Generally, the government imposes the ceiling when the equilibrium rent is considered too high. If, however, the rent ceiling ($600.00) is above the equilibrium rent ($500.00), people are already paying a rent lower than legal maximum. As a result, nothing changes and so this price ceiling is ineffective. Topic: Housing Markets and Rent Ceilings Skill: Conceptual

3)

What is the difference between a rent ceiling below the equilibrium rent and a rent ceiling above the equilibrium rent? Answer: Suppose the equilibrium rent without any rent ceiling is $1,000 per month. If the rent ceiling is set below $1,000, say at $800 per month, the ceiling then makes it unlawful to rent for more than $800. An apartment shortage results because the quantity demanded exceeds the quantity supplied at $800. A rent ceiling above the equilibrium rent $1,000, say $1,200 per month, has no effect. Why? Because no apartments would

Topic: A Regulated Housing Market Skill: Analytical

4)

Explain why in cities such as New York City that have rent ceiling laws, so many people who work in the city commute from outside the city. Answer: Rent ceilings decrease the quantity supplied of housing. So many people who work in the city and would like to live in the city cannot find housing. As a result, they must live outside the city and commute to work. Topic: Rent Ceiling Skill: Analytical

5)

Suppose that apartments rent for $1,300 a month in San Francisco, California and $850 a month in Los Angeles, California. If the state of California passes a state-wide rent ceiling for apartments of $1,100 a month, what occurs in the two cities? Answer: The rent ceiling is below the equilibrium rent in San Francisco. A shortage of apartments occurs as the quantity of apartments demanded increases and the quantity supplied decreases. The shortage becomes even larger as time passes because, with the rent ceiling, landlords have no incentive to maintain existing apartments or to build new ones. A black market for apartments will emerge, with bribery and “key money” becoming common. In Los Angeles, the outcome differs. The rent ceiling is above the equilibrium rent and has no impact on the quantity demanded or the quantity supplied. If, however, Los Angeles grows so that the demand for housing increases enough, the time may come when the rent ceiling is below the equilibrium rent in Los Angeles, in which case the Los Angeles apartment market also would be marked by a shortage and black market.

38

CHAPTER 6

Topic: A Regulated Housing Market Skill: Conceptual

Topic: Price Ceiling Skill: Conceptual

6)

9)

Why do rent ceilings lead to shortages and black markets? Answer: Rent ceilings that are set below the market equilibrium price for apartments lead to shortages for two reasons. First, the lower price increase the quantity of apartments demanded. Second, the quantity of apartments supplied decreases and, over time, the supply decreases even more because of the lower rent. When the quantity demanded exceeds the quantity supplied, a shortage is created. Faced with the inability to obtain the product they want, in this case, housing, people use bribes and other inducements in the black market to increase their chances of moving up the waiting list that emerges in the rent-controlled apartment market. Not everyone participates in the black market, but rent ceilings will favor those people willing to engage in illegal transactions. Topic: A Regulated Housing Market Skill: Conceptual

7)

What are the major economic effects of rent ceilings? Answer: Rent ceilings reduce the rental payments of those tenants able to find apartments at the controlled price and decrease the rental income received by landlords. They lead to a shortage of apartments, preventing some tenants from finding a suitable place to live. Another effect can be increased discrimination as landlords can choose tenants based on race, age, gender, or other criteria. Finally, rent ceilings result in black markets emerging and also increase the cost of searching for an apartment. Topic: A Regulated Housing Market Skill: Analytical

8) Do rent ceilings result in any deadweight loss? Answer: Yes. Because the number of apartments rented is decreased, some consumer surplus is lost and some producer surplus is lost. Part of these loses are gained by the tenants in the form of reduced rental payments. But the sum of the loses not gained by the tenants is the deadweight loss.

Might a price ceiling have no effect on a market’s price and quantity? If so, explain how; if not, explain why not. Answer: A price ceiling might have no effect on a market’s price or quantity. It will have no effect if it is set above the equilibrium price. Topic: Price Ceiling Skill: Conceptual

10) In order for a price ceiling to have an effect on the market, must it be set above or below the equilibrium price? Why? Answer: In order to have an effect, the price ceiling must be set below the equilibrium price. A price ceiling is the highest price that can legally be charged for the product. If it is set above the equilibrium price, the equilibrium price is less than the ceiling price and hence is legal. Therefore the price and the quantity are not affected. However if the ceiling price is less than the equilibrium price, the equilibrium price becomes illegal. As a result, the price ceiling will have an impact on the market by lowering the price. With the lower price, the quantity demanded increases while the quantity supplied decreases so that a shortage of the product emerges. Topic: Price Ceiling Skill: Conceptual

11) Suppose the government imposes a price ceiling that is less than the equilibrium price. Discuss the effect, if any, on the price and quantity if the government later removes the price ceiling. Answer: If the price ceiling is less than the equilibrium price, the price ceiling has an effect on the market. When the price ceiling is imposed, it increases the quantity demanded and decreases the quantity supplied, thereby creating a shortage of the good. When the government removes the price ceiling, the reverse effects occur. The price rises back to the equilibrium price. With the higher price, the quantity demanded decreases and the quantity supplied increases. The shortage is eliminated because the quantity produced equals the quantity consumed and both are equal to the equilibrium quantity.

MARKETS IN ACTION

39

Topic: Price Ceiling Skill: Conceptual

Topic: Search Skill: Recognition

12) In the 1980s, one of the most common sights in the socialist countries, such as the former Soviet Union and North Korea, were long lines for bread, sugar, and other necessities. These countries had price ceilings on these necessities. Some of the socialist nations, such as the former Soviet Union, have moved to a market economy by lifting the price ceilings, while others, such as North Korea, have retained their price ceilings. What prediction do you make about the presence (or absence) of long lines today in the former Soviet Union and North Korea? Explain your answer. Answer: The long lines have disappeared in the former Soviet Union but are still present in North Korea. The socialist countries had price ceilings on necessities that were below the equilibrium price. These ceilings created shortages, thus the long lines. In the countries that have lifted the price ceilings, the price has risen toward the equilibrium price, thereby eliminating the shortages and long lines. In the nations that have retained the price ceilings, the shortages and hence the long lines have persisted.

14) What is search activity and how does it influence consumer surplus when a rent ceiling creates a housing shortage? Answer: Search activity is the time spent looking for someone with whom to do business. When a rent ceiling creates a shortage of housing, search activity increases. As people spend more time searching for rental housing, the opportunity cost of housing, which is the rent plus the value of the search time, increases and therefore consumer surplus shrinks.

Topic: Price Ceiling Skill: Conceptual

Topic: The Minimum Wage Skill: Recognition

13) What is a black market and how does it influence the market for rental housing if a rent ceiling creates a housing shortage? What determines the level of a black market rent? Answer: A black market is an illegal market in which the price exceeds the legally imposed price ceiling. When a rent ceiling creates a housing shortage, frustrated renters offer a rent above the legal level in order to get an apartment. The level of a black market rent depends on how tightly the rent ceiling is enforced. With loose enforcement, the black market rent is close to the unregulated rent. With strict enforcement, it is close to the maximum price that renters are willing to pay given the rentceiling quantity of housing supplied.

Topic: The Minimum Wage Skill: Recognition

15) What is a minimum wage? What are the effects of a minimum wage set below the equilibrium wage rate? Answer: A minimum wage is a regulation that makes it illegal to pay or receive a wage rate lower than a specified level. If a minimum wage is set below the equilibrium wage, the minimum wage has no effect because it does not make the equilibrium wage rate illegal.

16) Is the minimum wage an example of a price floor or a price ceiling. Why? Answer: The minimum wage is an example of a price floor. A price floor sets the lowest price at which it is legal to trade a particular good, service, or factor of production. The minimum wage is a price floor imposed in the labor market. The minimum wage sets the lowest price for which labor can be traded. It is illegal to buy (or to sell) labor for less than the minimum wage. Topic: The Minimum Wage Skill: Conceptual

17) What happens to the quantity of labor supplied, the quantity of labor demanded, and the number of unemployed workers if the minimum wage rate is increased? Answer: An increase in the minimum wage increases the quantity of labor supplied because more people are willing to work at the higher wage rate. It decreases the quantity of labor demanded because firms hire fewer workers since the cost of the workers (their wage rate) has increased. The in-

40

CHAPTER 6

crease in the quantity supplied combined with the decrease in quantity demanded leads to an increase in the number of unemployed workers. Topic: The Minimum Wage Skill: Conceptual

18) Will an increase in the minimum wage create more unemployment if the supply and demand for labor are highly elastic or highly inelastic? Answer: An increase in the minimum wage creates less unemployment when the supply and demand for labor are highly inelastic. If the supply and demand for labor are highly elastic, an increase in the minimum wage leads to more unemployment because the quantity of labor supplied increases significantly and the quantity of labor demanded decreases significantly. If they are inelastic, an increase in the minimum wage leads to only a small decrease in the quantity of labor demanded and only a small increase in the quantity of labor supplied, so that the resulting unemployment is not very large. Topic: The Minimum Wage Skill: Conceptual

19) Why would an increase in the minimum wage to $15 per hour lead to more unemployment for teenage and low-skilled workers? Answer: Many teenage and low-skilled workers earn far less than $15 per hour. Hence a price floor in the labor market that “guaranteed” a wage of $15 per hour, that is, a minimum wage of $15 per hour, would have a double-edged effect. First, the quantity of labor supplied would increase because more people would be willing and able to work for the higher wage. Indeed, some high school students would drop out of school in order to obtain work if they thought they would be paid $15 an hour. Second, many businesses would be unwilling and unable to pay $15 per hour and would therefore lay off many teenage and lowskilled workers. In other words, the quantity of labor supplied increases and the quantity of labor demanded decreases. The resulting surplus of labor means a higher unemployment rate for the very people the minimum wage is designed to help.

Topic: The Minimum Wage Skill: Conceptual

20) Explain why low-skilled workers find that their employment opportunities are less with a minimum wage. Answer: A minimum wage set above the equilibrium wage rate decreases the quantity of labor demanded, so employment decreases with a minimum wage. This effect is particularly strong in the market for low-skilled labor, because it is lowskilled labor that is paid low wage rates. Thus it is low-skilled workers for whom the decrease in employment is severe. Topic: The Minimum Wage Skill: Conceptual

21) How does the elasticity of demand for labor affect the deadweight loss from an increase in the minimum wage? Why? Answer: The more elastic the demand for labor, the greater the deadweight loss from increasing the minimum wage. Essentially, the more elastic the demand for labor, the larger the decrease in the quantity of labor demanded when the minimum wage rate rises. As a result, the larger the decrease in employment and hence the greater is the deadweight loss. Topic: Inefficiency of Price Floors Skill: Conceptual

22) Discuss the inefficiencies created by a price floor. Answer: A price floor prevents the efficient allocation of resources. If a price floor is set above the equilibrium price, the quantity of the good or service supplied increases and the quantity demanded decreases, so that a surplus results. The minimum wage is a price floor. If the minimum wage is set above the equilibrium wage rate, the surplus of labor means increased unemployment. Similar to all price floors, the minimum wage creates inefficiency. The minimum wage leads to inefficiency because at the minimum wage, marginal benefit to a firm from hiring another worker exceeds the marginal cost to a worker from working. Topic: The Minimum Wage in Practice Skill: Conceptual

23) Who are minimum wages designed to help? Do they succeed? What is their effect on society? Answer: Minimum wages are designed to improve the incomes of low-wage earners. They do successfully

MARKETS IN ACTION

raise the wages of those fortunate enough to find jobs at the minimum wage; however they reduce the chance of finding a job by decreasing the quantity of labor demanded. The low-wage earners who cannot find a job with a minimum wage in place are harmed. Minimum wages harm society because they create a deadweight loss. In addition to the deadweight loss, minimum wages impose other costs on society, such as: they increase the amount of time it takes to find a job (search activity). they encourage illegal hiring at wages below the minimum wage rate. they block voluntary exchange at the equilibrium wage. Topic: The Minimum Wage in Practice Skill: Conceptual

24) We know that the minimum wage causes unemployment. So, why does the government impose one? Answer: The main reason for the minimum wage is because its supporters outnumber its opponents. Some supporters, such as labor unions, have a self-interested motivation for seeing a high minimum wage. A high minimum wage increases the cost of low-skilled labor, which is a substitute for high-skilled union labor. Hence firms decrease the quantity of low-skilled labor they demand and increase their demand for union labor, thereby increasing the employment and wages of union labor. Other supporters point to the fact that the minimum wage helps raise the incomes of lowskilled workers who retain jobs. Also, if the demand and supply of low-skilled workers is inelastic, the resulting unemployment will be low and so supporters might believe that helping some low-skilled workers is worth harming others. Topic: Price Floor Skill: Recognition

25) Explain why a price floor set below the equilibrium price is ineffective. Answer: Price floors are legal minimums; it is legal to charge the floor price, or higher. If the equilibrium price falls in the legal range, that is, the equilibrium price is above the price floor so that the price floor is below the equilibrium price, then

41

the price floor has no effect on the market because the market price does not change. Topic: Price Floor Skill: Conceptual

26) Explain how a price floor set above the equilibrium market price for a good can cause a surplus of that good. Answer: At the market equilibrium price, the quantity demanded equals the quantity supplied. If a price floor is set above the equilibrium price, the price floor has two separate effects and both help create a surplus of the good. First, the higher price increases the quantity supplied. Second, the higher price decreases the quantity demanded. On both counts—the increase in the quantity supplied and the decrease in the quantity supplied—a surplus is created. Topic: Price Floor Skill: Conceptual

27) Must a price floor always lead to a surplus? Explain your answer Answer: No, a price floor does not necessarily result in a surplus. In particular, a price floor set below the equilibrium price has no effect on the equilibrium price or quantity. Topic: Price Floor Skill: Conceptual

28) “A price floor that is less than the equilibrium price leads to a shortage of the good.” Is this assertion true or false? Explain your answer. Answer: The assertion is false. A price floor is the minimum price that can legally be charged. If the price floor is less than the equilibrium price, it has no effect because the equilibrium price is already in compliance with the law. Hence a price floor set below the equilibrium price has no effect. However, a price floor set above the equilibrium price will have an effect and will create a surplus of the good. Topic: Price Ceiling and Price Floor Skill: Conceptual

29) What occurs if a price floor is set above the equilibrium price? What occurs if a price ceiling is set below the equilibrium price? Answer: A price floor set above the equilibrium price will result in a surplus of the product. Because the price floor is above the equilibrium price, the

42

CHAPTER 6

price rises above the equilibrium value. With the higher price, the quantity supplied increases and the quantity demanded decreases. At the floor price, the quantity supplied exceeds the quantity demanded and hence a surplus is the outcome. A price ceiling set below the equilibrium price also affects the market. The price ceiling is below the equilibrium price, so the price falls, from what it was at its equilibrium, to the ceiling amount. The fall in price increases the quantity demanded and decreases the quantity supplied. As a result, at the price equal to the ceiling floor, the quantity demanded exceeds the quantity supplied and hence there is a shortage. Topic: Price Ceiling and Price Floor Skill: Conceptual

30) Compare and contrast a price ceiling and a price floor. Answer: A price ceiling is the maximum legal price that may be charged. A price floor is the minimum legal price that may be charged. For a price ceiling or a price floor to have an effect, they must make the equilibrium price illegal. So, an effective price ceiling is set below the equilibrium price while an effective price floor is set above the equilibrium price. A price ceiling creates a shortage of the good while a price floor creates a surplus. Both create an inefficient market outcome because the marginal benefit of the last unit consumed no longer equals the marginal cost of the last unit produced. Topic: Tax Incidence Skill: Conceptual

31) How is the incidence of a sales tax between the buyer and the seller determined? Answer: It all depends on the price elasticities of supply and demand. Buyers pay more of the tax the more inelastic the demand and the more elastic the supply. Sellers pay more of the tax the more elastic the demand and the more inelastic the supply.

Topic: Tax Incidence Skill: Conceptual

32) When a tax is imposed on sellers of a good, the resulting rise in the equilibrium price is usually less than the amount of the tax itself. Why doesn’t the equilibrium price rise by the full amount of the tax? Answer: Because firms collect taxes from consumers and send them to the government, the taxes drive a wedge between the price or dollar amount firms receive from consumers and the price or dollar amount they get to keep for selling the good. Firms would like to raise the price by the full amount of the tax. In this case, the price firms receive would stay the same and so the firms would continue to supply the same amount of the product. But as the price rises, the quantity demanded decreases. Hence, if the price rose by the full amount of the tax, the quantity supplied would not change and the quantity demanded would decrease. There would be a surplus of the product. The surplus forces the price downward, thereby making the rise in price less than the full amount of the tax. As a result, in the equilibrium the price generally does not rise by the full amount of the tax. Topic: Tax Incidence Skill: Conceptual

33) The social security tax is a tax that Congress imposes equally on both employers and employees. Does this mean that the burden of this tax is shared equally between firms and workers? Explain. Answer: No. A tax has the same effect regardless of whether it’s imposed on buyers (employers) or sellers (employees). The division of the burden of a tax between firms and workers depends on the elasticities of labor demand and labor supply, not on the tax law. That is, the market for labor, not Congress, decides how the burden of the Social Security tax is divided by firms and workers.

MARKETS IN ACTION

43

Topic: Tax Incidence Skill: Conceptual

Topic: Tax Incidence and the Elasticity of Demand Skill: Conceptual

34) Suppose that the government wants the burden of the cigarette tax to fall equally on buyers and sellers and declares that a $1.00 tax be imposed on each. Is the burden of the tax shared equally? Why or why not? Answer: Not necessarily. A tax has the same effect regardless of whether it’s imposed on buyers or sellers. The division of the burden of a tax between buyers and sellers depends on the elasticities of demand and supply, not on the tax law.

38) Explain under what conditions a sales tax on a specific good would be paid entirely by buyers. Answer: Buyers pay all of a tax when the demand for the product is perfectly inelastic or when the supply is perfectly elastic.

Topic: Tax Incidence and Elasticity Skill: Conceptual

35) Explain the relationship between the incidence of a tax and elasticity. Answer: It does not matter whether a tax is imposed on the buyers or the sellers. Once the tax is imposed on the market, the buyers and the sellers generally will both pay a share of the tax because the buyers will pay a higher price and the sellers will receive a lower price. The Incidence of the tax between the buyers and the sellers depends in the elasticities of demand and supply. Because elasticity is a measure of sensitivity to a change in price, the party that is less sensitive to price changes will end up paying a larger proportion of the tax. Topic: Taxes Skill: Conceptual

36) Can the incidence of a sales tax ever be so that buyers pay all of the tax or so that sellers pay all of the tax? Answer: Yes, each is possible. If demand is perfectly price inelastic, for example in the case of an extreme necessity, the tax will be paid entirely by the buyer. If supply is perfectly inelastic, for example in the case of the momentary supply curve, the tax will be paid entirely by the seller. Topic: Tax Incidence and the Elasticity of Demand Skill: Conceptual

37) How does the elasticity of demand affect the incidence of a tax? Answer: For a given elasticity of supply, the more elastic the demand, the smaller the share of a tax that falls on the buyer and hence the larger the share of a tax that falls on the seller.

Topic: Tax Incidence and the Elasticity of Demand Skill: Conceptual

39) Suppose the demand for saline solution is perfectly inelastic for contact lens wearers. If the government imposes a tax on saline solution, what occurs? Be sure to tell what happens to the price paid by the buyers and discuss the incidence of the tax. Answer: If demand is perfectly inelastic, then no matter what the price, buyers will not decrease their quantity demanded when the price rises. Hence the tax will be passed along entirely to the consumers. So, in the case of saline solution, the price paid by the buyers will rise by the full amount of the tax and so buyers pay the entire amount of the tax. Topic: Tax Incidence and the Elasticity of Demand Skill: Conceptual

40) Suppose the government decides to tax salt. The demand for salt is inelastic and the supply is quite elastic. Who bears the pays most of this tax? Answer: Buyers of salt pay most of the tax because their demand for salt is inelastic. Topic: Tax Incidence and the Elasticity of Demand Skill: Analytical

41) Which would be a better source of tax revenue for the government, a good with elastic or a good with an inelastic demand? Explain your reasoning. Answer: Inelastic goods are better sources of tax revenue because, as price rises, the equilibrium quantity does not decrease by as much as that of a good with elastic demand. The government’s tax revenue, which depends on the equilibrium quantity of the good, is larger when a good with an inelastic demand is taxed. Topic: Tax Incidence and the Elasticity of Supply Skill: Analytical

42) Suppose the price elasticity of demand for Mexican food is 1.23 and the price elasticity of supply is 0.47. If the government imposes a tax on Mexi-

44

can food, do buyers or sellers pay most of the tax? Why? Answer: Sellers pay most of the tax because supply is inelastic while demand is elastic. There are many substitutes for the good, so unless sellers are willing to pay most of the tax, buyers spend their money on other types of food. Topic: Taxes, Deadweight Loss Skill: Analytical

43) Is the deadweight loss from a sales tax on a product larger the more inelastic the demand for the good? Answer: No, the deadweight loss from a sales tax is smaller the more inelastic the demand for the good. Topic: Farm Revenues Skill: Conceptual

44) What is the relationship between the weather, as it affects growing conditions, and farm revenues? Answer: Bad weather shifts the supply curve leftward and raises prices. Because the demand for farm products is price inelastic, however, total revenue increases. Great weather can create a bumper crop and shift the supply curve rightward but, because demand is inelastic, total revenue decreases. So, there is a paradox: bad weather, small crop, higher total revenue; great weather, larger crop, lower total revenue. Topic: Subsidy Skill: Recognition

45) A student wrote: “A subsidy raises marginal benefit above marginal cost and eliminates the deadweight loss from underproduction.” If you were the instructor, how would you correct this statement? Answer: The student errs in several important respects. Here is a corrected statement: “A subsidy raises marginal benefit cost above marginal cost benefit and eliminates the creates a deadweight loss from underproduction overproduction.” Topic: Subsidy Skill: Recognition

46) “Farm subsidies in the European Union spill over to the rest of the world.” Explain this assertion. Answer: Farm subsidies in the European Union lower the price at which European farmers can offer their output for sale on the world market, which

CHAPTER 6

lowers the price in the rest of the world. Faced with lower prices, farmers in other countries decrease production and receive smaller revenues. Topic: Production Quota Skill: Recognition

47) How does a production quota influence farm prices and output? Answer: If a production quota is set above the equilibrium quantity, it has no effect. If a production quota is set less than the equilibrium quantity, it decreases output and raises the price. Topic: Production Quota Skill: Recognition

48) A student wrote: “A production quota is inefficient because it results in overproduction. At the quota quantity, marginal cost is equal to the market price and marginal benefit is less than the market price, so marginal cost exceeds marginal benefit.” If you were the instructor, how would you correct this statement? Answer: The student errs in several important respects. There are errors of omission and commission. Here is a corrected statement: “A production quota is inefficient because (if set below the equilibrium quantity) it results in overproduction underproduction. At the quota quantity, marginal cost benefit is equal to the market price and marginal benefit cost is less than the market price, so marginal cost benefit exceeds marginal benefit cost.” Topic: Markets for Illegal Goods Skill: Conceptual

49) When the government passes a law making a particular good illegal, does it matter for the black market price and quantity if the penalties for breaking the law are imposed on the buyers or on the sellers? Answer: Yes, it matters. Imposing penalties on the buyers shifts the demand curve leftward. Imposing penalties on the sellers shifts the supply curve leftward. Either penalty decreases the quantity, but the black market price will exceed the legal market price if only the seller is penalized and will be below the legal market price if only the buyer is penalized.

MARKETS IN ACTION

Topic: Markets for Illegal Goods Skill: Analytical

50) In 1920 a constitutional amendment was passed that outlawed the production, sale, purchase, and consumption of alcoholic beverages. “Prohibition” encouraged bootlegging and black markets for whiskey, wine, and beer. The amendment was eventually repealed in 1933. In 1920, what alternative economic policy was available to the government as a means of reducing alcohol consumption nationwide? Answer: The government could have kept alcoholic beverages legal and then placed a heavy tax on them. This policy would have shifted the supply curve leftward, increased the (legal) price, and decreased the (legal) quantity. Unfortunately, the higher the tax, the more likely it would have been that black markets would have developed (as with outright prohibition) to avoid the tax.

45

CHAPTER 7

UTILITY AND DEMAND satisfaction a person gets from consumption of a good or service.

Topic: Consumption Possibilities Skill: Recognition

1)

You are studying with a friend and your friend says “A budget line shows the various combinations of two goods that can be purchased with the buyer’s income at current prices.” Is your friend’s assessment correct or not? Answer: Your friend’s description of the budget line is accurate. Topic: Consumption Possibilities Skill: Recognition

2)

“Consumption possibilities are constrained by total utility.” True or false? Explain. Answer: False. Consumption possibilities are constrained by the consumers’ income and by the prices of goods and services they buy. Total utility does not constrain consumption, it is a measure of total benefit that consumers get from consumption of goods and services. Topic: Consumption Possibilities Skill: Recognition

3)

How is a budget line similar to a production possibilities frontier? How do they differ? Answer: Both the budget line and the production possibilities frontier illustrate limits. The budget line describes the limits to consumption possibilities; the production possibilities frontier describes the limits to production possibilities. A consumer is unable to consume combinations of goods that lie beyond his or her budget line. Similarly, a nation is unable to produce combinations of goods that lie beyond its production possibilities frontier. Topic: Total Utility Skill: Recognition

4)

Is “utility” another word for the cost we give up when we consume a good? Answer: No, utility does not mean the opportunity cost of consuming a good. Utility is the benefit or

Topic: Total Utility Skill: Conceptual

5)

If utility could actually be measured in numerical units, how would you compute the total utility derived from consuming five hamburgers using marginal utilities? Answer: The total utility from consuming any product is the sum of the marginal utilities of each unit of the product. So, if the five hamburgers yielded marginal utilities of 12, 8, 7, 5, and 3, respectively, the total utility from hamburgers would equal 35. Topic: Total Utility and Marginal Utility Skill: Conceptual

6) How are total and marginal utility related? Answer: Marginal utility is the change in total utility from consuming one more unit of a good. Topic: Total Utility and Marginal Utility Skill: Conceptual

7)

Are your total and marginal utility of ice cream schedules determined by the price of ice cream? Answer: No. Your total and marginal utility are determined purely by how much or how little you like ice cream. If you like it a lot, your marginal utility will be higher than otherwise for each additional ice cream scoop you consume and that will cause your total utility to be higher. Topic: Total Utility and Marginal Utility Skill: Conceptual

8)

“Diminishing marginal utility means a downward sloping total utility curve.” True or false? Explain. Answer: False. Diminishing marginal utility means a downward sloping marginal utility curve. A total utility curve stays upward-sloping as far as marginal utility is positive.

48

CHAPTER 7

Topic: Marginal Utility Skill: Recognition

Topic: Maximizing Utility Skill: Conceptual

9) What is “marginal utility?” Answer: Marginal utility is the change in total utility that results from a one-unit change in the quantity of a good consumed. In other words, it is the added utility a consumer gains by consuming one additional unit of a good or service.

14) If a consumer is consuming a combination of goods and services on his budget line, has he allocated his entire budget? Answer: Yes, the consumer has allocated his entire budget because he is spending his entire budget.

Topic: Diminishing Marginal Utility Skill: Recognition

10) What does “diminishing marginal utility” mean? Answer: As the consumption of a good or service increases, the marginal utility decreases. For example, the first cup of coffee you drink in the morning will give you great satisfaction. As you keep on drinking more and more cups of coffee, the satisfaction from additional cups will diminish. Topic: Diminishing Marginal Utility Skill: Conceptual

11) When you have diminishing marginal utility of Coca Cola, is your total utility curve for Coke necessarily downward sloping? Answer: No. If you have diminishing but positive marginal utility, your total utility curve will be upward sloping. If you have diminishing and negative marginal utility, then your total utility curve will be downward sloping. Topic: Diminishing Marginal Utility Skill: Conceptual

12) “As Rob consumes more dates over the course of a day, it is likely that his marginal utility from date consumption will rise.” Is the previous statement likely correct or incorrect? Answer: The statement is incorrect. The principle of diminishing marginal utility means that Rob marginal utility from eating dates decreases as he eats more dates. Topic: Maximizing Utility Skill: Conceptual

13) Explain the concept of consumer equilibrium. Answer: A consumer equilibrium is a situation in which a consumer has allocated all his or her available income in the way that, given the prices of goods and services, the combination of goods and services maximizes the consumer’s total utility.

Topic: Equalizing Marginal Utility per Dollar Spent Skill: Recognition

15) What are the two conditions that are met if a consumer is maximizing utility? Answer: There are two conditions: (1) The consumer spends all his/her income. (2) The consumer equalizes the marginal utility per dollar spent on all goods. Topic: Equalizing Marginal Utility per Dollar Spent Skill: Recognition

16) You are studying with a friend, and your friend says “To maximize utility, a consumer must consume the combination of goods so that the marginal utility of good X equals the marginal utility of good Y.” Explain whether your friend’s statement is correct or incorrect. Answer: Your friend’s statement is incorrect. To maximize utility, a consumer consumes the combination of goods and services so that the marginal utility per dollar spent on the goods is equal. Topic: Equalizing Marginal Utility per Dollar Spent Skill: Recognition

17) What is the utility-maximizing rule? Answer: The utility-maximizing rule states that for a person to maximize utility, the person must allocate (spend) his or her entire budget and consume the quantities of goods and services so that the marginal utility per dollar spent on each good or service is equal to the marginal utility per dollar spent on all the other goods and services. Topic: Equalizing Marginal Utility per Dollar Spent Skill: Conceptual

18) Why is utility maximized when the marginal utility per dollar spent on all goods is equal? Answer: Suppose an individual is choosing between two goods. When comparing the marginal utility per dollar spent on each good, the consumer will be better off by choosing the good in which the per dollar marginal utility is higher. The good that is not chosen is the forgone opportunity, the

UTILITY AND DEMAND

49

value of which is the marginal utility per dollar spent on that good. Choosing the good with the higher per dollar marginal utility is tantamount to choosing the good with the lower opportunity cost.

marginal utility per dollar spent on the Hyundai is 6. A Hyundai gives the consumer more utility per dollar spent. Thus when maximizing total utility, the consumer compares the marginal utility per dollar spent not the marginal utility.

Topic: Equalizing Marginal Utility per Dollar Spent Skill: Conceptual

Topic: Equalizing Marginal Utility per Dollar Spent Skill: Conceptual

19) Explain why total utility is maximized when the marginal utility per dollar spent on all goods is equal. Answer: It is easiest to prove that total utility is maximized when the marginal utility per dollar is equal across goods by looking at a situation in which the marginal utility per dollar is NOT equal across goods. If the marginal utility per dollar is 5 for good X and 8 for good Y, then the consumer could reallocate one dollar away from good X (losing 5 units of utility) and towards good Y (gaining 8 units of utility). The net result is a gain of 3 units of utility, for an increase in total utility. Hence whenever the marginal utility per dollar spent is not equal, the consumer can always rearrange his or her consumption and increase his or her total utility. Therefore the total utility cannot be at its maximum when the marginal utility per dollar differs for different goods. Only when all goods have the same marginal utility per dollar is it impossible for the consumer to reallocate funds and become better off. Hence at this point the consumer must have the maximum total utility.

21) Suppose that Arnold spends all his income on bratwursts and piano lessons and his marginal utility per dollar spent on bratwursts is lower than that on piano lessons. Is Arnold maximizing his utility? Why or why not? Answer: No. If Arnold spends a dollar less on bratwursts and a dollar more on piano lessons, his utility gain from more piano lessons will be greater than his utility loss from the decreased consumption of bratwursts and hence his total utility will increase.

Topic: Equalizing Marginal Utility per Dollar Spent Skill: Conceptual

20) To maximize utility, why does a consumer consume the combination of goods that equates marginal utility per dollar spent on the different goods rather than just equating the marginal utility of the different goods? Answer: The consumer equates the marginal utility per dollar spent on the goods because goods have different prices. For instance, the marginal utility of a Porsche might be 120,000 units and the marginal utility of a Hyundai might be 60,000 units. The consumer certainly prefers the Porsche to the Hyundai because the former has a greater marginal utility than the latter. However, we did not consider the prices. If the Porsche costs $120,000 and the Hyundai costs $10,000, the marginal utility per dollar spent on the Porsche is 1 and the

Topic: Equalizing Marginal Utility per Dollar Spent Skill: Conceptual

22) Suppose that Alyssa spends all her income on video games and DVDs and her marginal utility per dollar spent on video games equals that on DVDs. Is Alyssa maximizing her utility? Now, suppose that the price of a DVD falls. Should Alyssa change the combination of goods she consumes? If yes, how? Explain. Answer: If Alyssa spends all her income on video games and DVDs and her marginal utility per dollar spent on video games equals that on DVDs, both utility maximization conditions are met and therefore Alyssa maximizes her utility. If the price of a DVD falls, Alyssa’s marginal utility per dollar spent on DVDs becomes greater than that on video games. Therefore, if she spends a dollar more on DVDs and a dollar less on video games, her utility gain from the increased consumption of DVDs will be greater than her utility loss from the decreased consumption of video games and hence her total utility will increase. So, to maximize her utility, Alyssa should increase her consumption of DVDs and decrease her consumption of video games.

50

Topic: Predictions of Marginal Utility Theory Skill: Analytical

23) If Jack is a utility maximizer, shouldn’t a fall in the price of popcorn cause an increase in his marginal utility of popcorn? Answer: No. If the price of popcorn drops, Jack will buy more popcorn. With diminishing marginal utility, his total utility from popcorn will increase but his marginal utility will fall. Topic: Paradox of Value Skill: Recognition

24) What is the water and diamonds paradox of value and what is its resolution? Answer: The paradox is that water is critical to life— has high total utility—but sells for a low price. Diamonds, in contrast, are nonessential—have low total utility—but sell for a high price. The paradox is resolved by realizing that the price we are willing to pay is linked to the marginal utility from the next glass of water or diamond, not the total utility of all units of each previously obtained. We already have much water, so the marginal utility of the next water is low. But most people have few diamonds, so the marginal utility of the next diamond is high. Topic: Paradox of Value Skill: Conceptual

25) Explain the paradox of value. Answer: The paradox of value questions why water, which is essential for life, costs less than diamonds, which are not. This puzzle can be resolved by realizing the difference between marginal utility and total utility. Water has high total utility, but because we consume so much water, the last unit has little value, that is, the water has low marginal utility. Diamonds have little total utility, but because we consume few diamonds, the last unit of diamonds has high marginal utility. Topic: Paradox of Value Skill: Conceptual

26) “Water is very inexpensive. Thus the marginal and total utility of water is small.” Analyze the previous statements. Answer: Parts of the statement are true but a key part is false. Water is, indeed, very inexpensive and so its marginal utility is small. Because water is so inexpensive, the quantity people buy is large, which is why its marginal utility is small. However, wa-

CHAPTER 7

ter is essential to life. Hence the total utility of water is immense. Topic: Paradox of Value Skill: Conceptual

27) Does the fact that diamonds, which we do not need to survive, are more expensive than water, which is a necessity, constitute a violation of utility maximization? Answer: No, the fact that diamonds are less useful but more expensive than water reflects the paradox of value. The resolution to this paradox comes from the observation that utility maximization involves marginal utility (specifically, marginal utility per dollar spent) not total utility. Water has a low marginal utility, because people have a lot of it, but a very high total utility, because it is essential to life. Diamonds have a high marginal utility, because people have fewer diamonds, but a relatively low total utility because diamonds are not necessary to life. Hence the low marginal utility of water is reflected in the low price of water, while the high marginal utility of diamonds is reflected in the high price of a diamond. Topic: Paradox of Value Skill: Conceptual

28) Why does gold, which is a relatively nonessential item, have a higher price than water, which is essential to life? Answer: Gold has a high price and a high marginal utility, while water has a low price and a low marginal utility. Water is relatively abundant in supply, while gold is relatively scarce. Hence people consume a lot of water and only a little gold. As a result, the marginal utility of water is low and the marginal utility of gold is high. The utility maximizing rule, that the marginal utility per dollar spent on gold must equal that of water, in combination with the high marginal utility of gold and low marginal utility of water, implies that the price of gold must be higher than that of water.

UTILITY AND DEMAND

Topic: Paradox of Value Skill: Conceptual

29) A consumer allocates his budget according to rules of utility maximization. What are the rules of utility maximization and how do they explain the paradox of value, which is that diamonds are expensive but useless, while water is inexpensive but essential? Answer: The rules of utility maximization are to allocate the entire available budget and consume the combination of goods and services that make the marginal utility per dollar spent equal for all goods and services. Equating the marginal utility per dollar spent is what explains the paradox of value. People consume a great deal of water. Because they consume a lot of water, the marginal utility of water is quite low. However, people have only a few diamonds. Hence the marginal utility of a diamond is quite high. Thus for the marginal utility per dollar spent of water to equal that of diamonds requires that the price of water be low and the price of diamonds high. Note, however, that this condition says nothing about the total utility from water or the total utility from diamonds. Because water is essential to life, its total utility is tremendous. Because diamonds are not essential, their total utility is much smaller. Topic: Paradox of Value Skill: Conceptual

30) “Because the price of a diamond is much greater than the price of a gallon of water, the consumer surplus from diamonds is greater than the con-

51

sumer surplus from water.” Is the previous analysis correct? Explain your answer. Answer: No, the analysis is incorrect. The paradox of value points out the error. Diamonds are scarce, so they have a high price and a high marginal utility. Water is abundant, so it has a low price and a low marginal utility. But the total utility from water (vastly) exceeds that from diamonds and so the consumer surplus from water (vastly) exceeds that from diamonds. Topic: Paradox of Value Skill: Analytical

31) Does the fact that diamonds are so expensive imply that the demand curve for diamonds has a positive slope? Use marginal utility theory to answer the question. Answer: No. In order for the demand curve for diamonds to have a positive slope, the marginal utility of each diamond consumed would have to increase as more diamonds are consumed. Diamonds are no different than any other good. As more diamonds are consumed, the marginal utility of a diamond decreases. Therefore in order to increase the quantity of diamonds people demand, the price of diamonds must fall, which indicates a downward-sloping demand curve for diamonds.

CHAPTER 8

POSSIBILITIES, PREFERENCES, AND CHOICES

Topic: Consumption Possibilities, Budget Line Skill: Conceptual

Topic: Consumption Possibilities, Budget Line Skill: Conceptual

1)

What information do you need to draw your budget line? What is your budget line? What does it show for any pair of goods? Answer: To draw a budget line for any pair of goods, you need to know your income and the price of each good. The budget line shows all combinations of the two goods that you can afford. The slope of the budget line is the ratio of the prices and shows, therefore, the rate at which the market requires you to give up some of one for more of the other.

4)

Topic: Consumption Possibilities, Budget Line Skill: Recognition

5) What does the slope of the budget line equal? Answer: The slope of the budget line equals an opportunity cost. It is the same as the opportunity cost of the good measured along the x-axis. It also equals a relative price. In particular, it is the same as the relative price of the good measured along the x-axis.

2) What is a household’s real income? Answer: A household’s real income is the household’s income expressed as a quantity of goods the household can afford to buy. Measured using a budget line, the household’s real income is the maximum quantity of the specific goods or services plotted on the budget line. Topic: Consumption Possibilities, Budget Line Skill: Conceptual

3) Why is the budget line negatively sloped? Answer: Given the consumer’s budget and the prices for two goods, if a consumer buys more of one good, he or she must buy less of the other. The budget line shows the trade-off between the two goods, that is, as more of one is purchased, less of the other can be purchased.

Suppose a budget line is drawn between pizza, on the horizontal axis, and tacos, on the vertical axis. How does a change in the price of a pizza affect the budget line? Answer: When the price of the good measured on the horizontal axis, pizza, falls, the budget line rotates outward and becomes flatter. And, when the price of a pizza rises, the budget line rotates inward and becomes steeper. Topic: Consumption Possibilities, Budget Line Skill: Analytical

Topic: Consumption Possibilities, Change in Price Skill: Conceptual

6)

If the price of a good rises and the consumer’s budget remains the same, what happens to the consumer’s consumption possibilities? Answer: The consumer’s consumption possibilities decrease. This decrease is reflected by the change in the budget line, which rotates inward. The inward movement means that consumption possibilities that had previously been affordable are no longer affordable and hence the possible consumption possibilities have decreased.

54

Topic: Consumption Possibilities, Change in Income Skill: Conceptual

7)

If your income increases, what is the effect on your budget line? Answer: Your budget line will shift outward and not change its slope. Topic: Consumption Possibilities, Change in Income Skill: Conceptual

8)

“If Bill’s income increases from $30,000 per year to $41,000 per year and he consumes pickup trucks and lamb chops with his money, Bill’s budget line shifts outward and the increase in income means he can consume more trucks and more lamb.” Is this statement true or false? Briefly explain your answer. Answer: The statement is true. The increase in Bill’s income shifts his budget line outward and enables Bill to consume more trucks and more lamb chops. Topic: Consumption Possibilities, Change in Prices and Income Skill: Conceptual

9)

Explain how changes in the price of goods and the consumer’s budget affect the budget line. Answer: If the price of the good measured on the xaxis increases, the budget line rotates inward as the x-intercept decreases and the budget line becomes steeper. Conversely, if the price of the good measured on the x-axis decreases, the budget line rotates outward, the x-intercept increases, and the budget line becomes flatter. If the price of the good measured on the y-axis increases, the y-intercept decreases as the budget line rotates inward and becomes flatter. However, if the price of the good measured on the y-axis decreases, the budget line rotates outward as the yintercept increases, and the budget line becomes steeper. Finally, if the budget increases, the budget line shifts outward (both intercepts increase) and the slope remains unchanged while if the budget decreases, the budget line shifts inward (both intercepts decrease), and the slope remains unchanged.

CHAPTER 8

Topic: Preferences and Indifference Curves Skill: Recognition

10) What does the term “indifferent” mean and what is its relationship to an indifference curve? Answer: Indifferent means that the consumer does not care which outcome a person receives. An indifference curve measures the combinations of goods and services among which the consumer is indifferent as to which he or she consumes. Topic: Preferences and Indifference Curves Skill: Recognition

11) “If Ivan says he is indifferent between the consumption of a new pair of jeans or a set of earrings, he means that he does not want either product.” Is the previous analysis correct? Explain your answer. Answer: The analysis is incorrect. The term “indifferent” means that Ivan does not care if he gets another pair of jeans or another set of earrings. Topic: Indifference Curves Skill: Conceptual

12) Why do consumers prefer higher indifference curves (farther to the right) to lower indifference curves? Answer: When comparing two indifference curves, it is always possible to find consumption combinations on the higher indifference curve that have more of both goods than any particular point on the lower indifference curve. Consumers prefer consuming more goods rather than fewer, so they prefer the higher indifference curve because it offers more consumption of everything. Topic: Indifference Curves Skill: Conceptual

13) “Every point on a budget line has an indifference curve passing through it.” Explain whether the previous statement is correct or not. Answer: The statement is correct. The consumer has a preference map that consists of an infinite number of indifference curves, one through every possible consumption combination. So there is an indifference curve passing through every point on a budget line.

POSSIBILITIES, PREFERENCS, AND CHOICES

55

Topic: Marginal Rate of Substitution Skill: Recognition

Topic: Diminishing Marginal Rate of Substitution Skill: Conceptual

14) What is the meaning of the term “marginal rate of substitution”? Answer: The marginal rate of substitution between two goods is the rate at which a consumer is willing to give up some of one in exchange for more of the other, and remain indifferent between the old and the new combination. It is also the slope of an indifference curve between the two goods.

17) “The marginal rate of substitution of the good measured along the x-axis increases as a consumer moves downward along an indifference curve.” Is the previous statement correct or not? Answer: The statement is incorrect because the marginal rate of substitution decreases as a consumer moves downward along an indifference curve.

Topic: Marginal Rate of Substitution Skill: Recognition

15) What is the marginal rate of substitution and how does it relate to an indifference curve? Answer: The marginal rate of substitution is the rate at which a person will give up the good measured along the y-axis in order to get more of the good measured along the x-axis. The marginal rate of substitution equals the magnitude of the slope of the indifference curve. Because the indifference curve is not linear, the marginal rate of substitution changes as the consumer moves along the indifference curve. In particular, as more of the good measured along the x-axis is consumed, its marginal rate of substitution diminishes. In general, as more of a good is consumed, its marginal rate of substitution decreases. Topic: Marginal Rate of Substitution Skill: Analytical

16) Why does your marginal rate of substitution between chocolate and vanilla ice cream decline continuously as you move rightward on your indifference curve between the two? Answer: With chocolate on the vertical axis and vanilla on the horizontal, as you move rightward along your indifference curve, the more chocolate you give up and the less you have remaining. As a result, you grow to value your chocolate more and more and your vanilla less and less. You therefore become willing to give up less chocolate to get more vanilla.

Topic: Diminishing Marginal Rate of Substitution Skill: Conceptual

18) “As Jake consumes more dates over the course of a day, it is likely that his marginal rate of substitution of dates for other goods will rise.” Is the previous statement correct or incorrect? Answer: The statement is incorrect. The principle of diminishing marginal rate of substitution means that as Jake consumes more dates, he is willing to give up less other goods in order to get an additional date. Topic: Degree of Substitutability Skill: Conceptual

19) What will an indifference curve look like between two commodities that are perfect complements, such as right shoes and left shoes? Answer: The indifference curve will be L-shaped. This shape indicates that more of either product, without any more of the other, would not make the consumer better off. Topic: Degree of Substitutability Skill: Conceptual

20) Suppose that Dell computers and Gateway computers are very close substitutes. Then the indifference curves between Dell computers and Gateway computers are almost straight lines. Is this analysis correct or incorrect? Explain. Answer: The analysis is correct. When two goods are perfect substitutes, their indifference curves are straight lines. A Dell computer and a Gateway computer are very close substitutes, so their indifference curves are close to straight lines.

56

CHAPTER 8

Topic: Predicting Consumer Behavior, Best Affordable Point Skill: Conceptual

Topic: Predicting Consumer Behavior, Best Affordable Point Skill: Conceptual

21) In an indifference curve/budget line framework, how does a consumer decide which of all possible combinations of goods to purchase? Answer: The consumer will select the combination of goods and services that he or she most prefers and which he or she can afford to buy. Taking the latter consideration first, the consumer will buy a combination of goods and services that lie upon the budget line because all of these combinations are affordable. From the combinations that lie upon the budget line, the consumer will select the combination that is on the highest indifference curve because this is the combination that is most preferred.

24) Hermione and Ron are at a sweet shop in London. Hermione looks at the prices of ice cream and chocolate bars and says to Ron: “I can tell you what your marginal rate of substitution between ice cream and chocolate bars is at your best affordable point.” “No, you can’t,” says Ron, “You don’t know my preferences and how much money I have.” “I don’t need to know all this because I know the prices,” Hermione replies. Is she right? Explain. Answer: Yes, Hermione is correct. All information she needs is the relative price of ice cream in terms of chocolate bars (or vice versa), which determines the slope of Ron’s budget line. The marginal rate of substitution at Ron’s best affordable point is the slope of his indifference curve at this point. But because the best affordable point is the point where the budget line is tangent to the highest attainable indifference curve, the slope of the indifference curve at this point equals the slope of the budget line. So if Hermione knows the relative price, she knows the the slope of Ron’s budget line. Then she also knows the slope of his indifference curve at the best affordable point, which is the marginal rate of substitution at this point.

Topic: Predicting Consumer Behavior, Best Affordable Point Skill: Conceptual

22) Describe the consumer equilibrium in the indifference curve/budget line model. Answer: A consumer equilibrium occurs when the consumer is spending his or her entire income (is on the budget line), and is on the highest attainable indifference curve (is consuming the “best” point on the budget line). The equilibrium occurs at the point where the budget line just touches the highest indifference curve at one point. At this point, the budget line and indifference curve have the same slope, so the marginal rate of substitution (the slope of the indifference curve) is equal to the relative price (the slope of the budget line). Topic: Predicting Consumer Behavior, Best Affordable Point Skill: Conceptual

23) In a budget line/indifference curve figure, how do you identify the best affordable combination of any two goods? Answer: Any combination of goods along the budget line is affordable. The best of these combinations is on the highest indifference curve. This best combination is where the indifference curve is tangent to the budget line, so this combination is the best affordable combination.

Topic: Predicting Consumer Behavior, Demand Curve Skill: Analytical

25) Can the process of consumer choice as illustrated with a budget line/indifference curves approach explain the downward sloping demand curves that consumers have for goods, such as Pepsi? Answer: Yes. Measure the quantity of Pepsi consumed on the horizontal axis of the budget line/indiffer-ence curve diagram. Holding income and other prices constant, a series of decreases in the price of a Pepsi rotates the budget line outward. As the consumer then moves to successively higher indifference curves, the quantity of Pepsis consumed will increase successively, thereby demonstrating that lower prices increase the quantity demanded.

POSSIBILITIES, PREFERENCS, AND CHOICES

Topic: Predicting Consumer Behavior, Demand Curve Skill: Analytical

26) “Every point on a demand curve represents a consumer equilibrium in the indifference curve model.” Explain why the previous statement is correct or not. Answer: The statement is correct. The demand curve is derived using the indifference curve/budget line diagram. Hence every point on the demand curve is the result of a consumer equilibrium in the indifference curve/budget line diagram. Topic: Predicting Consumer Behavior, Substitution Effect Skill: Recognition

27) What is the substitution effect? Answer: The substitution effect is the effect of a change in price on the quantity bought when the consumer (hypothetically) remains indifferent between the original situation and the new one. Topic: Predicting Consumer Behavior, Substitution and Income Effect Skill: Recognition

28) What is the relationship between the income effect and the substitution effect for a normal good and what is it for an inferior good? Answer: For a normal good, the income effect reinforces the substitution effect. In other words, if the price falls so that a consumer’s real income increases, both the substitution effect and the income effect lead the consumer to increase his or consumption of the good. For an inferior good, the income effect reduces the substitution effect. In this case, when the price of a good falls, the substitution effect leads to an increase in the consumption of the good and the income effect leads to a decrease in the consumption of the good. Topic: Work-Leisure Choices Skill: Conceptual

29) What variables are related by an income-time budget line? What determines the slope of the line? Answer: An income-time budget line describes the relationship between leisure and labor income.

57

The slope of the line is determined by the wage rate. Topic: Work-Leisure Choices Skill: Analytical

30) What is the opportunity cost of leisure? Answer: The opportunity cost of leisure is the labor income forgone. Topic: Work-Leisure Choices Skill: Analytical

31) How does the substitution effect of a wage rate hike affect Katie’s labor supply decision? Answer: When the wage rate increases, Katie’s opportunity cost of leisure increases. Therefore, she takes less leisure or, in other words, she chooses to work more. Topic: Labor Supply, Income Effect Skill: Analytical

32) How does the income effect of a wage rate hike affect Katie’s labor supply decision? Answer: When the wage rate increases, Katie’s income increases allowing her to afford more normal goods. Leisure is a normal good, so she takes more leisure or, in other words, she chooses to work less. Topic: Labor Supply, Substitution and Income Effect Skill: Analytical

33) How do the substitution and income effects of a wage rate decrease affect Katie’s labor supply decision? Answer: When the wage rate falls, the opportunity cost of leisure falls, so Katie chooses more leisure and less work. This is the substitution effect. But the fall in the wage rate means that Katie’s income decreases, thereby allowing her to afford less leisure and other goods. Therefore, she takes less leisure or, in other words, she chooses to work more. So, the income and substitution effects work in the opposite direction.

CHAPTER 9

ORGANIZING PRODUCTION

Topic: Firm Skill: Recognition

Topic: Economic Profit and Normal Profit Skill: Conceptual

1) What is a firm? Answer: A firm is an institution that hires factors of production and organizes those factors to produce and sell goods and services.

5)

Topic: Explicit and Implicit Costs Skill: Conceptual

2)

What is the fundamental difference between explicit and implicit costs to the firm? Answer: Explicit costs are payments to others for resources that the firm does not already own. Examples are wages for workers and the electric bill to the local utility company. Implicit costs are imputed or implicit payments to the firm for resources it already owns. Examples would be the highest forgone salary of the firm’s owner and the forgone rent on a building that the firm owns and uses for its own business. Topic: Implicit Costs Skill: Conceptual

3)

Jake opens a pig farm in Idaho. To start his farm, he uses his entire $50,000 of savings from his savings account. The bank was paying him $2,500 interest on his saving. Explain why the $2,500 is one of Jake’s costs. Answer: When Jake provides the $50,000 to finance his farm’s operations, he forgoes the $2,500 in interest. Thus the opportunity cost of the $50,000 is the interest income forgone and so the $2,500 is an implicit cost of running the farm. Topic: Normal Profit Skill: Conceptual

4)

What is a normal profit? Is it part of the firm’s opportunity costs, total revenue, or neither? Answer: The normal profit is the return to entrepreneurship. It is part of the firm’s opportunity costs.

What is the difference between a normal profit and an economic profit? Answer: A normal profit is the return to entrepreneurship and is part of the opportunity cost of operating a business. An economic profit is the difference between the firm’s total revenue and its total opportunity cost. Because the normal profit is part of the firm’s opportunity cost, an economic profit is a profit above and beyond a normal profit. If the firm earns an economic profit, the entrepreneur running the firm receives the normal profit plus the economic profit. Topic: Economic Profit Skill: Conceptual

6) What is economic profit? Answer: A firm’s economic profit is its total revenue minus its total opportunity cost, which is the sum of explicit costs and implicit costs. Topic: Technological Efficiency and Economic Efficiency Skill: Conceptual

7)

Can a firm achieve technological efficiency without achieving economic efficiency? Answer: Yes. Technological efficiency means achieving the most output possible from any given combination of inputs. There might, however, be a different combination of inputs that can produce the same quantity of output at a lower cost. Economic efficiency is attained when the lowest cost combination of inputs is chosen, and when that combination of inputs is used to produce the largest possible quantity of output. Topic: Incentive System Skill: Recognition

8) What is an incentive system? Answer: An incentive system is a method of organizing production that uses a market-like mechanism inside the firm. Instead of issuing commands, sen-

60

CHAPTER 9

ior managers create compensation schemes that will induce workers to perform in ways that maximize the firm’s profit. Topic: Command Systems Skill: Recognition

9) What is a command system? Answer: A command system is a method of organizing production that uses a managerial hierarchy. Commands pass downward through the hierarchy, and information passes upward. Topic: The Principal-Agent Problem Skill: Conceptual

10) What is the principal-agent problem as applied to corporations? Answer: The principals are the owners of the corporation who want to see their profits maximized. The agents are the hired managers of the corporations who might serve their own goals instead. The principal-agent problem is to devise ways to make managers (the agents) act in the best interest of the owners (the principals). Topic: Coping With the Principal-Agent Problem Skill: Conceptual

11) What are three major ways that corporations can cope with the principal-agent problem? Answer: First, employees can be made part-owners of the corporation. Second, employees can be given incentive pay linked to achieving targets that serve the interests of the owners. Third, managers can be given long-term contracts that encourage them to pursue long-term strategies that can yield higher profit over time. Topic: Types of Business Organization Skill: Recognition

12) What is the main advantage and the main disadvantage of organizing a businesses as either a proprietorship or as a partnership rather than as a corporation? Answer: The main advantage of a proprietorship and partnership is that income is taxed only once by the personal income tax imposed on the owner’s income. The main disadvantage is that the owner(s) have unlimited liability.

Topic: Corporation Skill: Recognition

13) What are the main advantages and disadvantages of organizing a business as a corporation? Answer: The main advantages are limited liability to the stockholders and better access to lower cost capital by issuing stocks and bonds. The main disadvantage is that some of the corporate income is taxed twice. First, the corporation pays a corporate income tax. Second, the owners of the corporation pay a personal income tax on any capital gains they might earn. Topic: Corporation Skill: Recognition

14) “Since the tax on dividends has been eliminated, corporate profits are no longer taxed twice.” Do you agree? Explain. Answer: No. Stockholders still pay a capital gain tax on the profit they earn when they sell a stock for a higher price than they paid for it. Stocks generate capital gain when a corporation retains some of its profit and reinvests it in profitable activities. So retained earnings are taxed twice: first as corporate profits and second as stockholders’ capital gain they generate. Topic: Market Structure Skill: Recognition

15) Is the number of sellers in the market the only thing that is different in each of the four market types economists study? Answer: No, the number of sellers is not the only factor that differs; the degree of similarity of each firm’s product also plays a role. For instance, in a perfectly competitive market, each firm sells an identical product whereas in a monopolistically competitive market, each firm sells a slightly different product. Topic: Market Structure Skill: Recognition

16) What are the four types of markets? Give a brief description of each type. Answer: The four types of markets are perfect competition, monopoly, monopolistic competition, and oligopoly. Perfect competition has many firms selling identical products to many buyers, with no barriers to entry or exit.

ORGANIZING PRODUCTION

Monopoly has one firm selling a good with no close substitutes and a barrier that blocks the entry of new firms. Monopolistic competition has many firms making similar but not identical products with no barriers to entry or exit. Oligopoly has a small number of generally large firms producing either identical or differentiated products. Topic: Market Structure, Monopolistic Competition Skill: Conceptual

17) A student wrote: “Monopolistic competition is a market structure in which a small number of firms compete by making an identical product.” If you were the instructor, how would you correct this statement? Answer: The answer is wrong in several respects: Monopolistic competition is a market structure in which a small large number of firms compete by making an identical product similar but slightly different products. Topic: Market Structure, Monopoly Skill: Conceptual

18) Describe the conditions under which monopoly exists. Give two examples. Answer: Monopoly occurs when one firm produces a unique good or service for which there are no close substitutes. No competing firms can enter the market because there are barriers to entry. These barriers can be legal or natural. Your local water company and the U.S. Postal service are monopolies. Topic: The Herfindahl-Hirschman Index Skill: Recognition

19) What is the Herfindahl-Hirschman Index and what does it measure? Answer: The Herfindahl-Hirschman Index, or HHI, is an index used to measure the extent to which a market is dominated by a small number of firms. The HHI equals the sum of the squared percentage market shares of each of the 50 largest firms in the market. A monopoly will have a HHI of 10,000 whereas perfect competition will have a small HHI.

61

Topic: The Herfindahl-Hirschman Index Skill: Conceptual

20) If the Herfindahl-Hirschman Index for an industry is 8,528, is the industry competitive or concentrated? Answer: With a Herfindahl-Hirschman Index of 8,528, the industry is quite concentrated or, in other words, the industry is not very competitive. Topic: The Four-Firm Concentration Ratio and Herfindahl-Hirschman Index Skill: Conceptual

21) What is the difference between a four-firm concentration ratio and a Herfindahl-Hirschman Index? Answer: A four-firm concentration ratio sums the market shares of the four largest firms in an industry while the Herfindahl-Hirschman Index sums the squares of the market shares of the 50 largest firms. Topic: The Four-Firm Concentration Ratio and Herfindahl-Hirschman Index Skill: Conceptual

22) How would a merger between Coca-Cola and Pepsi Cola affect the four-firm concentration ratio for the soft drink market? How would it affect the Herfindahl-Hirschman Index for the soft drink market? Answer: Both Coca-Cola and Pepsi Cola are among the four largest firms in the soft drink market, so a merger between the two firms would (drastically) raise the four-firm concentration ratio and (drastically) raise the Herfindahl-Hirschman Index. Topic: Limitations of Concentration Measures; Geographic Scope Skill: Conceptual

23) “The concentration measures for the United States calculated by the Department of Commerce understate the degree of concentration in the newspaper industry.” Do you agree? Explain. Answer: Yes. The concentration measures calculated by the Department of Commerce take a national view of the market while the newspaper industry consists of mainly local markets. The calculated concentration measures for newspapers are low, but there is a high degree of concentration in the newspaper industry in most cities.

62

Topic: Why Firms?; Economies of Scope Skill: Conceptual

24) What would be an example of economies of scope achieved in the automobile insurance industry? Answer: The auto insurance industry can achieve economies of scope, for example, by also offering to insure homes, boats, and people’s lives. There could be cost savings, compared to selling each kind of insurance by different specialized companies, because one sales force can sell all kinds of policies. Also, there would not have to be multiple buildings for the headquarters and branch offices, computer networks, or cars for the sales agents.

CHAPTER 9

CHAPTER 10

OUTPUT AND COSTS

Topic: Short Run Versus Long Run Skill: Conceptual

Topic: Total Product Curve Skill: Conceptual

1)

4)

What is the difference between the short run and the long run? Answer: The short run is a time period so short that the firm cannot change the quantity of at least one resource, say plant size for a manufacturer or land for a farmer. The only way to increase output, therefore, is by combining more units of the variable resources with the fixed resource. The long run is a time period sufficiently long that a firm can change the quantity of all resources. Nothing is fixed so output can be produced with whatever combination of resources the firm chooses. Topic: Short Run Versus Long Run Skill: Conceptual

2)

What resources can a firm change in the short run? In the long run? Answer: In the short run, the firm can change its variable inputs. These would be inputs such as labor. The firm cannot change its fixed inputs, such as its capital stock. The inputs the firm cannot change are called its plant. In the long run, the firm can change all of its inputs. Indeed, the long run is defined as the period of time long enough so that the firm can change all of its inputs. Topic: Short Run Skill: Conceptual

3)

What do economists mean when they say that a firm’s plant is fixed? Answer: The phrase that a “firm’s plant is fixed” refers to the fact that, in the short run, there are some resources that are fixed, that is, the quantity employed cannot be changed. These resources, which include factors such as the firm’s capital, are collectively referred to as the firm’s plant.

What does a firm’s short-run total product curve show and what is its significance? Answer: A short-run total product curve shows the way the maximum output increases when the input of a variable factor (such as labor) increases, holding all other inputs fixed. Its significance is that it shows the productivity of the variable input as output is increased. The latter, in turn, is an important determinant of a firm’s short run costs. Topic: Law of Diminishing Returns Skill: Recognition

5)

What is the law of diminishing returns? Why is this proposition called a “law”? Answer: The law of diminishing returns states that as a firm uses more of a variable input, with a given quantity of fixed inputs, the marginal product of the variable input eventually decreases. This proposition is called a “law” because there is no case where adding more labor, capital, materials, fuel or any other variable input will cause output to increase indefinitely. Eventually, adding any variable input to some fixed input will lead to smaller and smaller additions to output. If the law of diminishing returns did not always hold true, farmers could grow the entire world’s supply of food on one acre of land, by just adding more water and more fertilizer (variable inputs) to the acre (the fixed input). Topic: Law of Diminishing Returns Skill: Conceptual

6)

“The law of diminishing returns applies is the same as the decreasing returns to scale.” Do you agree? Explain. Answer: The statement is incorrect. The law of diminishing returns states that as a firm uses more of a variable input, with a given quantity of fixed inputs, the marginal product of the variable input eventually diminishes. In the long run all inputs

64

CHAPTER 10

are variable, and decreasing returns to scale occurs when an increase in all inputs by the same percentage results in a smaller percentage increase in output. Topic: Average Product of Labor Skill: Recognition

7) What does the average product of labor equal? Answer: The average product of labor equals the total product (the total output) divided by the quantity of labor. Topic: Relationship Between the Marginal Product and the Average Product of Labor Skill: Conceptual

8)

“If the marginal product of labor curve slopes downward, then the average product of labor curve necessarily must slope downward.” Explain whether the previous statement is correct or incorrect? Answer: The statement is incorrect. The relationship between the marginal product and average product of labor is that when the marginal product of labor exceeds the average product of labor, the average product of labor curve slopes upward and when the marginal product of labor is less than the average product of labor, the average product of labor curve slopes downward. So, even if the marginal product of labor curve slopes downward, as long as it is above the average product of labor curve, the average product of labor curve slopes upward. Topic: Relationship Between the Marginal Product and the Average Product of Labor Skill: Conceptual

9)

Describe the relationship between the marginal and average products of labor as the employment of labor increases in the short run. Answer: Holding other inputs such as capital constant, additional workers have increasing and then diminishing marginal returns. So, the marginal product of additional workers rises and then falls, even though remaining positive. As that happens, when the marginal product is greater than the average product, the average product is increasing. When the marginal product is less than the average product, the average product is decreasing. The average product equals the marginal product when the average product is at its maximum.

Topic: Relationship Between the Marginal Product and the Average Product of Labor Skill: Conceptual

10) A student wrote: “When the average product of labor exceeds the marginal product, the marginal product is increasing.” If you were the instructor, how would you correct this statement? Answer: There are several errors. The corrected sentence is “When the average marginal product of labor exceeds the marginal average product, the marginal average product is increasing.” Topic: Total Cost Skill: Conceptual

11) What are the two components of a firm’s cost in the short run, and what are their definitions? Answer: The two parts are the total fixed cost (TFC) and the total variable cost (TVC). TFC is the cost that does not vary as output varies. Examples include rent on a building or interest on a business loan. TVC is the cost that varies as output varies. Examples include a firm’s payroll for labor or payments for raw materials. Topic: Fixed Cost and Variable Cost Skill: Conceptual

12) Jake is a corn farmer in Nebraska. He rents his land on a long-term lease for $250,000 a year. He pays his farm hands $28,000 a year. Is his rent a fixed cost or a variable cost? Are the wages he pays his workers a fixed cost or a variable cost? Briefly explain your answers. Answer: Jake’s rent is a fixed cost. He is renting his land on a long-term lease and so the land is a fixed input. The wages Jake pays are a variable cost. Labor is a variable input so the wages are a variable cost. Topic: Fixed Cost and Variable Cost Skill: Conceptual

13) “In the short run, even when output is zero, the firm still has some variable costs it must pay.” Is the statement correct or incorrect? Briefly explain your answer. Answer: The statement is incorrect. When output is zero, there are no variable costs because there are no variable inputs being employed. However, in the short run there might be fixed resources, in which case the firm would still have some fixed costs it must pay.

OUTPUT AND COSTS

Topic: Fixed Cost Curve Skill: Conceptual

14) When plotted against the total output, what does the total fixed cost curve look like? Answer: The total fixed cost curve is a horizontal straight line, indicating that at any level of output the total fixed cost does not change as output changes. Topic: Average Total Cost Skill: Conceptual

15) Explain why average total costs initially decrease and then increase as output increases. Answer: Initially, as output increases, both average fixed cost and average variable cost decrease. So, average total cost decreases at first. But as output increases further, diminishing returns set in and average variable cost begins to increase. Eventually, average variable cost increases more quickly than average fixed cost decreases, so average total cost increases. Topic: Average Total Cost Curve Skill: Analytical

16) Downsizing is the practice of laying off workers in an attempt to decrease average total cost. Can laying off workers decrease average total cost? Is it possible for the firm to downsize and have its average total cost increase? Explain your answer. Answer: Yes, downsizing by laying off workers can decrease the firm’s average total cost. And it is possible for a firm to downsize and increase its average total cost. Suppose that the firm initially is producing more output than the level that minimizes average total cost. In this case, downsizing by laying off workers decreases the amount of output the firm produces and the firm moves along its average total cost curve to a lower average total cost. However, the firm can lay off too many workers. If it lays off too many workers, it decreases its output so much that it moves along its average total cost curve enough that its average total costs wind up higher than they were initially. Topic: Average Fixed Cost Skill: Recognition

17) How do we calculate average fixed cost and why does average fixed cost fall as output increases? Answer: Average fixed cost is calculated by dividing total fixed cost by the quantity of output. Because total fixed cost stays the same for all levels of out-

65

put, as the quantity produced increases, the average fixed cost decreases. Topic: Marginal Cost Skill: Recognition

18) “Marginal cost is the increase in total cost that results from a one-unit increase in a variable input.” True or false? Explain. Answer: False. Marginal cost is the increase in total cost that results from a one-unit increase in output, not a one unit increase in an input. Topic: Marginal Cost Skill: Recognition

19) “Marginal cost eventually increases because of the law of diminishing returns.” True or false? Explain. Answer: The statement is true. The law of diminishing returns means that each additional worker produces a successively smaller addition to output. So to get an additional unit of output, ever more workers are required. Because more workers are required to produce one additional unit of output, the cost of the additional unit of output—marginal cost—must increase. Topic: Relationship Between Marginal Cost and Average Total Cost Skill: Conceptual

20) The average total cost curve is U-shaped. At the quantity of output where average total cost is at its minimum, is the marginal cost curve above the average total cost curve, below the average total cost curve, or intersecting the average total cost curve? Answer: When the average total cost is at its minimum, the average total cost equals the marginal cost. Therefore at the quantity of output where the average total cost is at its minimum, the marginal cost curve intersects the average total cost curve. Topic: Relationship Between Marginal Cost and Average Total Cost Skill: Conceptual

21) What is the difference between average total cost and marginal cost and are they ever equal to each other? Answer: Average total cost is total cost divided by output. Marginal cost is the change in total cost divided by the change in output. Marginal cost

66

CHAPTER 10

equals average total cost when the average total cost is at its minimum. Topic: Why the Average Total Cost Curve is UShaped Skill: Recognition

22) Which average cost curves are U-shaped? Answer: Both the average total cost and the average variable cost curves are U-shaped. The only average cost curve that is not U-shaped is the average fixed cost curve. Topic: Cost Curves and Product Curves Skill: Conceptual

23) What is the relationship between the marginal product of labor and the marginal cost? Answer: Initially, as each additional unit of labor is employed, the marginal product of labor increases. The cost of each worker is the same--the wage rate. Because each worker’s cost is the same, but each additional worker produces more output than the previous worker, the cost of producing an additional unit of output decreases. The cost of producing an additional unit of output is the marginal cost. Therefore when the marginal product of labor is increasing, the marginal cost is decreasing. At some point, the marginal product of labor will reach a peak, at which level of output marginal cost is minimized. After that point, the law of diminishing returns means that the marginal product decreases. When the marginal product decreases, the marginal cost increases; each worker costs the same to hire, but an additional worker produces less output than the previous worker. Therefore when the marginal product of labor is decreasing, the marginal cost is increasing. Topic: Change in Technology Skill: Conceptual

24) Explain how new technologies, which increase productivity, affect the average variable cost, average total cost, and marginal cost curves. Answer: A technological change that increases productivity shifts the total product, average product, and the marginal product curves upward. Because the new technologies enable existing inputs to produce more output, this effect means that the average variable cost, average total cost, and marginal cost curves shift downward, reflecting the decrease in average and marginal costs. However, the new technologies often need to use more capi-

tal and less labor. In this case, the firm’s fixed cost increases and its variable cost decreases. The increase in fixed cost leads to an increase in total cost while the decrease in variable cost leads to a decrease in total cost. Although the net effect is ambiguous, generally the total cost increases at low levels of output and decreases at higher levels of output. In this case, the average total cost curve shifts upward at lower levels of output and shifts downward at higher levels of output. Topic: Change in the Price of a Resource Skill: Conceptual

25) When the Rent-A-Limo Company negotiates its new labor contract it finds that the wages it must pay drivers have increased. How does this wage hike affect the Rent-A-Limo Company’s average fixed cost, average variable cost, average total cost, and marginal cost? Answer: The drivers’ wages are a variable cost because the number of drivers needed varies with how many limousines are rented. The increase in wages will have no effect on the average fixed cost but the average variable cost will increase. The increase in average variable cost leads to an increase in the average total cost and an increase in the marginal cost. Topic: Relationship Between Cost Curves and Product Curves Skill: Conceptual

26) How do the marginal and average products of labor affect a firm’s marginal and average variable costs in the short run? Answer: Over the range of output for which the marginal product is increasing, marginal cost is decreasing and vice versa. At the level of output at which the marginal product is at its maximum, marginal cost will be at its minimum. And, over the range of output for which the average product is increasing, average variable cost is decreasing and vice versa. Finally, the level of output at which the average product is at its maximum is the same level at which average variable cost is at its minimum. Topic: Long-Run Average Cost Curve Skill: Recognition

27) In the long run all costs are variable costs. Why? Answer: The long run is defined as the time frame over which all resources can be varied. The costs

OUTPUT AND COSTS

of the variable inputs are the variable costs. Taken together, these definitions imply that in the long run all inputs are variable inputs so all costs are variable costs. Topic: Long-Run Average Cost Curve Skill: Recognition

28) Which curve shows the lowest average total cost at which it is possible to produce each output when the firm has time to change both its labor force and plant size? Answer: The long-run average cost curve shows the lowest average total cost at which it is possible to produce each output when the firm has had time to change all its inputs. Topic: Long-Run Average Cost Curve Skill: Recognition

29) What is the long-run average cost curve? What are the three ranges of output and in what order do they occur? Briefly define each of the three ranges. Answer: The long-run average cost curve shows the lowest average cost at which it is possible to produce each output when the firm has time to change both its labor force and its plant size. The long-run average cost curve is U-shaped and shows three possible production ranges. In order, these ranges are: economies of scale, constant returns to scale, and diseconomies of scale. Economies of scale are when a firm increases its plant size and labor employed by the same percentage and its output increases by a larger percentage so that its average total cost decreases. Constant returns to scale are when a firm increases its plant size and labor employed by the same percentage and its output increases by the same percentage so that its average total cost does not change. And, diseconomies of scale are when a firm increases its plant size and labor employed by the same percentage and its output increases by a smaller percentage so that its average total cost increases. Topic: Economies of Scale Skill: Recognition

30) What are economies of scale? What is the main source of economies of scale? Answer: Economies of scale are when a firm increases its plant size and labor employed by the same percentage and its output increases by a larger percentage so that its average total cost decreases. Economies of scale result because of specializa-

67

tion. When a firm increases its capital stock, the capital can be more specialized and hence produce more output than less specialized, more generalized capital. Similarly, when a firm increases its labor force, it can hire more specialists, each of whom is better at doing his or her assigned task than a less specialized individual. As a result of specialization, the firm’s average product increases so that its average total cost decreases. Topic: Increasing Marginal Returns and Economies of Scale Skill: Conceptual

31) Explain the difference between increasing marginal returns and economies of scale. Answer: Increasing marginal returns apply to the case where a firm has increased only the amount of labor it uses while keeping capital fixed. If increasing marginal returns occur, an additional worker is more productive than the previous worker hired. In this case, the marginal product of the new worker exceeds the marginal product of the previous worker. Economies of scale apply when a firm increases both the amount of labor it uses and the amount of capital it uses. If there are economies of scale, when a firm increases labor and capital by the same percentage, its output increases by a larger percentage. Topic: Diseconomies of Scale Skill: Recognition

32) What are diseconomies of scale and why might they occur? Answer: Diseconomies of scale mean that as the firm increases all its inputs by the same percentage, its output increases by a smaller percentage. As a result, the firms’ long-run average cost rises. Diseconomies of scale can arise from the sheer size of a firm. As a firm grows larger, it becomes increasingly more difficult to manage. Communicating information up and down the management hierarchy as well as communicating between managers and workers on the same level becomes more costly. As a result, the firm experiences diseconomies of scale, that is, its long-run average cost increases as it expands the scale of its operation by producing yet more output.

68

Topic: Constant Returns to Scale Skill: Conceptual

33) A firm increases both its plant and its labor force by the same percentage and its average total costs remain unchanged. Is the firm experiencing increasing returns to scale, constant returns to scale, or decreasing returns to scale? Answer: The firm is experiencing constant returns to scale. Topic: Short-Run and Long-Run Average Cost Curves Skill: Conceptual

34) Are the short-run average total cost curve and the long-run average cost both U-shaped for the same reasons? If so, carefully explain these reasons. If not, explain why each curve is U-shaped. Answer: The curves are U-shaped for quite different reasons. In the short run, the firm’s plant, that is, its capital, is fixed. Therefore the only way to increase output is by increasing the quantity of the variable resource, labor. Initially as labor increases, there are gains from specialization and division of labor, which leads the marginal product to rise and the average total cost to fall as more workers are employed. Eventually, however, the firm runs up against its capital constraint and workers must share tools and building space. As this happens, decreasing marginal returns begin and average total cost eventually begins to rise.

CHAPTER 10

In the long run, both labor and capital are variable. As a firm expands its use of both resources, gains from specialization of both labor and capital cause average total costs to fall. Eventually, however, the business becomes so large it is difficult to coordinate and control. When this happens, average total cost begins to rise. So, the short-run average total cost is U-shaped because more workers must make do with the same amount of plant. The long-run average cost curve is U-shaped because the very scale of the firm’s operations make it difficult to control efficiently and effectively. Topic: Diseconomies of Scale Versus Diminishing Marginal Returns Skill: Conceptual

35) What is the difference between diminishing marginal returns and diseconomies of scale? Answer: Diminishing marginal returns is a short run concept. It occurs when increases in output become smaller and smaller as more units of a variable resource are combined with some fixed resource. Diseconomies of scale is a long run concept. It occurs when, as output increases, longrun average cost increases.

CHAPTER 11

PERFECT COMPETITION the producer can sell all he or she produces at the going market price. In this case, a producer will not lower the price he or she charges because no additional sales can be garnered. Hence it is nonsensical to undercut the market price because the lower price means lower revenue and hence lower profit.

Topic: Perfect Competition Skill: Recognition

1)

What are the requirements for perfect competition? Answer: The requirements are: many firms selling an identical product; many buyers; no restrictions on entry into the industry; established firms have no advantages over new entrants; and sellers and buyers have good information about prices of each firm’s product. Topic: Price Takers Skill: Recognition

2)

“A perfectly competitive firm is called a price maker because all the firms together must make the market price.” Is the previous statement correct or incorrect? Briefly explain your answer. Answer: The statement is false. A perfectly competitive firm is called a “price taker” because the firm must take whatever price the market determines. Any single firm’s actions cannot affect the market price. Topic: Price Takers Skill: Conceptual

3)

“Perfectly competitive firms have total control over the price they set for their product.” Explain why the previous statement is correct or incorrect. Answer: The statement is incorrect. Perfectly competitive firms are price takers, which means that they have no control over the price of their product. They must “take” the price given to them by the market as a whole, that is, they must take the price determined by the market demand and market supply. Topic: Price Takers Skill: Conceptual

4)

Does a perfectly competitive producer have any incentive to undercut the current market price? Explain your answer Answer: A perfectly competitive producer has no incentive to undercut the market price because

Topic: Price Takers Skill: Conceptual

5)

Why are perfectly competitive ranchers in Montana price takers? Answer: Because one farmer’s beef is identical to another farmer’s, each farmer’s beef is a perfect substitute for all other farmers’ beef. In addition, there are over one million ranchers in the United States. As a result, no individual rancher can impact the market price by increasing or decreasing production. Therefore each rancher faces a perfectly elastic demand. Each can sell all of the beef desired at the market price, but not one penny more. Once the market sets the price, the rancher must take as given whatever the price might be. Topic: Price Takers Skill: Conceptual

6)

If a perfectly competitive firm manufacturing chairs produces 100 more chairs, what happens to the market price of a chair? Answer: The price will not change. Any one perfectly competitive firm is such a small part of the market that a change in its output has virtually no effect on the price. This result is why the firm’s marginal revenue equals its price: No matter how much (or how little) the firm produces, the marginal revenue from one more unit always equals the price of the product.

70

CHAPTER 11

Topic: Market Demand/Firm Demand Skill: Conceptual

Topic: Market Demand/Firm Demand Skill: Conceptual

7)

10) Do firms in perfect competition advertise their products? Why or why not? Answer: Firms in perfect competition do not advertise their product. The purpose of advertising is to increase the demand for the firm’s product by convincing consumers that it’s better than that of competitors. Perfectly competitive firms sell a standard product, so consumers won’t believe that, for example, wheat sold by Farm A is somehow better than wheat sold by Farm B. So if a perfectly competitive firm advertised, this would only increase its costs, with no effect on demand and price.

Why is the demand for the perfect competitor’s good perfectly elastic even though the market demand is not? Answer: Each firm takes the market price as given. Because each firm’s good can be perfectly substituted with any other firm’s product, consumers will only pay the market price. Hence, any deviation above the market price causes the firm’s sales to plunge to zero and any deviation below causes the firm’s sales to soar to the entire amount sold in the market. Topic: Market Demand/Firm Demand Skill: Conceptual

8)

Hubert’s Copy Services is in perfect competition. Hubert currently charges 10 cents per page, which is the going market price. He thinks that he can increase his profit by raising the price. Is it possible? Why or why not? Answer: If Hubert raises his price, his profit will not increase. As a perfectly competitive firm, Hubert’s is a price taker. It produces a tiny proportion of the copy services in the area, and buyers are well informed about the prices charged by other firms. So, if the market price is 10 cents per page and Hubert asks, say, 12 cents per page, his customers will go to the next copy service and Hubert will lose all his sales. Topic: Market Demand/Firm Demand Skill: Conceptual

9)

Jane’s Copy Services is in perfect competition. Jane currently charges 10 cents per page, which is the going market price. Jane thinks that she can increase her profit if she lowers her price to 8 cents per page to increase the demand for her service. Is Jane right? Why or why not? Answer: If Jane lowers her price, her profit will not increase. As a perfectly competitive firm, Jane can sell as much of her service as she wants at the going market price. And to maximize her profit, she should choose the quantity at which her marginal cost equals the market price. If Jane lowers the price, her total revenue and hence profit will fall.

Topic: Marginal Revenue Skill: Recognition

11) What is marginal revenue? Answer: Marginal revenue is the change in total revenue that results from a one-unit increase in quantity sold. Topic: Normal Profit Skill: Recognition

12) What is a normal profit? Answer: A normal profit is the return a firm’s owner could obtain in the best alternative business. As a result, it is an opportunity cost to the firm. Topic: Profit-Maximizing Output Skill: Analysis

13) Why does the profit-maximizing level of production occur at the point where marginal revenue equals marginal cost? Answer: If a firm produces at a level where marginal revenue is greater than marginal cost, more profit could be gained by increasing output. Why? Because the added revenue (the marginal revenue) from producing another unit exceeds the added cost (the marginal cost) of producing the unit. Therefore the firm should increase production until no more additional profit can be earned, which occurs where marginal revenue equals marginal cost. Similarly, if a firm produces at a level where marginal cost exceeds marginal revenue, the firm’s profit would rise by decreasing output. In this case, the saved costs (the marginal cost) exceed the lost revenue (the marginal revenue). Therefore the

PERFECT COMPETITION

firm should decrease production until the point at which marginal cost equals marginal revenue. Topic: Marginal Analysis Skill: Analysis

14) Martha’s Cleaning Services is a perfectly competitive firm that currently cleans 30 offices a week and charges $20 per office, which is the going market price. Martha’s marginal cost is $15. What should Martha do to increase her economic profit? Clean more offices? Raise her price? Explain your answer. Answer: Martha should increase the number of offices she cleans until her marginal cost equals the market price. This way she will maximize her profit. As a perfectly competitive firm, Martha is a price taker and cannot raise or lower her price without losing profit. Topic: Marginal Analysis Skill: Analysis

15) Ellen’s Painting Services is a perfectly competitive firm that currently paints 10 houses a month and charges $100 per house, which is the going market price. Ellen’s marginal cost is positively related with the quantity of service she provides and is currently $120. What should Ellen do to increase her economic profit? Paint more houses? Raise her price? Explain your answer. Answer: Ellen should decrease the number of houses she paints until her marginal cost equals the market price. This way she will maximize her profit. As a perfectly competitive firm, Ellen is a price taker and cannot raise or lower her price without losing profit.

71

Topic: Shutdown Point Skill: Conceptual

17) If the price received by a perfectly competitive firm is less than its average variable cost, what will the firm do in the short run? Why? Answer: If the price is less than the average variable cost, the firm will shut down in the short run. By shutting down, the firm will incur an economic loss equal to its fixed cost. Whereas if the firm operated, its economic loss would be larger. Therefore the firm minimizes its loss by shutting down. Topic: Shutdown Point Skill: Conceptual

18) Define the shutdown point. Explain why the firm shuts down in the short run if the price falls below this point. Answer: The shutdown point is the point at which the price equals minimum average variable cost. If the price falls further, the firm does not even cover its variable costs if it operates. Its loss if it operated thus exceeds the loss of shutting down and so the firm shuts down. Topic: Shutdown Point Skill: Conceptual

19) Will a perfectly competitive firm ever produce in the short run even though it is suffering an economic loss? Answer: Yes, a perfectly competitive firm will continue to produce even though it is suffering an economic loss if the price exceeds the minimum average variable cost. In this case, even though the firm has an economic loss, if it shut down, its economic loss would be larger.

Topic: Shutdown Point Skill: Recognition

Topic: Shutdown Point Skill: Conceptual

16) “A perfectly competitive firm will shut down if the price falls below its average total cost.” Do you agree? Explain. Answer: A perfectly competitive firm will not shut down as far as the price is above its average variable cost. If the price is below the ATC but above the AVC, the firm can cover part of its fixed cost if it continues to operate. If the firm shuts down, it incurs an economic loss equal to total fixed cost. So as long as the price is above the AVC, the firm will have a smaller economic loss if it continues to operate than if it shuts down.

20) What must be the case if a perfectly competitive firm’s economic loss is less by shutting down rather than by producing and selling some output? Answer: If a firm’s economic loss is greater when it produces and sells some output than when it shuts down, it is the case that the price of the product is less than the average variable cost. When the price is less than the average variable cost, the firm’s economic loss is less if it shuts down than if it produces and sells output.

72

Topic: Economic Profits and Economic Losses in the Short Run Skill: Conceptual

21) Can a perfectly competitive firm earn an economic profit in the short run? Can it incur an economic loss? Answer: In the short run, a perfectly competitive firm can earn an economic profit or incur an economic loss. Indeed, the firm also can earn a normal profit in the short run. Basically, any profit or loss outcome is possible in the short run. Topic: Economic Profits and Economic Losses in the Short Run Skill: Conceptual

22) If the market price is less than a perfectly competitive firm’s average total cost, what sort of profit or loss is the firm earning? Answer: If the price is less than the average total cost, the firm is incurring an economic loss. Topic: Economic Profits and Economic Losses in the Short Run Skill: Analytical

23) What is the relationship between the price, P, and the average total cost, ATC, for a firm in perfect competition that earns an economic profit? That earns a normal profit? That incurs an economic loss? Answer: If the price is greater than the average total cost, P > ATC, the firm earns an economic profit. If the price equals the average total cost, P = ATC, the firm earns a normal profit. If the price is less than the average total cost, P < ATC, the firm incurs an economic loss. Topic: The Firm’s Short-Run Supply Curve Skill: Recognition

24) What is a perfectly competitive firm’s short-run supply curve? Answer: A perfectly competitive firm’s short-run supply curve is its marginal cost curve above the minimum average variable cost. Topic: The Firm’s Short-Run Supply Curve Skill: Analytical

25) If the market price faced by a perfectly competitive firm increases, in the short run how does the firm respond? Answer: If the market price rises, a perfectly competitive firm increases its output. The firm moves

CHAPTER 11

upward along its marginal cost curve, thereby increasing the quantity the firm will supply. Topic: Long-Run Adjustments Skill: Conceptual

26) Explain the process that drives the economic profit to zero in the long run for a perfectly competitive firm. Answer: In the long run, new firms will enter a perfectly competitive market if they can earn an economic profit. The increased supply causes the price to fall. As long as an economic profit exists, new firms continue to enter, and the price continues to fall until eventually the economic profit equals zero. In the long run, firms leave a perfectly competitive market if they are suffering an economic loss. By exiting, the price rises and the economic loss of the surviving firms shrinks. Eventually enough firms exit so that the price rises to the point that the survivors no longer incur an economic loss, earning instead a normal profit. Topic: Economic Profits and Economic Losses in the Short Run and the Long Run Skill: Conceptual

27) Describe the different possible profit outcomes for a perfectly competitive firm in the short run versus the long run. Explain why they occur. Answer: In the short run, a perfectly competitive firm can earn an economic profit, normal profit or economic loss. A firm earns an economic profit when P > ATC. It earns a normal profit (zero economic profit) when P = ATC. And earns an economic loss when P < ATC. If firms are earning an economic profit, new firms will enter and compete away the existing firms’ economic profit until all firms earn a normal profit. At this point, no new firms will enter the industry and a long-run equilibrium occurs. If firms are already earning a normal profit, no new firms will enter the market, and this condition continues into the long run. And, if some firms are earning an economic loss, some will exit the industry. This exit decreases the supply and drives up the price, thereby allowing the remaining firms to earn a normal profit. So, in the long run, a perfectly competitive firm will only earn a normal profit.

PERFECT COMPETITION

73

Topic: Long-Run Adjustments; Entry Skill: Conceptual

Topic: Long-Run Adjustments; Entry Skill: Conceptual

28) When will new firms enter a perfectly competitive market? When does entry stop? Answer: New firms will enter a perfectly competitive market as long as the existing firms are earning an economic profit. Essentially the new firms enter in order to earn an economic profit themselves. Entry will stop when it is no longer possible to earn an economic profit, which occurs when the existing firms are earning a normal profit.

32) The U-pick berry market is perfectly competitive. Suppose that all U-pick blueberry farms have the same cost curves and all are earning an economic profit. What happens as time passes? What is the long-run equilibrium outcome? Answer: The presence of economic profit attracts new firms into the U-pick blueberry market. As the new firms, that is, new farmers enter the market, the supply of U-pick blueberries increases. The increase in the supply drives the price lower and decreases the economic profits of the existing farmers. New farmers continue to enter the market as long as there is the possibility of an economic profit. Eventually enough new firms enter so that the price is driven so low that the economic profit is eliminated. All the firms earn a normal profit, which will keep them in business but will provide no incentive for new firms to enter the market. At this point, the long-run equilibrium has been reached.

Topic: Long-Run Adjustments; Entry Skill: Conceptual

29) What role does economic profit play in a competitive market? Answer: Economic profit acts as a signal to entrepreneurs telling them where above normal returns are being earned. Economic profit serves society by directing resources to those activities for which society is currently paying a premium. Topic: Long-Run Adjustments; Entry Skill: Conceptual

30) “For a perfectly competitive market, an economic profit attracts new firms. But when these firms enter the market, the price falls and the economic profit is eliminated.” Are the previous statements correct or incorrect? What is the long-run profit or loss outcome for firms in a perfectly competitive market? Answer: The statements are correct. As the statements point out, in the long run the economic profit of perfectly competitive firms is eliminated by entry. Similarly, an economic loss will be eliminated by exit. Therefore the long-run equilibrium profit for a perfectly competitive firm is a normal profit. Topic: Long-Run Adjustments; Entry Skill: Conceptual

31) In the long run, perfectly competitive firms cannot earn an economic profit. Why? Answer: An economic profit attracts entry by new firms. As new firms enter the market, the market supply increases and the market supply curve shifts rightward. The increase in supply decreases the price. And, as the price falls, the economic profit is eliminated.

Topic: Long-Run Adjustments; Entry Skill: Analytical

33) During the middle of the 1990s, the price of pork rose. After a couple of years the price decreased back to about the level before the initial increase. What might have led to these events? Answer: There are thousands upon thousands of hog farmers in the United States. In the middle of the 1990s, a general decrease in the demand for beef led to a large increase in the demand for pork. As a result of the increase in market demand, the price of pork increased dramatically. Hog farmers were getting a high price and earning large economic profits. In the long run, the word got out that economic profit was possible in this arena. Over time, more hog farmers entered the market, which led to a large increase in the supply of pork. As supply increased, the price of pork dropped. Thus the higher price was the short-run result of an increase in demand. The falling price reflected the adjustment to the long-run equilibrium, as new hog farmers entered the market. The long run was ultimately reached and the price of pork was more or less the same as before the increase in demand.

74

CHAPTER 11

Topic: Long-Run Adjustments; Exit Skill: Conceptual

Topic: Long-Run Adjustments; Exit Skill: Conceptual

34) Describe how economic losses are eliminated in a perfectly competitive industry. Answer: If firms are incurring economic losses, some will exit in the long run. When firms exit, the market supply decreases and the market supply curve shifts leftward. When supply decreases, the price rises. As the price rises, the surviving firms increase production and their economic losses are eliminated.

37) How does a decrease in the demand for wheat ultimately lead to normal profits for wheat growers in the long run? Answer: If the demand for wheat decreases, the price of wheat falls and many wheat farmers incur economic losses. These losses lead to some farmers shutting down their operations. As these farmers exit the market, the supply of wheat decreases. A decrease in the supply of wheat pushes wheat prices back up. The process of exit and rising prices continues until finally the price of wheat rises enough so that the surviving wheat farmers are earning a normal profit. At this time, which occurs in the long run, the economic losses have disappeared so that no further exit occurs. The wheat market is back in its long-run equilibrium.

Topic: Long-Run Adjustments; Exit Skill: Conceptual

35) Pumpkin growing is a perfectly competitive industry. Suppose that pumpkin growers are all generally suffering an economic loss. What happens as time passes? What is the long-run equilibrium outcome? Answer: Because the firms are generally experiencing an economic loss, as time passes some firms will exit the market, perhaps by switching to other crops, or perhaps by closing entirely. As farmers leave the market, the supply of pumpkins decreases. The supply curve shifts leftward and the price of a pumpkin rises. As the price rises, the economic losses of the remaining firms decrease. Eventually enough farmers leave the market so that the price rises sufficiently so that the remaining firms earn a normal profit and no longer suffer an economic loss. At that point, the long-run equilibrium is reached because there is no further incentive for any firms to leave the market. Topic: Long-Run Adjustments; Exit Skill: Conceptual

36) Suppose a farmer raising beef is earning a normal profit. Then, because of a scare about mad cow disease, the demand for beef decreases drastically. What happens to the profits of the beef farmer in the short run and in the long run? Answer: In the short run, the fall in beef prices will decrease the farmer’s profits. With the fall in price, the farmer will incur an economic loss. If the price is high enough to cover the farmer’s variable costs, the farmer will continue to operate. In the long run, if demand continues to remain depressed, some farmers will exit the market until the remaining farmers earn a normal profit once again.

Topic: Long-Run Adjustments; Exit Skill: Conceptual

38) Entry by competitive firms decreases the market price, while exit by competitive firms increases the market price. Explain why firms enter or exit an industry and why these price changes occur. Answer: Competitive firms will enter an industry where economic profits exist in an attempt to earn an economic profit. As new firms enter, the supply increases and the supply curve for the product shifts rightward. The increase in supply drives the price lower. Firms exit an industry when economic losses are incurred. As they leave, supply decreases and the supply curve shifts leftward. The decrease in the supply forces the price higher. Entry and exit continue until the remaining firms in the industry are earning a normal profit. Topic: Long-Run Equilibrium Skill: Conceptual

39) In the long run, a perfectly competitive firm earns zero economic profit. What incentive does the firm have to stay in business if it is making zero economic profit? Answer: Zero economic profits do not mean no profit whatsoever. The firm is still making a normal profit. A normal profit compensates the firm’s owners enough to keep the firm in business because it is equal to the owner’s opportunity cost. Hence the firm has the incentive to stay in business.

PERFECT COMPETITION

Topic: Long-Run Equilibrium Skill: Conceptual

40) “In the long run, a perfectly competitive firm cannot earn an economic profit” True or false? Explain. Answer: The statement is true. Economic profit attracts new firms. As firms enter the market, the market supply increases. The market supply curve shifts rightward and the price falls until the economic profit disappears. Topic: Long-Run Equilibrium Skill: Conceptual

41) When a perfectly competitive firm is in long-run equilibrium, what is the relationship between the firm’s marginal cost, average total cost, marginal revenue, and price? Answer: Marginal cost and average total cost equal the market price, which is also the marginal revenue. Topic: Long-Run Equilibrium Skill: Conceptual

42) With regard to its profits and losses, how is the short run different from the long run for a perfectly competitive firm? Answer: The firm can earn an economic profit, incur an economic loss in the short run, or earn a normal profit in the short run. In the long run, however, the only possible outcome is a normal profit. An economic profit attracts entry by new firms and economic losses lead to exit by some firms. Thus, after entry or exit is complete in the long run, the remaining firms will earn a normal profit. Topic: External Economies and Diseconomies Skill: Recognition

43) What is meant by the term “external diseconomies”? Why are they called “external”? Answer: External diseconomies occur when, as the market output increases, a firm’s costs increase. These cost hikes are called “external” because they are beyond the control of—external to—any specific firm. Topic: External Economies and Diseconomies Skill: Conceptual

44) Explain the relationship between external diseconomies and costs? Answer: External diseconomies affect a firm’s costs. External diseconomies are factors beyond a firm’s

75

control that increase the firm’s costs as industry output increases. They come into play in the long run as firms change their plant size or new firms enter an industry. External diseconomies cause the price of the good to increase in the long run. Topic: External Economies and Diseconomies Skill: Conceptual

45) If an industry is characterized by external diseconomies, what can you determine about its long-run market supply curve? What if the industry has long-run external economies? In this case, what can you determine about its long-run market supply curve? Answer: If an industry has external diseconomies, its long-run market supply curve is upward sloping. Similarly, if an industry has external economies, its long-run market supply curve is downward sloping. Topic: External Economies and Diseconomies Skill: Conceptual

46) How do external economies and diseconomies affect the long-run market supply curve? Answer: External economies occur when an increase in the total market output lowers firms’ costs. External diseconomies occur when an increase in the total market output raises firm’s costs. Therefore in a market with external economies, the long-run market supply curve is downward sloping and in a market with external diseconomies, the long-run market supply curve is upward sloping. Topic: A Permanent Change in Demand Skill: Analytical

47) Starting from a long run equilibrium, what effect does a leftward shift in the market demand curve have on a perfectly competitive firm’s profit in both the short run and the long run? Answer: The leftward shift in the demand curve will cause the price and hence the profit to fall in the short run. The firms incur an economic loss, which causes some firm to exit in the long run. As firms exit, the short-run market supply curve shifts leftward, which causes the price to rise. The higher price reduces the economic loss, until eventually the price is high enough so that the firms earn a normal profit.

CHAPTER 12

MONOPOLY

Topic: How Monopoly Arises Skill: Recognition

Topic: Legal Barriers To Entry Skill: Recognition

1) What are the conditions that define a monopoly? Answer: There is only one firm producing a good or service with no close substitutes for the good or service sold and there are barriers to entry that prevent competing firms from entering the market.

4)

Topic: Barriers To Entry Skill: Recognition

2) Describe the two general types of barriers. Answer: Barriers to entry can be divided into legal barriers and natural barriers. Legal barriers occur when government action blocks competition in a market. For instance, the government could grant a public franchise, government license, patent, or copyright. In all cases, the government action prevents other firms from entering the market. Alternatively, a firm might buy up a significant portion of a natural resource. For instance, DeBeers controls over 80 percent of the world’s diamond market. The other type of barrier to entry is a natural barrier. A natural barrier to entry occurs when economies of scale are so large that they make it possible for one firm to meet the entire market demand at a lower price than could two or more firms. In this case, the market will “naturally” evolve to become a monopoly as a larger firm uses its cost advantage to cut its price and drive its high-cost, smaller competitors out of business. Topic: Legal Barriers To Entry Skill: Recognition

3) What is a legal barrier to entry? Answer: A legal barrier to entry arises when entry is restricted because a natural resource is owned by one person or the government has granted a franchise, patent, license or copyright to one person or firm.

Patents provide a firm with a monopoly on a given product. What is the economic rationale for granting patents? Answer: The rationale for patents is that without a patent, competitors could copy an innovation and drive the economic profit to zero. As a result, firms would have little incentive to innovate. Topic: Legal Barriers To Entry Skill: Recognition

5)

Are some monopolies created by government legislation that gives a firm the unique right to produce a good or service? Answer: Yes, some monopolies are created by government legislation, such as patent or copyright laws and the granting of public franchises. Monopolies that are created because of legal barriers to entry are called legal monopolies. Topic: Legal Barriers To Entry Skill: Conceptual

6)

Competition keeps prices lower for consumers. So why do we have patent laws? Answer: Patent laws are necessary to promote innovation. Without such laws an inventor might spend countless hours and a great deal of money developing a new product, put the product out into the market only to have a competitor copy it without incurring any of the time or costs to develop it. In the long run, this prospect would serve as a mighty disincentive to innovate and so would drastically reduce the supply of new products that come into the market. Topic: Legal Monopoly Skill: Recognition

7) What is a legal monopoly? Answer: A legal monopoly is a market in which competition and entry are restricted by the granting of a public franchise, government license, patent, or copyright.

78

CHAPTER 12

Topic: Natural Monopoly Skill: Recognition

Topic: Marginal Revenue and Elasticity Skill: Conceptual

8) What is a natural monopoly? Answer: A natural monopoly is a market in which one firm can supply the entire market at a lower price than two or more firms can.

12) What does the marginal revenue equal when a monopoly’s total revenue is maximized? What is the elasticity of demand when the total revenue is maximized? Answer: When the total revenue is at its maximum, the marginal revenue equals 0 and the elasticity of demand equals 1.

Topic: Demand Skill: Conceptual

9)

A monopolist, unlike a perfect competitor, has total control in its market because it is the single producer. Why, then, must a single-price monopolist decrease its price if it wants to increase its output? Answer: Because the monopolist does control the market, the monopolist sets the price at the maximum level that sells all the output the monopolist produces. This maximum price is determined from the demand for the product. The demand curve shows that the only way to increase the quantity consumers will buy is to lower the price. As a result, when a monopolist wants to produce more output, demanders will not buy the additional output at the initial price. As the demand curve indicates, in order to sell the extra production, the monopolist must lower its price. Topic: Price and Marginal Revenue Skill: Recognition

10) What is the relationship between the marginal revenue curve and the demand curve for a singleprice monopolist? Answer: For a single-price monopolist, price exceeds marginal revenue. The price is obtained from the demand curve, so for a single-price monopolist, the marginal revenue curve lies below the demand curve. Topic: Price and Marginal Revenue Skill: Recognition

11) How does marginal revenue compare to price for a single-price monopoly? Answer: Marginal revenue is less than price.

Topic: Marginal Revenue and Elasticity Skill: Conceptual

13) Comment on the following: “A monopolist is a firm that can raise its price without experiencing a decrease in its total revenue.” Answer: This statement describes a firm for which demand is price inelastic. Because a profit maximizing monopolist sets its price on the elastic portion of the demand curve, this statement is false. Topic: Marginal Revenue and Elasticity Skill: Conceptual

14) “A profit-maximizing monopoly never produces an output in the inelastic range of its demand curve.” True or false? Explain. Answer: The statement is true. The profit maximizing condition is MC = MR. If demand is inelastic, marginal revenue is negative. But marginal cost is never negative. So it’s impossible to maximize profit producing an output in the inelastic range of the demand curve. Topic: Single-Price Monopoly’s Output and Price Decisions Skill: Conceptual

15) What is the relationship between price, marginal revenue, and marginal cost when a single-price monopoly is maximizing profit? Answer: Marginal cost equals marginal revenue and both are below the price. Topic: Single-Price Monopoly’s Output and Price Decisions Skill: Conceptual

16) A monopolist can set any price it wants. So why does it still produce at a point where MC = MR, just like a perfectly competitive firm? Answer: The point where MC = MR maximizes any firm’s profit for the same reason it maximizes a perfectly competitive firm’s profit. In particular, for small amounts of output it is the case that the

MONOPOLY

MR exceeds the MC. Any unit for which MR > MC is a profitable unit to produce and so the firm wants to produce all of these units. As it increases its output, its total profit increases even as the difference between MR and MC shrinks. But as long as MR > MC, the unit is profitable and therefore is produced. Eventually the firm gets to the point where MR = MC. The firm does not want to go beyond this level of output, because for every unit beyond it MC > MR. Producing these units would cost the firm profit. So, once it starts producing, the firm won’t stop producing additional units of output before it reaches the level for which MR = MC. Then, once it reaches this point, it won’t go beyond this amount. Therefore the condition MR = MC determines the profit maximizing level of output. Topic: A Single Price Monopoly’s Output and Price Decisions Skill: Conceptual

17) Why do perfectly competitive firms maximize their profits by producing so that the price is equal to marginal cost, but monopolists maximize their profits by setting a price that is greater than marginal cost? Answer: Both types of firms maximize profits by producing so that their marginal revenue equals their marginal cost. For perfect competitors, price also equals marginal revenue, and therefore, at maximum profits, price equals marginal cost. Topic: Single-Price Monopoly’s Output and Price Decisions Skill: Conceptual

18) “A single-price monopolist will always charge a price that is on the elastic range of its demand.” Explain why the previous statement is correct or incorrect. Answer: The statement is correct. Only when the demand is elastic is marginal revenue positive. (When demand is unit elastic, marginal revenue is zero and when demand is inelastic marginal revenue is negative.) Because marginal cost is always positive, a single-price monopolist will always produce in the elastic range of its demand because it produces where marginal cost equals marginal revenue. Hence the price that the monopolist sets will always be on the elastic range of its demand.

79

Topic: Single-Price Monopoly’s Output and Price Decisions Skill: Analytical

19) Why will a profit-maximizing, single-price monopolist NOT produce the amount of output that maximizes its total revenue? Answer: When total revenue is at its maximum, the demand is unit elastic and marginal revenue equals zero. However, to maximize its profit, a single-price monopolist produces so that its marginal revenue equals its marginal cost. If marginal revenue equals zero, then in order for this level of output to maximize the monopolist’s profit, marginal cost also must equal zero. But marginal cost will never equal zero because to produce another unit always incurs some costs. Because marginal cost cannot equal zero, it is impossible for a profit-maximizing single-price monopolist to produce the amount of output that maximizes its total revenue. Topic: A Monopoly’s Economic Profit Skill: Conceptual

20) Can an unregulated monopoly earn an economic profit in the long run? Explain your answer. Answer: An unregulated monopoly can earn an economic profit in the long run. The fact that the monopoly is protected by a barrier to entry allows the firm to earn an economic profit in the long run. If the monopoly is earning an economic profit, other competitors want to enter the market but the barrier to entry keeps them out. Topic: A Monopoly’s Economic Profit Skill: Conceptual

21) What factor(s) enable a monopoly to earn an economic profit in the long run? Answer: Monopolies can earn an economic profit in the long run because of barriers to entry. Barriers to entry keep other firms from entering the industry and driving down the price and profits. Topic: A Monopoly’s Economic Profit Skill: Conceptual

22) What kind of profit can a monopoly earn in the short run? In the long run? Explain your answers. Answer: In the short run, a monopoly can earn an economic profit, a normal profit, or incur an economic loss. In other words, any sort of profit outcome is possible in the short run. In the long run, a monopoly can earn an economic profit or a nor-

80

CHAPTER 12

mal profit. A monopoly will not incur an economic loss in the long run because it would shut down. The key result that differentiates it from firms in other types of markets is that a monopoly can earn an economic profit in the long run. It can do so because there are barriers to entry. These barriers prevent other firms from entering the market and usurping part of the economic profit. Topic: Single-Price Monopoly and Competition Compared Skill: Conceptual

23) What does the deadweight loss from monopoly measure? Answer: The deadweight loss from monopoly measures the loss to society of having a product produced by a monopoly rather than a perfectly competitive industry. The deadweight loss results from the point that a monopolist produces less output than would a perfectly competitive industry. The monopolist produces less output in order to raise the price of the product and thereby earn an economic profit. Topic: Single-Price Monopoly and Competition Compared Skill: Conceptual

24) “A single-price monopolist charges a higher price and produces more output than a perfectly competitive industry.” Is the previous statement correct or incorrect? Explain your answer. Answer: The statement is partially correct because a monopolist charges a higher price than a perfectly competitive industry. However the statement also is partially incorrect because the monopolist produces less output than a perfectly competitive industry. Topic: Single-Price Monopoly and Competition Compared Skill: Conceptual

25) Compare the outcome in a market with a singleprice monopoly to that in a perfectly competitive market. Answer: The monopoly charges a higher price and produces less output than in a perfectly competitive market. The monopoly creates a deadweight loss by producing less than the efficient quantity of output, whereas a perfectly competitive market produces the efficient quantity of output and so

has no deadweight loss. The monopoly decreases consumer surplus and increases producer surplus from what they would be if the market is perfectly competitive. Topic: Single-Price Monopoly and Competition Compared Skill: Conceptual

26) Explain how a single-price monopoly determines its output and price. Compare this process to how a perfectly competitive firm determines its output and price. Answer: Single-price monopolies follow the same profit-maximizing rule as perfectly competitive firms and set their output level where marginal revenue equals marginal cost. However, unlike perfectly competitive firms, monopolies have control over price and can charge as much as the market will bear; therefore, given the quantity they produce, the monopoly chooses its price from the demand curve. Topic: Single-Price Monopoly and Competition Compared Skill: Conceptual

27) Which creates a larger deadweight loss, perfect competition or a single-price monopoly? Answer: The single-price monopoly creates a larger deadweight loss because perfect competition does not create a deadweight loss. Topic: Redistribution of Surpluses Skill: Conceptual

28) How do the price, output, consumer surplus, economic profit, and total surplus for a singleprice monopoly compare to that of a competitive industry? Answer: For the monopolist, price is higher, output is lower, consumer surplus is less, economic profit is larger, and total surplus is smaller relative to a competitive industry. Topic: Redistribution of Surpluses Skill: Conceptual

29) “Compared to a competitive market, a singleprice monopoly decreases the consumer surplus and increases the economic profit.” Is the previous statement correct or incorrect? Explain your answer. Answer: The statement is correct. A single-price monopoly produces less than a competitive market

MONOPOLY

and sets a higher price, both of which decrease the consumer surplus but increase the economic profit. Topic: Redistribution of Surpluses Skill: Conceptual

30) Suppose the government breaks up a single-price monopoly and turns it into a perfectly competitive industry. What will happen to price and the quantity produced? What will happen to the monopoly’s economic profit and the deadweight loss associated with the monopoly? Answer: The price will fall and output will increase to the efficient level. The monopoly’s economic profit will revert to the consumers as consumer surplus as the price falls and the quantity increases. The deadweight loss will be eliminated as output and the consumer surplus increase. Topic: Rent Seeking Skill: Recognition

31) What is rent seeking? How does rent seeking affect the deadweight loss from monopoly? Answer: Rent seeking is the act of obtaining special treatment by the government to create economic profit or to divert consumer surplus or producer surplus away from someone else. Often, rent seeking takes the form of lobbying to increase the economic profit of the lobbyist. Rent seeking increases the deadweight loss from monopoly. With rent seeking, not only does the monopoly create the (standard) deadweight loss, but also resources are used up in the process of rent seeking itself. Topic: Rent Seeking Skill: Conceptual

32) “Because of rent seeking, a monopoly may end up earning only a normal profit.” Is the previous statement correct or incorrect? Why? Answer: The statement is correct. Competition among rent seekers might cause the successful person striving to create a monopoly to pay so much in order to create the monopoly that, when taking into account the cost of creating the monopoly, the person earns only a normal profit. Topic: Rent Seeking Skill: Conceptual

33) Often to secure a monopoly, one must erect barriers to entry to obtain a monopoly. What is this

81

activity called in general and what does it entail specifically? Answer: Rent seeking is the term used to describe activities that are performed for the purpose of obtaining or retaining a monopoly. Specific activities might include lobbying, campaign contributions, bribery, lawsuits. Topic: Price Discrimination Skill: Recognition

34) What is price discrimination? Give examples of price discrimination. Answer: Price discrimination is the practice of selling different units of a good or service for different prices. For example, airlines’ customers pay different prices for the same trip, pizzerias charge a lower price for the second pizza bought, or Microsoft charges a lower than regular price when it sells its Office software to students. Topic: Price Discrimination Skill: Recognition

35) What is price discrimination? Can a perfectly competitive firm price discriminate? Explain you answer. Answer: Price discrimination is the practice of selling different units of a good or service for different prices. A perfectly competitive firm cannot influence the price of its product—it is a price taker. So a perfectly competitive firm cannot price discriminate. Topic: Price Discrimination Skill: Recognition

36) What are the characteristics of a market that allow a monopolist to successfully price discriminate between groups? Answer: A monopolist can price discriminate between groups when the groups are easily identifiable, can be separated, have different average willingness to pay for the good, and when the good or service cannot be resold. Topic: Price Discrimination Skill: Recognition

37) Define price discrimination. What factors must be present in order for a monopolist to price discriminate? Why do firms price discriminate? Answer: Price discrimination is selling a good or service at a number of different prices. In order to price discriminate, the firm must be able to iden-

82

CHAPTER 12

tify and separate different types of buyers. In particular, the firm must be able to identify which buyers are willing to pay a higher price than other buyers. And the firm must sell a product that cannot be resold. Therefore it must not be possible for a buyer who pays a low price to resell the product to a buyer who is willing to pay a higher price. Firms price discriminate because it increases their profit. By price discriminating the firm can charge a buyer a price that is closer to the maximum price the buyer is willing to pay. By setting the price closer to the maximum a buyer is willing to pay the firm can gain added total revenue and thereby added economic profit. Topic: Price Discrimination Skill: Conceptual

38) Give an example of price discrimination. Answer: Though there are many examples of price discrimination, price discrimination occurs when senior citizens get cheaper drugs, students get discounts at movies, or children under two can enter museums free. Topic: Price Discrimination Skill: Conceptual

39) “Price discriminators lose money by being nice to their customers.” Is the previous statement correct or incorrect? Answer: The statement is incorrect—price discrimination allows the firm to increase its economic profit. Topic: Price Discrimination Skill: Conceptual

40) “Price discrimination allows a monopolist to increase his or her economic profits by capturing part of the consumer surplus and turning it into economic profit.” Is the previous statement correct or incorrect? If the statement is correct, why is it important in understanding firms’ behaviors? If it is incorrect, why is it incorrect? Answer: The statement is correct. The statement is important because it explains why firms want to price discriminate, namely because they can convert some of the consumer surplus into extra economic profit. Hence firms endeavor to price discriminate because if they can do so, they can increase their economic profit.

Topic: Price Discrimination Skill: Conceptual

41) Why do some firms practice price discrimination? Relate your answer to the common practice of public colleges charging lower tuition to in-state students and higher tuition to out-of-state students. Answer: Price discrimination helps businesses capture more consumer surplus and hence increase their economic profit. Basically the firm charges more to people who are willing to pay more. For a public college, out-of-state students will likely have a higher willingness to pay for attending that college because, by leaving their home state, they are demonstrating that they truly want to attend the college. If the college charged in-state residents the same tuition as out-of-state residents, the college would miss the chance to maximize revenue from each group. Charging in-state residents the same high price as out-of-state residents would lead to a massive drop in quantity demanded and thus lower total revenue. By separating their customers based on differing demand conditions, the college earns more total revenue. Topic: Efficiency With Price Discrimination Skill: Conceptual

42) Compare the consumer surplus in a perfect competition with that of a single-price monopoly and with a price-discriminating monopoly. Answer: The consumer surplus in a perfect competition is larger than the consumer surplus in a single-price monopoly. And, the consumer surplus in a single-price monopoly is larger than that with a price-discriminating monopoly. Indeed, for a monopoly able to perfectly price discriminate, there is no consumer surplus. Topic: Perfect Price Discrimination Skill: Conceptual

43) What is perfect price discrimination? Is perfect price discrimination efficient? Why or why not? Answer: Perfect price discrimination occurs if a firm is able to sell each unit of output for the highest price anyone is willing to pay for it. With perfect price discrimination, output increases to the point at which price, and hence marginal benefit, equals marginal cost. So perfect price discrimination achieves efficiency.

MONOPOLY

83

Topic: Perfect Price Discrimination Skill: Conceptual

Topic: Monopoly Policy Issues Skill: Recognition

44) Even though a perfect price discriminator can extract all of the consumer surplus, how can it be efficient? Answer: Efficiency has nothing to do with who gets the surpluses; that is, efficiency has nothing to do with whether the consumer surplus (or producer surplus) is large or small. Instead, a market is efficient as long as there is no deadweight loss. Because there is no deadweight loss with perfect price discrimination, even though the amount of consumer surplus is as small as possible—zero!— and the producer surplus is as large as possible, nonetheless the market is efficient.

47) What are economies of scope? How do they affect the social desirability of monopoly? Answer: Economies of scope occur when the production of two goods together can be performed at a lower average cost than if the products had been produced separately. If a monopolist can enjoy economies of scope, it might set a lower price than would be the case in a competitive market.

Topic: Perfect Price Discrimination Skill: Conceptual

45) Compare and contrast the effect of perfect competition to the effect of perfect price discrimination on: a) efficiency. b) consumer surplus. c) economic profit in the long run. Answer: a) Both perfect competition and perfect price discrimination create efficiency. b) Consumers receive consumer surplus with perfect competition. However, there is no consumer surplus with perfect price discrimination. c) Perfectly competitive firms cannot earn an economic profit in the long run. A perfectly price discriminating monopoly earns the maximum amount of economic profit. Topic: Perfect Price Discrimination Skill: Conceptual

46) Which produces more output: a perfectly price discriminating monopoly or a single-price monopoly? Answer: The monopoly practicing perfect price discrimination produces more output. Indeed, a perfectly price discriminating monopoly produces the efficient level of output.

Topic: Regulating Natural Monopoly Skill: Conceptual

48) What are the two ways a natural monopoly can be regulated? What are the disadvantages and advantages of each? Answer: Natural monopolies can be regulated using a marginal cost pricing rule, so that the firm must set its price equal to its marginal cost, or by using an average cost pricing rule, so that the firm must set its price equal to its average total cost. The advantage of the marginal cost pricing rule is that the resulting output is efficient; the disadvantage is that the firm suffers an economic loss. The advantage of the average cost pricing rule is the firm earns a normal profit; the disadvantage is that it produces an inefficient quantity of output. Topic: Regulating Natural Monopoly Skill: Conceptual

49) Compare and contrast the marginal cost and average cost pricing rules for regulating natural monopolies. Answer: Marginal cost pricing sets the price equal to the marginal cost. It does so by determining the price using the intersection of the marginal cost curve and the demand curve. Marginal cost pricing results in an efficient level of output but the firm incurs an economic loss. Average cost pricing sets the price equal to the average total cost. It does so by determining the price using the intersection of the average cost curve and the demand curve. Average cost pricing results in an inefficient level of output and zero economic profit, that is, a normal profit. Topic: Regulating Natural Monopoly Skill: Conceptual

50) Electric utilities are often considered natural monopolies and are regulated. When would the price be highest: when the utility is not regulated, when

84

it is regulated using an average cost pricing rule, or when it is regulated using a marginal cost pricing rule? When would its price be lowest? Answer: The price would be highest if the utility was left unregulated and could set the profitmaximizing price. The price is lowest if the utility is regulated using a marginal cost pricing rule. Topic: Regulating Natural Monopoly Skill: Conceptual

51) “When electric utilities are regulated using an average cost pricing rule, they will earn a normal profit.” Is the previous statement correct or incorrect? Why? Answer: The statement is correct. An average cost pricing rule requires that the firm set its price equal to its average total cost. When price equals average total cost, the firm earns a normal profit.

CHAPTER 12

Topic: Regulating Natural Monopoly Skill: Conceptual

52) When a natural monopoly is regulated using an average cost pricing rule, what can you say about the firm’s profit and the market’s efficiency? Answer: Using an average cost pricing rule, the monopoly is earning a normal profit. However, there is an inefficient quantity of output produced so that the market is inefficient and there is a deadweight loss. Topic: Regulating Natural Monopoly Skill: Conceptual

53) When a natural monopoly is regulated using a marginal cost pricing rule, what can you say about the firm’s profit and the market’s efficiency? Answer: Using a marginal cost pricing rule, the monopoly is incurring an economic loss. However, there is an efficient quantity of output produced so that the market is efficient with no deadweight loss.

CHAPTER 13

MONOPOLISTIC COMPETITION AND OLIGOPOLY

Topic: Monopolistic Competition Skill: Recognition

Topic: Monopolistic Competition Skill: Recognition

1)

4)

List four characteristics of monopolistic competition. Answer: There are a large number firms; each produces a slightly different product; firms compete on price, quality and marketing; and firms are free to enter and exit. As a result of these characteristics, there are no dominant firms and each firms have a small market share. Because the firms produce differentiated products, firms can charge a markup and, in the long run, firms have excess capacity. Topic: Monopolistic Competition Skill: Recognition

2)

What are the differences between monopolistic competition and oligopoly? Answer: There two essential differences. First, in monopolistic competition a large number of firms compete, while in oligopoly few firms compete. Second, in monopolistic competition firms are free to enter and exit, whereas in oligopoly natural or legal barriers inhibit the entry of new firms. Topic: Monopolistic Competition Skill: Recognition

3)

“One of the defining features of monopolistic competition is product variety.” Is the previous statement correct or incorrect? Answer: The statement is correct. In monopolistic competition firms compete by producing similar but slightly differentiated products.

How do the characteristics of perfect competition and monopolistic competition differ? Answer: In monopolistic competition, the products sold are similar but differentiated, thereby enabling firms to compete on the basis of product development and marketing to further differentiate their products. In perfect competition the products are identical, thereby eliminating the opportunity for firms to compete by differentiating their product. Topic: Monopolistic Competition Skill: Conceptual

5)

Why is collusion about the price and amount of output impossible in monopolistic competition? Answer: The smaller the number of firms, the more likely collusion is to occur. Monopolistic competition has too many firms for collusion to be successful.

Topic: Monopolistic Competition Skill: Conceptual

6)

Is collusion possible in monopolistic competition? Why or why not? Answer: Collusion is not possible in monopolistic competition. It is not possible because there are many firms in monopolistic competition, reaching an agreement to restrict output and boost price is impossible. Topic: Product Differentiation Skill: Recognition

7)

What is product differentiation? What market structure is characterized by product differentiation? Answer: A firm practices product differentiation if it makes a product that is slightly different from the

86

CHAPTER 13

products of competing firms. A differentiated product has close substitutes, but not perfect substitutes. Product differentiation is a distinguishing feature of monopolistic competition. Topic: Monopolistic Competition; Demand Curve Skill: Conceptual

8)

What do demand and marginal revenue curves look like in monopolistic competition? How do they compare to the demand and marginal revenue curves in perfect competition and monopoly? Answer: In monopolistic competition, the product is differentiated. This fact gives each firm some control over price, so each firm’s demand curve is downward sloping. Because there are many close substitutes for these firms’ goods, demand is elastic. These firms must lower their price to sell more; therefore the marginal revenue curve is beneath the demand curve. In perfect competition, the product is homogeneous, which makes firms price-takers, able to sell as much as they wish at the market price. Therefore, marginal revenue equals price, and the marginal revenue curve and the demand curve are the same and are horizontal. In monopoly, there is only one firm. The firm faces the market demand, which is steep, because there are no close substitutes for the good. The firm must lower its price to sell more, so for a single-price monopoly, the marginal revenue curve is beneath the demand curve. Topic: Monopolistic Competition; Profit Maximization Skill: Conceptual

9)

“A firm in monopolistic competition maximizes its profit by producing where its price is equal to its marginal cost.” Is the previous statement correct or incorrect? Answer: The statement is incorrect. A firm in monopolistic competition maximizes its profit by producing where its marginal revenue equals its marginal cost. Because the marginal revenue is less than the price for a firm in monopolistic competition, it definitely is not the case that the firm produces where its price equals its marginal cost!

Topic: Monopolistic Competition; Profit Maximization Skill: Conceptual

10) How does a firm in monopolistic competition determine its price and quantity? What type of profit can it earn in the short run and the long run? Answer: The firm produces where its marginal cost equals its marginal revenue. Then the price is determined from the demand curve and is the highest price at which people will buy the quantity produced. The firm can earn an economic or a normal profit in the short run. (It could also suffer an economic loss in the short run.) In the long run, the firm cannot earn an economic profit; it can only earn a normal profit. Topic: Monopolistic Competition; Long-Run Economic Profit Skill: Conceptual

11) What type of profit can a firm in monopolistic competition earn in the long run? Explain your answer. Answer: In the long run, a firm in monopolistic competition can earn only a normal profit. It cannot earn an economic profit because there are no barriers to entry. So if a firm in monopolistic competition is earning an economic profit, in the long run new firms enter the market, produce a similar product, and decrease the demand for the initial firm’s product. Entry continues until the firms earn only a normal profit. Topic: Monopolistic Competition; Long-Run Economic Profit Skill: Conceptual

12) Why are firms in monopolistic competition unable to earn an economic profit in the long run? Answer: While firms in monopolistic competition do not produce an identical product, such as perfectly competitive firms, they face the same problem other competitive firms face: freedom of entry. When firms in monopolistic competition are earning an economic profit, other firms will enter the market. Entry decreases the demand for the products of the existing firms and thereby decreases their economic profit. Firms will continue to enter the market until the economic profit equals zero.

MONOPOLISTIC COMPETITION AND OLIGOPOLY

87

Topic: Monopolistic Competition; Long-Run Economic Profit Skill: Conceptual

Topic: Monopolistic Competition; Long-Run Excess Capacity Skill: Conceptual

13) Why does a firm in monopolistic competition earn a normal profit in the long run? Answer: Entry into monopolistically competitive markets is easy because there are no barriers to entry. So when firms in monopolistic competition have an economic profit, other firms are attracted to enter the industry, thereby decreasing profits for everyone. Entry continues as long as there is an economic profit so, in the long run when all entry is completed, the firms earn only a normal profit.

16) Explain why firms in monopolistic competition have excess capacity in the long run. Answer: A firm’s efficient scale of production is the level of production where average total cost is at a minimum. In the long run, monopolistically competitive firms do not produce where ATC is at the minimum. They would need to increase output to reach this point. But if they did so, their profit would decrease. So for firms in monopolistic competition, the profit-maximizing output is less than the efficient scale.

Topic: Monopolistic Competition; Long-Run Excess Capacity Skill: Conceptual

Topic: Output and Price in Monopolistic Competition Skill: Conceptual

14) What is excess capacity? What industry has excess capacity in the long run: perfect competition or monopolistic competition? Answer: The efficient scale of output is the level that minimizes the average total cost. Excess capacity occurs if the firm produces less than the efficient scale. In this case, the firm could boost its output and lower its average total cost. Firms in monopolistic competition have excess capacity. Firms in perfect competition produce at the minimum of the average total cost. They produce at the efficient scale of output and so do not have excess capacity.

17) In monopolistic competition, firms sell a differentiated product. In perfect competition, firms sell an identical product. How do these markets differ as a result? Answer: Because of this difference, firms in monopolistic competition face a downward sloped demand curve and have price setting power. Firms in perfect competition do not—their demand curves are horizontal. Because of this difference, in the long run, perfectly competitive firms produce at minimum average cost while monopolistically competitive firms have excess capacity.

Topic: Monopolistic Competition; Long-Run Excess Capacity Skill: Conceptual

15) Define the efficient scale of production and then for a firm in monopolistic competition discuss its excess capacity. Answer: The efficient scale is the quantity produced when average total cost is at its minimum. In the long run, a firm in monopolistic competition operates below the efficient scale when maximizing profits. The difference between the efficient scale and the quantity produced by a firm is the excess capacity. Because the firm is not producing at the efficient scale, its average total cost is higher than if it produced at the efficient scale.

Topic: Output and Price in Monopolistic Competition Skill: Conceptual

18) A monopoly firm can earn economic profit in the long run. A firm in monopolistic competition cannot. What creates this difference? Answer: The key to long-run economic profits is a barrier to entry. The monopoly firm has a barrier to entry that protects its economic profit from the entry of new firms. A monopolistically competitive firm’s industry features easy entry and exit and so there is nothing to protect any economic profit. Topic: Innovation and Product Development Skill: Conceptual

19) How do product development and marketing affect a firm in monopolistic competition? Answer: Product development and marketing have two effects on a firm. First, because these activities

88

CHAPTER 13

are costly, they increase the firm’s costs and shift its costs curves upward. Second, they can increase the demand for the firm’s products. Topic: Marketing Skill: Conceptual

20) Why would a firm in a monopolistically competitive industry ever advertise? Answer: Similar to virtually every other business decision, advertising carries with it benefits and costs. While advertising causes the fixed costs to increase, and thereby shifts the average total cost curve upward, advertising also might increase the demand for the company’s product by temporarily making people believe that the product is better than some other firm’s product. Firms in monopolistic competition frequently advertise extensively in order to differentiate their product from those of their competitors and thereby increase the demand for their particular version of the product. Topic: Marketing Skill: Conceptual

21) Explain the role of advertising in monopolistic competition. Describe how advertising by all firms in a monopolistically competitive industry impacts a firm’s ATC curve, its MC curve, its demand curve, and its MR curve. Answer: In order to maintain (or regain) economic profit, a firm in monopolistic competition must continually develop new products that are unique and/or of high quality (or make consumers believe this). Advertising lets firms signal this information. So all firms in monopolistic competition tend to advertise extensively. Advertising is a fixed cost and it shifts the ATC curve upward. Even though it shifts to the ATC curve upward, the total average cost might be lower if it increases the amount sold by enough. Because advertising is a fixed cost, it has no effect on the marginal cost, so the MC curve does not change. Because all firms advertise, advertising might or might not increase demand for a specific firm. When all firms advertise, the demand curve and marginal revenue curve for a specific firm become more elastic.

Topic: Marketing Skill: Conceptual

22) Why are selling costs high in monopolistic competition? Answer: In monopolistic competition, there are a large number of small firms producing differentiated products. Each firm’s output is a substitute for other firms’ output; therefore demand for any firm’s product is very elastic. If firms can further differentiate their product in the eyes of the consumer, they might be able to both increase demand and decrease demand elasticity, at least temporarily. In this case, the firm could then charge a higher price and, temporarily at least, earn an economic profit. This differentiation occurs through innovation, product development and marketing, which contribute to selling costs. Topic: Efficiency of Monopolistic Competition Skill: Conceptual

23) Explain how selling costs in monopolistic competition affect the efficiency of monopolistic competition. Answer: The additional selling costs from product differentiation and marketing increase consumer choice by providing variety. This benefits society and weighs in favor of the efficiency of monopolistic competition. However, selling costs can add to the product’s price. Also, at times the product differentiation is more apparent than real. These factors harm society and count against the efficiency of monopolistic competition. Topic: Markup Skill: Conceptual

24) What is a firm’s markup? What does it show? Answer: A firm’s markup is the amount by which price exceeds marginal cost. A markup shows that buyers pay more than the firm’s marginal cost. Topic: Oligopoly Skill: Recognition

25) Describe the characteristics of an oligopoly. Answer: There are a small number of firms that act interdependently. They are tempted to form a cartel and collude to increase profits. They can compete on price only (if they produce identical products) or compete on price, product quality and marketing (if they produce slightly different products). Natural or legal barriers prevent the entry of new firms.

MONOPOLISTIC COMPETITION AND OLIGOPOLY

Topic: Natural Oligopoly Skill: Recognition

26) What is a natural oligopoly? How does it arise? Give an example. Answer: A natural oligopoly is an industry in which a small number of large firms can supply the entire market at a lower price than could a larger number of smaller firms. Natural oligopoly arises when economies of scale and limited market demand create natural barriers to entry. For example, suppose the minimum efficient scale for a taxi company is 30 rides per day and the ATC at this level of output is $10 per ride. If the quantity of taxi rides demanded at $10 is 60 rides per day, there is only room in the market for two taxi companies. With more taxi companies, either the price would have to fall below $10 per ride or the ATC would have to rise above $10 per ride. In both cases the firms would incur an economic loss and would exit until only two firms are left. Topic: Oligopoly Skill: Conceptual

27) “Because firms in an oligopoly are so large, they do not need to consider each other’s actions.” Is the previous statement correct or incorrect? Explain your answer. Answer: The statement is incorrect. Oligopoly is an industry in which only a few firms compete. Because there are only a few firms, the hallmark of oligopoly is mutual interdependence, that is, one firm’s action will affect the other firms. The fact that in oligopoly each firm’s actions affect its rivals is unlike the case in perfect competition or monopolistic competition, in which there are so many firms that one firm’s actions have no effect on its rivals, or monopoly, in which there is only one firm and hence no rivals. Topic: Oligopoly Skill: Conceptual

28) What market structures other than oligopoly have the characteristic of one firm’s actions affecting the actions of its competitors? Explain your answer. Answer: No other market structure has the characteristic that one firm’s actions can affect the actions of its competitors. In monopoly, there are no competitors to affect. And in perfect competition and monopolistic competition, there are so many

89

competitors that any one firm’s actions have no measurable impact on its competitors. Oligopoly is unique in that it is the only market structure in which one firm’s actions affect the actions of its competitors. Topic: Cartel Skill: Recognition

29) What is a cartel? Answer: A cartel is a group of firms acting together to limit output, raise price and increase economic profit. Cartels are illegal in the United States. Cartels operate in a market structure with oligopolies. If firms can stick to the cartel agreement, the firms can earn an economic profit. However, cartels tend to break down because firms are tempted to cheat on their cartel partners and increase their own profit at the expense of their partners. Topic: Cartel Skill: Conceptual

30) Explain what a cartel is and the difficulties faced in maintaining a cartel. Answer: A cartel is a group of firms acting together to decrease output, raise price, and increase economic profit. The difficulty faced by a cartel is the fact that each member has the incentive to cheat on the cartel and increase its output. If a member increases its output and the rest of the cartel members do not, the cheating member’s profits will increase substantially. Each member reasons that if it is the only cheater, it can significantly increase its profit and so each firm has an incentive to cheat. Topic: Cartel Skill: Conceptual

31) In the United States, why are cartels among firms usually kept secret? Answer: Cartels are typically kept secret because they are illegal. In the United States and many other countries, it is illegal for firms to collude to form a cartel. It is illegal because when firms collude they do so in order to restrict output, raise prices, and capture consumer surplus in order to increase their economic profit.

90

CHAPTER 13

Topic: Cartel Skill: Conceptual

Topic: Game Theory Skill: Recognition

32) What is the legal status of a cartel among firms in the United States? Answer: Cartels are illegal in the United States and in many other countries.

36) What is a payoff matrix in game theory? Answer: A payoff matrix is a table that shows the payoffs for each player for every possible combination of actions by the other players.

Topic: Colluding To Maximize Profits Skill: Conceptual

Topic: Game Theory Skill: Conceptual

33) “If firms in an oligopoly enter into a collusive agreement to operate as a monopoly, the industry produces the most output and if they operate as perfect competitors, the industry produces the least output.” Is the previous statement correct or incorrect? Why? Answer: The statement is incorrect; it reverses the outcomes. If the firms in an oligopoly operate as a monopoly, the industry produces the least output and if they operate as perfect competitors, the industry produces the most output.

37) What is game theory and what light does it shed on the issues faced by duopolists? Answer: Game theory is a tool economists use to analyze the behavior of oligopolistic firms because game theory is a tool to study strategic behavior. Game theory shows that because these firms are interdependent, the decisions they make to promote their own self-interest can wind up harming all the firms. The dilemma faced by duopolists is illustrated using game theory: Firms looking to earn for themselves the maximum possible profit can wind up earning less profit than if they had behaved less self-interestedly and more cooperatively.

Topic: Colluding To Maximize Profits Skill: Conceptual

34) “If firms in duopoly collude and operate as a monopoly, the industry produces more output compared to the Nash equilibrium.” True or false? Explain. Answer: The statement is false. In the Nash equilibrium, both firms cheat and output is the same as in perfect competition. If the firms operate as a monopoly, the industry’s profit-maximizing level of output is below the competitive level. Topic: Colluding To Maximize Profits Skill: Conceptual

35) What is the best outcome for society: When firms in an oligopoly enter into a collusive agreement to operate as a monopoly or when they act as perfect competitors? Briefly explain your answer. Answer: The best outcome for society is when the firms act as perfect competitors. Perfect competition produces the efficient quantity of output. A monopoly restricts the quantity of output it produces and creates a deadweight loss, which harms society So society is better off if the firms compete rather than collude and operate as a monopoly.

Topic: Nash Equilibrium Skill: Recognition

38) What is a Nash equilibrium? Answer: A Nash equilibrium occurs when the each player takes the best action given the actions of the other players. Topic: Nash Equilibrium Skill: Conceptual

39) What is a Nash equilibrium? Is this equilibrium necessarily the best outcome for the players? Give an example. Answer: John Nash proposed the concept of an equilibrium in a game where each player takes the best possible action given the action of other players. A Nash equilibrium is not necessarily the best one for the players. This can be seen in the prisoners’ dilemma. Typically the prisoners’ dilemma is a game where two prisoners are given rules and payoffs to encourage them to confess to a crime. The prisoners, acting in their own self interest, confess to the crime to minimize their jail time and so confession is the Nash equilibrium. But if the players can communicate with each other, they can improve their position. If they can communicate, they both deny the crime and so both wind up doing less time in jail.

MONOPOLISTIC COMPETITION AND OLIGOPOLY

Topic: Nash Equilibrium Skill: Conceptual

40) “A Nash equilibrium occurs when both parties to a game end up worse off as a result of the decisions that are made.” Is the previous definition of a Nash equilibrium correct or incorrect? Answer: The definition is incorrect. A Nash equilibrium is an equilibrium in which each player takes the best possible action given the action of the other player. Topic: Equilibrium of the Prisoners’ Dilemma Skill: Conceptual

41) What is the real dilemma facing the prisoners in the prisoners’ dilemma game? Answer: The real dilemma facing the prisoners in the prisoners’ dilemma game is that when each prisoner plays his or her best strategy, the best outcome is not achieved. Topic: Cartel; Incentive To Cheat Skill: Conceptual

42) OPEC, the Organization of Petroleum Exporting Countries, was formed in Baghdad in 1960. Since its formation, this cartel has suffered from a major problem with respect to the quota (limit) of output it assigns each member nation. What is OPEC’s goal and what sort of quota do you think the cartel assigns? How and why do nations cheat on their quota? What happens when a nation cheats on its quota? Answer: In order to keep oil prices high, as has been the case since 1999, OPEC creates a target level of output designed to achieve a particular high price. OPEC’s goal is to set a price high enough so that its member nations earn the maximum economic profit. Once the target output is set, OPEC assigns a production quota to each member. As long as each member adheres to its quota the price will remain high and stable. However, from time to time, individual nations cheat on the agreement by producing more oil than they are allowed. Nations cheat because they realize that if they alone cheat, the impact on oil prices will be slight but the impact on their profit will be large. Once this oil shows up on world markets, the supply of oil increases and prices begin to fall. Then, once prices begin to fall other members begin to panic and they start selling more oil too in order to get the highest price they can before a collapse takes

91

place. If every nation cheats, the supply will increase more than if just a few do and the collapse in price becomes a self-realizing prophecy. Topic: Cartel; Incentive To Cheat Skill: Conceptual

43) Why do most collusive agreements have difficulty surviving? Answer: Most collusive agreements have difficulty surviving because each firm individually can increase its profits by lowering its price and increasing its output. Thus the incentive to cheat on the agreement is great for all firms. Topic: Cartel; Incentive To Cheat Skill: Conceptual

44) Why do oligopoly firms find it difficult to cooperate and not cheat on a cartel agreement? Answer: Firms in an oligopoly have large market shares. When they change their output or price, the firm affects not only its own revenue and profit but also the revenue and profit of other firms. For example, if a firm cheats on a cartel agreement by lowering its price, it will capture a larger market share. The competitors will see a decrease in their total revenue and their profit but the cheating firm’s profit increases. If the firms cooperate, they could act like a monopoly and have the maximum joint profit but each firm has the temptation to cheat and produce more than its share. This temptation is strong because cheating will increase the cheater’s revenue and profit substantially. Topic: Cartel; Incentive To Cheat Skill: Conceptual

45) What is the dilemma faced by firms in collusive agreement to restrict output and boost price? Answer: Because there are just a few large firms in an oligopoly, output and pricing decisions made by one firm affect the demand for other firms’ goods. To maximize the total joint profit, the firms must cooperate, act like a monopoly so as to restrict output and earn monopoly profits. Each firm, though, has an incentive to cheat on an agreement to restrict output because if it increases production it can (temporarily, at least) earn higher profits. But if all firms increase production, total profits will fall and the market will move toward the competitive equilibrium.

92

CHAPTER 13

Topic: Efficiency of Oligopoly Skill: Conceptual

Topic: Contestable Market Skill: Conceptual

46) Does an oligopoly produce the efficient quantity of output or does it create a deadweight loss? Do the firms want to produce the efficient quantity of output? Explain your answer. Answer: An oligopoly might or might not operate efficiently. It operates efficiently if the firms cheat on any agreement and increase output so that it is the same as the perfectly competitive level. In this case, price equals marginal cost and the outcome is efficient. There is no deadweight loss. From the firms’ perspectives, this outcome is undesirable because the firms earn only a normal profit. If the firms can play repeated games, detecting and punishing overproduction, the oligopoly is more likely to restrict output to the monopoly level. This outcome is inefficient because marginal cost does not equal marginal benefit. A deadweight loss is created. From the firms’ perspective, this outcome is more desirable because the firms earn an economic profit.

47) Why would a profit maximizing monopolist in a contestable market set its price at a level below that which maximizes short run profits? Answer: A firm in a contestable market is not protected by barriers to entry. Thus while it is currently the only firm in the market, it might worry that other firms will enter the market. In this case, setting a relatively lower price is known as limit pricing. It is a pricing strategy that deters entry by sending a signal to potential entrants that entering the industry would result in economic losses. Topic: Contestable Market Skill: Conceptual

48) How is a contestable market similar to a perfectly competitive one? Answer: A contestable market is similar to a perfectly competitive market in that there is free entry and exit. As a result, the active firm(s) cannot earn an economic profit in the long-run because potential entrants will enter any time economic profits exceed zero.

CHAPTER 14

REGULATION AND ANTITRUST LAW

Topic: Externalities Skill: Recognition

Topic: Market Intervention Skill: Conceptual

1) What is an externality? Answer: An external cost is a cost borne by a party not involved in a transaction. An external benefit is a benefit enjoyed by a party not involved in a transaction.

6)

What are the two main economic reasons why governments exist? Answer: First, governments establish and maintain property rights and set the rules for the redistribution of income and wealth. Second, governments provide mechanisms for allocating scarce resources when the market economy results in inefficiency.

What is the difference between economic regulation and antitrust law? Answer: Economic regulation generally involves the determination of prices that regulated firms can charge and control over entry into the industry. Regulation is administered by government regulatory agencies such as state public utility commissions and many federal commissions such as the FCC. Antitrust law has to do with the prohibition of monopoly type behavior, such as price-fixing, or certain mergers that would result in monopoly power. The major antitrust laws are the Sherman Act, the Clayton Act, and the Federal Trade Commission Act. They are enforced through the judicial system.

Topic: Public Choice and the Political Marketplace Skill: Recognition

Topic: Regulation Skill: Recognition

Topic: The Economic Theory of Government Skill: Recognition

2)

3) Who are the actors in the political marketplace? Answer: The actors in the political market place are voters, firms, politicians, and bureaucrats. Topic: Public Choice and the Political Marketplace Skill: Recognition

4)

Who are the demanders and who are the suppliers in the political market place? How do the demanders “pay” the suppliers? Answer: The demanders are voters and firms. The suppliers are politicians and bureaucrats. The demanders “pay” the suppliers with their votes, campaign contributions, and taxes. Topic: Political Equilibrium Skill: Recognition

5)

What is meant by the term “political equilibrium”? Answer: A political equilibrium occurs when the choices of voters, politicians, and bureaucrats are compatible, and no group can improve its position by making a different choice.

7)

Although regulatory agencies vary in scope, size, and in the economic aspects they control, and exist at all levels of government, what features do all such agencies have in common? Answer: All regulatory agencies have political appointees, bureaucracies, industry experts, budgets, and operating rules for controlling prices as well as other aspects of economic performance. Regulatory agencies also control production quantities, the nature of the good or service, and the markets that will be served. However they rarely specify the exact details of the firms’ production processes. Topic: Scope of Regulation Skill: Recognition

8)

In the United States since the 1970s, has regulation tended to increase or decrease? Answer: Since the middle of the 1970s, there has been a tendency to deregulate, which means that regulation has tended to decrease.

94

CHAPTER 14

Topic: Economic Theory of Regulation Skill: Conceptual

Topic: Capture Theory Skill: Conceptual

9)

13) What is the idea behind the capture theory of regulation? Answer: Capture theory assumes that the marginal cost of regulation is high, but there is a specific group that receives a high marginal benefit (a special interest group) from the regulation. If the group has low organization costs, the regulations likely will be imposed. If the costs of regulation are spread thinly, the regulation does not decrease votes (and so aren’t costly to politicians). As a result, regulations can help producers, the special interest group, to maximize their profit.

What is meant by the “economic theory of regulation”? Answer: The economic theory of regulation is part of the theory of public choice and is based on the supply of and demand for regulation. Regulation is demanded by certain consumers and/or certain firms and is supplied by government. Given the supply and demand, a political equilibrium emerges at some quantity of regulation. Topic: Social interest Skill: Recognition

10) Explain the social interest theory of regulation. Answer: The social interest theory of regulation is that politicians supply the regulation that achieves an efficient allocation of resources. According to this theory, the political process works well to find inefficiencies and eliminate them through regulations. Topic: Social interest Skill: Recognition

11) “Under the social interest theory of regulation, regulators attempt to maximize profits for the public owners of the firms being regulated.” Is the previous statement correct or incorrect? Answer: The statement is incorrect. The social interest theory is the theory that regulators seek an efficient use of resources. The capture theory of regulation is the theory that regulators attempt to maximize the producers’ economic profit. Topic: Capture Theory Skill: Recognition

12) “The theory that regulation seeks an efficient use of resources is called the capture theory of regulation.” Is the previous statement correct or incorrect? Answer: The statement is incorrect. The capture theory is the theory that the regulated producers “capture” the regulators and so the regulators help the producers maximize their economic profit. The theory that regulation seeks an efficient use of resources is the “social interest” theory.

Topic: Social interest and Capture Theory Skill: Recognition

14) What are the two major economic theories of regulation and what is the difference between them? Answer: First is the social interest theory, which sees regulation as supplied by government to achieve efficiency and eliminate deadweight loss. Second is the capture theory, which sees regulation as supplied to satisfy the demand of producers who want to maximize their producer surplus or economic profit. Topic: Social interest and Capture Theory Skill: Recognition

15) What is the social interest theory of regulation? How does it differ from the capture theory of regulation? Answer: The social interest theory of regulation is that regulators seek an efficient use of resources. The capture theory asserts that producers “capture” the regulators so that the regulation is designed to help the producers maximize their economic profit. Topic: Social interest and Capture Theory Skill: Recognition

16) Describe the difference between social interest theory of regulation and the capture theory or regulation. Answer: Both are theories of regulation but they differ according to what they see as to the goal of the regulation. The social interest theory assumes that regulation that seeks an efficient use of resources. It asserts that the political process works to eliminate deadweight loss by using appropriate regula-

REGULATION AND ANTITRUST LAW

tions. The capture theory proposes that producers bend the regulators to their will so that resources are not used efficiently because regulated market outcomes favor producers. Everyone else but producers bears the cost of this regulation. Because this cost is a small amount per person, no one finds it worthwhile to propose legislation to avoid it. Topic: Natural Monopoly Skill: Recognition

17) What is a natural monopoly and what problem does natural monopoly pose for regulators? Answer: A natural monopoly is a firm that can supply the market at lower cost than two or more firms. It can do so because it has declining long-run average total cost over the entire range of market output. Because the ATC is declining, the marginal cost must be below the ATC. Therefore, if the regulator forces the firm to price at MC to achieve efficiency (such as is done by a perfectly competitive firm), the natural monopolist will fail to cover its total cost. It incurs an economic loss and requires a government subsidy to survive. Topic: Natural Monopoly Skill: Conceptual

18) Why are water companies considered a natural monopoly? Answer: Once one water company incurs the cost of establishing a physical connection to one customers’ home or place of business, the marginal cost of providing service falls rapidly over time as more and more service is provided. For instance, once a main pipe is buried under a street, adding additional customers on the street is relatively cheap. Economies of scale make it very cost prohibitive for another firm to enter the market, leaving one water company as the provider of the service in that area. Topic: Natural Monopoly, Marginal Cost Pricing Rule Skill: Conceptual

19) “A single-price natural monopoly that is regulated to set price equal to marginal cost will always incur an economic loss.” True or false? Explain. Answer: The statement is true. A natural monopoly’s average total cost is falling as its output increases. This means that marginal cost is below average total cost. Because price equals marginal cost, price

95

is less than average total cost so that the firm incurs an economic loss. Topic: Natural Monopoly, Marginal Cost Pricing Rule Skill: Conceptual

20) “If a natural monopoly is regulated using a marginal cost pricing rule, the firm earns a normal profit.” Is the previous statement correct or incorrect? Answer: The statement is incorrect. If a firm is regulated using a marginal cost pricing rule, the firm incurs an economic loss. Topic: Natural Monopoly, Marginal Cost Pricing Rule Skill: Conceptual

21) “If Michigan’s electric utilities were allowed to use marginal cost pricing, it would lead to economic profits for these utilities.” Is the previous statement correct or incorrect? Answer: The statement is incorrect. Imposing marginal cost pricing on natural monopolies results in the firms incurring economic losses not economic profits. Topic: Natural Monopoly, Marginal Cost Pricing Rule Skill: Conceptual

22) If a natural monopoly is regulated using the marginal cost pricing rule, how will that affect prices, outputs, profits, and the distribution of surpluses? What are the pros and cons to this method of regulation? Answer: The marginal cost pricing rule sets the regulated price equal to the price where the marginal cost curve intersects the demand curve. This price is lower than the monopoly price, and results in a higher level of output. The monopoly’s economic profit is eliminated; in fact, this rule results in the firm making economic losses, as marginal cost is less than average total cost for a natural monopoly. Because output increases to the point where marginal cost equals price, consumer surplus is maximized. The advantage of this method of regulation is that it results in the efficient level of output. The disadvantage of this method is that means the firm will incur an economic loss. Unless subsidized by the government, the firm will eventually exit the industry, as no firm can operate at a loss in the long run.

96

Topic: Natural Monopoly, Average Cost Pricing Rule Skill: Conceptual

23) What is an average cost pricing rule? Why do regulatory agencies use it for natural monopolies? Answer: Average cost pricing means that the firms will equate their price to their average total cost. It is used by regulatory agencies, because if a natural monopoly is forced to charge a perfectly competitive price (using a marginal cost pricing rule), the firm will not be able to cover its costs. No firm can operate on losses, so average cost pricing is considered a second-best solution: it allows the natural monopoly to make a normal profit but does not allow it to set its price as high as it would were it unregulated. Topic: Natural Monopoly, Cost Inflation Skill: Conceptual

24) Why do some utilities have an incentive to exaggerate their costs of production? Answer: Because utilities are generally allowed to charge prices that cover their average cost of production, a utility might want to incur higher than normal costs (maybe from lush carpets, hunting lodges, tickets to sporting events, and so forth). If the utility exaggerates its costs, the regulators are likely to simply order higher rates to cover the higher costs, and so the utility executives can enjoy benefits (the carpet, hunting lodge, sporting events) without incurring any personal costs. Topic: Natural Monopoly, Cost Inflation Skill: Conceptual

25) How can managers of natural monopolies exaggerate their costs? Answer: By increasing on-the-job luxury items such as sumptuous office suites, limousines, golf competitions at expensive locations, company jets, and other non-necessary expenditures, the managers can exaggerate their costs over what is truly necessary to produce the product. Topic: Rate of Return Regulation Skill: Conceptual

26) What potential problem is there with rate of return pricing? Answer: The monopolist might exaggerate its costs and mislead the regulator. In this case, the regulator allows the firm to increase its price, so the

CHAPTER 14

monopolist has no incentive to operate efficiently and cost effectively. Topic: Rate of Return Regulation Skill: Conceptual

27) Describe the main problem with rate of return regulation and name an alternative regulatory scheme that has been devised to deal with that problem. Answer: The main problem with rate of return regulation is that a firm might be inclined to inflate its costs, because its price is set at a level that permits the firm to recoup all its costs. Therefore, the firm might incur unnecessary costs that serve the interests of its managers, such as lavish offices, company cars, travel, entertainment, etc. The alternative regulation schemes is price cap regulation. With price cap regulation, the regulating agency sets the maximum price the company can charge. The company is allowed to charge any price below the cap and can keep all (or some) of any economic profit it can earn. Topic: Price Cap Regulation Skill: Conceptual

28) What incentive does price cap regulation attempt to give the firm? How does it give the firm this incentive? Answer: Price cap regulation is intended to motivate the firm to operate efficiently and keep its costs under control. It does so setting the maximum price the company can charge and then allowing the firm to keep part (or perhaps all) of any economic profit it can earn if it cuts its costs. Topic: Price Cap Regulation Skill: Conceptual

29) Explain the difference between price cap regulation in a natural monopoly and the effect of a price ceiling in a competitive market. Answer: In regulating a natural monopoly, a price cap regulation is a price ceiling in which a rule specifies the highest price that the firm is allowed to charge. A price cap lowers the price and increases output. This type of regulation gives a firm an incentive to operate efficiently and to keep its costs under control. In a competitive market, a price ceiling establishes the highest price that all firms in the market are allowed to charge. But the major issue is that in a competitive market, the competitive equilibrium already is generally efficient. And,

REGULATION AND ANTITRUST LAW

to be effective, the price ceiling needs to be below the market equilibrium price. A shortage of the good occurs because firms are willing to supply less output than they would produce in the absence of the price ceiling. As a result, inefficiency is created. Topic: Natural Monopoly, Regulation Skill: Conceptual

30) Briefly describe and discuss the different ways a natural monopoly can be regulated: Marginal cost pricing, average cost pricing, rate of return regulation, and price cap regulation. Answer: Marginal cost pricing: The regulated price is set equal to marginal cost. In this case, the efficient quantity is produced so there is no deadweight loss. Consumer surplus is maximized. The firm incurs an economic loss unless it can raise revenues in an additional way, such as using price discrimination or a two-part tariff. Average cost pricing: The regulated price is set equal to average cost. While this form of regulation does not produce an efficient outcome, it allows firms to earn a normal profit. There is a deadweight loss. Rate of return regulation: The regulated price enables a regulated firm to earn a specified target percent return on its capital. If a regulator could observe the firm’s total cost and also know that the firm minimized total cost, the regulation would be the same as average cost pricing. In some cases, however, the firm is able to “capture” the regulator, which enables the firm to exaggerate it costs and so set its price and produce the amount of output that it would were it an unregulated monopoly. Price cap regulation: The regulator sets a price ceiling. The firm can charge any price it wants below the price cap and keep some or all of any economic profit it earns. This regulation induces the firm to operate efficiently and control costs. If the firm earns a profit that is too high, the regulator might impose earnings share regulation, which require the firm to make refunds to customers when profits rise above a target level.

97

Topic: Cartel Regulation Skill: Conceptual

31) Do firms in a cartel support regulation that limits the amount each firm can produce? Explain your answer. Answer: Firms in a cartel definitely support regulation that limits the amount each firm can produce. The major problem faced by firms in a cartel is “cheating” by firms on the cartel agreement to limit production. Firms cheat by increasing their production beyond the assigned limit because each firm knows that if it, and it alone cheats, its profits will increase dramatically. Regulation that limits production can eliminate cheating and ensure that the cartel, and hence the firms within it, earn an economic profit. Topic: Social interest or Capture? Skill: Conceptual

32) Is there any evidence in support of the capture theory of regulation? Answer: Yes. In the airline and trucking industries, rates of return were much higher when they were regulated than after they were deregulated. After deregulation, the gain in consumer surplus in both industries was greater than the gain in producer surplus. Both of these pieces of evidence suggest that regulation of these industries served the interests of the producers, thereby supporting the capture theory. Topic: Sherman Act Skill: Conceptual

33) What is the Sherman Act and what is its purpose? Answer: The Sherman Act of 1890 was the first major piece of federal antitrust legislation. It prohibits two things. First, it prohibits any combination, trust, or conspiracy to restrict interstate or international trade. Second, it prohibits monopolization or any attempt to monopolize interstate or international trade. Topic: Sherman Act Skill: Conceptual

34) Does section 2 of the Sherman Act make it a felony to “attempt” to monopolize an industry or must the attempt succeed before it is a felony? Answer: Section 2 of the Sherman Act makes attempting to monopolize an industry a felony. It is not necessary for the attempt to succeed.

98

Topic: Clayton Act Skill: Recognition

35) “The Clayton Act repealed the Sherman Act so that only the Clayton Act remains in force.” Is the previous statement correct or incorrect? Answer: The statement is incorrect. The Sherman Act remains part of the law of the land. Topic: Clayton Act Skill: Recognition

36) What are the actions that are prohibited according to the Clayton Act and its amendments. What conditions must be met for these actions to be prohibited? Answer: The Clayton Act prohibits certain practices only if they substantially lessen competition or create monopoly. These practices are: 1) Price discrimination. 2) Tying arrangements. 4) Requirements contracts. 5) Exclusive dealing. 6) Territorial confinement. 7) Acquiring a competitor’s shares or assets. 8) Becoming a director of a competing firm. Topic: Clayton Act Skill: Conceptual

37) What is meant by the term “exclusive dealing”? Give an example of an exclusive deal. When is it illegal? Answer: Exclusive dealing is a contract that prevents a firm from selling competing items. For instance, Taco Bell has a contract with Pepsi that only Pepsi products will be sold at Taco Bell. Hence Pepsi has arranged an exclusive deal with Taco Bell. Exclusive deals are illegal under the Clayton Act only if they substantially lessen competition or create a monopoly. Topic: Clayton Act Skill: Conceptual

38) If Sony required all its retailers not to sell televisions from other companies, Sony would be engaging in what kind of activity? Is Sony’s requirement legal or does it violate the Clayton Act? Answer: Sony is engaged in an exclusive deal. The question of whether Sony’s requirement is legal depends on whether it substantially lessens competition or creates a monopoly. If it does either, it

CHAPTER 14

is illegal under the Clayton Act. If it does neither, it is legal under the Clayton Act. Topic: Price Fixing Skill: Recognition

39) Explain how the courts have ruled on price fixing. Answer: Price fixing is always a violation of the antitrust law. Price fixing, in and of itself, is a violation of the law. If the government can prove the existence of price fixing, the accused firms are guilty because there are no mitigating circumstance allowed. Topic: Price Fixing Skill: Conceptual

40) If price fixing is necessary because without it a firm will go bankrupt, is the price fixing legal? Answer: No, price fixing is always illegal. Regardless of whether a firm will go bankrupt or not, price fixing is illegal. Topic: Resale Price Maintenance Skill: Recognition

41) What is resale price maintenance? Is resale price maintenance legal in the United States? Answer: Resale price maintenance occurs when a manufacturer agrees with a distributor on the price at which the product will be resold. For instance, Sony could arrange with Best Buy the price for which Sony televisions are sold. Resale price maintenance (also called vertical price fixing) agreements are illegal under the Sherman Act. But it isn’t illegal for a manufacturer to refuse to supply a retailer who doesn’t accept guidance on what the price should be. Topic: Current Merger Rules Skill: Recognition

42) What are the current merger guidelines as developed and administered by the Federal Trade Commission? Answer: The current merger guidelines are based on the Herfindahl-Hirschman index, which is the sum of the squares of the market shares of the fifty largest firms in an industry. If the HHI is less than 1000, the market is considered unconcentrated and mergers will usually go unchallenged. If the HHI is greater than 1000 but less than 1800, the market is considered moderately concentrated and mergers may be challenged if the HHI would rise by more than 100 points. If the HHI exceeds

REGULATION AND ANTITRUST LAW

1800, the market is considered concentrated and mergers may be challenged if the HHI rises by more than 50 points. Topic: Current Merger Rules Skill: Conceptual

43) In a market with a Herfindahl-Hirschman Index of 2,000, according to their guidelines will the Department of Justice challenge a merger that would increase the index by 50? Answer: Yes, according to their guidelines the Department of Justice will challenge a merger that increases the Herfindahl-Hirschman Index by more than 50 points if the initial index is greater than 1,800. Topic: Current Merger Rules Skill: Conceptual

44) “If an industry’s Herfindahl-Hirschman Index is below 1,000, a merger between any two firms in

99

that industry will be disallowed.” Comment on the accuracy of the previous statement. Answer: The statement is incorrect in at least three dimensions. First, the lower the HerfindahlHirschman Index, the more competitive the industry and hence the more likely the government will allow a merger to occur. Second, even if the Herfindahl-Hirschman Index is high, a merger that increases it only a small bit will not be challenged. And third, the Herfindahl-Hirschman Index is only part of the information considered when the government is determining whether to challenge a merger.

CHAPTER 15

EXTERNALITIES

Topic: Externality Skill: Recognition

Topic: Sources of Environmental Problems Skill: Recognition

1)

4)

Explain the difference between a negative production externality and a negative consumption externality. Answer: A negative production externality occurs when a firm produces a good and the cost of the good falls on parties other than the firm. A negative consumption externality occurs a person consumes a good and that consumption imposes a cost on others.

How has air quality changed in the United States since 1980? Answer: Generally air quality in the United States has improved. With only a couple of exceptions (nitrogen dioxide and ozone) most other sources of pollution have decreased. Topic: Environmental Problems Skill: Recognition

Explain the difference between a positive production externality and a positive consumption externality. Answer: A positive production externality occurs when a firm produces a good and that production provides benefits to parties other than the firm. A positive consumption externality occurs when a person consumes a good and that consumption provides benefits to others who did not consume the good.

How do economists estimate the cost of pollution in dollars? Explain with a simple example. Answer: Economists use market prices to put a dollar value on the cost of pollution. For example, suppose there are two similar lakes, one polluted and the other clean. On each lake, there are 100 identical houses. The houses on the clean lake rent for $2,000 a month, and those on the polluted lake rent for $1,500 a month. If the pollution is the only significant difference between the two locations, the difference in rent is the cost of pollution. For the 100 houses this cost is $50,000 per month.

Topic: Environmental Problems Skill: Recognition

Topic: External Costs Skill: Recognition

3)

6) What is marginal external cost? Give an example. Answer: Marginal external cost is the additional cost that is imposed on someone other than the producer of the good or service. An example is the sulfur dioxide and other chemicals emitted by utility companies in the Midwest. These pollutants travel north with the winds, causing acid rain that harms vegetation and kills fish in the Northeast of the United States.

Topic: Externality Skill: Recognition

2)

What has happened to the demand for a pollution-free environment within recent years? Why? Answer: The demand for a pollution-free environment has increased for two main reasons. First, households’ incomes have increased and a clean environment is a normal good. Second, people have become more aware of the negative effects of pollution. For both counts the demand for a cleaner environment has increased over the past years.

5)

Topic: Marginal External Cost Skill: Recognition

7)

Discuss the difference between a private cost and a social cost. Answer: A private cost is the cost of producing a good or service that is borne by the producers. In cer-

102

CHAPTER 15

tain instances, however, some of the costs of production are borne by someone other than the producer. This cost is called an external cost. The social cost is the total cost borne by all of society. Thus the social cost equals the private cost, the costs borne by the producers, plus the external cost, the costs borne by everyone else. Topic: Marginal External Cost Skill: Recognition

8) What is marginal external cost of production? Answer: Marginal external cost of production is the cost of producing an additional unit of a good or service that falls on people other than the producer. Topic: Marginal External Cost Skill: Recognition

9)

“Marginal external cost is another term for the entire marginal cost incurred by the entire society.” True or false? Explain. Answer: The statement is false. The entire marginal cost incurred by the entire society is marginal social cost. Marginal external cost is the cost of producing or consuming an additional unit of a good or service that falls on people other than the producer or consumer. Topic: Marginal External Cost Skill: Conceptual

10) When a forest is logged, it is possible for the logging to create “soil runoff,” a situation in which the soil, no longer protected by trees, erodes and silts a river miles downstream from the logging area. Is soil runoff created by logging in Montana that ruins a river an example of a private cost to the lumbering company or an external cost? Answer: The soil runoff is an external cost. The lumber company does not pay the cost of the ruined river, so the cost is not a private cost to the company. Instead, the cost is borne by fishermen or other users of the river who can no longer use the river, or by society, which pays to clean up the river so that the fishermen and other users can utilize it once again.

Topic: Marginal External Cost Skill: Conceptual

11) Why do firms ignore external costs when they pollute? Answer: Firms have no incentive to incorporate external costs into their decision making. They do not bear the costs—the costs are external to them—and so these costs have no effect on their profits. Topic: Marginal External Cost Skill: Conceptual

12) Fully explain why a producer who is causing external costs does not have the incentive to reduce these costs. Answer: The answer lies in the fact that the costs are external. These are not costs borne by the producer. The producer responds to the costs he or she must pay and external costs are paid by someone else. Hence the producer has no incentive to decrease the activity that is creating the external cost because the activity costs the producer nothing. Topic: Marginal Social Cost Skill: Recognition

13) The marginal social cost of burning garbage in Houston is the sum of the marginal private cost and the marginal external cost.” Is this assertion correct or incorrect? Answer: The statement is correct. The marginal social cost of any activity is the sum of the marginal private cost plus the marginal external cost. Topic: Marginal Social Cost Skill: Recognition

14) If the production of a good creates an external cost, is the supply curve the same as the marginal social cost or the same as the marginal private cost curve or both? Answer: The supply curve is always the same as the marginal private cost curve. In the case of an external cost, however, the marginal private cost curve is not the same as the marginal social cost curve. In this case, the supply curve is the same as only the marginal private cost curve and is not the same as the marginal social cost curve.

EXTERNALITIES

103

Topic: An Unregulated Market Skill: Conceptual

Topic: An Unregulated Market Skill: Conceptual

15) Why does an external cost lead to inefficient overproduction? Answer: If there is an external cost, then the market supply curve represents only the private costs rather than the total costs of production. Private producers respond only to the costs that they pay. In the case of an external cost, because producers are not paying all of the costs, they produce too much of the good from the social perspective, that is, there is overproduction. Thus government policies, such as taxes, emission charges, or marketable permits, that mean the producers pay all the costs of their production move the market toward efficiency.

18) “If the market for aluminum is perfectly competitive and aluminum factories emit pollution, the marginal social cost of aluminum is less than the market price.” True or false? Explain. Answer: The statement is false. Pollution is an external cost, and marginal social cost equals marginal private cost plus marginal external cost. In an unregulated competitive market, the market price equals marginal private cost. As a result, the marginal social cost of aluminum is its market price, which equals its marginal cost, plus the marginal external cost of pollution. So the marginal social cost of aluminum is higher than the market price.

Topic: An Unregulated Market Skill: Conceptual

16) If the production of a good causes pollution (an external cost) is the unregulated competitive market equilibrium of that product efficient? Answer: The presence of pollution means that the supply curve of the firm, which is also the firm’s marginal cost curve, only represents the firm’s private costs and ignores all external costs. The equilibrium between supply and demand curves leads to overproduction of this good. The amount produced, therefore, is not efficient because the level of production occurs where the marginal private cost equals the marginal social benefit rather than where the marginal social cost equals the marginal social benefit. Topic: An Unregulated Market Skill: Conceptual

17) Burning coal to generate electricity can create pollution. If the market for generating electricity is competitive and is allowed to operate without any government intervention, is the equilibrium quantity of coal burned equal to, more than, or less than the efficient quantity? Answer: The equilibrium quantity of coal burned is more than the efficient quantity. In a market for a good with an external cost, an unregulated competitive market will have an equilibrium quantity that exceeds the efficient quantity.

Topic: Efficiency Skill: Conceptual

19) “If production of a good creates an external cost, then, when production is such that the marginal private costs are equal to the marginal private benefits, the market outcome will be inefficient.” Explain whether this assertion is correct or incorrect. Answer: The assertion is correct. Efficiency requires that the marginal social benefit equal the marginal social cost. If the production of a good creates an external cost, then the marginal private cost does not equal the marginal social cost. So, even if the marginal private benefit equals the marginal social benefit, setting the marginal private cost equal to the marginal private benefit creates inefficiency because the marginal social benefit does not equal the marginal social cost. Topic: Efficiency Skill: Conceptual

20) Why is it not efficient to eliminate all pollution? Answer: Eliminating pollution is a task that uses resources, so there is an opportunity cost. If the marginal social cost of eliminating another unit of pollution is greater than the marginal social benefit, it would not be an efficient use of resources to eliminate the pollution because there are other activities with a higher value to society. In other words, pollution elimination is similar to any other good: pollution should be eliminated until the marginal social benefit of pollution reduction equals the marginal social cost of pollution reduction.

104

CHAPTER 15

Topic: Property Rights Skill: Recognition

Topic: Coase Theorem Skill: Conceptual

21) What do we mean by “property rights” and why are they important? Answer: Property rights are legally established titles to ownership, use, and disposal of factors of production and goods and services that are enforceable in the courts. Property rights are important because ownership of a resource provides a strong incentive to its owner to see that the resource they own is not damaged, overused, or degraded.

24) “According to the Coase theorem, if Gabriel wants the local television station to cease having helicopters fly over his house at night, he will be more likely to be able to reach an agreement with the station if the property right to the airspace is clearly defined and the transaction costs of negotiating are high.” Is this statement true or false? Explain your answer. Answer: The statement is false. The first part of the statement is correct, insofar as the property right to the airspace needs to be clearly defined. However, the second part is incorrect. In particular, if the costs of negotiating are high, Gabriel and the television station are less likely to be able to come to an agreement.

Topic: Coase Theorem Skill: Recognition

22) What is the Coase Theorem? Answer: The Coase Theorem states that if property rights exist, if the number of parties involved is small, and transaction costs are low, then private transactions are efficient; the market can efficiently deal with externality problems. Topic: Coase Theorem Skill: Recognition

23) What is the Coase theorem? What conditions need to be present for this theorem to work? Answer: The Coase theorem states that if property rights over a resource are clearly defined and are enforceable, then private transactions are efficient and the outcome is not affected by who is assigned the property right. The conditions that need to be present for the Coase theorem to work are: ♦ property rights are clearly defined; ♦ the transactions costs of the negotiations must be small, eg, there are a small number of parities involved. If these conditions are met, then the Coase theorem applies and there are no externalities.

Topic: Coase Theorem Skill: Conceptual

25) Using the Coase Theorem, why might a firm that currently pollutes a river no one owns pollute the river less if it owned the river? Answer: The firm would pollute less if it owned the river because its neighbors could compensate the firm if it limits its pollution. If the firm pollutes beyond this limit, it would lose that payment from neighbors and, thereby, incur the cost of polluting. Topic: Government Policies Toward External Costs Skill: Conceptual

26) Use the idea of external costs to explain why some cities have laws against late-night rock concerts. Answer: While many people might like to see a midnight show featuring AC/DC or Creed, many others argue that a rock concert blaring away early in the morning imposes external costs on many nearby residents who do not want to be disturbed late at night. Therefore regulations on the time that music can be “emitted” are common. To the extent that the regulations are motivated by the externality, such regulations increase efficiency.

EXTERNALITIES

Topic: Government Policies Toward Pollution Skill: Recognition

27) What three tools can governments employ to limit pollution? Answer: Three tools that governments employ to limit pollution are: emission charges; marketable permits; and taxes. Topic: Government Policies Toward Pollution Skill: Recognition

28) List and briefly define the three methods government can use to cope with an external cost, such as pollution. Answer: The three methods are emission charges, marketable permits, and taxes. Emission charges set a price per unit of pollution that the firm pays to the government. Marketable permits are permits given to firms that allow them to pollute by an amount assigned by the permit. The permits can be bought and sold among firms. Taxes can be imposed on polluting firms. If this tax equals the marginal external cost, efficiency can be attained. Topic: Pigovian Tax Skill: Conceptual

29) The production of paper creates pollution, an external cost. What happens to the production of paper if the government imposes a tax on paper producers equal to the marginal external cost of the pollution? Answer: When the government imposes a tax equal to the marginal external cost of the pollution, the production of paper decreases. Prior to the tax being imposed, the external cost was ignored by the producers because they did not pay this cost. However, once the tax is in place, the external cost now becomes a cost that the producer must pay. As a result, the producers’ costs increase and so the supply of paper decreases. Topic: Pigovian Tax Skill: Conceptual

30) Explain how a Pigovian tax works. Answer: A Pigovian tax is a method of dealing with externalities. The government sets the tax rate equal to the marginal external cost, making polluting firms behave in the same way as they would if they bore the cost of the externality directly because their costs are the same as if they paid for

105

the pollution directly. In this case, the amount of pollution is reduced to the efficient level. Topic: Marketable Permits Skill: Conceptual

31) What are marketable permits? Suppose there are two firms in an area, each emitting tons of sulfur. The government decides on a target level of 200 tons of sulfur, and gives each firm a permit to emit 100 tons of sulfur. Suppose Firm A is very efficient and can reduce pollution by 100 tons with an abatement cost of $500. Firm B has an older plant, so it will cost Firm B $1,000 to reduce emissions by 100 tons. What will occur with marketable permits? Answer: Marketable permits are a government issued permit that allows a firm to emit a certain amount of pollution. Firms are allowed to buy and sell these permits. Firm A can reduce pollution by 100 tons with an abatement cost of $500, while it costs Firm B $1,000 for the same reduction. Marketable permits can be bought and sold. Hence Firm A can reduce its emissions at the cost of $500 and sell its permit to Firm B for some price higher than $500. Firm B has an incentive to buy the permit for any price less than $1,000 rather than reduce its emissions at the cost of $1,000. Hence with marketable permits, Firm A will sell its permit to Firm B. Topic: Marginal Private Benefit and Marginal External Benefit Skill: Conceptual

32) Explain the difference between marginal social benefit and marginal external benefit. Answer: Marginal social benefit equals the marginal benefit enjoyed by society (by consumers of the good plus others). It equals marginal benefit plus marginal external benefit. Marginal external benefit is the benefit from an additional unit of a good that is enjoyed by someone other than the consumer. Topic: Marginal External Benefit Skill: Conceptual

33) Does inoculation against chicken pox have both private and external benefits? Answer: Yes, inoculation against chicken pox has both private and external benefits. The individual who is vaccinated benefits because he or she will

106

CHAPTER 15

not catch chicken pox. In addition, everyone who interacts with that person will benefit because they will not catch chicken pox from him or her. Topic: Marginal External Benefit Skill: Conceptual

34) “Education in elementary and high schools has external benefits because families who have a lot of children do not pay any more than families without children.” Is this statement correct or incorrect? Explain your answer. Answer: The statement is incorrect. Education has external benefits because educated people create benefits for everyone else. Who pays for the education is not an external benefit. Topic: Marginal External Benefit Skill: Conceptual

35) Describe some of the external benefits associated with education. What can government do to encourage production of the efficient amount of education? Answer: External benefits to education include the facts that college graduates: ♦ communicate more effectively with others, ♦ tend to be better citizens, ♦ have lower crime rates, ♦ are more tolerant of others’ views. To insure that the efficient amount of education is produced, the government could: ♦ provide education, as it does with public schools, ♦ subsidize education, as it does with state universities, or ♦ give consumers vouchers to reduce the cost of private schooling, as is happening in some states or with various college grant programs. Topic: Marginal External Benefit Skill: Conceptual

36) Does the existence of the University of Oklahoma affect citizens who do not attend the University? Answer: Education creates an external benefit. Hence people who do not attend the University of Oklahoma benefit from the education received by its students because this education has external benefits. For instance, the presence of hundreds of thousands of educated graduates from the University of Oklahoma raises public discourse on certain topics and makes the provision of high qual-

ity social activities, such as live theater, more common. Topic: Inefficiency Skill: Conceptual

37) “External benefits lead to overproduction so that more than the efficient quantity is produced.” Is the previous statement true or false? Answer: The statement is false. External benefits lead to underproduction so that less than the efficient quantity is produced. Topic: Inefficiency Skill: Conceptual

38) Why does the existence of an external benefit lead to the production of less than the efficient quantity? Answer: Buyers ignore the presence of an external benefit because the benefit they receive is the private benefit. As a result, buyers do not take account of all the benefits from a good or service. Because buyers do not take account of all the benefits, their demand for the good or service, which reflects their private benefit, is less than the marginal social benefit, is less than the marginal social benefit. So, with the demand being less than the marginal social benefit, the equilibrium quantity is less than the efficient quantity. Topic: Inefficiency Skill: Conceptual

39) Why would the amount of education provided be inefficient if subsidies to education were not provided? Answer: Private decision makers consider only private benefits and costs when purchasing education. As a result, the value that all of society places on the last unit of education purchased exceeds the cost of that education. This is not efficient because more education would generate social benefits that exceed social costs. Subsidies can overcome this problem by reducing the private cost of education, thereby leading to a greater quantity of education being provided. Topic: Inefficiency Skill: Conceptual

40) Why should people without children pay some tax to support local schools? Answer: Education generates an external benefit. Society as a whole, including people without chil-

EXTERNALITIES

dren, benefits from having better educated citizens. Such a tax provides the funding for a subsidy that helps achieve the efficient amount of education. Topic: Government Policies Skill: Recognition

41) What four devices can governments use to achieve a more efficient allocation of resources in the presence of external benefits? Answer: The government can use: (1) public provision, (2) private subsidies, (3) vouchers, and (4) patents and copyrights. Topic: Government Policies Skill: Conceptual

42) Why does government provide educational opportunities in the form of vouchers, subsidies, and public provision? Answer: Education generates external benefits. With a better educated population, everyone is better off. The more educated the population, the more productive it is. Educated individuals need less social services than uneducated individuals. Crime rates are lower the more educated the population. And a more educated population makes it more likely to provide other goods with external benefits, such as high quality newspapers. But because part of the social benefit from education is an external benefit, an unregulated market will create less than the efficient amount of education. Hence government policies such as vouchers, subsidies, and public provision are attempts to overcome inefficiency and move the market to the efficient amount of education. Topic: Subsidies Skill: Conceptual

43) “A government subsidy can increase the production of a product whereas a tax can decrease the production of a product. Hence a government subsidy is the appropriate policy to use with a product that has an external cost and a tax is the appropriate policy to use with a product that has an external benefit.” Comment on the accuracy of the previous two sentences. Answer: The first sentence is correct but the second sentence is incorrect. For a product with an external cost, overproduction, that is, producing more than the efficient quantity is the problem. A tax

107

could be imposed in order to decrease the quantity toward the efficient amount. Conversely, for a product with an external benefit, underproduction, that is, producing less than the efficient quantity is the problem. For this situation, a subsidy could be granted in order to boost production toward the efficient level. Topic: Subsidies Skill: Conceptual

44) A private subsidy has what effect on the amount of a good or service produced? Is a subsidy an appropriate policy to offset the inefficiency from an external cost or an external benefit? Answer: A private subsidy increases the production of the good or service that is subsidized. Because it increases the production, a private subsidy is the appropriate policy to overcome the inefficiency that is the result of an external benefit. (A good or service with an external benefit is underproduced by a competitive, unregulated market.) Topic: Subsidies Skill: Conceptual

45) Explain the process by which a private subsidy corrects an external benefit. Answer: If a good or service has an external benefit, an unregulated competitive market will produce less than the efficient quantity. A private subsidy, which is money given to firms, reduces the cost of production. With the subsidy, firms are willing to produce more at any given price. If the subsidy equals the marginal external benefit, production with the subsidy will be at the efficient level. Topic: Patents and Copyrights Skill: Conceptual

46) From the perspective of society, what are the advantages and disadvantages of a patent or a copyright? Answer: The advantage of patents and copyrights is that they encourage invention and innovation. The disadvantage is that the holder of the patent or copyright has a monopoly. This creates inefficiency because a monopoly has a incentive to produce less and charge more for the good or service.

CHAPTER 16

PUBLIC GOODS AND COMMON RESOURCES are rival. Public goods are nonrival. Common resources are rival.

Topic: Nonexcludable Skill: Recognition

1)

What does it mean for a good to be “nonexcludable”? Are private goods nonexcludable? Are public goods nonexcludable? Are common resources nonexcludable? Answer: A good is nonexcludable if it is technologically impossible to prevent someone from consuming it even if they have not paid for it. Private goods are excludable, not nonexcludable. Public goods are nonexcludable. Common resources are nonexcludable. Topic: Nonexcludable Skill: Recognition

2) What is a nonexcludable good? Give an example. Answer: A good is nonexcludable if it is impossible (or extremely costly) to prevent someone from benefiting from it. An example of a nonexcludable good is fish in the ocean. Topic: Nonrival Skill: Recognition

3) What is a nonrival good? Give an example. Answer: A good is nonrival if its use by one person does not decrease the quantity available for someone else. An example of a nonrival good is the Internet. Topic: Nonrival Skill: Recognition

4)

When describing goods and services, what is meant by the terms “rival” and “nonrival?” Are private goods rival or nonrival? Are public goods rival or nonrival? Are common resources rival or nonrival? Answer: A good is rival if its use by one person decreases its consumption by other people. A good is nonrival if its use by one person does not decrease its consumption by other people. Private goods

Topic: Public Goods Skill: Conceptual

5)

The Wall Street Journal (February 2001) reported that the government of Thailand “plans to launch a chain of more than 3,000 Thai restaurants world-wide over the next five years, with the largest number, more than 1,000, slated for the United States.” The Thai government will have a 30 percent minority stake in the restaurants and the rest will be given to Thai owners. The country’s deputy commerce minister explains that the government will play an active role in drawing up menus, making sure that genuine Thai food is served and ensuring that 70 percent of supplies for the restaurants are imported from Thailand. Because the Thai government will be part owner of these restaurants, are these restaurants public goods? Answer: Although the government of Thailand has a minority stake in these restaurants, these restaurants are both rival and excludable. Hence these restaurants are like all restaurants, specifically, they are private goods. In defining a public good, the source of financing is irrelevant. To be a public good, the good must be nonrival and nonexcludable and a restaurant is neither. Topic: Public Goods Skill: Conceptual

6)

Is a sailboat purchased in Victoria, British Columbia, a private good or a public good? Answer: The sailboat is a private good. The sailboat has none of the characteristics of a public good. First, people can easily be excluded from using the boat. Basically, the only people allowed on the boat are the people the owner allows on the boat. Second, the owner’s use of the boat prevents other people from simultaneously using the boat. That

110

CHAPTER 16

is, the owner’s consumption of the boat decreases the consumption by other consumers. Topic: Public Goods Skill: Conceptual

7)

What are the differences between public goods and private goods? Answer: Public goods are ♦ nonrival, which means that consumption by one person doesn’t decrease the quantity available for others’ consumption, and ♦ nonexcludable, so that it is impossible or very costly to prevent people from consuming the good even if they did not pay for the good. In contrast, private goods are ♦ rival, so that consumption by one person decreases the amount available for consumption by another person, and ♦ excludable, which means that it is possible to prevent someone from enjoying the benefits of consuming the good unless they pay for the good. Topic: Public Goods Skill: Conceptual

8)

You are studying with a friend and your friend says, “private goods are excludable and nonrival, while public goods are nonexcludable and rival.” Do you agree? Answer: No, private goods are excludable and rival, while public goods are nonexcludable and nonrival. Contrary to what your friend asserted, it is common resources rather than public goods that are nonexcludable and rival. Topic: Common Resource Skill: Recognition

9)

Some resources are private and others are common. Define a common resource, explain the definition, and provide an example of a private resource and a common resource. Answer: A common resource is rival and nonexcludable. Because the resource is rival, if one person uses some of the resource, the person decreases the use of the resource by other people. And because the resource is nonexcludable, it is not possible to prevent someone from using the resource. An example of a private resource is a gold mine: This resource is rival but is excludable. It is rival because one persons’ use of the gold decreases the consumption of other people. And it is excludable

because the owner of the gold mine can keep anyone out of the mine that he or she wants to keep out of the mine. An example of a common resource is fish in the ocean. This resource is rival and nonexcludable. It is rival because one person’s catch of a fish decreases the catch of other people. And it is nonexcludable because a fisherman cannot keep anyone from fishing in the river, lake, or ocean. Topic: Natural Monopoly Skill: Conceptual

10) “The Internet is nonrival, which means it’s a public good.” Do you agree or disagree? Explain. Answer: The statement is incorrect. A public good is both nonrival and nonexcludable. Although the Internet is nonrival, it is excludable, which means that the Internet is a natural monopoly, rather than a public good. Topic: The Free-Rider Problem Skill: Recognition

11) What is the free rider problem with respect to public goods? Answer: Because public goods, such as national defense, are nonexcludable, a person can enjoy its benefits without paying for it. Such a person is known as a free rider. Because all individuals have the incentive to be free riders, a private, unregulated market will produce less than the efficient quantity of a public good because a private firm providing the good cannot collect from free riders. Topic: The Free-Rider Problem Skill: Recognition

12) What is the free-rider problem? What results from the free-rider problem? What is a solution to the free-rider problem? Answer: The free-rider problem occurs when people enjoy the benefits from a good or service without paying for it. The free-rider problem occurs with public goods because no one can be excluded from using them without paying for them. As a result, the free-rider problem is that the private market produces too little of these goods. To produce the efficient quantity, the government can intervene by taxing people and using those funds to pay for the public good.

PUBLIC GOODS AND COMMON RESOURCES

111

Topic: Benefit of a Public Good Skill: Conceptual

Topic: Cost of a Public Good Skill: Conceptual

13) What is the marginal benefit of a public good and how is it different from the marginal benefit of a private good? Answer: The marginal benefit of a public good is the maximum amount that all the people together are willing to pay for one more unit of it, that is, the sum of individual marginal benefits from the good. The marginal benefit of a private good is the maximum amount that a person is willing to pay for one more unit, that is, an individual marginal benefit from the good.

16) “The principle of increasing marginal cost does not apply to public goods.” Is this statement correct or not? Answer: The statement is false. For both public goods and private goods, the marginal cost increases as more of the good is produced.

Topic: Benefit of a Public Good Skill: Conceptual

14) “To find the economy’s marginal benefit curve for a public good, we sum the individual marginal benefit curves horizontally.” True or false? Explain. Answer: The statement is false. To find the economy’s marginal benefit curve of a public good, we sum the individual marginal benefit curves vertically because we need to obtain the sum of what everyone together is willing to pay for each unit of a public good. Topic: Benefit of a Public Good Skill: Conceptual

15) Why is society’s marginal benefit curve for a public good found by vertically summing individual marginal benefit curves and not by horizontal summation as is the case for private goods? Answer: In the case of a private good, producers must supply one different item to all individuals who demand one; that is, if 10,000 pizzas are demanded, 10,000 must be produced. Horizontal summation tells us how much in total is demanded at a certain price. Because public goods are nonrival, producers need only supply one unit in total to all individuals who demand one. Vertical summation tells us how much society is willing to pay for each unit produced.

Topic: Private Provision of a Public Good Skill: Conceptual

17) Why isn’t national defense provided by free markets? Answer: National defense is a public good. This fact means that it is hard to exclude anyone from consuming the benefits of national defense. If left to the free market, it would be very difficult for private companies to charge people for protection. People would be unwilling to pay for the service because they can get it without paying. The freerider problem would make the ability to earn a profit a difficult or impossible prospect and hence little or no defense would be provided. Topic: Efficient Quantity of a Public Good Skill: Conceptual

18) Protection of rivers in Idaho is a public good. If the marginal cost of protecting rivers in Idaho exceeds the marginal benefit of river protection, is there more than, less than, or the efficient amount of river protection taking place? Answer: If the marginal cost exceeds the marginal benefit, there is more than the efficient amount of protection taking place. The efficient amount is the quantity such that the marginal benefit equals the marginal cost. Any other amount creates a deadweight loss. Topic: Efficient Quantity of a Public Good Skill: Conceptual

19) How is the efficient quantity of public goods determined? Answer: The efficient quantity of a public good is determined in the same way as the efficient quantity of any good: by comparing marginal cost and marginal benefit, and producing where one equals the other. The marginal cost of a public good is calculated similarly as the marginal cost for a private good. It is based on the cost of producing one more unit of the good. The marginal benefit, however, is calculated differently for a public good

112

CHAPTER 16

than for a private good. For a public good, the economy’s marginal benefit is determined by summing the individual marginal benefits at each quantity of the public good. Once the marginal cost and marginal benefits are known, the efficient quantity is equal to the quantity such that the marginal cost equals the marginal benefit. Topic: Principle of Minimum Differentiation Skill: Recognition

20) What is the principle of minimum differentiation? Answer: The principle of minimum differentiation is the tendency for competitors to make themselves identical to appeal to the maximum number of clients or voters. This principle describes the behavior of political parties as well as that of business firms. Topic: Principle of Minimum Differentiation Skill: Conceptual

21) How does the principle of minimum differentiation relate to the free-rider problem? Answer: The free-rider problem requires that the government intervene so that an efficient level of a public good is produced. Often the government itself provides the public good. In this case, politicians are the government officials that are responsible for deciding which and what amounts of public goods are produced. In order to collect the most votes, politicians choose policies proposing the provision of certain public goods that are similar to competing politicians. The tendency for competitors to choose similar policies is the principle of minimum differentiation. In this case, the principle of minimum differentiation means that political parties propose to produce similar quantities of public goods because these proposals attract the maximum number of votes. Topic: Rational Ignorance Skill: Conceptual

22) What is rational ignorance? Answer: Rational ignorance is the voter’s decision not to acquire information about an issue because the marginal cost of doing so exceeds the expected marginal benefit. Thus the voter remains (rationally) ignorant about the issue.

Topic: Rational Ignorance Skill: Recognition

23) Why is it rational for individuals to be ignorant about public goods but not about private goods? Answer: In the case of a private good, the individual enjoys all of the benefits and pays all of the costs so it is important to gather product information. In the case of a public good, the individual receives only a fraction of the benefits and pays only a fraction of the costs. Moreover, it is uncertain whether one individual’s gathering information will have any impact on the public goods provided anyway. Topic: Rational Ignorance Skill: Conceptual

24) Explain why some amount of ignorance can be rational. Answer: Acquiring information is not costless. The rational voter will acquire information only if the expected benefit of the information exceeds the cost of acquiring the information. Often the expected benefit is less than the cost, so the person is “rationally ignorant” about some issues. Topic: Rational Ignorance Skill: Conceptual

25) Why is watching sports more important to many prospective voters than watching the news? In your answer, mention the role played by rational ignorance and the cost of gathering information. Answer: It takes a great deal of time to learn about all of the issues that affect our lives. The marginal cost of information gathering can be very high for many people, especially when we are very busy with work, school, personal matters, and so forth. Quite likely the cost of gathering information about sports is less than that of politics. Most assuredly, it is easy to determine the key elements of sports (e.g., who won, who lost, standings, rankings, and so on). It is much more difficult to gather the relevant information about all the details of various policy proposals, plus it is significantly more likely that the proposals will be changed than is the likelihood that the outcome of an already played game will be changed. Thus people are more likely to remain rationally ignorant of political events while staying informed about sporting events.

PUBLIC GOODS AND COMMON RESOURCES

Topic: Inefficient Overprovision Skill: Conceptual

26) Under what conditions will the political process provide an inefficient amount of a public good? Answer: The government will provide an inefficient amount of a public good when voters are rationally ignorant of the marginal benefits and marginal costs of the good. In this case, bureaucrats, intent on maximizing the size of their budgets, will provide an inefficiently large quantity of the good. Topic: Problem of the Commons Skill: Recognition

27) What is the problem of the commons? Answer: The problem of the commons is the absence of incentives to prevent the overuse and depletion of a commonly owned resource like fish in the oceans and open pasture land upon which anyone can graze sheep. A common resource is rival, so that one person’s use of the resource decreases other people’s use, but nonexcludable, so no one can be prevented from using the common resource. Topic: Problem of the Commons Skill: Recognition

28) “The problem with a common resource is that no one gets to use the resource.” Comment on the preceding assertion. Answer: The assertion is totally false; the problem with a common resource is that too many people use the resource so that it is overused and depleted. A common resource is rival, so that one person’s use of the resource decreases other people’s use, but nonexcludable, so that no one can be prevented from using the common resource. As a result, everyone has the incentive to use the resource and no one has the incentive to conserve the resource. Topic: Problem of the Commons Skill: Conceptual

29) Why are fish in the ocean an example of a resource that suffers from the problem of the commons but cattle grazing in a farmer’s pasture do not suffer from the problem of the commons? Answer: Fish in the ocean are a common resource because they are rival, so that one person catching a fish prevents others from catching the same fish, and they are nonexcludable, so that no one can be

113

prevented from fishing. As a result, over-fishing is likely to occur. But cattle grazing in a farmer’s pasture are not a common resource. The cattle are rival, so that one person butchering a steer prevents others from butchering the same steer, but the cattle are excludable, so that no one can butcher a steer unless the cattle’s owner is compensated. As a result, the owner has the incentive to conserve the resource, that is, the owner has the incentive to insure that the cattle are not overused. Topic: Marginal Private Benefit Versus Marginal Social Benefit Skill: Recognition

30) How does the marginal social benefit curve of a common resource compare to the marginal private benefit curve of the common resource? If there is a difference, why is there a difference? If there is not a difference, why is there not a difference? Answer: The marginal social benefit curve of a common resource lies below the marginal private benefit curve. The marginal private benefit from a common resource takes account of only the benefit accruing to the user of the common resource. But when an additional person uses a common resource, the additional person decreases the marginal benefit of other users. The marginal social benefit from a common resource adjusts the marginal private by subtracting the decrease in the other users’ marginal benefits. So, the marginal social benefit from use of a common resource is less than the marginal private benefit. Topic: Quota Skill: Conceptual

31) How does the government determine the quota amount that will produce an efficient use of a common resource? Answer: The government must determine the marginal social benefit and the marginal (social) cost of the common resource. By equating marginal social benefit to marginal cost, the efficient use of a common resource is determined. A quota is set to this level of production. Producers are assigned a portion of this quota. As long as no one cheats on his or her quota, the efficient outcome occurs.

114

CHAPTER 16

Topic: Individual Transferable Quota Skill: Recognition

Topic: Individual Transferable Quota Skill: Recognition

32) What are ITQs? Where are they used? Answer: An ITQ, or individual transferable quota, is a production limit that is assigned to an individual who is free to transfer the quota to someone else. ITQs are used to deal with the problem of commons where producers are hard to monitor and where marginal costs differ across producers.

34) If the number of ITQs issued equals the efficient production level, what is the price of an ITQ? Answer: The price of an ITQ equals the marginal private benefit at the quota quantity minus the private marginal cost.

Topic: Individual Transferable Quota Skill: Recognition

33) What is an individual transferable quota (ITQ)? Answer: An ITQ is a production limit that is assigned to an individual who is free to transfer the quota to someone else. An ITQ is used in a market where there are common resources that might otherwise be overused.

CHAPTER 17

DEMAND AND SUPPLY IN FACTOR MARKETS

Topic: Derived Demand Skill: Recognition

Topic: Marginal Revenue Product Skill: Conceptual

1) What is a derived demand? Answer: The demand for factors of production is called a derived demand because its is derived from the demand for goods and services.

4)

Topic: Derived Demand Skill: Conceptual

2) Why is the demand for labor a “derived demand?” Answer: The demand for labor arises because we need to use labor to produce a good or service. The demand for a good, for example an apple, is a direct demand. However, the demand for an apple picker is a derived demand. It is derived from our need to use labor to produce the good we demand, the apple. Topic: Marginal Revenue Product Skill: Conceptual

3)

What is the marginal revenue product of labor? What is the formula that can be used to calculate? How does the marginal revenue product affect how much labor a firm hires? Answer: The marginal revenue product of labor tells the additional amount of revenue the firm earns by hiring an extra unit of labor, so it is the value to a firm of hiring one more unit of labor. The marginal revenue product equals the marginal revenue of a unit of output multiplied by the marginal product of labor. The firm maximizes its profit by hiring up to the point where the marginal revenue product equals the wage rate.

“As the quantity of labor hired increases, the marginal revenue product stays constant as long as the wage rate is constant for all workers.” Is the previous statement correct or incorrect? Explain your answer. Answer: The statement is incorrect. The marginal revenue product equals the marginal revenue of a unit of output multiplied by the marginal product. As more workers are employed, the marginal product decreases; and as more of a good is produced, except for a firm in perfect competition, the marginal revenue decreases. So on both counts the marginal revenue product decreases. Topic: Marginal Revenue Product Skill: Conceptual

5)

“The decision to employ an additional unit of labor depends on whether the marginal revenue product is greater than or smaller than the wage rate.” Is the previous statement correct or incorrect? Answer: The statement is correct. If the marginal revenue product exceeds the wage rate, the worker is hired; if the marginal revenue product is less than the wage rate, the worker is not hired. Topic: Marginal Revenue Product and the Demand for Labor Skill: Conceptual

6)

What is the relationship between a firm’s marginal revenue product curve for labor and its demand for labor curve? Explain why this relationship exists. Answer: The marginal revenue product curve for labor is the same as the demand for labor curve. The curves are the same because of the firm’s profit-maximizing decisions. In order to maximize its profit, a firm hires the number of workers that sets the wage rate equal to the marginal revenue

116

CHAPTER 17

product. So at any wage rate, the firm uses its marginal revenue product curve to determine how many workers to hire. But that is what the demand for labor curve shows: how many workers the firm hires at any wage rate. As a result, the marginal revenue product for labor is the same as the firm’s demand for labor curve. Topic: Profit Maximization Skill: Recognition

7)

What are two conditions for profit maximization? How are they related? Answer: Profit is maximized when, at the quantity of labor hired, marginal revenue product equals the wage rate and when, at the output produced, marginal revenue equals marginal cost. These two requirements are basically different sides of the same coin. When the first condition, hire the quantity that sets the marginal revenue product equal to the wage rate, is met, the second condition, produce so that marginal revenue equals marginal cost, also is met. And when the second condition is met, the first condition also is met. Topic: Profit Maximization Skill: Conceptual

8)

Why does a firm maximize its profits by hiring so that MRP = W? Answer: The MRP represents the additional revenue generated by the last worker. The wage is the cost of the last worker. If MRP > W, then an additional worker adds more to revenue than to cost and so hiring the worker boosts the firm’s profit. Conversely, if MRP < W, then decreasing labor reduces costs more than it reduces revenue and thereby decreasing the quantity of labor it employs raises the firm’s profit. Only when MRP = W can no other level of labor create a greater profit. Topic: Profit Maximization Skill: Conceptual

9)

“In order for Charlie Trotter’s, an upscale restaurant in Chicago, to maximize profit from the employment of chefs, the restaurant should hire chefs up to the point where the marginal revenue product equals the wage rate for chefs.” Is the statement correct or incorrect? Answer: The statement is correct. In order for any firm to maximize its profit, it hires workers until

the wage rate equals the marginal revenue product. Topic: Profit Maximization Skill: Conceptual

10) Why does a profit-maximizing firm hire labor up to the point where the marginal revenue product equals the wage rate? Answer: If a company stopped adding workers at the point where the wage rate was less than the marginal revenue product of labor, the firm could hire more workers and its profit would increase. Why? Because the return from the workers, the marginal revenue product, exceeds the cost of hiring the workers. Since the marginal product of labor decreases as more workers are employed, as more workers are added, the marginal product of labor and hence the marginal revenue product decreases. Eventually the firm will reach the point at which the marginal revenue product equals the wage rate. If still more workers are employed, then the wage the firm must pay the workers exceeds the marginal revenue product. These workers would contribute losses to the firm. So only when the marginal revenue product equals the wage rate can the firm not increase its profit by changing the quantity of workers it employs. Topic: Changes in the Demand for Labor Skill: Conceptual

11) What factors other than the wage rate influence the demand for labor? How is demand affected by changes in these factors? Answer: The demand for labor is influenced by changes in the price of the firm’s output, the prices of other factors of production, and technology. The demand for labor is based on the marginal revenue product, which is equal to the marginal product times the price. When the price of the firm’s output increases, the marginal revenue product increases, making hiring more workers profitable. So when the price of the firm’s output increases, so does its demand for labor. The prices of other factors of production also influence the demand for labor. The different factors of production can be substitutes for each other. For instance, if the price of capital falls relative to the wage rate, a firm will substitute capital for labor, which will decrease the demand for la-

DEMAND AND SUPPLY IN FACTOR MARKETS

bor. However, if the cheaper capital results in a large increase in production, it is possible that the demand for labor will increase. Technology can either increase or decrease the demand for labor, depending on whether the technology allows firms to replace workers with capital (which decreases the demand for labor) or makes the labor more productive (which increases the demand for labor). Topic: Changes in the Demand for Labor Skill: Conceptual

12) What factors shift the demand for labor curve? Briefly describe the effect of each. Answer: Three factors shift the demand for labor curve: the price of the firm’s output, the prices of other factors of production, and technology. When the price of the firm’s output rises, the firms’ demand for labor increases and its demand for labor curve shifts rightward. These changes occur because when the price of output rises, the marginal revenue product increases, which means the value to the firm of hiring additional workers rises. When the prices of other factors of production change, the demand for labor changes. When the price of capital rises, the firm is motivated to use more labor and less capital. The firm’s demand for labor increases and the demand for labor curve shifts rightward. When the price of capital falls, more capital is demanded and, unless the fall in the price of capital increases the scale of the firm’s operations, less labor is demanded. The demand for labor curve shifts leftward. Changes in technology affect the demand for labor in a more complex way. Depending on the type of advance in technology, the demand for different types of labor might increase or decrease. If the demand for a type of labor increases, its demand curve shifts rightward; if the demand for another type of labor decreases, its demand curve shifts leftward.

117

Topic: Changes in the Demand for Labor; Price of the Product Skill: Conceptual

13) What effect does an increase in the price and marginal revenue of the output the firm produces have on its demand curve for labor? Why? Answer: An increase in the price and marginal revenue of its output shifts the labor demand curve rightward. This shift occurs because the value of the additional output produced by each unit of labor has become more valuable as a result of the price and marginal revenue increase. Topic: Labor Supply Curve Skill: Conceptual

14) Why does the labor supply curve eventually bend backwards? Answer: The wage rate represents the opportunity cost of leisure. Starting from a relatively low wage, as the wage increases, leisure becomes more expensive and so workers substitute labor for leisure. As a result, a higher wage means less leisure and more labor and the supply curve is positively sloped. At a sufficiently high wage, however, the worker’s income is high enough that, even though leisure has gone up in price, the worker can afford to “buy” more leisure by working less. So, at a high wage, the labor supply curve becomes negatively sloped. Topic: The Supply of Labor Skill: Conceptual

15) Is it possible for an individual to decrease the quantity of labor he or she supplies when the wage rate increases? Briefly explain your answer. Answer: Yes, it is possible for an individual to decrease the quantity of labor supplied when the wage rate increases. It is this reaction that leads to the backward-bending labor supply curve, which shows that above a certain wage, as the wage rate increases, the individual’s quantity of labor supplied decreases. Topic: The Supply of Labor Skill: Conceptual

16) Explain why an individual’s labor supply curve is backward bending. Answer: An individual’s labor supply curve bends backward when an increase in the wage rate decreases the quantity of labor the worker supplies. The labor supply curve bends backwards because

118

CHAPTER 17

workers have alternative uses for their time that they prefer to working. So if a worker is offered a high enough wage, the worker decreases the quantity of labor supplied in order to take additional time off from work in order to enjoy more leisure time. Topic: The Supply of Labor Skill: Conceptual

17) In order to spend more time with her children, a young mother decides to work less hours as her pay increases. What does her labor supply curve look like? Answer: The young mother’s labor supply curve is backward bending. In other words, her labor supply curve is positively sloped at lower wage rates, indicating that a higher wage increases the quantity of labor she supplies. But when her wage reaches a certain amount, further increases in the wage decrease the quantity of labor she supplies and her labor supply curve becomes negatively sloped. Topic: Changes in the Supply of Labor Skill: Conceptual

18) As the population of Las Vegas increased, what happened to the supply of labor in Las Vegas? Answer: As the population of Las Vegas increased, the supply of labor increased and the supply curve of labor shifted rightward. Topic: Changes in the Demand for Labor and the Supply of Labor Skill: Conceptual

19) The Small Bookshop hires teenagers who love books. How does each of the following changes affect the supply and demand for workers for the Small Bookshop? a) The Big Bookshop, a chain bookstore down the street, offers a higher wage rate. b) More people buy books from Amazon.com. c) People watch more television and read fewer books. d) More older people who love to read move into the area where the Small Bookshop is located. Answer: a) The supply of workers for Small Bookshop decreases and the supply curve shifts leftwards.

b)

c) d)

The demand for books from Small Bookshop decreases. The price of books at the Small Bookshop falls and so the demand for workers decreases. Similar to part (c), the demand for workers decreases. An increase in the population of readers increases the demand for books and thus increase the demand for workers at the Small Bookstore.

Topic: Labor Market Equilibrium Skill: Analytical

20) How does technological change affect jobs and wage rates? Answer: Technological change creates and destroys jobs. On the average, it creates more jobs than it destroys and the new jobs pay a higher wage rate than the old jobs that are destroyed. Topic: Present Value Skill: Analytical

21) The interest rate is 5 percent. How does $500 to be received a year from today compare in value to $500 right now? Answer: With an interest rate of 5 percent, $500 deposited in the bank right now will be worth $525 one year from today. (To calculate this amount, the future value is $500(1 + r) = $500(1+.05) = $525.) The $500 received one year from today, however, has a present value of $500/(1 + r) = $500/(1+.05) = $476.19. So, $500 today is worth more than $500 a year from now. Topic: Net Present Value Skill: Analytical

22) Suppose the next unit of capital would generate marginal revenue products of $100 per year over the next three years. The interest rate is 5 percent. Will the firm purchase a unit of capital if it costs $275? Answer: The future stream of $100 per year over the next three years is discounted at 5 percent to yield: $95.24, $90.70, and $86.38. The sum of these amounts is $272.32. Thus the next unit of capital is worth $272.32 to the firm and so it will not buy the capital if the capital costs more than $272.32.

DEMAND AND SUPPLY IN FACTOR MARKETS

Topic: Demand For Capital Skill: Conceptual

23) Why and how does the interest rate affect the demand for capital? Answer: The decision to buy a unit of capital is based upon the present value of the future marginal revenue products generated by the capital equipment. The interest rate is used to discount those future marginal revenue products. An increase in the interest rate means that the future marginal revenue project projections are more heavily discounted. This effect decreases the present value of that future stream and hence decreases the quantity of capital demanded. Topic: Supply of Capital Skill: Recognition

24) What factors affect saving? Answer: Saving is affected by: income; expected future income; and interest rates. Saving increases with income and decreases with expected future income. The interest rate represents the opportunity cost of current consumption. Thus, saving increases when the interest rate rises. Topic: Supply of Capital; Income Skill: Conceptual

25) How does a typical person’s saving change over his or her lifetime? Answer: For young people, expenses typically exceed income and so younger people tend to borrow. For these people, their current income is low compared to their future income. In middle age, most people’s incomes reach their peak and saving is substantial. Then, upon retirement, income drops dramatically and most people begin to spend part of their accumulated wealth. Topic: Supply of Capital Skill: Conceptual

26) What are the three factors that influence the supply of capital? Briefly explain how an increase in each affects the supply of capital curve. Answer: The three factors are income, expected future income, and the interest rate. An increase in income increases saving and shifts the supply of capital curve rightward. An increase in expected future income decreases saving and shifts the supply of capital curve leftward. And an increase in the interest rate increases the amount people save

119

and leads to a movement along the supply of capital curve. Topic: Financial Market Equilibrium Skill: Conceptual

27) If the quantity of financial capital supplied exceeds the quantity of financial capital demanded, what takes place? Answer: If the quantity of financial capital supplied exceeds the quantity of financial capital demanded, the interest rate falls in order to restore equilibrium. As the interest rate falls, the quantity of financial capital supplied decreases and the quantity of financial capital demanded increases. Topic: Renewable and Nonrenewable Resources Skill: Conceptual

28) Discuss the difference between renewable and nonrenewable resources. Given an example of each. Answer: A renewable natural resource can be used repeatedly; a nonrenewable natural resource can be used only once, and once a particular unit is used, it cannot be used again. Forestland, cropland, rivers, lakes, and wind power are examples of renewable natural resources. Examples of nonrenewable resources include coal and oil. Topic: Supply of an Nonrenewable Natural Resource Skill: Conceptual

29) Why is the flow supply of an nonrenewable natural resource perfectly elastic? Answer: The flow supply of an nonrenewable natural resource is perfectly elastic at the present value of next year’s expected price. If today’s price is below that, the owner is better off storing his or her current supply and bringing it to market next year. Why? If the owner sells the resource this year and saves the proceeds, the owner will have less next year than if the owner waited and sold the resource next year at the higher price. Topic: Hotelling Principle Skill: Conceptual

30) What is the Hotelling Principle. Have resource prices behaved as the principle predicts? Answer: The Hotelling Principle states that the price of a nonrenewable natural resource will rise at a rate equal to the interest rate. In reality, resource prices have not behaved this way. In addition to

120

CHAPTER 17

interest rates, technology and cost of extracting the resources affect the price of resources and so in reality, the prices of nonrenewable natural resources have generally fallen during the past years. Topic: Price of Natural Resources Skill: Conceptual

31) How do new technologies effect the price of nonrenewable resources? Answer: New technologies have two general effects. First, new technologies that lead to the discovery of previously unknown reserves of a nonrenewable natural resource increase supply, which decreases the price of the resource. Second, new technologies that enable a more efficient use of a nonrenewable resource decrease demand, and also lead to a decrease in the price. Topic: Economic Rent Skill: Recognition

32) Define economic rent. Answer: Economic rent is income received by the owner of a factor over and above the amount required to induce the owner to offer the factor for use. Economic rent is received by any factor (land, labor, capital, or entrepreneurship) that receives an income greater than its opportunity cost. Topic: Economic Rent Skill: Conceptual

33) Comment on the following: “Because the supply of land is perfectly inelastic, its owners merely receive the land’s opportunity cost as payment.” Answer: Quite the contrary. Because the supply is perfect inelastic, the land is there even at a zero price. Thus, all income received by the land owner is economic rent not payments to cover the land’s opportunity cost.

Topic: Economic Rent Skill: Conceptual

34) Is economic rent equal to the opportunity cost of supplying land? Answer: No, economic rent is not equal to the opportunity cost of supplying land. The opportunity cost of supplying land (or, for that matter, any other resource) is the amount of income necessary to induce the supply of a given quantity of a resource. Economic rent is income over and above the opportunity cost. Topic: Economic Rent Skill: Conceptual

35) Critics of the salaries of professional athletes contend that athletes would play for far lower salaries. How can this statement be true? Answer: The statement can be correct because a large fraction of the salaries of athletes is economic rent. Take Michael Jordan for an example. Mr. Jordan has a degree in geography from the University of North Carolina. If, while he was an NBA basketball player, the owners had revolted against rising salaries and slashed his and all other players pay from millions per year to say $200,000 per year, Mr. Jordan would have had to consider the opportunity cost of playing basketball at that pay. If his second-best choice would have been to be a really tall geography teacher for $45,000 per year, chances are he would have continued to play basketball at the lower pay rate. Therefore one can conclude that much of what professional athletes make for their highly demanded and rare talents is economic rent.

APPENDIX 17

MARKET POWER IN THE LABOR MARKET

Topic: How Unions Try to Change the Demand for Labor Skill: Conceptual

1)

What tools can unions use to increase the demand for union labor? Answer: The main tools that unions use to increase the demand for union labor are: increased training and education for its members; lobbying for import restrictions; supporting minimum wage laws; supporting immigration restrictions; and increasing the demand for the goods produced. Topic: How Unions Try to Change the Demand for Labor Skill: Conceptual

2)

Why does an increase in the minimum wage increase the demand for union labor? Answer: An increase in the minimum wage raises the cost of unskilled labor, which is typically nonunionized. By making this type of labor, which can be substituted for union labor, more expensive, an increase in the minimum wage increases the demand for union labor.

Topic: Monopsony Skill: Conceptual

3)

Explain why the marginal cost of labor facing a monopsony is greater than the wage rate. Answer: If the monopsony wants to hire more labor, it must offer a higher wage to the additional unit of labor PLUS it must offer that same higher wage to all units of labor that were previously hired at a lower wage. Topic: Monopsony and the Minimum Wage Skill: Conceptual

4)

Why does a monopsony increase employment when faced with an effective minimum wage law? Answer: A monopsony hires the quantity of labor so that the marginal revenue product of labor (MRP) is equal to the marginal cost of labor (MCL). This condition means that initially the MRP is above the wage (W). When a minimum wage is imposed, the marginal cost of labor decreases because it becomes equal to the minimum wage. Thus, the firm will hire more workers up to the point at which MRP = W.

CHAPTER 18

ECONOMIC INEQUALITY

Topic: Economic Inequality in the United States Skill: Recognition

Topic: Lorenz Curve Skill: Conceptual

1)

5)

In the United States, do the poorest 20 percent of the households receive more or less than 5 percent of total income? Answer: The poorest 20 percent of households receive less than 5 percent of total income. Topic: Economic Inequality in the United States Skill: Recognition

2)

Which is distributed more equally: income or wealth? Answer: The income distribution is much more equal than the wealth distribution. The poorest 40 percent of wealth holders own only 0.2 percent of wealth and the richest 5 percent own 59.4 percent of wealth. For income, the poorest 40 percent of households receive 12.3 percent of total income and the richest 20 percent of households receive 50.1 percent of total income. Topic: Lorenz Curve Skill: Recognition

3) What is a Lorenz curve and what does it show? Answer: A Lorenz curve graphs the cumulative percentage of income (or wealth) against the cumulative percentages of households. The curve shows how equally income (or wealth) is distributed across households. Topic: Lorenz Curve Skill: Conceptual

4)

“If the distribution of income was equal, the Lorenz curve would be a straight, 45º line.” Is the previous statement correct or incorrect? Answer: The statement is correct.

What do the Lorenz curves for wealth and income in the United States look like? Which is closer to the line of equality? Answer: Both the Lorenz curve for wealth and the Lorenz curve for income are bowed outward from the line of equality because neither wealth nor income is equally distributed. The Lorenz curve for income is closer to line of equality than the Lorenz curve for wealth because income, while unequally distributed, is more equally distributed than is wealth. Topic: Wealth Versus Income Skill: Recognition

6)

What is the difference between wealth and income? Answer: Wealth is a stock of assets, that is, wealth is what is owned at a point in time. Income is a flow of earnings over time. Thus a person’s wealth can be measured on, say December 31, 2004 whereas a person’s income is measured over a time period, say during 2004. Topic: Wealth Versus Income Skill: Conceptual

7)

How do income and wealth change over a person’s lifetime? How does this affect the distribution of income at a point in time? Answer: On the average, incomes start out low, increase as the household’s workers gain experience and other human capital, peak when the household’s workers reach retirement age, and then fall after retirement. On the average, wealth also starts out low, increases as income increases and workers save for retirement, peaks at the point of retirement, then falls after retirement. Because there are always people at all stages of life, the distribution of income at a point in time overstates the actual inequality. The discrepancy would be greater for

124

CHAPTER 18

countries with large percentages of young and retired workers. Topic: Measured Wealth Skill: Conceptual

8)

What is the main reason that the distribution of measured wealth is more unequal than the distribution of income? Answer: The distribution of income statistics include the returns from human capital, but the distribution of measured wealth statistics do not include the stock of human capital. Topic: Distribution of Lifetime Income Skill: Conceptual

9)

What effect does the age of a household have when estimating the degree of inequality in income among households? Answer: Household income changes over the life cycle of the household, which leads to an overestimation of inequality among households. At young ages, income is relatively low, peaks in middle ages, and then falls after retirement. The standard measures of income distribution do not account for age differences among households. They, therefore, overestimate the inequality because the distribution of lifetime income is more equal than the distribution of annual income. Topic: Gini Ratio Skill: Recognition

10) What is the Gini ratio? Answer: The Gini ratio is based on the Lorenz curve and equals the ratio of the area between the line of equality and the Lorenz curve to the entire area beneath the line of equality. Topic: Trends in the Distribution of Income Skill: Recognition

11) “Over the past two decades, the distribution of income in the United States has become more equal.” Is the previous statement correct or incorrect? Briefly explain your answer. Answer: The statement is incorrect. In recent years, higher income groups have gained relatively more than the lower income groups. The Gini ratio has increased, which means that the share of income going to the richest households has increased and the share of income going to the poorest households has decreased.

Topic: Trends in the Distribution of Income Skill: Recognition

12) How has income inequality changed in recent years? What factors account for the changes? Answer: Over the last couple of decades, income inequality has increased. In recent years, higher income groups have gained relatively more than the lower income groups, perhaps because of rapid technological progress. These technological advances have benefited the better educated more than the less educated. Topic: Trends in the Distribution of Income Skill: Recognition

13) In recent years, has the distribution of income in the United States become more equal or less equal? Which quintile share has changed most? Answer: The distribution of income in the United States has become less equal. The major change is that the share of income received by richest 20 percent of household has increased. Topic: Who Are the Rich and the Poor? Skill: Recognition

14) What are the main influences on a family’s income? Answer: The main influences on a family’s income are educational achievement, household size, marital status, age of householder, race, and region of residence. Topic: Who Are the Rich and the Poor? Skill: Recognition

15) Of age, marital status, family size, education, and race, which is the single biggest factor affecting the household income distribution? Answer: The single most important factor affecting the distribution of income is education, with more highly educated people earning significantly more than less-educated people. Topic: Who Are the Rich and the Poor? Skill: Recognition

16) Describe the characteristics of the typical lowest income household in the United States and the characteristics of the typical richest household in the United States. Answer: The typical lowest income household has less than a 9th grade education, is headed by single black woman over 65. The family lives in the South. The highest income household lives in the

ECONOMIC INEQUALITY

West. It is headed white married couple between 45 and 54 years of age. Both parents have college educations. The family has two children. Topic: Poverty Skill: Recognition

17) Is the poverty rate the lowest among Hispanic households, black households, or white households? Answer: The poverty rate is lowest among white households. About 10 percent of white households live in poverty whereas about 22 percent of Hispanic and 31 percent of black households live in poverty. Topic: Human Capital Skill: Recognition

18) What is “human capital”? How is it important in the determination of a worker’s wage rate? Answer: Human capital is the skill, education, and training that a person acquires. The more human capital a person has, the higher the person’s productivity and hence the higher the person’s value of the marginal product. A higher value of the marginal product increases the demand for the worker and leads to a higher wage rate. Topic: Human Capital Skill: Recognition

19) How is human capital acquired? Answer: Human capital can be acquired either through formal education and/or through on-thejob training. Topic: Demand for High-Skilled and Low-Skilled Labor Skill: Conceptual

20) How does the demand for high-skilled workers compare to the demand for low-skilled workers? Why does this difference exist? Answer: The demand for high-skilled workers exceeds the demand for low-skilled workers because the value of the marginal product of high-skilled workers exceeds the value of the marginal product of low-skilled workers.

125

Topic: Supply of High-Skilled and Low-Skilled Labor Skill: Conceptual

21) At any wage rate, the quantity of welders willing to work is less than the quantity of tomato pickers. Why? Answer: For the welder the cost of acquiring his or her skill is much higher than the cost incurred by the tomato picker. The cost of learning how to pick fruits and vegetables is so low that the supply of individuals who can do that activity is quite large. Welders must learn their trade on the job and/or through vocational training, which raises the costs of learning this skill and decreases the supply of welders. Topic: Supply of High-Skilled and Low-Skilled Labor Skill: Conceptual

22) How does an increase in the cost to acquire a skill affect the vertical distance between the supply curves of high-skilled and low-skilled workers? Answer: The vertical distance between the supply of high-skilled labor curve and the supply of lowskilled labor curve is the amount necessary to compensate high-skilled workers for the cost of acquiring the skill. Hence an increase in the cost of acquiring the skill will increase the amount needed to compensate the workers and so will increase the vertical distance between the two supply curves. Topic: Wage Rates of High-Skilled and Low-Skilled Labor Skill: Conceptual

23) Describe the effect education and training have on outcomes in the labor market. Answer: Education and training affect both the supply and demand sides of the labor market. On the supply side, because education and training takes time and money to acquire, the number of people available for high-skilled work is less than those available for low-skilled work. As a result, the supply curve of high-skilled labor is left of the supply curve of low-skilled labor. On the demand side, education and training increases the productivity of workers, giving them a higher value of the marginal product. The higher value of marginal product means that the demand curve for high-skilled labor lies right of the demand curve for low-skilled labor.

126

CHAPTER 18

With a decrease in supply and an increase in demand, the wage rate for high-skilled workers is higher than the wage rate for low-skilled workers. The effect on the quantity of labor employed, however, is indeterminate. Topic: Wage Rates of High-Skilled and Low-Skilled Labor Skill: Conceptual

24) What factors explain why high-skilled workers are paid more than low-skilled workers? Answer: High-skilled workers have a higher marginal revenue product than that of low-skilled workers. This fact causes the demand for high-skilled labor to exceed that of low-skilled labor and is one reason why high-skilled workers’ wages are higher than low-skilled workers’ wages. In addition, high-skilled workers require compensation for having acquired the skill. Thus, the supply of high-skilled labor is less than the supply of lowskilled labor, which also causes the wages paid high-skilled workers to exceed those paid lowskilled labor. Topic: Wage Rates Skill: Analytical

25) Wage rates in the United States differ between jobs. Use demand and supply to explain the distribution. Answer: The supply of qualified labor for some occupations is very limited, such as athletes, actors/actresses, or scientists. This limited supply will result in a high market equilibrium wage rate. In addition, the demand for these skilled workers is quite high, which also results in an increase in the equilibrium wage rate. On the other hand, the supply of labor in other occupations is readily available, especially when fewer skills are required. The result of this effect is low market equilibrium wage rates for the low-skilled workers. In addition, the demand for low-skilled workers is not great, which further reinforces the low wage rate these workers are paid. Topic: Discrimination Skill: Conceptual

26) Some economists argue that discrimination based on race and sex cannot lead to persistent wage differentials. What is their argument? Answer: In short, wage differentials will lead to price differentials. The price of the output produced by

workers suffering discrimination will be less than the price of output produced by favored workers. If the goods are the same, this price differential cannot persist. An increase in the demand for the good produced by the discriminated-against group will raise the demand for their labor and thereby increase their wage. Topic: Bequests Skill: Conceptual

27) What factors make bequests a source of increased economic inequality? Answer: Two factors make bequests a source of increased economic inequality: debts cannot be bequeathed and assortative mating. The law is that debts cannot be passed on to family members. As a result, only positive bequests, which can add to a family’s wealth increasing future generations’ wealth, can be made. Most households receive either no inheritance or else a very small one; however, a few receive enormous inheritances. As a result, wealthy families tend to remain wealthy. In addition, assortative mating, the fact that people tend to marry within their own socioeconomic class, also leads to persisting inequality in wealth. In particular, wealthy people marry wealthy people, so that wealth is not passed to poorer families. As a result, inherited wealth becomes more concentrated in wealthy families. Topic: Assortative Mating Skill: Recognition

28) What is meant by the term “assortative mating” and how does it affect the distribution of wealth? Answer: Assortative mating is the tendency for people to marry within their own socioeconomic class. Wealthy people tend to marry other wealthy people; wealth remains concentrated and this fact contributes to the skewed distribution of wealth. Topic: Income Redistribution Skill: Recognition

29) What are the three ways that the government redistributes income in the United States? Briefly discuss each. Answer: The government redistributes income using income taxes, income maintenance programs, and subsidized services. Income taxes redistribute income because they are generally progressive so that higher-income households pay more in income taxes than lower-income households. In ad-

ECONOMIC INEQUALITY

dition, poor households receive money from the income tax through the earned income credit. Income maintenance programs are programs such as social security, unemployment compensation, and welfare. These programs tend to give more income to poorer households than richer households. Finally, the government provides subsidized services, such as below-cost health care and below-cost schooling. Topic: Income Redistribution Skill: Recognition

30) What are the three main tools employed by the U.S. government to redistribute income? Answer: The three main tools are: income taxes which are typically progressive; income maintenance programs like unemployment compensation; and subsidized services like Medicaid and Medicare. Topic: Income Redistribution Skill: Recognition

31) What are three programs that redistribute income in the United States by making direct payments to people? Who benefits from each program? Answer: Three programs are Social Security, unemployment compensation, and welfare programs. Social Security is a public insurance system under which employers and employees pay payroll taxes. Recipients of social security include older persons, retirees, disabled persons and Medicare patients. Each state offers unemployment benefits to those people without a job, but are looking for work. Unemployed workers benefit from these programs. Finally, government welfare programs include supplementary security income (SSI) (for old and/or disabled people); temporary assistance for needy households (TANF) for people in poverty; a food stamp program (also helps the poorest families); and Medicaid (pays for medical costs for those families receiving SSI and TANF). Topic: Market Income and Money Income Skill: Conceptual

32) What is the difference between market income and money income? Which is more equally distributed? Answer: Market income is the income the household earns in the market for factors of production, for example, the household’s labor income. Money income is market income plus money benefits

127

paid by the government to the household. Money income is distributed more equally than market income. Topic: Big Tradeoff Skill: Recognition

33) With respect to income redistribution programs, what is meant by “The Big Tradeoff”, and what causes it? Answer: The big tradeoff refers to the tradeoff between equity and efficiency. A dollar collected from a rich person passes through a “leaky bucket” before getting to a poor person. The poor person doesn't receive all of that dollar. The cost of administering the program takes some of that money. More importantly, redistribution creates a disincentive to work, which decreases efficiency. Topic: Big Tradeoff Skill: Conceptual

34) Explain how redistributing income creates a deadweight loss. Answer: When income is redistributed from a richer person to a poorer person, both parties are affected. The richer person, who loses income, will pay a higher tax on earned income. This higher tax decreases the opportunity cost of leisure time (time spent not working) so the person will take more leisure time and work less. For the poorer person who gains income, the income transferred (say, welfare payments) will be reduced when the person earns more income. As a result, the poorer person also has less incentive to work. Because both the richer person and the poorer person have less incentive to work when the government administers income redistribution, there is created a deadweight loss, that is, a decrease in efficiency to society. Topic: Big Tradeoff Skill: Conceptual

35) A government redistribution of income program is proposed with the aim of making the income distribution the same as the 45º line in a Lorenz curve diagram. What implications does the idea of the “big tradeoff” have for the program? Answer: If the distribution of income was the same as the 45º line in a Lorenz curve diagram, then there would be no income differences across all households; everyone would have the same income. While this proposition might be appealing on an

128

CHAPTER 18

emotional level, the big tradeoff points out a significant drawback with this program and this outcome. The problem with the program arises because to achieve the desired result, the government would need to remove income from people who otherwise would be in the upper end of the income distribution and give the income to people who otherwise would be near the bottom end of the income distribution. This redistribution would drastically reduce the incentives of

successful people to create new products, technologies and jobs. It would tell people that education, skill and training, and hard work will not be rewarded. People would rationally assume that there is no relationship between effort and outcome and therefore work effort would decrease sharply.

CHAPTER 19

UNCERTAINTY AND INFORMATION

Topic: Uncertainty Skill: Recognition

Topic: Cost of Risk Skill: Recognition

1) Explain the concept of uncertainty. Answer: Uncertainty is a situation in which more than one event may occur but we don’t know which one. For example, when farmers plant their crops, they are uncertain about the weather during the growing season and hence are uncertain about their annual income.

6) How can we measure the cost of risk? Answer: The cost of risk is the amount by which expected wealth must be increased to give the same expected utility as a no-risk situation. Topic: Gains From Insurance Skill: Conceptual

2) Explain the economic concept of risk. Answer: In economics, risk is a situation in which more than one outcome may occur and the probability of each possible outcome can be estimated.

When would a risk averse individual not insure a risky event? Answer: Risk averse individuals are always willing to pay more than the minimum cost of insurance up to a point. If the insurance is priced above the maximum price the individual is willing to pay, he or she will not buy the insurance.

Topic: Probability Skill: Recognition

Topic: Reservation Price Skill: Recognition

3) Explain the concept of probability. Answer: A probability is a number between 0 and 1 that measures the chance of some possible event occurring. A zero probability means that the event will not happen. A probability of one means that the event will occur for sure.

8) What is a buyer’s reservation price? Answer: A buyer’s reservation price is the highest price that the buyer is willing to pay for a good.

Topic: Risk Skill: Recognition

Topic: Expected Utility Skill: Recognition

4) What is expected utility? Answer: Expected utility is the average utility arising from all possible outcomes. Topic: Risk Averse Skill: Recognition

5) What does it mean to be risk averse? Answer: A person is risk averse if his or her expected utility from a risky situation is less than the utility he or she would receive from having, with certainty, the expected wealth from the risky situation.

7)

Topic: Optimal Search Skill: Recognition

9) What is the optimal-search rule? Answer: Search for a lower price until the expected marginal benefit of additional search equals the marginal cost of search. Topic: Private Information Skill: Recognition

10) What is private information and what problems does it create? Answer: Private information is information that is available to one person but too costly for anyone else to obtain. Private information creates two problems: moral hazard and adverse selection.

130

CHAPTER 19

Topic: Moral Hazard Skill: Recognition

Topic: Adverse Selection Skill: Recognition

11) Explain the concept of moral hazard. Give an example. Answer: Moral hazard exists when one of the parties to an agreement has an incentive after the agreement made to act in a manner that brings additional benefits to himself or herself at the expense of the other party. Moral hazard arises because it is too costly for the injured party to monitor the actions of the advantaged party. For example, Bob hires Jack as a salesperson and pays him a fixed wage. Jack has an incentive to put the least possible effort, benefiting himself and lowering Bob’s profits.

14) Explain the concept of adverse selection. Give an example. Answer: Adverse selection is a tendency for people to enter into agreements in which they can use their private information to their own advantage and to the disadvantage of the less informed party. For example, if a car dealership offers its salespeople a fixed wage, it will attract lazy salespeople. Hardworking salespeople will prefer not to work for this dealership because they can earn more by working a dealership that pays by results.

Topic: Moral Hazard Skill: Conceptual

15) What adverse selection problems are created by banks that offer loans at interest rates that are very much above the market interest rate? Answer: Low-risk borrowers will be able to borrow elsewhere at the market rate and will not bother to apply at the high interest rate banks. High-risk borrowers, who have been turned down elsewhere, will be the only applicants for loans at the high interest rate banks. Thus, a high interest rate runs the risk of having only high-risk applicants apply.

12) “People buy insurance do protect themselves from moral hazard.” True or false? Explain. Answer: The statement is false. In fact, insurance creates a moral hazard problem because a person with insurance coverage for a loss has less incentive than an uninsured person to avoid such a loss. For example, a person with fire insurance for his house has less incentive to take precautions against fire than a person with no fire insurance does. Topic: Moral Hazard Skill: Conceptual

13) Most college professors are granted tenure after six years of employment. Tenure implies a lifetime appointment. What problem does this situation create, and how can colleges minimize the problem? Answer: Receiving a lifetime appointment creates a moral hazard. Knowing that one cannot be fired creates an incentive to shirk work. Colleges minimize the problem by looking for signals such as publications and an ongoing commitment to one’s discipline during the six year probation period with the belief that those who work hard for these six years are also likely to work hard throughout the future.

Topic: Adverse Selection Skill: Conceptual

Topic: Used Car Market Skill: Conceptual

16) Consider a market for used cars. Suppose there are only two kind of cars: lemons and good cars. A lemon is worth $1,500 both to its current owner and to anyone who buys it. A good car is worth $6,000 to its current and potential owners. Buyers can’t tell whether a car is a lemon until after they have bought the car. What do economists call the problem that buyers of used cars face? What kind of cars (lemons, good cars, or both) are traded? Explain and substantiate your answer. Answer: Because buyers can’t tell the difference between a lemon and a good car, they are willing to pay only one price for a used car. And they are not willing to pay $6,000 since there is some probability that they are buying a lemon worth only $1,500. If buyers are not willing to pay $6,000 for a used car, the owners of good cars are not willing to sell them: their car is worth $6,000 to them. So only the owners of lemons are willing to sell. But if only the owners of lemons are sell-

UNCERTAINTY AND RISK

ing, all the used cars available are lemons. Thus the market for used cars is a market for lemons. Economists call the problem that buyers of used cars face adverse selection. Topic: Used Car Market Skill: Conceptual

17) Consider a market for used cars. Suppose there are only two kind of cars: lemons and good cars. A lemon is worth $1,500 both to its current owner and to anyone who buys it. A good car is worth $6,000 to its current and potential owners. Buyers can’t tell whether a car is a lemon until after they have bought the car. What do economists call the problem that buyers of used cars face? What is the price of a used car? Explain and substantiate your answer. Answer: Because buyers can’t tell the difference between a lemon and a good car, they are willing to pay only one price for a used car. And they are not willing to pay $6,000 since there is some probability that they are buying a lemon worth only $1,500. If buyers are not willing to pay $6,000 for a used car, the owners of good cars are not willing to sell them: their car is worth $6,000 to them. So only the owners of lemons are willing to sell. But if only the owners of lemons are selling, all the used cars available are lemons, so the maximum price worth paying is $1,500. Thus the market for used cars is a market for lemons, and the price of a used car is $1,500. Economists call the problem that buyers of used cars face adverse selection. Topic: Private Information; Signal Skill: Conceptual

18) How can a warranty at the seller’s expense signal that a product is high quality? Answer: If the product is low quality, the warranty would be very costly for the seller to uphold. Low quality products would need numerous repairs for which the seller would be required to pay. Thus,

131

warranties are not likely to be offered on low quality products. Topic: Diversification Skill: Conceptual

19) What are costs and benefits of diversification? Answer: Diversification helps reduce the probability of extreme outcomes. It makes the investor more certain about a narrower band of outcomes. The benefit from diversification is that the chance of an extremely bad outcome is reduced. The cost of diversification is that the chance of an extremely good outcome is also reduced. While diversification does not affect the expected result, it allows the risk averse individual to sleep better at night. Topic: Efficient Market Skill: Recognition

20) What is an efficient market? Answer: An efficient market is a market in which the actual price of a good embodies all currently available relevant information. As a result, nobody can predictably make a profit by buying low and selling high. Topic: Efficient Market Skill: Conceptual

21) In an efficient market, is it possible to forecast changes in price? Why or why not? Answer: In an efficient market, it is not possible to forecast changes in price. In an efficient market no predictable profit opportunities are available. If traders predict that price will rise tomorrow, they will buy today. As a result, demand will increase and price will rise today until it reaches the level expected tomorrow. So there will be no price change tomorrow.

CHAPTER 20

TRADING WITH THE WORLD**

Topic: Patterns and Trends in International Trade Skill: Recognition

Topic: Patterns and Trends in International Trade Skill: Recognition

1)

4)

Explain recent trends in U.S. international trade. In your answer, include the role played by manufactured goods and services. Answer: Most U.S. exports and imports are manufactured goods. Trade in goods accounts for most of U.S. international trade; trade in services (travel and transportation) accounts for the rest. Since 1960, trade has accounted for an increasingly large fraction of total output in the United States. In 2003, as has been the case for about the past two decades, the value of U.S. imports exceeded that of U.S. exports. Topic: Patterns and Trends in International Trade Skill: Recognition

2)

“Agricultural products are the largest part of U.S. exports.” Is the previous sentence correct or incorrect? Answer: The sentence is incorrect. Manufactured goods account for most of U.S. exports and agricultural products account for only 8 percent. Topic: Patterns and Trends in International Trade Skill: Recognition

3)

Discuss the major individual nations and regions with whom the United States trades. Answer: The United States’ largest trading partner is Canada. In 2003, 20 percent of our exports went to Canada and 17 percent of our imports came from Canada. The second leading trading is Japan, with 8 percent of our exports and 9 percent of our imports. The region with the most trade with the United States is the European Union, with 24 percent of our exports and 23 percent of our imports. Second is Latin America, with 20 percent of our exports and 18 percent of our imports.

*

* This is Chapter 33 in Economics.

Is Germany America’s largest trading partner?

Answer: No, Canada is America’s largest trading part-

ner. Topic: Trade in Services Skill: Recognition

5)

Is it possible to export services? If so, give some examples. If not, explain why not. Answer: It is possible to export services and, in fact, services account for about 30 percent of U.S. exports. Exports of services include payments for transportation of goods by U.S. carriers (such as the payment made to UPS for shipping parcels between Japan and Thailand) and payments for meals made by foreign visitors to the United States. Topic: Balance of Trade Skill: Recognition

6)

Since 1960 in the United States, how have imports and exports changed as a percent of total output? Answer: As a percent of total output, both imports and exports have been increasing in the United States since 1960. Topic: Balance of Trade Skill: Recognition

7)

What is the balance of trade? What is the U.S. balance of trade? Answer: The balance of trade is the value of exports minus the value of imports. The United States has a balance of trade deficit, which means that the value of U.S. imports exceeds the value of U.S. exports.

134

CHAPTER 20

Topic: Balance of Trade Skill: Recognition

Topic: Gains From Trade Skill: Conceptual

8)

Define “balance of trade.” What is meant by a trade surplus and a trade deficit? How can a nation finance a trade deficit? Last year, did the United States have a trade deficit or a trade surplus? Answer: The trade balance equals the value of exports minus the value of imports. If the value of exports exceeds the value of imports, the nation has a trade surplus. If the value of exports is less than the value of imports, the nation has a trade deficit. The trade deficit is financed by either borrowing from foreigners or selling some assets. Last year, as in recent decades, the United States had a trade deficit.

11) “When countries specialize in producing the good in which they have a comparative advantage and then trade with each other, only the country with the absolute advantage gains.” Is the previous statement correct or incorrect? Briefly explain your answer. Answer: The statement is incorrect. Absolute advantage, which is essentially the ability to produce more of a good than another country, has nothing to do with the gains from trade. All nations gain from free international trade regardless of whether they possess an absolute advantage or not.

Topic: Comparative Advantage Skill: Conceptual

12) “Because the United States is the largest economy in the world and can produce anything it needs domestically, there are no gains from trade for the United States.” Is the previous statement correct or incorrect? Answer: The statement is incorrect. The United States, like any other nation, gains from trade when it specializes according to comparative advantage.

9)

Consider two countries, A and B, that produce apples and bananas, respectively. Using your own numbers for production, give an example of country A having a comparative advantage in apples. Answer: Country A will have a comparative advantage in apples if the opportunity cost of producing apples in Country A is less than in Country B. So, regardless of the numbers used, Country A will have a comparative advantage as long as the number of bananas given up to get one more apple is less in Country A than in Country B. Topic: Comparative Advantage Skill: Conceptual

10) Define comparative advantage and discuss its role in international trade. Answer: Comparative advantage is the factor that drives international trade. A country has a comparative advantage in the production of a good if the country can produce it at a lower opportunity cost than any other country. Because the cost of production of a good is lower in the nation with the comparative advantage in the good, that country will export the good. The country will then gain by buying the goods from other nations that those nations produce at the lowest opportunity cost, that is, those goods in which the other nations have a comparative advantage.

Topic: Gains From Trade Skill: Conceptual

Topic: Gains From Trade Skill: Conceptual

13) Why do nations engage in international trade? Answer: Nations engage in international trade because they gain from trade. International trade results in a more efficient use of resources and thereby increases world output. As a result, it increases the amount of goods and services available for consumption in all nations and thereby makes all countries better off. Topic: Gains from Trade; Consumption Possibilities Skill: Conceptual

14) Explain why specialization and trade increases a country’s overall level of consumption. Answer: A country specializes in the activities in which it has the lowest opportunity cost. By trading, it can obtain goods and services at a lower opportunity cost than it would cost to produce the goods and services domestically. Hence the nation acquires goods and services at a lower cost than before and so the nation’s consumption increases. In

TRADING WITH THE WORLD

fact, trade allows the nation to consume at a point beyond its production possibilities frontier. Topic: Gains from Trade; Consumption Possibilities Skill: Conceptual

15) Explain how countries can gain from international trade. Answer: A country can gain from international trade by specializing in the production of the products in which it has a comparative advantage and then trading with other nations. By specializing and trading, a nation can increase its consumption to a point beyond its production possibilities frontier. The added consumption is the gains from trade. Topic: Gains from Trade; Consumption Possibilities Skill: Conceptual

16) Explain how free trade benefits countries. Answer: Each country should produce and export the goods and services in which they have a comparative advantage. By producing the good or service in which their opportunity cost is lower and engaging in free trade in which each country exports the goods and services in which they have a comparative advantage, there will be an increase in consumption opportunities for each and every country. Topic: Gains from Trade; Consumption Possibilities Skill: Conceptual

17) During 2003, as oil and gas prices continued to increase, a growing number of Americans called for the United States to become less reliant on Middle-Eastern oil. Would it make sense for the United States to try to become totally self-reliant in the production of oil? Why or why not? Answer: It would be foolish for the United States to try to become totally self-reliant in oil production. There is a reason that approximately 58 percent of our oil comes from OPEC nations: MiddleEastern countries can produce oil at a far lower opportunity cost than U.S. producers. In the Middle East the vast reserves of oil, combined with more lax regulations imposed by the government, have combined to drive down the per barrel opportunity cost of oil extraction to very low levels. Hence the United States gains from

135

trade with these nations. Even though the price of oil in 2003 was higher than it was in 2001, that price was much less than what would be the opportunity cost of producing enough oil domestically so that the United States was totally selfreliant. Topic: Gains from Trade in Reality Skill: Conceptual

18) Explain why countries export and import the same good. Answer: There are two reasons for trade in similar goods: diversified tastes and economies of scale. Diversified tastes refers to the fact that people demand many similar but slightly different products. So, a country might export minivans and import small sports cars. Economies of scale occur when average total cost declines with output. If there are substantial economies of scale, a nation can specialize in the production of one of the similar goods and capture economies of scale by trading the good throughout the world. Topic: Restricting Trade Skill: Conceptual

19) What are the two tools used by governments to restrict trade? Answer: One tool used by governments to restrict trade is tariffs. A tariff is a tax imposed by an importing country when an imported good crosses its international boundary. The second tool is nontariff barriers. Nontariff barriers are any action, other than a tariff, that restricts international trade. Examples of nontariff barriers are quotas and voluntary restriction agreements. Topic: History of Tariffs Skill: Recognition

20) Give a brief description of the history of tariffs in the U.S. Answer: Today, U.S. tariffs are low compared to their historical levels. The average U.S. tariff was highest in the early 1930s when the Smoot-Hawley tariff was passed. The average tariff at that time was about 20 percent. Since then there has been a general downward trend so that today the average tariff is less than 5 percent. The downward trend was fairly rapid until 1950 and has slowed since then.

136

CHAPTER 20

Topic: Trade Restrictions Skill: Recognition

Topic: Effects of a Tariff Skill: Conceptual

21) What methods do governments use to restrict international trade? Briefly define each of the methods. Do any of these methods restrict trade without harming domestic consumers? Answer: Governments restrict trade through the use of tariffs and quotas. A tariff is a tax imposed on a good when it is imported into the nation. A quota is a limit on the amount of a good that may be imported. Both of these methods restrict trade but they both harm domestic consumers by raising the price and decreasing the quantity consumed.

25) What are the effects of a tariff? Answer: A tariff is a tax on goods that are imported into the country imposing the tariff. The tariff decreases the supply of the imported good, so the tariff raises the domestic price of the good and decreases the quantity imported. Because the price of the good rises, the quantity produced in the country imposing the tariff increases. Topic: Quotas Skill: Conceptual

22) “Tariffs today in the United States are much higher than in the past.” Is the previous statement correct or incorrect? Answer: The statement is most decidedly incorrect. U.S. tariffs today are lower compared to what they were in the past.

26) Explain the effects of a quota. Answer: A quota is a quantitative restriction on the maximum amount of a good that can be imported. Because quotas limit the supply of the good, they raise the prices of imported goods and decrease the quantities imported. Unlike a tariff, however, the government gets no revenue from a quota; the revenue from the higher price goes to importers.

Topic: Effects of a Tariff Skill: Conceptual

Topic: Quotas Skill: Conceptual

23) How does a tariff affect the domestic price of the import, the domestic consumption, the domestic production, and the quantity imported? Answer: A tariff raises the price of the good. As a result, domestic consumption decreases as domestic consumers decrease the quantity they demand. And, also as a result, domestic production increases as domestic producers increase the quantity they supply. Because domestic consumption decreases and domestic production increases, the quantity imported decreases.

27) Currently, the United States has a quota on the amount of sugar that is allowed to be imported into the United States. What would happen to the price of sugar in the United States if the quota was removed? What would happen to U.S. consumption and U.S. production of sugar? Answer: If the quota is removed, the price of sugar in the United States would fall, U.S. consumption of sugar would increase, and U.S. production of sugar would decrease.

Topic: History of Tariffs Skill: Recognition

Topic: Effects of a Tariff Skill: Conceptual

24) The United States imposes a tariff on foreign limes. How does the tariff affect the U.S. price of a lime and the production of limes in the United States? Answer: The tariff raises the price of limes in the United States. As a result of the higher price, U.S. lime production increases.

Topic: Quotas Skill: Conceptual

28) How does a quota affect the domestic price of the import, the domestic consumption, the domestic production, and the quantity imported? Answer: A quota raises the price of the good because it decreases the amount that can be imported. As a result, domestic consumption decreases as domestic consumers decrease the quantity they demand. And, also as a result, domestic production increases as domestic producers increase the quantity they supply.

TRADING WITH THE WORLD

Topic: Quotas Skill: Conceptual

29) How does a tariff affect the government’s revenue? How does a quota affect the government’s revenue? Answer: A tariff is a tax on an imported good. Like all taxes, a tariff increases the government’s revenue. However, a quota is quantitative restriction on the amount of a good that can be imported. As such, a quota has no effect on the government’s revenue. Topic: Nontariff Barriers Skill: Conceptual

30) What are the two main forms of nontariff barriers? When these are imposed, who receives the difference between the world price of the good and the domestic price? Answer: The two main forms of non tariff barriers are quotas and voluntary export restraints. A quota is a quantitative restriction on the import of a good where the maximum quantity that can be imported is specified. A voluntary export restraint is an agreement between two countries where trade limits are agreed upon. With a quota, the importers receive the difference between the world price of the good and the domestic price. With a voluntary export restraint, the exporters receive the difference between the world price of the good and the domestic price. Topic: Why Is International Trade Restricted? Skill: Conceptual

31) Discuss reasons why we see trade restrictions. Are any of these reasons valid? Answer: There are several reasons advanced to restrict trade. These include: national security (the idea that the nation should protect industries that are necessary for its defense); the infant-industry argument (that the nation should protect a young industry that will reap learning-by-doing gains in productivity and eventually be able to compete successfully in the world market); dumping (the nation should protect an industry from foreign competitors who sell goods below cost); protection saves jobs (the claim that imports cost U.S. jobs); cheap foreign labor (the assertion that tariffs are necessary to compete with cheap foreign labor); diversity and stability (nations specialized in the production of one good might be subject to

137

economic fluctuations); lax environmental standards (the claim that protection is needed to compete against nations with weak environmental standards); national culture (protection is necessary to protect the nation’s national culture); and, rich nations exploit developing countries (the suggestion that protection prevents developed nations from forcing people in poor nations to work for slave wages). Economists reject all of these claims as valid reasons for protection. Topic: Why Is International Trade Restricted? Skill: Conceptual

32) Three arguments used to promote trade barriers are the national security argument, the infantindustry argument, and the dumping argument. Explain each of these arguments and evaluate whether each one has any flaws. Answer: The national security argument is that countries must protect industries that produce defense equipment and armaments as well as those industries upon which the defense industries rely. The problem with this argument is that all industries contribute to national defense, though some do so more indirectly than others. For instance, the pillow industry can claim that it contributes to national defense because without good pillows, plant workers will be unable to sleep well and hence their efficiency will be lowered. The pillow industry therefore can make a claim for protection— which, incidentally, it did in the 1950s using the argument just described! The infant-industry argument is that it is necessary to protect a new industry to enable it to grow into a mature industry that can compete in world markets. The problem with this argument is that if an industry can eventually compete, then its backers should be willing to fund it until that time. In addition, if the industry has benefits that spill over to other industries, then the more efficient government policy is to subsidize the industry rather than protect it from competition. Finally, the dumping argument asserts that protection is needed to protect domestic industries from foreign dumping practices designed to eliminate competition. (Dumping is selling a good for a price that is less than its cost of production.) The problems with this argument are twofold. First, it is extremely difficult to determine if a firm is dumping because determining the cost of

138

CHAPTER 20

production is difficult. Second, even if a firm is dumping, its success in establishing a monopoly is in doubt and its success in maintaining its global (!) monopoly is even more doubtful. Hence dumping to obtain a monopoly is likely a very uncommon practice. Topic: The Case Against Protection; National Security Skill: Conceptual

33) What is the national security argument for restricting international trade? What is its flaw? Answer: The national security argument is that the nation must protect and maintain industries that produce defense equipment and armaments as well as those industries upon which the defense industries rely. The problem with this argument is that all industries contribute to national defense, though some do so directly while others do so indirectly. For instance, the sugar industry contributes to national defense because factory workers and armed service men and women drink colas made using sugar, but protecting the sugar industry for reasons of national defense is ludicrous. Topic: The Case Against Protection; Dumping Skill: Recognition

34) What is dumping? Answer: Dumping is the situation in which a firm sells its export goods and services for a lower price than its cost of production. Topic: The Case Against Protection; Cheap Foreign Labor Skill: Conceptual

35) Because wage rates are so low in Africa, why don’t Microsoft, Cisco and other major corporations close down their American operations and move to Africa? Answer: Wage rates must be weighed against productivity. It is not just wages that influence where production occurs. Wages divided by the productivity of the workers gives the average cost of production. In Africa, workers have low levels of skill, education, and training so their productivity is much less than in the United States. Therefore the cost of production would be far higher in Africa than in America. So even though U.S. wage rates are high, industries stay here because the cost of production is lower because U.S. productivity is so high.

Topic: The Case Against Protection, Exploits Poor Countries Skill: Conceptual

36) Some people assert that protection from foreign competition prevents rich countries from exploiting developing countries. What is this argument in more detail and what is its flaw? Answer: The argument claims that rich nations will exploit poor nations by importing goods from the poor nations and that the workers in the poor nations are paid slave wages to produce these goods. This argument has a truly fatal flaw. Free trade increases the demand for the goods produced by workers in developing countries. Thus in order to produce more of these goods, the firms must hire more workers. Hence the demand for the labor used to produce the goods increases, which means that the wage rates paid the workers in the developing countries rises. Thus rather than exploiting the workers in the poor nations, free international trade is one of their few hopes for better lives! Topic: Rent Seeking Skill: Conceptual

37) Explain how governments restrict international trade and who benefits as well as who loses from the restrictions. Answer: Governments use tariffs and nontariff barriers, such as quotas, to restrict trade. Tariffs and quotas both boost the domestic price of the protected good. Consumers in that country lose because of the higher price. The domestic suppliers, however, gain from the higher price. Tariffs are source of revenue for the government that imposes it on imported goods, so the domestic government gains from a tariff. Quotas, on the other hand, do not create revenue for government so the government does not gain from a quota. Topic: Rent Seeking Skill: Conceptual

38) What is “rent seeking?” How does it apply to restricting imports? Answer: Rent seeking is lobbying and other political activity that seeks to capture the gains from trade. When imports are restricted, some people gain from the restrictions. Rent seekers, such as domestic producers of import-competing goods or services, lobby the government to impose import restrictions because restrictions allow the rent seekers to gain revenue and/or profit.

TRADING WITH THE WORLD

139

Topic: Compensating Losers Skill: Conceptual

Topic: Compensating Losers Skill: Conceptual

39) Economics demonstrates that opening up unrestricted free international trade is beneficial to all nations. However, are there any losers from such a policy change? Answer: Yes, there are losers from opening up to free trade. Domestic suppliers of imported goods lose from allowing free trade. Owners of the businesses lose as do workers who had jobs in the importcompeting industries. However, it is important to realize that although there are losers from free trade, there also are substantial gains and the gains exceed the loses so that the nation as a whole is made better off with free trade.

40) How does the United States attempt to compensate losers from lower trade restrictions? Answer: The U.S. government attempts to compensate workers who lose from lowering U.S. trade restrictions. For instance, the U.S. government set up a fund to support and retrain workers who lost their jobs because of NAFTA. Job losers can also collect unemployment compensation benefits.

NUMERIC AND GRAPHING QUESTIONS

CHAPTER 1

WHAT IS ECONOMICS?

GPA and decides to study an extra night each week. His GPA now rises to 3.5. Had Jerry not decided to study an extra night, he would have spent this night with his friends. What is Jerry’s marginal benefit from studying for one additional night a week? What is his marginal cost of increasing the study time by one night per week? Why does Jerry decide to study an extra night?

Numeric and Graphing Questions Topic: Marginal Benefit Skill: Analytical

1)

Suppose you are working four nights per week at your courses and your grade point average is 3.5. You want a higher grade and decide to study an extra night each week. Your GPA now rises to 3.8. What is your marginal benefit from studying for one additional night a week?

Answer:

Marginal benefit is the benefit that arises from an increase in an activity. Jerry’s marginal benefit is the 0.4 increase in his grade. Marginal cost is the opportunity cost of an increase in an activity. Jerry’s marginal cost is a night spent with his friends that he gives up. Jerry decides to stud an extra night because he values the marginal benefit from it (the 0.4 increase in his grade) more highly than its marginal cost (a night spent with his friends).

Answer:

Marginal benefit is the benefit that arises from an increase in an activity. Your marginal benefit is the 0.3 increase in your grade. It’s not the 3.8 grade because you already have the benefit from studying for four nights a week and should not count this benefit as resulting from the decision you are now making. Topic: Marginal Benefit/Marginal Cost Skill: Analytical

2)

Jerry is studying three nights per week and his grade point average is 3.1. He wants a higher

143

APPENDIX 1 GRAPHS IN ECONOMICS Topic: Time Series Skill: Recognition

Topic: Positive Relationship Skill: Recognition

1)

2)

The figure above shows the price of a DVD player from 1999 to 2003. a) What type of graph is illustrated above? b) What is the trend in the price of a DVD player? Answer:

a) The graph is a time-series graph because it plots time along the horizontal axis and the price of a DVD player along the vertical axis. b) The trend in the price of a DVD player is negative, that is, the price of a DVD player has generally decreased from one year to the next.

The figure above shows how the sales of the video game “Tomb Raider—Lara Retires” change when the advertising spent on the game changes. Is the relationship between advertising and the number of games sold positive, negative, or neither? Explain your answer. Answer:

The figure shows that there is a positive relationship between advertising and the number of video games sold. The relationship is positive because the two variables move together: If advertising increases, so, too, does the number of games sold.

146

APPENDIX 1

Topic: Negative Relationship Skill: Recognition

3)

The figure above shows how the relationship between the number of hours per week a high school student spends on the web and the student’s SAT score. Is the relationship between hours on the web and the SAT score positive, negative, neither? Explain your answer. Answer:

The figure shows that there is a negative relationship between hours on the web and the student’s SAT score. The relationship is negative because the two variables move in opposite directions: If hours on the web increase, the SAT score decreases. Topic: Maximum Skill: Recognition

4)

A graph has a point that is either a maximum or a minimum. To the left of the point, the slope of relationship is positive. To the right of the point, the slope is negative. Is the point a maximum point or a minimum point? Be sure to draw a figure that supports your answer.

Answer:

The point is a maximum point. Examine the figure above. The slope of a curved line at any point equals the slope of a straight line that touches the curved line at only that one point. Thus to the left of the maximum point, take point A. The slope of the straight line that touches the curved line at only point A is positive, so the slope of the relationship is positive. Similarly, take point B to the right of the maximum point. As the straight line shows, the slope of the relationship at point B is negative. Indeed, whenever there is a maximum point, the slope of the relationship to the left of the maximum is positive and the slope to the right is negative.

GRAPHS IN ECONOMICS

Topic: Slope Skill: Conceptual

147

Topic: Slope of a Straight Line Skill: Analytical

Katie’s income (dollars per year) 50,000 70,000 90,000 110,000

5)

Katie’s purchases (books per year) 14 16 18 20

In the figure above, what can you deduce about the slope of the curve? Answer:

The slope is positive and increasing in size as we move rightward along the curve. Topic: Slope Skill: Analytical

6)

If two points on a line are x = 2, y = 5 and x = 7, y = 10, what is the slope of this line? Answer:

The slope equals the change in the y-variable divided by the change in the x-variable. So, the slope equals (10 − 5) ÷ (7 − 2) = (5) ÷ (5) = 1.00.

7)

The table above shows how the number of books Katie buys each year depends on her income. a) What kind of relationship exists between Katie’s income and the number of books she purchases? b) Plot the relationship between Katie’s income and the number of books she purchases in the above figure. Measure income along the vertical axis and the number of books along the horizontal axis. Be sure to label the axes. c) What is the slope of the relationship between $50,000 and $70,000 of income? d) What is the slope of the relationship between $90,000 and $110,000 of income? e) Comment on the similarity or dissimilarity of your answers to parts (c) and (d). Answer:

a) There is a positive relationship. When Katie’s income increases, so too does her purchase of books.

148

APPENDIX 1

Topic: Slope of a Straight Line Skill: Analytical

X 2 4 6 8

b) The relationship is plotted in the figure above. c) The slope equals the change in the value of the variable measured on the vertical axis, income, divided by the change in the value of the variable measured along the horizontal axis, the number of books. Between $50,000 and $70,000 of income, the number of books purchased increases from 14 to 16. Hence income increases by $20,000 and the number of books increases by 2, so the slope equals $20,000/2 = 10,000. d) As with the previous answer, the slope equals the change in income divided by the change in books. Between $90,000 and $110,000 of income, the number of books purchased increases from 18 to 20. Hence income increases by $20,000 and the number of books increases by 2, so the slope equals $20,000/2 = 10,000. e) The slopes in parts (c) and (d) are equal. But, they must be equal because the relationship between Katie’s income and the number of books she purchases is linear. For a linear relationship, the slope is the same regardless of where it is measured.

8)

Y 20 16 12 8

Graph the data in the table above in the figure. Label the axes. a) Is the relationship between X and Y positive or negative? b) What is the slope when X = 4? c) What is the slope when X = 8?

GRAPHS IN ECONOMICS

Answer:

The figure labels the axes and graphs the relationship. a) The relationship between X and Y is negative. b) The slope equals –2. c) The slope equals –2.

149

Answer:

A horizontal line has a slope of zero. The figure above shows a horizontal line with a slope of zero. Topic: Slope of a Straight Line Skill: Analytical

Topic: Slope of a Straight Line Skill: Analytical

10) What does the slope of the line shown in the above figure equal? Answer:

9)

In the above diagram, draw a straight line with a slope of zero.

The slope equals the change in variable on the yaxis divided by the change in the variable on the xaxis, or (150 – 300)/(600 – 800) = 0.75.

150

APPENDIX 1

Topic: Slope of a Straight Line Skill: Analytical

Answer:

The slope equals the change in variable on the yaxis divided by the change in the variable on the xaxis, or (30 – 60)/(25 – 15) = –3.0. Topic: Slope of a Straight Line Skill: Analytical

11) What does the slope of the line shown in the above figure equal? Answer:

The slope equals the change in variable on the yaxis divided by the change in the variable on the xaxis, or (18 – 27)/(10 – 20) = 0.90. Topic: Slope of a Straight Line Skill: Analytical

12) What does the slope of the line shown in the above figure equal?

13) What does the slope of the line shown in the above figure equal? Answer:

The slope equals the change in variable on the yaxis divided by the change in the variable on the xaxis, or (5 – 10)/(60 – 100) = 0.125.

GRAPHS IN ECONOMICS

Topic: Slope of a Curved Line Skill: Analytical

151

Topic: Relationships Among More Than Two Variables Skill: Analytical

Hours studied (per week) 2 4 6 8 10

SAT Score 900 1000 1050 1075 1090

14) What does the slope of the curved line at point A shown in the above figure equal? Answer:

The slope of a curved line equals the slope of a straight line that touches the curved line at only that point. And, the slope of a straight line equals the change in variable on the y-axis divided by the change in the variable on the x-axis. Measure the slope of the straight line from point A to where the line crosses the y-axis, at 15. Thus the straight line has a slope of (30 – 0)/(10 – 15) = –6. Therefore the curve line at point A also has a slope equal to –6.

15) Jamie is preparing to take his SAT tests. The table above shows how Jamie’s score depends on the number of hours a week Jamie studies. a) Plot the relationship in the figure, putting the hours studied on the horizontal axis. b) Is the relationship you plotted positive or negative? c) What happens to the slope of the relationship as hours studied increase? d) Suppose Jamie can enroll in an SAT prep course and, by so doing, for every possible number of hours he studies, his score will be 100 points higher. Plot the new relationship between the number of hours studied and Jamie’s SAT score in the figure. e) How many variables are involved in the figure you just completed?

152

APPENDIX 1

Answer:

a) The figure above plots the relationship between the number of hours Jamie studies and his SAT score.

b) The relationship is positive: As Jamie increases the hours he studies, his SAT score increases. c) The relationship is nonlinear, so the slope of the relationship changes as the number of hours studied changes. In the figure, the slope of the relationship decreases in size as the number of hours studied increases. d) The figure above also plots the relationship between the hours Jamie studies and his SAT score if Jamie takes an SAT preparation course. e) There are three variables: The number of hours Jamie studies, whether or not he takes an SAT preparation course, and his SAT score.

CHAPTER 2

THE ECONOMIC PROBLEM

Topic: Production Possibilities Frontier Skill: Conceptual

1)

Topic: Production Possibilities Frontier Skill: Conceptual

Draw a production possibilities frontier between beans and peas. Label the unattainable points, the attainable points with fully employed resources, and the attainable points with unemployed resources. Answer:

2)

The production possibilities frontier, with the points labeled, is above. Any point beyond the production possibilities frontier is unattainable. Any point on the production possibilities frontier is attainable and resources are fully employed. Finally, any point within the production possibilities frontier is attainable and has unemployed resources.

The figure above shows a nation’s production possibilities frontier for apples and oranges. a) What combination of goods is represented by point A? b) What combination of goods is represented by point B? c) Which point represents an unattainable combination of goods? Answer:

a) 3 million bushels of apples and 3 million bushels of oranges b) 3 million bushels of apples and 4 million bushels of oranges c) Point B is an unattainable point. Topic: Production Possibilities Frontier Skill: Conceptual

3)

Before the first Gulf War in 1991, Kuwait had the capacity to produce a certain amount of oil from its oil wells. After the war, it found that capacity greatly diminished because the oil wells had been set on fire. Draw Kuwait’s PPF before and

154

CHAPTER 2

after the war, assuming that the only two goods produced are oil and food. Further assume that setting the oil wells on fire did not affect Kuwait’s ability to produce food. Explain why the PPF before the war is different from the PPF after the war.

Answer:

Answer:

When a PPF is drawn, we draw it for a fixed amount of natural resources, along with fixed amounts of the other factors of production such as labor, capital, etc. Fire reduced Kuwait’s natural resources temporarily, so the PPF after the war shifted inwards. However, because setting the oil wells on fire did not affect Kuwait’s ability to produce food, the maximum amount of food production, the point where the PPF intersects the vertical axis, did not change. Topic: Production Possibilities Frontier Skill: Analytical

Production point A B C D E F G 4)

Graph the PPF. What is the opportunity cost of producing the first four pizzas? What is the opportunity cost of producing the 10th pizza. What is the opportunity cost of producing the first 12 CDs? What is the opportunity cost of producing the 26th CD?

Pizza produced 0 4 8 12 16 20 24

and and and and and and and

CDs produced 42 40 36 30 22 12 0

The table above lists seven points on the production possibilities frontier for pizza and CDs.

The PPF graph is shown above . The opportunity cost of the first four pizzas is 2 CDs. The 10th pizza is the midpoint between 8 and 12 pizzas. The opportunity cost of gaining 4 additional pizzas by producing 12 pizzas instead of 8 pizzas is 6 CDs given up, so the opportunity cost of an additional pizza is 6÷ 4 = 1.5 CDs. The opportunity cost of the first 12 CDs is 4 pizzas. The 26th CD is the midpoint between 22 and 30 CDs. The opportunity cost of gaining 8 CDs by producing 30 CDs instead of 22 CDs is 4 pizzas given up, so the opportunity cost of an additional CD is 4 ÷ 8 = 0.5 pizzas.

THE ECONOMIC PROBLEM

155

Topic: Opportunity Cost Skill: Analytical

Production point A B C D E 5)

Milk (gallons) 0 2 4 6 8

and and and and and

Shirts (number) 100 90 70 40 0

A (very, very small) country produces milk and shirts and its production possibilities frontier is in the table above. The nation is currently producing at point B. What is the opportunity cost of two additional gallons of milk? At point C? At point D? What do your results show?

Movement from A to B B to C C to D D to E

Topic: Opportunity Cost Skill: Analytical

Production point A B C D E 6)

Pages typed 0 40 70 90 100

and and and and and

Web pages created 4 3 2 1 0

Jean can either type her term paper or create Web pages during the limited time she has available. The table above shows her PPF. a) Can Jean type 90 pages and create 2 Web pages? b) Use the above numbers to calculate the opportunity cost of a typed page as she increases her time typing and decreases time creating a Web page. Answer:

a) Jean cannot type 90 pages and create 2 Web pages because, as row D shows, that combination is beyond her PPF.

Decrease in Web pages 1 1 1 1

Opportunity cost 1/40 1/30 1/20 1/10

b) The opportunity cost is the ratio of the decrease in the number of Web pages divided by the increase in the number of typed pages. The table above gives the opportunity cost for typed pages. Topic: Opportunity Cost Skill: Analytical

Answer:

At point B, the opportunity cost of 2 additional gallons of milk is 20 shirts. At point C, the opportunity cost of 2 additional gallons of milk at 30 shirts. At point D, the opportunity cost of 2 additional gallons of milk is 40 shirts. The opportunity cost of 2 additional gallons of milk increases as more milk is produced.

Increase in typed pages 40 30 20 10

Production point A B C D 7)

Wheat Soybeans (bushels) (bushels) 1,500 and 0 1,000 and 2,250 500 and 3,500 0 and 4,000 The table above gives the production possibilities frontier for a nation that produces wheat and soybeans. Use the information in that table to complete the table below, which has in it the opportunity costs of moving from one production point to another. Do not forget to note the units of the opportunity costs.

Movement from A to B B to C

Opportunity cost

Movement from D to C C to B

Opportunity cost

Answer:

Opportunity Opportunity cost cost (bushels of (bushels of soybeans per wheat per bushel of Movement bushel of Movement from wheat) from soybeans) A to B 0.22 D to C 1.00 B to C 0.40 C to B 2.50 The table above gives the opportunity costs. The units of the opportunity costs are in the column headings.

156

CHAPTER 2

Farmer Brown’s opportunity cost of additional beef as Farmer Brown moves from point A to B to C to D. Also use the data to calculate Farmer Brown’s opportunity cost of additional wheat as Farmer Brown moves from point D to C to B to A. Based on these costs, does Farmer Brown use resources that are more productive in one activity than the other? Explain your answer.

Topic: Opportunity Cost Skill: Analytical

Answer:

8)

The figure above represents the production possibilities frontier for a country. a) The nation is currently producing at point B and wants to move to point C. What is the opportunity cost of the move? b) The nation is currently producing at point B and wants to move to point A. What is the opportunity cost of the move? Answer:

a) By moving from point B to point C, the production of automobiles decreases by 1 million, from 3 million to 2 million. The 1 million decrease in automobiles is the opportunity cost of the movement. b) By moving from point B to point A, the production of cameras decreases by 3 million, from 3 million to 0 million. The 3 million decrease in cameras is the opportunity cost of the movement. Topic: Increasing Opportunity Cost Skill: Analytical

Production point A B C D 9)

Beef (pounds) 0 2 4 6

and and and and

Wheat (bushels) 9 7 4 0

The table above presents the production possibilities of Farmer Brown. Use these data to calculate

The opportunity cost of a pound of beef is 1 bushel of wheat between points A and B, 1 1/2 bushels of wheat between points B and C, and 2 bushels of wheat between points C and D. The opportunity cost of a bushel of wheat is 1/2 pound of beef between points D and C, 2/3 pound of beef between points C and B, and 1 pound of beef between points B and A. Farmer Brown does use resources that are more productive in one activity than the other because the opportunity costs of producing beef and wheat increase as more beef and wheat are produced. If the resources were equally productive in both activities, the opportunity costs would be constant. Topic: Marginal Cost Skill: Analytical

Production point A B C D E

Fish (pounds) 0 1 2 3 4

and and and and and

Berries (pounds) 50 45 35 20 0

10) The table above shows the production possibilities frontier for the nation of Isolanda. a) Find the marginal cost of a pound of fish using the above PPF. b) How does the marginal cost of a pound of fish change as more fish are caught?

THE ECONOMIC PROBLEM

Answer:

Fish (pounds) 0.5 1.5 2.5 3.5

Marginal cost (pounds of berries) 5 10 15 20

a) The table above shows the marginal cost of a pound of fish. Remember that marginal cost is calculated at the mid-point, that is, midway between the two production possibilities. For instance, moving from production point A to production point B has an opportunity cost of 5 pounds of berries. Because this movement gains 1 pound of fish, the marginal cost is 5 pounds of berries. We then attribute this marginal cost to the point midway between points A and B, which is 0.5 pounds of fish. b) As more fish are caught, the marginal cost of a fish increases.

157

shows the maximum number of tacos that people are willing to give up to get another slice of pizza. Topic: Marginal Cost Curve Skill: Analytical

12) Draw and describe a marginal cost curve for slices of pizza where the opportunity cost of producing a slice of pizza is measured as tacos forgone. Answer:

Topic: Marginal Benefit Curve Skill: Analytical

11) Draw and describe a marginal benefit curve for slices of pizza where the opportunity cost of consuming a slice of pizza is measured as tacos forgone. Answer:

A marginal cost curve is an upward sloping line with the marginal cost of pizza on the vertical axis and the quantity of pizza on the horizontal axis. A point on the curve shows the number of tacos that must be given up to get another slice of pizza. Topic: Allocative Efficiency Skill: Conceptual

13) If, on a nation’s production possibilities frontier, the marginal benefit of a slice of pizza is 500 tacos while the marginal cost of producing a slice of pizza is 750 tacos, to be allocatively efficient, what should be done? Answer:

The marginal benefit of a slice of pizza is less than its marginal cost. Therefore to be allocatively efficient, less pizza and more tacos should be produced.

A marginal benefit curve is a downward sloping line with the marginal benefit of the slice of pizza on the vertical axis and the quantity of slices of pizza on the horizontal axis. A point on the curve

158

CHAPTER 2

Topic: Allocative Efficiency Skill: Analytical

Quantity (millions of bushels of apples) 5 10 15 20 25

Marginal Benefit (oranges per bushel) 10 8 6 4 2

Marginal Cost (oranges per bushel) 1 3 6 9 12

14) Using the values for the marginal benefit and the marginal cost of a bushel of apples given in the table above, what is the allocatively efficient quantity of apples? Suppose 10 million bushels of apples are produced. Should the quantity be increased or deceased? What if 20 million bushels are produced; should the quantity be increased or decreased? Answer:

The allocatively efficient quantity of apples is 15 million bushels because this is the quantity at which the marginal benefit equals the marginal cost. If 10 million bushels of apples are produced, the marginal benefit exceeds the marginal cost, so more apples should be produced. If 20 million bushels of apples are produced, the marginal cost exceeds the marginal benefit and so fewer apples should be produced. Topic: Allocative Efficiency Skill: Analytical

marginal cost and marginal benefit of producing trucks in terms of the forgone cars. a) What is the marginal benefit of the 25th truck? b) What is the marginal cost of the 25th truck? c) Should the 25th truck be produced? Why or why not. d) What is the marginal benefit of the 75th truck? e) What is the marginal cost of the 75th truck? f) Should the 75th truck be produced? Why or why not? g) What is the allocatively efficient quantity of trucks? Answer:

a) 3 cars b) 1 car c) This truck should be produced because people value the 25th truck at 3 cars, but it only costs 1 car to make the truck. d) 1 car e) 3 cars f) This truck should not be produced because people value the 75th truck at 1 car, but it costs 3 cars to make the truck. g) 50 trucks is the allocatively efficient quantity of trucks. Topic: Comparative Advantage Skill: Analytical

China’s production in 1 day Cloth Cheese

8 16

Pakistan’s production in 1 day Cloth Cheese

4 12

16) The table above shows the amounts of cloth and cheese that China and Pakistan can produce in an hour. Which country has the comparative advantage in cloth and which country has the comparative advantage in cheese? Answer:

15) Suppose a factory can be designed to produce either trucks or cars. The figure above shows the

In China, to produce 8 cloths has an opportunity cost of 16 cheeses, so the opportunity cost of 1 cloth is (16 cheese)/(8 cloths) = 2 cheeses per cloth. In Pakistan, to produce 4 cloths has an opportunity cost of 12 cheeses. Hence the opportunity cost of 1 cloth is (12 cheeses)/(4 cloths) or 3 cheeses per cloth. Because China’s opportunity

THE ECONOMIC PROBLEM

159

cost of a cloth is lower, China has the comparative advantage in producing cloth. In China, to produce 16 cheeses has an opportunity cost of 8 cloths, so the opportunity cost of 1 cheese is (8 cloths)/(16 cheeses) = 1/2 cloth per cheese. In Pakistan, to produce 12 cheeses has an opportunity cost of 4 cloths. Hence the opportunity cost of 1 cheese is (4 cloths)/(12 cheeses) or 1/3 cloth per cheese. Because Pakistan’s opportunity cost of a cheese is lower, Pakistan has the comparative advantage in producing cheese. Topic: Comparative Advantage Skill: Analytical

Omar’s production in 1 day John’s production in 1 day Computers fixed Lines of code

12 800

Computers fixed Lines of code

4 200

17) Omar and John can fix computers or write computer programs. The table above shows the number of computers they can fix and the lines of code they can write in a day. a) Who, if anyone, has the absolute advantage? b) Who has the comparative advantage in fixing computers? Why? c) Who has the comparative advantage in writing programs? Why? Answer:

a) Omar has an absolute advantage in fixing computers and writing code because he can fix 12 per day compared to John who can fix only 4 per day, and can write 800 lines of code per day compared to John who can write only 200 lines a day. b) John has the comparative advantage in fixing computers. He has the comparative advantage because his opportunity cost of fixing one computer is 50 lines of computer code. Omar does not have a comparative advantage in fixing computers because his opportunity cost of fixing a computer is higher at 66.7 lines of code. c) Omar has the comparative advantage in writing programs. His opportunity cost of writing one line of code is .015 of a computer fixed. John does not have the comparative advantage in writing programs because his opportunity cost of writing one line of code is 0.02 computers fixed. (Alternatively, to write 1 line of

code costs Omar the opportunity to repair 1.5 percent of a computer and costs John the opportunity to repair 2.0 percent of a computer.) Topic: Comparative Advantage Skill: Analytical

18) Jake takes 40 minutes to fry a chicken and 10 minutes to toast a slice of bread. His brother Elwood takes 60 minutes to fry a chicken and 4 minutes to toast a slice of bread. Calculate each brother’s opportunity cost. Who has a comparative advantage in which activity? Explain. Will the brothers gain if they specialize? Answer:

Jake can spend an hour to fry 1.5 chickens or toast 6 slices of bread, which means 1 chicken costs him 6 slices of bread ÷ 1.5 chickens = 4 slices of bread per chicken while 1 slice of bread costs him 1.5 chickens ÷ 6 slices of bread = 0.25 of a chicken per slice of bread. Elwood spends an hour to fry 1 chicken or toast 15 slices of bread, which means 1 chicken costs him 15 slices of bread ÷ 1 chicken = 15 slices of bread per chicken while 1 slice of bread costs him 1 chicken ÷ 15 slices of bread = 0.067 of a chicken per slice of bread. Thus, Jake has a comparative advantage (a lower opportunity cost) in frying chickens, whereas Elwood has a comparative advantage in toasting bread. The brothers can gain from specialization. For example, suppose each of them spends 2 hours to fry chickens and 2 hours to toast bread. Then, Jake will produce 3 chickens and 12 slices of bread and Elwood will produce 2 chickens and 30 slices of bread, so that together they will produce 5 chickens and 42 slices of bread. But, if Jake specializes in his comparative advantage, frying chickens and spends all 4 hours frying chickens, he will produce 6 chickens. And if Elwood specializes in his comparative advantage, toasting bread, and spends all 4 hours toasting bread, he will produce 60 slices of bread. Then, together they will produce 6 chickens and 60 slices of bread. The gain from specialization is 1 extra chicken and 18 extra slices of bread. Topic: Comparative Advantage Skill: Analytical

19) Mary takes 4 minutes to make a sandwich and 6 minutes to mix a cocktail. Her sister Ash takes 4

160

CHAPTER 2

minutes to make a sandwich and 4 minutes to mix a cocktail. Calculate each sister’s opportunity cost. Which of the two sisters has an absolute advantage in making sandwiches? In mixing cocktails? Which of the two has a comparative advantage in making sandwiches? In mixing cocktails? Answer:

Mary can spend an hour to make 15 sandwiches or mix 10 cocktails, which means 1 sandwich costs her 10 cocktails ÷ 15 sandwiches = 0.67 of a cocktail per sandwich while 1 cocktail costs her 15 sandwiches ÷ 10 cocktails = 1.5 sandwiches per cocktail. Ash can spend an hour to make 15 sandwiches or mix 15 cocktails, which means 1 sandwich costs her 15 cocktails ÷ 15 sandwiches = 1 cocktail per sandwich and 1 cocktail costs 1 sandwich per cocktail. The sisters are equally productive in sandwiches (15 sandwiches per hour) and therefore neither of them has an absolute advantage in sandwiches. But Ash has an absolute advantage in cocktails. She can mix 15 cocktails per hour while Mary can only mix 10. To see the sisters’ comparative advantage we compare their opportunity costs. Mary’s opportunity cost of a sandwich (0.67 of a cocktail per sandwich) is lower than Ash’s (1 cocktail per sandwich), so Mary has a comparative advantage in sandwiches. Ash’s opportunity cost of a cocktail (1 sandwich per cocktail) is lower than Mary’s (1.5 sandwiches per cocktail), so Ash has a comparative advantage in cocktails.

in one day than can Nation A. Nation B has the comparative advantage in computer production and Nation A has the comparative advantage in software. The nation with the lowest opportunity cost of producing a good has the comparative advantage in that good. In Nation A, to produce 100 computers has the opportunity cost of 140 units of software forgone, so the opportunity cost of 1 computer equals (140 units of software)/(100 computers) = 1.4 units of software per computer. In Nation B, similar calculations show that the opportunity cost for a computer is 1.25 units of software per computer. Nation B’s opportunity cost is lower, so Nation B has the comparative advantage in computers. For software, in Nation A the opportunity cost of a unit of software is (100 computers)/(140 units of software) = 0.71 computers per unit of software while in Nation B the opportunity cost is (120 computers)/(150 units of software) = 0.80 computers per unit of software. Nation A’s opportunity cost is lower, so Nation A has the comparative advantage in software. Topic: Comparative Advantage Skill: Analytical

Topic: Comparative Advantage Skill: Analytical

Nation A’s production in 1 day

Nation B’s production in 1 day

Computers Software

Computers Software

100 140

120 150

20) Two nations can produce computers and software in the amounts given in the table above. Does either nation have an absolute advantage in producing the products? Which nation has a comparative advantage in computers? Which nation has a comparative advantage in software? Explain your answers. Answer:

Nation B has an absolute advantage in producing both goods because it can produce more of both

21) The figure above shows Prakash’s and Gail’s production possibilities frontiers for writing books and magazine articles. a) What is Prakash’s opportunity cost of a book? What is Gail’s opportunity cost? Who has the comparative advantage in writing books? b) Who has the comparative advantage in writing magazine articles?

THE ECONOMIC PROBLEM

c) According to their comparative advantages, who should write books and who should write magazine articles? Answer:

a) In a year, Prakash can write 2 books or 40 magazine articles. Hence the opportunity cost of 1 book is (40 magazine articles) ÷ (2 books) = 20 magazine articles per book. In a year, Gail can write 3 books or 30 magazine articles. Hence the opportunity cost of 1 book is (30 magazine articles) ÷ (3 books) = 10 magazine articles per book. Gail’s opportunity cost of writing books is lower than Prakash’s, so

161

Gail has the comparative advantage in writing books. b) Prakash has the comparative advantage in writing magazine articles. c) Gail has the comparative advantage in writing books, so she should write books. Prakash has the comparative advantage in writing magazine articles, so he should write magazine articles.

CHAPTER 3

DEMAND AND SUPPLY

Topic: Price and Opportunity Cost Skill: Analytical

Topic: Change in Demand, Income Skill: Analytical

1)

3)

The price of a computer is $1,000 and the price of a car is $12,000. What is the relative price of a car? What is the relative price of a computer? Answer:

Soft drinks are a normal good. Draw a graph showing the effect of an increase in income on the demand for soft drinks. Answer:

The relative price of a car is $12,000 per car ÷ $1,000 per computer = 12 computers per car. The relative price of a computer is $1,000 per computer ÷ $12,000 per car = 0.083 of a car per computer. Topic: Change in Demand, Prices of Related Goods Skill: Analytical

2)

Soft drinks and milk are substitutes for consumers. Draw a graph showing the effect of an increase in the price of milk on the demand for soft drinks. Answer:

The increase in the income increases the demand for soft drinks, as illustrated in the figure above. Topic: Surplus Skill: Analytical

The increase in the price of milk increases the demand for soft drinks, as illustrated in the figure above.

Price (dollars per pound of cat food) 1.00 1.50 2.00 2.50 3.00 4)

Quantity demanded (tons of cat food per year) 52 46 43 40 35

Quantity supplied (tons of cat food per year) 15 26 34 30 44

The above table gives the demand and supply schedules for cat food. If the price is $3.00 per pound of cat food, will there be a shortage, a sur-

164

CHAPTER 3

plus, or is this price the equilibrium price? If there is a shortage, how much is the shortage? If there is a surplus, how much is the surplus? If $3.00 is the equilibrium price, what is the equilibrium quantity? Answer:

At a price of $3.00 per pound of cat food, there is a surplus. The surplus equals 44 tons (the quantity supplied) minus 35 tons (the quantity demanded), or 9 tons of cat food. Topic: Shortage Skill: Analytical

Price (dollars per pound of cat food) 1.00 1.50 2.00 2.50 3.00 5)

Topic: Equilibrium Skill: Analytical

Price (dollars per pound of cat food) 1.00 1.50 2.00 2.50 3.00 6)

Quantity demanded (tons of cat food per year) 52 46 43 40 35

Quantity supplied (tons of cat food per year) 15 26 34 30 44

The above table gives the demand and supply schedules for cat food. If the price is $1.00 per pound of cat food, will there be a shortage, a surplus, or is this price the equilibrium price? If there is a shortage, how much is the shortage? If there is a surplus, how much is the surplus? If $3.00 is the equilibrium price, what is the equilibrium quantity? Answer:

At a price of $1.00 per pound of cat food, there is a shortage. The shortage equals 52 tons (the quantity demanded) minus 15 tons (the quantity supplied), or 37 tons of cat food.

Quantity demanded (tons of cat food per year) 52 46 43 40 35

Quantity supplied (tons of cat food per year) 15 26 34 30 44

The above table gives the demand and supply schedules for cat food. What is the equilibrium price and quantity? Answer:

The equilibrium price is $2.50 per pound of cat food because that is the price at which the quantity demanded equals the quantity supplied. The equilibrium quantity of cat food is 40 tons per year. Topic: Predicting Changes in Price and Quantity; Demand Decreases, Supply Increases Skill: Analytical

Price (dollars per pound of cat food) 1.00 1.50 2.00 2.50 3.00 7)

Quantity demanded (tons of cat food per year) 52 46 43 40 35

Quantity supplied (tons of cat food per year) 15 26 34 30 44

The above table gives the demand and supply schedules for cat food. If the supply increases by 20 tons at every price, what is the new equilibrium price and quantity? Answer:

The equilibrium price is $1.50 per pound of cat food because that is the price at which the quantity demanded equals the (new) quantity supplied. The equilibrium quantity of cat food is 46 tons per year.

DEMAND AND SUPPLY

Topic: Shortage Skill: Analytical

165

Topic: Shortage Skill: Analytical

9) 8)

The diagram above illustrates the market for apartments in Victoria, British Columbia. a) If the current rent is $300 per month, is there a shortage or surplus in the apartment market and how much is the shortage or surplus? b) What is the equilibrium rent and quantity of apartments? Answer:

a) If the rent is $300 per month, there is a shortage of 30,000 apartments. b) The equilibrium rent is $400 per month and the equilibrium quantity is 40,000 apartments.

In the figure above, if the price is $8 a unit, is there a shortage or surplus and what is the amount of any shortage or surplus? What is the equilibrium price and quantity? Answer:

At a price of $8 there is a surplus because the quantity supplied exceeds the quantity demanded. The amount of the surplus is 4 units per month. The equilibrium price is $4 a unit and the equilibrium quantity is 3 units per month Topic: Predicting Changes in Price and Quantity; Demand Changes Skill: Analytical

10) Last year a very severe ice storm hit the north counties of New York state, and the states of Vermont and Maine. Electric poles were down and no one had power for days. It was reported that the price of kerosene heaters skyrocketed and the number purchased increased during this time. Using a supply and demand diagram, show the impact of the ice storm on the market for kerosene heaters.

166

CHAPTER 3

Answer:

The demand for kerosene heaters increased, so the demand curve shifted rightward, as illustrated above. As a result, the price rises and quantity increases. Topic: Predicting Changes in Price and Quantity; Demand Changes Skill: Analytical

11) Consumers can use either natural gas or heating oil to warm their houses. Suppose the price of natural gas increases. Use a demand and supply diagram to show the impact of the higher price of natural gas on the market for home heating oil.

Answer:

Natural gas and home heating oil are substitutes. The increase in the price of natural gas increases the demand for home heating oil, so the demand curve for home heating oil shifts rightward, as illustrated above. As a result, the price rises and quantity increases. Topic: Predicting Changes in Price and Quantity; Demand Changes Skill: Analytical

Price (dollars per disk) 0.50 1.00 1.50 2.00 2.50 3.00

Quantity demanded (millions of disks per month) 50 40 30 20 10 0

Quantity supplied (millions of disks per month) 0 15 30 45 60 75

12) Suppose the market for CD-Rs has the demand and supply schedules shown in the table above. Graph the demand and supply curves. What is the equilibrium price and the equilibrium quantity in this market? Suppose the current price is $2.00. What is the quantity of CD-Rs sold? Explain. Is there a shortage or a surplus? How big is it? Explain.

DEMAND AND SUPPLY

167

What is the new equilibrium price and the new equilibrium quantity of CD-Rs?

Answer:

Answer:

The graph is shown above. The equilibrium price is $1.50. The equilibrium quantity is 30 million disks per month. The quantity sold is 20 million disks per month. Although at $2.00 suppliers want to sell 45 million disks, the buyers want to buy only 20 million and so 25 million disks won’t be sold. Since the quantity supplied, 45 million disks, is greater than the quantity demanded, 20 million, there is a surplus, 25 million disks. Topic: Predicting Changes in Price and Quantity; Demand Changes Skill: Analytical

Price (dollars per disc) 0.50 1.00 1.50 2.00 2.50 3.00

Quantity demanded (millions of disks per month) 50 40 30 20 10 0

Quantity supplied (millions of disks per month) 0 15 30 45 60 75

13) Suppose the market for CD-Rs has the demand and supply schedules shown in the table above. Graph the demand and supply curves and mark in the equilibrium price and quantity. Now, suppose a technological advance increases the quantity of disks supplied at each price by 25 million.

The graph is shown above. The initial equilibrium price is $1.50 and the initial equilibrium quantity is 30 million disks per month. The technological advance shifts the supply curve rightward by 25 million disks from S0 to S1. As a result, the equilibrium price falls to $1.00 and the equilibrium quantity increases to 40 million disks. Topic: Predicting Changes in Price and Quantity; Demand Changes Skill: Analytical

Quantity Quantity demanded supplied (millions of (millions of disks per disks per month) month) 0.50 50 0 1.00 40 15 1.50 30 30 2.00 20 45 2.50 10 60 3.00 0 75 14) Suppose the market for CD-Rs has the demand and supply schedules shown in the table above. Graph the demand and supply curves and mark the equilibrium price and quantity. Now, suppose a decrease in the price of a CD burner increases the quantity of disks demanded at each price by Price (dollars per disc)

168

CHAPTER 3

20 million. What are the new equilibrium price and equilibrium quantity of CD-Rs? Answer:

The figure above shows the supply and demand curves. The initial equilibrium price is $1.50 and the initial equilibrium quantity is 30 million disks per month. The decrease in the price of a CD shifts the demand curve rightward by 25 million disks to D1. As a result, the equilibrium price rises to $2.00 and the equilibrium quantity increases to 45 million disks.

CHAPTER 4

ELASTICITY

Topic: Calculating Elasticity Skill: Analytical

1)

Answer:

The price elasticity of demand = (percentage change in the quantity demanded) ÷ (percentage change in the price). Using the midpoint method to calculate the percentages, the percentage change in the quantity demanded = (700 − 500) ÷ (600) × 100 = 33.3 percent and the percentage change in the price is ($5 − $7) ÷ ($6) = −33.3 percent. We ignore the negative sign so that the price elasticity of demand equals (33.3 percent) ÷ (33.3 percent), or 1.00.

Suppose the price of flour increases from $0.80 to $1.00 a pound and the quantity demanded decreases from 100 pounds to 95 pounds. Using the midpoint method, what is the price elasticity of demand for flour? Is the demand for flour elastic or inelastic? Answer:

The price elasticity of demand is 0.23. (The price elasticity is calculated from [(100 pounds − 95 pounds) ÷ 97.5 pounds] ÷ [($0.80 − $1.00) ÷ $0.90] = 0.23.) Because the elasticity is less than one, the demand is inelastic.

Topic: Calculating Elasticity Skill: Analytical

4) Topic: Calculating Elasticity Skill: Analytical

2)

If the price of suntan lotion increases from $6 to $8 per bottle and quantity demanded decreases from 900,000 bottles to 845,000 bottles, using the midpoint method, what is the price elasticity of demand for suntan lotion?

Answer:

The price elasticity of demand = (percentage change in the quantity demanded) ÷ (percentage change in the price). Use the midpoint method to calculate the percentages. So the percentage change in the quantity demanded is (10 million − 8 million) ÷ (9 million) = 22 percent and the percentage change in the price is ($7 − $5) ÷ ($6) = 33 percent. Therefore the elasticity of demand equals (22 percent) ÷ (33 percent) = 0.67. Demand is inelastic because the price elasticity of demand is less than 1.

Answer:

The price elasticity of demand = (percentage change in the quantity demanded) ÷ (percentage change in the price). Use the midpoint method to calculate the percentages. Thus the percentage change in the quantity demanded = (900,000 − 845,000) ÷ (872,500) = 6.3 percent and the percentage change in the price is ($8 − $6) ÷ ($7) = 28.6 percent. Therefore the elasticity of demand equals (6.3 percent) ÷ (28.6 percent) = 0.22. Topic: Calculating Elasticity Skill: Analytical

3)

When the price of a movie ticket falls from $7 to $5, the quantity of tickets demanded increases from 500 to 700 a day. What is the price elasticity of demand for movie tickets? (Use the midpoint method.)

If the price of a magazine increases from $5 to $7 and the quantity demanded of the magazines decreases from 10 million per month to 8 million per month, using the midpoint method, what is the price elasticity of demand? Show your work. Is the demand elastic, inelastic, or unit elastic?

Topic: Calculating Elasticity Skill: Analytical

5)

Suppose the price elasticity of demand for bouquets of flowers is 4.0. You are charging $8 per bouquet. If you want to increase the quantity of bouquets you sell by 20 percent, what price should you charge?

170

CHAPTER 4

Answer:

The price elasticity of demand = (percentage change in the quantity demanded) ÷ (percentage change in price). Using the numbers in the problem gives 4.0 = (20 percent) ÷ (percentage change in price), where 20 percent is the desired increase in the quantity of bouquets demanded. Therefore, rearranging the formula shows that (percentage change in price) = (20 percent) ÷ 4.0 = 5 percent. In order to increase the quantity demanded, the price needs to be decreased, so the price needs to be decreased by 5 percent. Next, 5 percent of $8 is ($8) × (5 percent) = 40¢. Thus the price needs to be decreased to $7.60 per bouquet.

Topic: Calculating Elasticity Skill: Analytical

A B C D E 7)

Topic: Calculating Elasticity Skill: Analytical

6)

The price elasticity of demand for adults for cigarettes is 0.4. If government wants to reduce smoking among adults by 15 percent, by what percentage should it raise the price of cigarettes? Teenagers have a higher price elasticity of demand for cigarettes than do adults. Suppose the price elasticity of teenager’s demand for cigarettes is 0.8. If the government imposes a tax on cigarettes that raises the price to reduce overall smoking by 15 percent, by what percentage will the government reduce teenage smoking?

Price (dollars) 100 80 60 40 20

Quantity demanded (units per week) 40 60 80 100 120

The table above gives the demand schedule for a good. Using the midpoint method, find the price elasticity of demand between points A and B, between B and C, between C and D, and between D and E. Answer:

The price elasticity of demand between points A and B is 1.80. Between points B and C, the elasticity of demand is 1.00. Between points C and D, the elasticity of demand is 0.56. And, between points D and E, the elasticity of demand is 0.27. Topic: Calculating Elasticity Skill: Analytical

Answer:

The government should raise the price of cigarettes by 37.5 percent. The price elasticity of demand = (percentage change in the quantity demanded) ÷ (percentage change in the price). So for the given elasticity, 0.4, and the desired percentage change in quantity, 15 percent, the formula shows that 0.4 = (15 percent) ÷ (percentage change in price). Solving for the percentage change in price gives (percentage change in price) = (15 percent) ÷ (0.4), which equals 37.5 percent. For teenagers, given their elasticity of demand for cigarettes, 0.8, and the price increase, 37.5 percent, the percentage change in the quantity demand equals (37.5 percent) × (0.8), which is 30.0 percent. So a 37.5 percent price hike in cigarettes will decrease the quantity of cigarettes demanded by teenagers by 30 percent.

8)

The figure above shows the demand curve for pizza. Using the midpoint method and moving from point A to point B, calculate the a) percentage change in price. b) percentage change in quantity demanded. c) price elasticity of demand. Answer:

a) Between points A and B, the price falls 40 percent.

ELASTICITY

171

b) Between points A and B, the quantity increases 10 percent. c) The price elasticity of demand is 0.25. Topic: Elasticity Along a Straight-Line Demand Curve Skill: Analytical

9)

In the figure above, at which point (a, b, or c) along the linear demand curve illustrated would demand be a) most elastic? b) most inelastic? Answer:

a) The demand would be most elastic at point a. b) The demand would be most inelastic at point c. Topic: Total Revenue and Elasticity Skill: Analytical

A B C D E

Price (dollars) 100 80 60 40 20

Quantity demanded (units per week) 40 60 80 100 120

10) The table above gives the demand schedule for a good. What is the total revenue at point A? At point B? At point C? At point D? At point E? Answer:

The total revenue is the price times the quantity demanded. Hence at point A, the total revenue is $4,000. At point B it is $4,800. At point C it is

$4,800. At point D it is $4,000. And at point E it is $2,400. Topic: Total Revenue and Elasticity Skill: Analytical

Price (dollars per admission) 10 8 6 4 2

Quantity demanded (thousands of visits per week) 100 200 300 400 200

11) The table above gives the demand schedule for museum visits. a) You, as the resident economist, have been given the task of maximizing the museum’s total revenue. What admission price should you charge? b) What is the elasticity of demand between $6 and $4? c) Moving along the demand schedule from $10 to $8 to $6 and ultimately to $4, how does the price elasticity of demand change in size? Answer:

a) The admission price you should charge is $6. The total number of visits will be 300,000 and total revenue is $6 × 300,000 = $1,800,000. No other price gives you this much total revenue. b) The price elasticity of demand equals [(300 visits − 400 visits) ÷ 350 visits] ÷ [($6 − $4) ÷ $5] = (0.29) ÷ (0.4) = 0.71. c) Moving along the demand schedule to lower prices, the elasticity of demand falls in size. Topic: Total Revenue and Elasticity Skill: Analytical

Price (dollars) 1.00 1.25 1.50 1.75

Total revenue (dollars) 110 125 125 100

12) Steve sells hotdogs from a vending cart downtown. The table above shows his daily total reve-

172

CHAPTER 4

nues at four different prices. Between which two prices is the demand for hotdogs a) elastic? b) unit elastic? c) inelastic? Answer:

a) Steve’s demand is elastic between $1.50 and $1.75. In this range, when Steve raises his price from $1.50 to $1.75 per hot dog, his total revenue falls, which means that the demand is elastic. b) Steve’s demand is unit elastic between $1.25 and $1.50. In this range, when Steve raises his price from $1.25 to $1.50 per hot dog, his total revenue does not change, which means that the demand is unit elastic. c) Steve’s demand is inelastic between $1.00 and $1.25. In this range, when Steve raises his price from $1.00 to $1.25 per hot dog, his total revenue rises, which means that the demand is inelastic. Topic: Total Revenue and Elasticity Skill: Analytical

13) Suppose bad weather decreases the wheat harvest by 12 percent. If the price elasticity of demand for wheat is 0.6, how would the crop failure affect the price of wheat? Would the crop decrease benefit or harm wheat farmers? Answer:

When the price elasticity of demand is 0.6, a 12 percent decrease in quantity brings about a 20 percent increase in the price. Wheat farmers’ total revenue increases because the demand is inelastic. So, taken as a group, wheat farmers’ total revenue increases. However, the benefit is distributed unequally. Those farmers whose crop was destroyed because of the bad weather are harmed whereas those farmers whose crop was spared the bad weather, benefited.

Answer:

In this case, the cross elasticity of demand = (percentage change in the quantity of Chevy trucks demanded) ÷ (percentage change in the price of a Ford truck). Use the midpoint method to calculate the percentages. Thus the percentage change in the quantity of Chevy trucks demanded = (144,000 − 112,000) ÷ (128,000) = 25 percent and the percentage change in the price of a Ford truck is ($19,000 − $18,000) ÷ ($18,500) = 5.4 percent. Thus the cross elasticity of demand equals (25 percent) ÷ (5.4 percent) = 4.625. Topic: Cross Elasticity of Demand Skill: Analytical

15) When the price of bananas rises 2 percent, the quantity demanded of peanut butter falls 4 percent. a) What is the cross elasticity of demand between these two goods? b) How are these goods related? c) If the price of bananas rises, how will that affect the demand curve for peanut butter? Answer:

a) The cross elasticity of demand equals −2. b) Because the cross elasticity of demand is negative, the cross elasticity indicates that the two goods are complements. c) If the price of bananas rises, the demand for peanut butter decreases and the demand curve for peanut butter shifts leftward. Topic: Cross Elasticity of Demand Skill: Analytical

16) Consider two goods: peanut butter and jelly. If the price of jelly increases from $2 a jar to $3 per jar and the quantity demanded of peanut butter decreases from 50 jars to 45 jars, what is the cross elasticity of demand? Are the goods substitutes or complements? Answer:

Topic: Cross Elasticity of Demand Skill: Analytical

14) When the price of Ford pickup trucks rises from $18,000 to $19,000, the quantity of Chevy trucks demanded increases from 112,000 to 144,000. What does the cross elasticity of demand between Ford and Chevy trucks equal?

The cross elasticity of demand equals −0.275. The value is negative so the goods are complements. Topic: Income Elasticity of Demand Skill: Analytical

17) A 10 percent increase in income brings about a 15 percent decrease in the demand for a good. What is the income elasticity of demand and is the good a normal good or an inferior good?

ELASTICITY

Answer:

The income elasticity of demand is −1.5. The good is an inferior good. Topic: Income Elasticity of Demand Skill: Analytical

18) If income increases from $50,000 to $60,000 while the demand for a good increases from 100 units to 125 units, what is the income elasticity of demand? Is the good a normal good or an inferior good? Answer:

The income elasticity equals 1.22. Because the income elasticity of demand is positive, the good is a normal good. Topic: Income Elasticity of Demand Skill: Analytical

19) The income elasticity of demand for movies in the United States is 3.41. If people’s incomes decrease by 1 percent, what is the decrease in the quantity of movies demanded? Answer:

The income elasticity of demand = (percentage change in quantity demanded) ÷ (percentage change in income). Using the numbers in the problem gives 3.41 = (percentage change in quantity demanded) ÷ (1 percent). Rearranging the formula shows (percentage change in quantity demanded) = (1 percent) × 3.41 = 3.41 percent. Therefore the quantity of movies demanded decreases by 3.41 percent. Topic: Income Elasticity of Demand Skill: Analytical

20) Jenny’s weekly income increases from $500 to $650. As a result, she goes out for dinner one day a week instead of one day every other week. What is Jenny’s income elasticity of demand for restaurant dinners? Answer:

The income elasticity of demand = (percentage change in the quantity demanded) ÷ (percentage change in income). Using the midpoint method to calculate the percentages, the percentage change in the quantity of meals demanded = (1.0 − 0.5) ÷ (0.75) × 100 = 66.67 percent and the percentage change in income ($650 − $500) ÷ ($575) × 100 = 26.1 percent. Therefore the income elasticity of

173

demand equals (66.67 percent) ÷ (26.1 percent) = 2.56. Topic: Income Elasticity of Demand Skill: Analytical

Income (dollars) 18,000 22,000

Quantity demanded (pounds per year) 60 140

21) The table above gives Sharon’s demand for ground beef at two different income levels. Use the midpoint method in this problem. a) What is the percentage change in Sharon’s income? b) What is the percentage change in the quantity demanded? c) What is Sharon’s income elasticity of demand for ground beef? d) Is ground beef a normal or an inferior good for Sharon? Answer:

a) The percentage change in Sharon’s income is 20 percent. b) The percentage change in the quantity of ground beef demanded is 80 percent. c) Sharon’s income elasticity of demand for ground beef is 4.00. d) Because the income elasticity is positive, ground beef is a normal good for Sharon. Topic: Calculating the Elasticity of Supply Skill: Analytical

22) If the price increases by 20 percent and the quantity supplied increases by 40 percent, what does the elasticity of supply equal? Answer:

The price elasticity of supply = (percentage change in the quantity supplied) ÷ (percentage change in price) = (40 percent) ÷ (20 percent) = 2.00. Topic: Calculating the Elasticity of Supply Skill: Analytical

23) Suppose the quantity supplied of computers increases from 2 million to 4 million units as the price of a computer increases from $600 to $700. What does the price elasticity of supply equal? Answer:

The price elasticity of supply = (percentage change in the quantity supplied) ÷ (percentage change in

174

CHAPTER 4

the price). Use the midpoint method to calculate the percentages. Thus the percentage change in the quantity supplied = (4,000,000 − 2,000,000) ÷ (3,000,000) = 66.7 percent and the percentage change in the price is ($700 − $600) ÷ ($650) = 15.4 percent. Therefore the elasticity of supply equals (66.7 percent) ÷ (15.4 percent) = 4.33.

Between C and D, the elasticity of supply is 0.71. Between D and E, the elasticity of supply is 0.60. Topic: Calculating the Elasticity of Supply Skill: Analytical

Topic: Calculating the Elasticity of Supply Skill: Analytical*

24) When the price of oil is $20 per barrel, the quantity of oil supplied is 74 million barrels per day. When the price of oil is $15 per barrel, the quantity of oil supplied is 71 million barrels per day. What is the elasticity of supply of oil? (Use the midpoint method.) Answer:

The price elasticity of supply = (percentage change in the quantity supplied) ÷ (percentage change in the price). Using the midpoint method to calculate the percentages, the percentage change in the quantity supplied = (71 million − 74 million) ÷ (72.5 million) × 100 = −4.14 percent and the percentage change in the price is ($15 − $20) ÷ ($17.50) × 100 = −28.57 percent. Therefore the elasticity of supply equals (−4.14 percent) ÷ (−28.57 percent) = 0.145. Topic: Calculating the Elasticity of Supply Skill: Analytical

A B C D E

Price (dollars) 100 80 60 40 20

Quantity demanded (units per week) 120 100 80 60 40

25) The table above gives the supply schedule for a product. Using the midpoint method, find the price elasticity of supply between points A and B, between B and C, between C and D, and between D and E. Answer:

Between A and B, the elasticity of supply is 0.82. Between B and C, the elasticity of supply is 0.78.

26) June makes holiday wreaths and sells them during the holiday season. The figure above shows her supply curve of wreaths per week. Use the midpoint method in this problem. a) Calculate the percentage change in quantity between points A and B. b) Calculate the percentage change in price between points A and B. c) Calculate the price elasticity of supply between points A and B. Answer:

a) The percentage change in quantity is 20 percent. b) The percentage change in price is 20 percent. c) The price elasticity of supply is 1.0.

CHAPTER 5

EFFICIENCY AND EQUITY

Topic: Willingness To Pay Skill: Analytical

Topic: Consumer Surplus Skill: Analytical

2)

Jenn is willing to pay $75 for a purse and the purse’s price is $60. What is Jenn’s consumer surplus? Answer:

The consumer surplus equals the difference between the marginal benefit of the good and the price actually paid. Jenn is willing to pay $75 for the purse, so its marginal benefit to her is $75. However, she only must pay $60, so Jenn has a consumer surplus of $75 − $60 = $15. Topic: Consumer Surplus Skill: Analytical

3)

1)

The figure shows the demand curve for hotel rooms at a local resort. a) If the hotel charges $120 per night, how many rooms will they rent? b) If there are only 40 rooms available, how much are customers willing to pay for a room? c) If 60 rooms are available, how much are customers willing to pay? d) What do the dollars in your answer to part (c) represent? Answer:

a) At $120 a night, the hotel will rent 20 rooms. b) If 40 rooms are available, customers are willing to pay $90 per room. c) If 60 rooms are available, customers are willing to pay $60 per room. d) The $60 represents the dollars worth of other goods and services that customers are willing to forgo to get one more night in a hotel room.

Jason needs help getting ready for the next test in his economics course and would like to hire Maria, an economics tutor to help him. Jason is willing to pay $30 for the first hour of tutoring, $25 for the second, $20 for the third, $15 for the fourth, and $10 for the fifth. The equilibrium price for tutoring is $15 per hour. For how many hours of tutoring will Jason hire Maria? Why this amount of hours? What is Jason’s consumer surplus, if any, from the tutoring? What is Maria’s consumer surplus from the tutoring? Answer:

Jason will hire Maria for 4 hours of tutoring. At $15 per hour, the marginal benefit to Jason of the first 4 hours exceeds the price. But the marginal benefit from the fifth hour is less than the price and so Jason will not hire Maria for 5 hours. Jason’s total consumer surplus is $30, the sum of $15 from the first hour plus $10 from the second plus $5 from the third plus $0 from the fourth. Maria has no consumer surplus in this market because she is the producer not the consumer.

176

CHAPTER 5

Topic: Consumer Surplus Skill: Analytical

4)

The figure above shows the demand curve for DVDs. Use it to illustrate the consumer surplus if the price of a DVD is $15. Answer:

Topic: Consumer Surplus Skill: Analytical

5)

The figure above shows the demand curve for pizza. a) What is the marginal benefit of the 20th pizza? b) What is the maximum price the consumer is willing to pay for the 20th pizza? c) If the price of a pizza is $6, what is the consumer surplus of the 20th pizza? d) If the price of a pizza is $10, what is the consumer surplus? e) If the price of a pizza is $6, what is the consumer surplus? Answer:

The consumer surplus is the area under the marginal benefit curve and above the price and is illustrated in the figure above.

a) The marginal benefit of the 20th pizza is $10. b) The maximum price the consumer is willing to pay for the 20th pizza is $10. c) If the price of a pizza is $6, the consumer surplus of the 20th pizza is $4. d) If the price of a pizza is $10, the consumer surplus is $40. e) If the price of a pizza is $6, the consumer surplus is $160.

EFFICIENCY AND EQUITY

177

Topic: Consumer Surplus Skill: Analytical

Topic: Consumer Surplus Skill: Analytical

6)

7)

The figure above shows Cindy’s demand for CDs per year. a) What is Cindy’s total consumer surplus if the price of a CD is $12? b) What is Cindy’s total consumer surplus if the price of a CD is $9? c) What happens to Cindy’s consumer surplus when the price of a CD falls? Answer:

a) Her consumer surplus when the price of a CD is $12 equals $15, the area of the triangle under the demand curve and above the price. b) Her consumer surplus when the price of a CD is $9 equals $60, the area of the triangle under the demand curve and above the price. c) As the price of a CD falls, Cindy’s consumer surplus increases. This result reflects the observation that consumers are better off when prices are lower.

The diagram above depicts the demand for, and market price of, buckets of raw oysters in Orlando. a) What is the consumer surplus of the person who buys the 100th bucket of oysters? b) What is the consumer surplus of the person who buys the 200th bucket of oysters? c) What is the consumer surplus of the person who buys the 300th bucket of oysters? d) What is the consumer surplus in the market? Answer:

a) The consumer surplus of the person who buys the 100th bucket of oysters is $6. b) The consumer surplus of the person who buys the 200th bucket of oysters is $3. c) The consumer surplus of the person who buys the 300th bucket of oysters is $0. d) The consumer surplus in the market is $1,350. Topic: Producer Surplus Skill: Analytical

8)

Maria helps tutor students taking economics. The equilibrium price for tutoring is $15 per hour. Maria has determined her opportunity cost per hour to be $6 for the first, $9 for the second, $12 for the third, $15 for the fourth, and $18 for the fifth. How many hours will Maria tutor? Why this amount of hours? What, if any, is Maria’s producer surplus?

178

CHAPTER 5

Answer:

Maria will tutor for 4 hours. For all of the first 4 hours of tutoring, Maria’s marginal cost of tutoring is less than or equal to the price she receives from tutoring. Maria will not tutor the fifth hour because for this hour her marginal cost exceeds the price. Maria makes a producer of surplus of $18, the sum of $9 on the first hour plus $6 on the second hour plus $3 on the third hour plus $0 on the fourth hour. Topic: Producer Surplus Skill: Analytical

d) If the price is $6 per pizza, the producer surplus is $20. e) If the price is $8 per pizza, the producer surplus is $80. f) If the price is $10 per pizza, the producer surplus is $180. Topic: Efficiency of Competitive Markets Skill: Analytical

10) Jason hires Maria to tutor him in economics. Jason is willing to pay $30 for the first hour of tutoring, $25 for the second, $20 for the third, $15 for the fourth, and $10 for the fifth. Maria has an opportunity cost per hour of $6 for the first, $9 for the second, $12 for the third, $15 for the fourth, and $18 for the fifth. What will be the equilibrium quantity of hours tutored and the equilibrium price? Explain why this quantity and price is the equilibrium. What is Jason’s consumer surplus and what is Maria’s producer surplus? Answer:

9)

The figure above shows the supply curve for pizzas. a) What is the marginal cost of the 20th pizza? b) What is the minimum supply price of the 20th pizza? c) If the price is $6 per pizza, what is the producer surplus on the 20th pizza? d) If the price is $6 per pizza, what is the producer surplus? e) If the price is $8 per pizza, what is the producer surplus? f) If the price is $10 per pizza, what is the producer surplus? Answer:

a) The marginal cost of the 20th pizza is $6. b) The minimum supply price of the 20th pizza is $6. c) If the price is $6 per pizza, the producer surplus on the 20th pizza is zero.

The equilibrium quantity will be 4 hours of tutoring and the equilibrium price will be $15 per hour. The equilibrium quantity will be 4 hours of tutoring because for any of these first 4 hours, the marginal social benefit (to Jason) exceeds or is equal to the marginal social cost (to Maria). The price will be $15 per hour because that is the maximum price Jason is willing to pay for the fourth hour of tutoring and $15 is the minimum price Maria will accept for the fourth hour of tutoring. Jason’s total consumer surplus is $30, the sum of $15 from the first hour plus $10 from the second plus $5 from the third plus $0 from the fourth. Maria makes a producer surplus of $18, the sum of $9 on the first hour plus $6 on the second hour plus $3 on the third hour plus $0 on the fourth hour.

EFFICIENCY AND EQUITY

179

Topic: Efficiency of Competitive Markets Skill: Analytical

Price (dollars per loaf) 0.60 0.80 1.00 1.20 1.40 1.60 1.80

Quantity demanded (loaves per day) 240 200 160 120 80 40 0

Quantity supplied (loaves per day) 0 80 160 240 320 400 480

11) The table above gives the demand and supply schedules for bread. Assume that the only people who benefit from bread are the people who consume it and the only people who bear the cost of bread are the people who produce it. a) What is the maximum price that consumers are willing to pay for the 80th loaf of bread? b) What is the minimum price that producers are willing to accept to produce 200 loaves of bread? c) What is the efficient quantity of bread? d) If the market is efficient, what is the consumer surplus? e) If the market is efficient, what is the producer surplus? f) If one firm owns all the bread outlets and sells 120 loaves per day, what is the deadweight loss (if any)?

d) The figure above will help answer the question. If the market is efficient, 160 loaves of bread are sold and bought at $1.00 per loaf. Consumer surplus is the value that consumers receive minus the price they pay summed over the quantity bought. Thus consumer surplus is the area of the indicated triangle, which equals 1/2 × ($1.80 − $1.00) × 160 = $64. e) Producer surplus is the is the price of bread minus the marginal cost of producing it, summed over the quantity sold. Thus producer surplus is the area of the indicated triangle, which equals 1/2 × ($1.00 − $0.60) × 160 = $32.

Answer:

a) The demand curve shows consumers’ willingness to pay. If 80 loaves of bread are available, consumers are willing to pay $1.40 for the last loaf. b) The supply curve shows the minimum supply price. The minimum price that will get producers to produce 200 loaves of bread is $1.10 per loaf. c) The efficient quantity of bread is 160 loaves per day. When 160 loaves per day are produced, the marginal social benefit ($1.00) equals the marginal social cost.

f) The efficient quantity of bread is 160 loaves per day because when 160 loaves per day are

180

CHAPTER 5

produced, the marginal social benefit, $1.00, equals the marginal social cost, $1.00. If 120 loaves per day are sold, the marginal benefit exceeds the marginal cost and therefore this level of output is not efficient. As the figure above shows, the deadweight loss is the area of the triangle and equals 1/2 × ($1.20 − $0.90) × (160 − 120), which is $6.

tity demanded equals the quantity supplied. There is no deadweight loss at the equilibrium quantity because it is the same as the efficient quantity. Topic: Efficiency of Competitive Markets Skill: Analytical

Topic: Efficiency of Competitive Markets Skill: Analytical

12) The figure above shows the market for pizza. a) If the price of a slice of pizza is $3, what is the consumer surplus of the 50th slice? b) If the price of a slice of pizza is $3, what is the producer surplus of the 50th slice c) What is the efficient quantity? What is the equilibrium quantity? What is the deadweight loss when the equilibrium quantity is produced? Answer:

a) Consumer surplus is the distance between the demand curve and the price of $3 when 50 slices are consumed. That difference is $4, the marginal social benefit, minus $3 or $1. b) Producer surplus is the distance between the supply curve and the market price when 50 slices are produced. That difference is $3 minus $2, the marginal social cost, or $1. c) The efficient quantity is 100 slices because that is the quantity for which the marginal social benefit equals the marginal social cost. The equilibrium quantity is 100 slices, because that is the quantity for which the quan-

13) The figure above shows the market for hot dogs. a) What is the maximum price consumers are willing to pay for the 25th hot dog? b) What is the efficient quantity? c) Suppose that the production was limited to 25 hot dogs. In the figure, indicate the amount of the deadweight loss. Answer:

a) Consumers are willing to pay a maximum of $4 for the 25th hot dog. b) The efficient quantity of hot dogs is 75 hot dogs because that is the quantity for which the marginal social benefit equals the marginal social cost.

EFFICIENCY AND EQUITY

c) The deadweight loss is indicated in the figure above. Topic: Efficiency of Competitive Markets Skill: Analytical

181

hour of tutoring, $25 for the second, $20 for the third, $15 for the fourth, and $10 for the fifth. Maria has an opportunity cost per hour of $6 for the first, $9 for the second, $12 for the third, $15 for the fourth, and $18 for the fifth. The initial equilibrium price for tutoring is $15 an hour and hence Maria tutors Jason for 4 hours. Now, Maria realizes that she is the only economics tutor because all the other tutors have graduated. Because she is the only tutor, she has a monopoly and, as a monopolist, Maria decides to charge a price of $25 instead of $15 an hour. a) At the price of $25 an hour, how many hours will Maria tutor Jason? b) At the initial equilibrium price of $15 an hour, what was Jason’s total consumer surplus and Maria’s total producer surplus? c) At the price of $25 an hour, what is Jason’s total consumer surplus and Maria’s total producer surplus? d) How does the sum of Jason’s consumer surplus plus Maria’s producer surplus compare at the initial equilibrium price of $15 an hour (part b) and at the new price of $25 an hour (part c)? Comment on any difference. Answer:

14) The figure above shows the supply and demand curves for pizza. What area in the graph above represents: a) consumer surplus? b) producer surplus? Answer:

a) AFB b) BFC Topic: Deadweight Loss Skill: Analytical

15) Jason wants to hire Maria to tutor him in economics. Jason is willing to pay $30 for the first

a) Maria will tutor Jason for 2 hours. b) At the initial price of $15, Jason’s total consumer surplus was $30, the sum of $15 from the first hour plus $10 from the second plus $5 from the third plus $0 from the fourth. Maria made a producer surplus of $18, the sum of $9 on the first hour plus $6 on the second hour plus $3 on the third hour plus $0 on the fourth hour. c) At the new price of $25 an hour, Jason’s total consumer surplus is $5, the consumer surplus from the first hour. Maria makes a producer surplus of $35, the sum of $19 on the first hour plus $16 on the second hour. d) When the price was $15 an hour, the sum of the consumer and producer surpluses was $48. When the price is $25, the sum of the consumer and producer surpluses is $40. The $8 difference between the initial situation and the situation in which Maria is a monopolist is the deadweight loss.

182

Topic: Deadweight Loss Skill: Analytical

CHAPTER 5

c) Why does the deadweight loss in part (b) occur? d) If 20,000 pizzas are produced, what area represents the deadweight loss? e) Why does the deadweight loss in part (d) occur? Answer:

16) The figure above shows the supply and demand for pizza. a) What is the efficient level of output? b) If 70,000 pizzas are produced, what area represents the deadweight loss?

a) 40,000 pizzas is the efficient quantity. b) If 70,000 pizzas are produced, the deadweight loss is area C. c) The deadweight loss occurs because we are producing some pizzas, specifically pizzas 40,001 through 70,000, for which the marginal social cost is greater than the marginal social benefit. d) If 20,000 pizzas are produced, the deadweight loss is area B. e) The deadweight loss occurs because we are not producing some pizzas, namely pizzas 20,001 through 40,000, for which the marginal social benefit is greater than the marginal social cost.

CHAPTER 6

MARKETS IN ACTION a) If the local government imposes a price ceiling of $1,500 per month, is there a shortage? If so, how much, if not, why not? b) If the local government imposes a price ceiling of $900 per month, is there a shortage? If so, how much, if not, why not?

Topic: A Regulated Housing Market Skill: Analytical

Rent (dollars per month) 1,000 800 600 400 200 1)

Quantity demanded (housing units) 150 200 250 300 350

Quantity supplied (housing units) 350 300 250 200 150

Answer:

a) There is not a shortage. The equilibrium rent is $1,200 a month. Because the rent ceiling is above the equilibrium rent, it has no effect. b) There is a shortage. At a rent of $900 per month, the quantity of units demanded is 100,000 and the quantity of units supplied is 60,000. Hence there is a shortage of 40,000 units.

The table above gives the demand and supply schedules for the housing market in a small town. If a rent ceiling of $200 a month is imposed, what is the quantity demanded, the quantity supplied, and the shortage of housing? Answer:

The quantity demanded is 350 units, the quantity supplied is 150 units, and the shortage is 200 units.

Topic: Price Ceiling Skill: Analytical

Topic: A Regulated Housing Market Skill: Analytical

3)

2)

The figure above illustrates the current market for apartments in Washington, D.C.

The above figure shows the market for a prescription drug. What is the equilibrium price of the drug? How many doses are purchased? Suppose the government imposes a price ceiling of $1.50 a dose. How many doses are purchased after the price ceiling is imposed? Answer:

The equilibrium price is $2.50 per dose. The number of doses purchased is the equilibrium

184

CHAPTER 6

quantity, 8,000 doses per day. After the price ceiling is imposed, the number of doses purchased is 4,000 per day, which is equal to the quantity of doses supplied at the price of $1.50 per dose.

Topic: The Minimum Wage Skill: Analytical

Topic: Price Ceiling Skill: Analytical

4)

In a diagram, illustrate the case of a price ceiling that affects the market’s price and quantity. Answer:

5)

The figure above has a price ceiling, Pc , that is set below the equilibrium price, P. As a result, a shortage equal to Qd – Qs emerges.

The figure above illustrates the current market for fast-food workers in Baltimore. a) Without any government intervention, what is the equilibrium wage rate and amount of employment? b) If the city government imposes a minimum wage of $3 an hour, what is the amount of employment? Does the minimum wage create any unemployment? Why or why not? c) If the city government imposes a minimum wage of $6 an hour, what is the amount of employment? Does the minimum wage create any unemployment? Why or why not? Answer:

a) The equilibrium wage rate is $4 an hour and the equilibrium amount of employment is 8,000 workers. b) Employment remains 8,000 workers. The minimum wage does not create any unemployment. The $3 per hour minimum wage does not bring about any unemployment because it is below the equilibrium wage rate. c) Employment decreases to 4,000 workers, the quantity of labor demanded at a wage rate of $6 per hour. At the wage rate of $6 per hour, the quantity of labor supplied is 12,000 workers and the quantity of labor demanded is 4,000 workers. Hence there are 8,000 workers unemployed. The minimum wage of $6 an hour creates unemployment because it is higher than the equilibrium wage rate. As a re-

MARKETS IN ACTION

185

sult, the quantity of labor supplied increases and the quantity of labor demanded decreases, leading to a situation of excess supply, or unemployment.

Topic: Price Floor Skill: Analytical

7)

Topic: Price Floor Skill: Analytical

Price (dollars per scooter) 70 80 90 100 110 6)

Quantity demanded (scooters per week) 120 110 100 90 80

In a supply and demand diagram, illustrate a price floor that affects the market’s price and quantity and a price floor that has no affect on the price and quantity. Answer:

Quantity supplied (scooters per week) 60 80 100 120 140

The table above gives the supply and demand schedules for scooters. Because of increased injuries sustained by children riding scooters, Citizens Against Scooter Accidents successfully lobbies the government to impose a price floor of $80.00 on scooters. a) What is the quantity demanded and supplied after the price floor has been imposed? b) Is there any shortage or surplus at this price floor? If so, how much? c) If the price floor is raised to $110.00, what is the quantity demanded and supplied? d) At a price floor of $110.00, is there any shortage or surplus? If so, how much?

The figure above shows a price floor that affects the price and quantity, labeled “effective” and a price floor that has no effect on the price or quantity, labeled “ineffective.” The effective price floor is greater than the equilibrium price, P, and hence keeps the price from falling to the equilibrium. The ineffective price floor is below the equilibrium price and so changes nothing.

Answer:

a) The quantity demanded and supplied is 100 scooters at $90, the equilibrium price. Because the price floor, which is the legal minimum price, is below the equilibrium price, it has no effect. The price stays at the equilibrium price of $90 per scooter and the quantity remains at the equilibrium quantity of 100 scooters. b) There is no surplus or shortage at a price floor of $90 per scooter. c) The quantity demanded at $110 is 80 scooters and quantity supplied is 140. d) There is a surplus. The surplus equals the quantity supplied, 140 scooters, minus the quantity demanded, 80 scooters, or a 60 scooters per week surplus.

Topic: Price Ceiling and Price Floor Skill: Analytical

Price (dollars per CD) 20 18 16 14 12 10 8 6 8)

Quantity demanded (millions of CDs per month) 4 8 12 16 20 24 28 32

Quantity supplied (millions of CDs per month) 25 22 19 16 13 10 7 4

The table above gives the demand and supply schedules for CDs.

186

CHAPTER 6

a) According to the table, what is the equilibrium price? b) Suppose the government imposes a price floor of $16 for a CD. What is the result? c) Suppose the government imposes a price ceiling of $8 for a CD. What is the result?

Answer:

Answer:

a) The equilibrium price is $14 for a CD and the equilibrium quantity is 16 million CDs per month. b) If the government imposes a price floor of $16 per CD, the price cannot fall to its equilibrium of $14 per CD. At the floor price of $16 for a CD, there is a surplus of CDs. The quantity supplied is 19 million and the quantity demanded is only 12 million, so there is a surplus of 7 million CDs. c) If the government imposes a price ceiling of $8 for a CD, the price cannot rise to its equilibrium level of $14 per CD. At the price of $8, there is a shortage of CDs. The quantity demanded is 28 million CDs but the quantity supplied is only 7 million CDs. Hence there is a 21 million CD shortage. Topic: Price Ceiling and Price Floor Skill: Analytical

9)

As illustrated, an effective price ceiling is set below the equilibrium price, P in the figure. When a price ceiling is set below the equilibrium price, a shortage results. An effective price floor is set above the equilibrium price, as illustrated. This sort of price floor causes a surplus. Topic: Tax Incidence Skill: Analytical

In a supply and demand diagram, illustrate an effective price ceiling, that is, a price ceiling that changes the price and quantity. In the same diagram, also illustrate an effective price floor, that is, a price floor that changes the price and quantity.

10) The figure above illustrates the market for antifreeze. Suppose the government decides to implement an $8 sales tax on the sellers for every gallon of antifreeze sold. a) In the figure, illustrate the effect the tax has on the market for antifreeze.

MARKETS IN ACTION

b) What is the equilibrium price of a gallon of antifreeze before the tax? What is the price paid by buyers after the tax? c) What is the equilibrium quantity of antifreeze before the tax? What is the equilibrium quantity after the tax? d) What is the revenue collected by the government from this tax? e) Do buyers or sellers bear the largest incidence of the tax? f) Illustrate the deadweight loss created by the tax.

187

Topic: Tax Incidence Skill: Analytical

Answer:

11) The figure above shows the market for gasoline. The government has imposed a tax on gasoline. a) What is the amount of the tax per gallon of gasoline? b) How much of the tax is paid by consumers? How much is paid by producers? Which is more elastic, the supply or demand for gasoline? Answer:

a) The tax shifts the supply curve, as illustrated above. b) Before the tax, the price was $4 per gallon. After the tax, buyers pay $10 per gallon. c) Before the tax the equilibrium quantity was 8,000 gallons of antifreeze. After the tax, the quantity is 4,000 gallons of antifreeze. d) The government collects $32,000 in tax revenue per week. e) Buyers bear the largest incidence of the tax because the price they pay rises by $6, from $4 per gallon to $10 per gallon. f) The deadweight loss is equal to the area of the darkened triangle in the figure.

a) The tax is $1.50 per gallon, the amount by which the supply curve has been shifted upward. b) Consumers pay $1 of the tax, as the price they pay rises from $1.50 to $2.50 per gallon. Producers pay $0.50 of the tax, as the price they receive falls from $1.50 to $1.00 per gallon. The supply is more elastic because suppliers pay a smaller fraction of the tax than do buyers.

188

CHAPTER 6

Topic: Tax Incidence and the Elasticity of Demand Skill: Analytical

price rose by the full amount of the tax so buyers pay the entire tax Topic: Tax Incidence and the Elasticity of Supply Skill: Analytical

12) The figure above shows the market for a lifesaving drug. Suppose the government imposes a $150 tax per dose on sellers of the drug. Show and describe the impact on the market. Who pays this tax? Answer:

13) The figure above shows the market for tickets to the Super Bowl the day of the game. Suppose the government imposes an entertainment tax of $100 per ticket. a) What is the equilibrium price of a Super Bowl ticket before the tax? What is the price paid by buyers after the tax? What is the price received by sellers after the tax? b) What is the equilibrium quantity of tickets before the tax? What is the equilibrium quantity after the tax? c) Do buyers or sellers bear the largest incidence of the tax? Answer:

The tax decreases the supply and shifts the supply curve leftward. As shown in the figure, the vertical difference between the old supply curve and the new one is the amount of the tax, $150 per dose. The price of the drug rises from $150 per dose before the tax to $300 per dose after the tax. The quantity remains constant, at 8,000 does per month. Because demand is perfectly inelastic, the

a) The equilibrium price before the tax is $200 for a ticket. After the tax, buyers still pay $200 per ticket. Sellers, however, receive only $100 (= $200 price paid by a buyer minus the $100 tax) per ticket. b) Before the tax, the equilibrium quantity was 80,000 tickets. After the tax the quantity is still 80,000 tickets. c) Sellers pay all the tax and buyers pay none of the tax. In other words, the price that a buyer pays does not change—it remains equal to $200. Hence buyers pay none of the tax. However, the price received by sellers falls from $200 per ticket to $100. Hence sellers

MARKETS IN ACTION

189

pay the full amount of the $100 tax, which was to be expected because the supply of tickets is perfectly inelastic. Topic: Tax Incidence and the Elasticity of Supply Skill: Analytical

14) In the Village of Punjab, Sheryl owns a well, which is the only source of drinking water. The supply of water is perfectly inelastic at a quantity of 1,000 gallons of water per day. At a price of $2.00 per gallon, the quantity demanded per day is 1,000 gallons. The government imposes a $0.50 per gallon tax. a) After the tax is imposed, what is the price paid by the villagers? What is the price received by Sheryl? b) How much revenue does the government collect? c) What fraction of the tax does Sheryl pay? What fraction is paid by the villagers?

a) How will the subsidy affect the price paid by buyers for a loaf of bread and the quantity of bread sold? Show your work on your graph. b) How will the subsidy affect the amount sellers receive for a loaf of bread? c) After the subsidy is granted, what is the marginal social cost of the last loaf of bread produced? What is its marginal social benefit? If there are no external costs or external benefits, is the bread market efficient once the subsidy is granted? Answer:

Answer:

a) Because the supply is perfectly inelastic, no matter what price Sheryl receives, she supplies 1,000 gallons of water per day. Because she supplied 1,000 gallons of water per day, the price paid by the villagers remains equal to $2.00, the price at which 1,000 gallons per day is the quantity demanded. Sheryl receives $1.50, the $2.00 paid by the villagers minus the $0.50 sent to the government as the tax. b) The government receives $500 in tax revenue. c) Sheryl pays 100 percent of the tax because the price she receives falls by the full amount of the tax. The villagers pay none of the tax because the price they pay does not change. Topic: Subsidy Skill: Analytical

Price (dollars per loaf) 1.00 1.50 2.00 2.50

Quantity demanded (loaves per day) 1,200 800 400 0

Quantity supplied (loaves per day) 0 400 800 1,200

15) The table above gives the demand and supply schedules for bread in Bakerville. Draw the demand and supply curves. Suppose a subsidy on bread of $1 per loaf is given to bread producers.

a) The figure above shows the bread market. Before the subsidy was granted, the equilibrium price was $1.75 a loaf and the equilibrium quantity was 600 loaves per day. After the subsidy is granted, supply increases and the supply shifts downward by $1 per loaf, as shown in the figure. The equilibrium price paid by buyers is $1.25 and the equilibrium quantity is 10,000 loaves of bread. b) After the subsidy is granted, the price buyers pay for a loaf of bread is $1.25. The sellers receive this amount and also receive the $1.00 per loaf subsidy, so the sellers receive $2.25 per loaf. c) With no externalities, the supply curve is also the marginal social cost (MSC) curve and the demand curve is also the marginal social benefit (MSB) curve. With the subsidy, 1,000 loaves are produced. The marginal social cost of the 1,000th loaf is $2.25. The marginal

190

CHAPTER 6

social benefit of the 1,000th loaf is $1.25. Because MSC > MSB, the quantity of bread produced is not efficient

CHAPTER 7

UTILITY AND DEMAND

Topic: Consumption Possibilities Skill: Analytical

1)

Joe has $100 a week to purchase either computer online service or film for his other hobby, photography. The price of on-line service is $5 an hour while the price of film is $10 a roll. Draw Joe’s budget line in the figure above. Answer:

Joe’s budget line is illustrated in the figure above.

Topic: Consumption Possibilities Skill: Analytical

2)

Suppose you have a $20 budget per week, the price of soda is $1 per bottle, and the price of pizza is $4 per slice. In the above figure, draw a budget line for soda and pizza, placing soda on the horizontal axis. Correctly label the axes. Answer:

The budget line is illustrated in the figure above.

192

CHAPTER 7

Topic: Consumption Possibilities Skill: Analytical

Topic: Marginal Utility Skill: Analytical

Quantity (rentals per week) 0 1 2 3 4 5 6 7 4)

Total utility 0 28 54 78 100 120 138 154

Marginal utility X ____ ____ ____ ____ ____ ____ ____

Amy has the total utility values given above for video rentals a week. Complete the table by calculating her marginal utilities. Answer:

3)

Quantity (rentals per week) 0 1 2 3 4 5 6 7

John likes to spend Thursday nights playing pool and drinking soda. John’s budget for Thursday nights is $10, a soda costs $2, and one game of pool costs $1. a) Draw a graph of John’s budget line in the figure above. b) In your graph, label the affordable and unaffordable areas. Answer:

Total utility 0 28 54 78 100 120 138 154

Marginal utility X 28 26 24 22 20 18 16

The completed table is above. Topic: Total Utility and Marginal Utility Skill: Analytical

a) The budget line is in the figure above. b) The affordable area is the lighter area and the budget line itself. The unaffordable area is the darker area.

Quantity (cones per week) 0 1 2 3 4 5 6 7 5)

Total utility 0 22 38 ____ 56 ____ 62 63

Marginal utility X ____ ____ 11 ____ 4 ____ ____

The table above gives the utility Andy receives from different quantities of vanilla ice cream cones. Complete the table.

UTILITY AND DEMAND

193

Answer:

Quantity (cones per week) 0 1 2 3 4 5 6 7

Total utility 0 22 38 49 56 60 62 63

Marginal utility X 22 16 11 7 4 2 1

Topic: Total Utility and Marginal Utility Skill: Analytical

6)

Total utility 0 ____ 35 45 52 57

Marginal utility X 20 ____ ____ ____ ____

The table above gives Jessie’s marginal utility and total utility from tacos. Complete the table. Answer:

Quantity (cones per week) 0 1 2 3 4 5

Total utility 0 20 35 45 52 57

The completed table is above.

Eclairs Quantity 0 1 2 3 4 5 7)

The completed table is above.

Quantity (cones per week) 0 1 2 3 4 5

Topic: Utility-Maximizing Choice Skill: Analytical

Marginal utility X 20 15 10 7 5

Total utility 0 126 243 342 414 441

Cream puffs Total Quantity utility 0 0 1 54.0 2 103.5 3 145.5 4 177.0 5 196.5

The table gives the total utility Jamal derives from the consumption of eclairs and cream puffs. Jamal has $12 to spend on these two confectionery goods. The price of an eclair is $3 and the price of a cream puff is $1.50. a) Jamal’s budget is $12. In order for Jamal to maximize his utility, how many eclairs and cream puffs should he buy? b) Suppose the price of an eclair increase to $6. Jamal’s income does not change and neither does the price of a cream puff. What combination of eclairs and cream puffs will Jamal buy now? c) Using your answers to parts (a) and (b), derive two points on Jamal’s demand curve for eclairs. Answer:

a) Jamal’s utility maximizing combination will allocate (spend) his budget and be such that the marginal utility per dollar spent on the last eclair equals the marginal utility per dollar spent on the last cream puff. Jamal’s utility maximizing combination is 3 eclairs and 2 cream puffs. At this combination, Jamal spends $9 on eclairs (3 eclairs × $3) and $3 on cream puffs (2 cream puffs × $1.50) for a total of $12. The marginal utility of the third eclair is 99 (= 342 − 243), so the marginal utility per dollar spent on eclairs is 99 ÷ 3 = 33. The marginal utility of the second cream puff is 49.50 (= 103.5 − 54.0), so the marginal utility per dollar spent on cream puffs is 49.5 ÷ $1.50 = 33. Hence the marginal utilities per dollar spent are equal, so the combination of 3 eclairs and 2 cream puffs maximizes Jamal’s utility.

194

CHAPTER 7

b) Jamal’s new utility maximizing combination is 1 eclair and 4 cream puffs. As in part (a), this combination allocates all of Jamal’s income and equalizes the marginal utiliy per dollar spent on the two goods. (Both are equal to 21.) c) When the price of an eclair is $3, Jamal buys 3 eclairs, so one point on Jamal’s demand curve is the price of $3 and the quantity demanded of 3 eclairs. When the price of an eclair increases to $6, Jamal buys only 1 eclair. So another point on Jamal’s demand curve is the price of $6 and quantity demanded of 1 eclair. Topic: Equalizing Marginal Utility per Dollar Spent Skill: Analytical

Pants Marginal Quantity utility 1 220 2 180 3 140 4 100 5 60 8)

Belts Quantity 1 2 3 4 5

Marginal utility 85 65 45 25 15

Tris is shopping for pants and belts. He has a budget of $100. The price of a pair of pants is $20 and the price of a belt is $5. His marginal utility schedules are above. What combination of pants and belts does Tris buy? Explain your answer. Answer:

Tris buys 4 pairs of pants and 4 belts because this is the combination that allocates (spends) all of his budget and has the marginal utility per dollar spent on a pair of pants equal to the marginal utility per dollar spent on a belt. For the first requirement, that Tris allocate his entire budget, the combination of 4 pants and 4 belts spends (4 pairs of pants) × ($20 per pair of pants) = $80 on pants and (4 belts) × ($5 per belt) = $20 on belts, for a total spent of $100. For the second requirement, the equality of the marginal utilities per dollar, the marginal utility per dollar spent on a pair of pants is (100 units) ÷ ($20) = 5 units per dollar and the marginal utility per dollar spent on a belt is (25 units) ÷ ($5) = 5 units per dollar.

Topic: Equalizing Marginal Utility per Dollar Spent Skill: Analytical

Ice Cream Romance novels Total Total Quantity utility Quantity utility 1 95 1 170 2 180 2 320 3 255 3 450 4 320 4 560 5 375 5 650 6 420 6 720 9) The table above shows Danielle’s utility from ice cream and romance novels. a) What is Danielle’s marginal utility from the 4th novel? b) The price of ice cream is $5 per gallon and a novel is $10. If Danielle’s budget for these two goods is $50 and she buys 2 gallons of ice cream, how many novels can she buy? If she buys 6 gallons of ice cream, what is her marginal utility per dollar spent on novels? c) Which combination of the two goods is better: 4 gallons of ice cream and 3 novels or 6 gallons of ice cream and 2 novels? Answer:

a) Danielle’s marginal utility from the 4th novel is 560 − 450 = 110. b) When Danielle buys 2 gallons of ice cream, she spends $5 × 2 = $10 on ice cream. So Danielle spends the rest of her budget, which is $50 • $10 = $40, on novels. Because the price of a novel is $10, she buys $40 ÷ $10 = 4 novels. When Danielle buys 6 gallons of ice cream, she spends $5 × 6 = $30 on ice cream. So Danielle spends the rest of her budget, which is $50 − $30 = $20, on novels. Because the price of a novel is $10, she buys $20 ÷ $10 = 2 novels. So her marginal utility from the second novel is 320 • 170 = 150 and, since the price of a novel is $10, the marginal utility per dollar is 150 ÷ $10 = 15. c) 4 gallons of ice cream and 3 novels is better. In both cases Danielle spends all her budget. But if she buys 4 gallons of ice cream and 3 novels, her total utility is 320 + 450 = 770, while if she buys 6 gallons of ice cream and 2 novels, her total utility is 420 + 320 = 740.

UTILITY AND DEMAND

195

Topic: Equalizing Marginal Utility per Dollar Spent Skill: Analytical

10) Ricardo likes to rent videos and attend concerts. The videos cost $4 and the concerts cost $40. Ricardo’s marginal utility from the last video is 20 units. Ricardo is maximizing his utility. What is his marginal utility from the last concert he attended? Answer:

The marginal utility from his last concert is 200 units. In order for Ricardo to maximize his utility, the marginal utility per dollar spent must be equal for all goods. The marginal utility per dollar for the last video is (20 units) ÷ ($4) = 5. So the marginal utility per dollar for the last concert also must equal 5, which, given its price of $40, means that concert has a marginal utility of 200, because (200 units) ÷ ($40) = 5. Topic: Equalizing Marginal Utility per Dollar Spent Skill: Analytical

11) Suppose Abe is allocating his entire income and he receives 100 units of utility per dollar from the tenth slice of pizza and 200 units of utility per dollar from his second soda. Is Abe maximizing his utility? Should Abe change his consumption of pizza and soda? Answer:

Abe is not maximizing his utility because his marginal utility per dollar spent pizza does not equal his marginal utility per dollar spent on soda. Abe should consume more soda (because it provides more satisfaction per dollar spent) and less pizza. Topic: Predictions of Marginal Utility Theory, Demand Curve Skill: Analytical

Bananas Quantity Marginal (pounds) utility 1 30 2 24 3 18 4 12 5 6 6 0

Apples Quantity Marginal (pounds) utility 1 40 2 34 3 24 4 16 5 8 6 0

12) The table above gives Sam’s marginal utility schedule for bananas and apples. Sam’s fruit budget is $10.

a) If bananas cost $1 per pound and apples cost $2 per bag, what is Sam’s marginal utility per dollar for all quantities of both goods? b) What is the utility maximizing combination of bananas and apples for Sam? c) If the price of bananas increases to $2 per pound, how does Sam’s marginal utility per dollar for bananas change? d) At the banana price of $2 per pound, what is the new utility maximizing combination of bananas and apples for Sam? e) List two points on Sam’s demand curve for bananas. Answer:

Bananas Marginal Quantity utility per (pounds) dollar 1 30 2 24 3 18 4 12 5 6 6 0

Apples Marginal Quantity utility per (pounds) dollar 1 20 2 17 3 12 4 8 5 4 6 0

a) The table above has Sam’s marginal utility per dollar for bananas and apples. b) Sam’s utility maximizing combination of bananas and apples is 4 pounds of bananas and 3 bags of apples. This quantity allocates (spends) his budget and equates the marginal utility per dollar spent on bananas and apples. Bananas Marginal Quantity utility per (pounds) dollar 1 15 2 12 3 9 4 6 5 3 6 0 c) The table with Sam’s new marginal utility per dollar for bananas is above. d) Sam’s new utility maximizing combination of bananas and apples is 2 pounds of bananas and 3 bags of apples.

196

CHAPTER 7

e) When the price of a pound of bananas is $1, the quantity demanded is 4 pounds and when the price rises to $2, the quantity demanded decreases to 2 pounds. Topic: Marginal Analysis Skill: Analytical

13) For a consumer, the marginal utility of good A is 25 and its price is $5. The marginal utility of good B is 60 and its price is $12. The consumer has allocated his entire budget. Is this consumer maximizing his total utility? Explain your answer. Answer:

Yes, the consumer is maximizing his utility. He has allocated his entire budget and he is consuming the combination of goods A and B such that the marginal utility per dollar spent on each is the same. Topic: Marginal Analysis Skill: Analytical

14) If Mark’s marginal utility per dollar spent on the last taco is 15 and his marginal utility per dollar spent on the last burrito is 20 units, what should Mark do to increase his total utility? Answer:

Mark should buy more burritos and fewer tacos. If Mark decreases his purchases of tacos by $1, he

loses 15 units of utility. But if he then spends the dollar on burritos, he gains 20 units of utility. Because his gain in utility exceeds his loss, Mark’s total utility increases as he buys more burritos and fewer tacos. Topic: Marginal Analysis Skill: Analytical

15) Suppose Jenny’s marginal utility of fish is 40 and her marginal utility from chips is 20. The price of fish is $10 and the price of chips is $1. What should Jenny do to maximize her utility? Explain your answer. Answer:

Jenny should buy more chips and fewer fish. Her marginal utility per dollar spent on chips is 20 and her marginal utility per dollar spent on fish is 4. If Jenny decreases her purchases of fish by $1, she loses 4 units of utility. But if she spends the dollar on chips, she gains 20 units of utility. Because her gain in utility exceeds her loss, Jenny’s total utility increases as she buys more chips and fewer fish.

CHAPTER 8

POSSIBILITIES, PREFERENCES, AND CHOICES

Topic: The Budget Equation Skill: Analytical*

1)

Joe spends $72 on camera films and sandwiches. The price of a roll of film is $3 and a sandwich is $4. Placing the quantity of sandwiches on the left side, what is Joe’s budget equation? If sandwiches are measured on the vertical axis, what is the slope of Joe’s budget line?

a) Alex’s income is $8 per day. What is the price of a hot dog? Of a hamburger? b) What is the slope of the budget line? Answer:

Answer:

The budget equation is QS = 18 – 0.75QF, where QS is the quantity of sandwiches and QF is the quantity of films. To determine the slope of Joe’s budget line, first notice that Joe’s budget line intersects the vertical axis when Joe buys 18 sandwiches (which equals his income, $72, divided by the price of a sandwich, $4). Second, Joe’s budget line intersects the horizontal axis when he buys 24 roles of films (which equals his income, $72, divided by the price of a role of film, $3). So calculating the slope of Joe’s budget line over the horizontal distance from 0 rolls of film to 24 rolls of film, the vertical fall is −18 sandwiches divided by the horizontal distance of 24 rolls of film, so the budget line’s slope of −18 sandwiches/24 roles of film, which is • 0.75 of a sandwich per roll of film. Topic: Consumption Possibilities, Budget Line Skill: Analytical

Possibility A B C D E 2)

Hamburgers (per day) 0 1 2 3 4

Hot dogs (per day) 8 6 4 2 0

The table above has different combinations of hamburgers and hot dogs that Alex can buy. After labeling the axes, graph Alex’s budget line in the figure, putting hot dogs on the x-axis.

The figure above has Alex’s budget line. a) The price of a hot dog is $1 and the price of a hamburger is $2. b) The slope of the budget line is 1/2 of a hamburger per hot dog.

198

CHAPTER 8

Topic: Consumption Possibilities, Change in Prices and Income Skill: Analytical

3)

Sherry is on vacation and wants to bring souvenirs home to family and friends. Her souvenir budget is $100, and she can choose between T-shirts, which cost $20 each, and key chains, which cost $5 each. a) Draw a graph of Sherry’s budget line in the figure above. b) What is the slope of Sherry’s budget line? How does that slope represent an opportunity cost? Suppose that Sherry now finds a store where Tshirts are on sale for $10 each. (Key chains still cost $5 each). c) Draw the new budget line in the figure. d) What is the slope of the new budget line? How has the opportunity cost changed?

Answer:

a) The budget line is in the figure above and is the budget line labeled BL1. b) The slope of the budget line is 20 key chains divided by 5 T-shirts, which equals 4 key chains per T-shirt. 4 key chains per T-shirt is an opportunity cost because it represents the quantity of key chains that must be given up to acquire one more T-shirt. c) The new budget line is in the figure above and is labeled BL2. d) The slope of the new budget line is 2 key chains per T-shirt, which means that the opportunity cost of one T-shirt is now 2 key chains, rather than 4. Thanks to the new store, the opportunity cost of a T-shirt has fallen.

POSSIBILITIES, PREFERENCS, AND CHOICES

Topic: Consumption Possibilities, Change in Prices and Income Skill: Analytical

4)

The figure above contains several budget lines for Sarah, who uses her income to purchase two goods, cheese and crackers. a) A movement between which two budget lines represents an increase in income? b) A movement between which two budget lines represents an increase in the price of a pound of cheese? c) A movement between which two budget lines represents an increase in the price of a box of crackers? Answer:

a) A movement from BL1 to BL3 shows an increase in income. b) A change from BL2 to BL1 shows an increase in the price of a pound of cheese. c) A change from BL3 to BL2 shows an increase in the price of a box of crackers.

199

Topic: Indifference Curve Skill: Conceptual

15) Tom is stranded on a deserted island where he can only consume coconuts and crabs. Two of his indifference curves are in the figure above. a) Would Tom prefer his consumption to be at point a or at point b? At point b or at point c? Explain your answers. b) Between points a and b, what is Tom’s marginal rate of substitution for a crab? Answer:

a) Tom is indifferent between consuming at point a or at point b because both points are on the same indifference curve, I1. Tom would prefer to consume at point c rather than point b because point c is on a higher indifference curve, I2 versus I1. b) Between points a and b, Tom’s marginal rate of substitution for a crab is 2 coconuts per crab.

200

CHAPTER 8

Topic: Indifference Curve Skill: Conceptual

Topic: Marginal Rate of Substitution Skill: Analytical

16) The figure above shows a preference map for Sarah, who likes hamburgers and milk shakes. a) Which two combinations contain the same amount of hamburgers but different quantities of shakes? b) Which combination(s) does Sarah most prefer? Why? c) Which combination(s) does Sarah least prefer? Why? d) Between which combinations is Sarah indifferent? Why?

7)

Answer:

a) Combinations a and b have the same amount of hamburgers but different amounts of milk shakes. b) Sarah most prefers combination d because it is on the highest indifference curve. c) Sarah least prefers combination a because it is on the lowest indifference curve. d) Sarah is indifferent between combinations b and c because they are on the same indifference curve.

The figure above shows one of Bob’s indifference curves for CDs and books. a) Is the indifference curve steeper at point a or point b? b) What is Bob’s marginal rate of substitution at point a? c) What is Bob’s marginal rate of substitution at point b? Answer:

a) The indifference curve is steeper at point a. b) The marginal rate of substitution at point a is 5 CDs per book, the slope of the line that just touches the indifference curve at this one point. c) The marginal rate of substitution at point b is 1 1/2 CDs per book, the slope of the line that just touches the indifference curve at this one point.

POSSIBILITIES, PREFERENCS, AND CHOICES

201

Topic: Consumption Possibilities, Budget Line Skill: Analytical*

Topic: Consumption Possibilities, Budget Line Skill: Analytical*

8)

9)

Nadya spends $200 a year to buy computer games and Barbie dolls. The price of a computer game is $20 and the price of a Barbie doll is $10. The figure above illustrates Nadya’s preferences. What combination of computer games and Barbie dolls does Nadya buy? Answer:

The figure above shows that Nadya buys 5 computer games and 10 Barbie dolls.

Nadya spends $200 a year to buy computer games and Barbie dolls. The price of a computer game is $40 and the price of a Barbie doll is $10. The figure above illustrates Nadya’s preferences. What combination of computer games and Barbie dolls does Nadya buy? Answer:

The figure above shows that Nadya buys 3 computer games and 8 Barbie dolls.

202

Topic: Predicting Consumer Behavior, Demand Curve Skill: Analytical

CHAPTER 8

d) What combination of dinners and movies will George select along budget line BL2? e) Use the information in this problem to give two points on George’s demand curve for dinners. Answer:

10) George has a $600 annual entertainment budget that he uses to buy trips to the movies and dinners at local restaurants. The figure above shows indifference curves and budget lines for these two goods. The price of a movie is $15. a) Along budget line BL1, what is the price of a dinner? b) What combination of dinners and movies will George select along budget line BL1? c) Budget line BL2 represents a change in the price of dinners from that along BL1. What is the new price of dinners along this budget line?

a) George can buy 20 dinners and his income is $600, so the price of a dinner is $30. b) George will buy 20 movies and 10 dinners because this combination is on his budget line, and hence is affordable, as well as on the highest attainable indifference curve, and hence is the “best” combination. c) If George spends all his income on dinners, he can purchase 40. Therefore the price of a dinner must be $15 (= $600 income/40 dinners = $15 per dinner). d) George will buy 20 movies and 20 dinners because this combination is on his budget line, and hence is affordable, as well as on the highest attainable indifference curve, and hence is the “best” combination. e) One point on George’s demand curve for dinners is a price of $30 per dinner and a quantity demanded of 10 dinners (from part b). Another point is a price of $15 per dinner and a quantity demanded of 20 dinners (from part d).

CHAPTER 9

ORGANIZING PRODUCTION

Topic: Explicit and Implicit Costs Skill: Analytical

1)

Professor Rush decided to quit teaching economics and opens a shoe store out at the mall. He gave up an annual income of $50,000 to open the store. A year after opening the shoe store, the total revenue for the year was $200,000. Rush’s expenses were $30,000 for labor, rent was $18,000, and utilities were $1,200. He also had to purchase new shoes from manufacturers, at a cost of $60,000, which was financed by cashing in his savings of $60,000 that had been in a bank earning 8 percent per year. The normal profit from operating a shoe store in the mall is $20,000. Determine Professor Rush’s explicit costs, implicit costs, and economic profit. Answer:

The explicit costs are the cost of labor, the rent, the utilities, and the cost of the shoes. Thus the explicit cost equals $30,000 + $18,000 + $1,200 + $60,000 = $109,200. The implicit costs are the forgone income from the job he quit, the forgone interest, and the normal profit. The forgone interest is (8 percent) × ($60,000) = $4,800, so the total implicit costs equals $50,000 + $4,800 + $20,000 = $74,800. The economic profit equals the total revenue minus the total opportunity cost and the total opportunity cost equals the sum of the explicit and implicit costs. Therefore the total opportunity cost is $109,200 + $74,800 = $184,000. Thus the economic profit = $200,000 − $184,000 = $16,000. Topic: Explicit and Implicit Costs Skill: Analytical

2)

Mr. Blowfish opened a seafood store in December 2003. He borrowed $60,000 from a bank at an annual interest rate of 8 percent. He used the funds he borrowed to purchase $60,000 of capital equipment. Over the year, he rented a building for $50,000 a year. During the first year of operation, Blowfish paid $45,000 to his employees,

$20,000 for utilities, and $25,000 for raw fish he bought from other firms. In December 2004, the market value of his capital was $50,000. Blowfish’s best alternative to running the seafood store is to work for a grocery store as a sales clerk for $20,000 a year. a) What is the economic depreciation of Blowfish’s capital? b) What are Blowfish’s implicit costs? c) What are Blowfish’s total opportunity costs? d) What is Blowfish’s economic profit? Answer:

a) Economic depreciation is the change in the market value of capital over a given period. The market value of Blowfish’s capital was $60,000 at the beginning of the period and $50,000 at the end. So the economic depreciation is $60,000 – $50,000 = $10,000. b) Blowfish’s implicit costs are the economic depreciation and his forgone income. The economic depreciation is $10,000. Blowfish’s forgone income is the $20,000 he could have earned working for a grocery store. So the total implicit costs are $10,000 + $20,000 = $30,000. c) Blowfish’s total opportunity costs include his explicit costs and implicit costs. The explicit costs are the wages paid, $45,000, rent, $50,000, utilities, $20,000, interest paid, $4,800, and raw fish bought, $25,000. So Blowfish’s explicit costs are $45,000 + $50,000 + $20,000 + $4,800 + $25,000 = $144,800. His implicit costs are $30,000. So Blowfish’s total opportunity cost is $144,800 + $30,000 = $174,800. d) Economic profit equals total revenue minus total opportunity costs. Blowfish’s total revenue is $165,000 and his total opportunity cost is $174,800. So his economic profit is $165,000 – $174,800 = –$9,800, which

204

CHAPTER 9

percent, so the industry would be regarded as competitive.

means that Blowfish incurred an economic loss of $9,800. Topic: Explicit and Implicit Costs Skill: Analytical

3)

Jessica is a young doctor who has just started her own practice. Her previous position paid her $80,000 a year. For office space, she uses a building which she owns and which she has rented in the past for $40,000 a year. Her total revenue from her new practice is $250,000. She pays $50,000 to other firms for materials and supplies, and she pays $40,000 in wages to her office nurse. Assume that Jessica’s building and equipment do not depreciate and that her normal profit is $20,000. a) Which of Jessica’s costs are explicit costs and what is their total? b) Which of Jessica’s costs are implicit costs and what is their total? c) What is the opportunity cost of all factors of production employed by Jessica? d) What is Jessica’s economic profit?

Topic: The Four-Firm Concentration Ratio Skill: Analytical

Company A B C D 5)

Topic: The Four-Firm Concentration Ratio Skill: Analytical

4)

The four largest firms in an industry account for the following value of industry sales: 12 percent, 8 percent, 5 percent and 4 percent. Calculate the four-firm concentration ratio. Would this industry be regarded as competitive or concentrated? Answer:

The four-firm concentration ratio is 29 percent. The four-firm concentration ratio is less than 40

The above table shows the percent of sales held by the four largest firms in an industry. a) Calculate this industry’s four-firm concentration ratio. b) Is this industry competitive? c) What market type does it most likely represent? Answer:

a) The four-firm concentration rate is 30 percent. b) Because the four-firm concentration ratio is relatively low, the industry is competitive. c) The industry is most likely monopolistic competition.

Answer:

a) Jessica’s explicit costs are the wages and the amount she paid to other firms for material and supplies. Hence her explicit costs equal $40,000 + $50,000 = $90,000. b) Jessica’s implicit costs are her forgone wages, the forgone rent on her building, and her normal profit. Hence her implicit costs equal $80,000 + $40,000 + $20,000 = $140,000. c) The opportunity cost equals the sum of explicit costs plus implicit costs, so Jessica’s opportunity cost equals $90,000 + $140,000 = $230,000. d) Economic profit equals total revenue minus total opportunity cost, or $250,000 − $230,000 = $20,000.

Market share (percent of sales) 12 10 5 3

Topic: The Herfindahl-Hirschman Index Skill: Analytical

6)

An industry has only four firms, who have market shares of 45 percent, 25 percent, 20 percent, and 10 percent. What is the Herfindahl-Hirschman Index? Answer:

The Herfindahl-Hirschman Index is 3,150.

ORGANIZING PRODUCTION

Topic: The Herfindahl-Hirschman Index Skill: Analytical

Company A B C D E F G H I J 7)

Market share (percent of sales) 32 17 15 10 7 7 5 4 2 1

Suppose there are ten firms that occupy the Odell, Oregon cherry pie market. The market share of each firm is listed in the above table. a) What is the four-firm concentration ratio in this market? b) What is the Herfindahl-Hirschman Index for this market? c) If Firm H and Firm A merge, what is the new Herfindahl-Hirschman Index for this market? d) A severe winter causes every firm except A, B, and E to close. With only these three firms operating, Firm A’s market share is 71 percent, Firm B’s market share is 23 percent, and Firm C’s market share is 6 percent. What is the Herfindahl-Hirschman Index for this market now?

205

Topic: The Four-Firm Concentration Ratio and the Herfindahl-Hirschman Index Skill: Analytical

Industry A (percent of sales) 5 7 4 16 11 10 8 19 13 4 3 8)

Industry B (percent of sales) 10 8 9 8 9 10 9 9 10 9 9

Listed in the above table are the market shares for the firms in two different industries. Each industry has only eleven firms. Find the four-firm concentration ratio and the Herfindahl-Hirschman Index for each industry. Answer:

Industry A has a four-firm concentration ratio of 59 percent and a Herfindahl-Hirschman Index of 1,186. Industry B has a four-firm concentration ratio of 39 percent and Herfindahl-Hirschman Index of 914. Topic: The Herfindahl-Hirschman Index Skill: Analytical

Answer:

a) The four-firm concentration rate is 74 percent. b) The Herfindahl-Hirschman Index is 1,782. c) Once firms H and A merge, the new Herfindahl-Hirschman Index is 2,038. d) The Herfindahl-Hirschman Index is now 5,606.

9)

Market share Company (percent of sales) A 20 B 20 C 15 D 10 E 10 F 8 G 7 H 5 I 5 There are 9 firms in a industry with market shares in the table above. Calculate the HHI for the industry. What kind of market does this operate in and why.

206

CHAPTER 9

Answer: 2

2

2

2

2

2

The HHI equals 20 + 20 + 15 + 10 + 10 + 8 2 2 2 + 7 +5 + 5 = 1388. With this HHI, the concentration ratio is small and so the market is monopolistic competition.

CHAPTER 10

OUTPUT AND COSTS

Topic: Total Product Skill: Analytical

Quantity of labor (workers) 0 1 2 3 4 5

Answer:

Total product (lawns mowed per day) 0 3 8 15 20 21

a) The figure graphs the total product curve. b) The average product when 1 worker is employed is 3 lawns mowed per day; when 2 workers are employed, it is 4 lawns per day; when 3 workers are employed, it is 5 lawns per day; when 4 workers are employed, it is 5 lawns per day; and when 5 workers are employed, it is 4.2 lawns per day.

1)

The above table has the total product schedule for Jesse’s Lawn Service. a) In the figure, label the axes and then graph the total product curve. b) Find the average product for the different amounts of employment.

208

CHAPTER 10

Topic: Average Product of Labor and Marginal Product of Labor Skill: Analytical

2)

Quantity of labor (workers) 0

Total product 0

Average product

1

24

____

2

50

____

3

74

____

4

94

____

5

110

____

6

122

____

7

130

____

8

134

____

9

134

____

Topic: Marginal Product of Labor Skill: Analytical

Quantity of labor (workers) 0 1 2 3 4 5

Marginal product ____ ____ ____ ____

3)

____ ____ ____ ____ ____

a) The first worker’s marginal product is 10 hair stylings. The second worker’s marginal product is 15 hair stylings. The third worker’s marginal product is 20 hair stylings. The fourth worker’s marginal product is 15 hair stylings. The fifth worker’s marginal product is 10 hair stylings. b) There are increasing marginal returns for the first 3 workers. After the third worker, there are decreasing marginal returns.

Answer:

Total product 0

Average product

1

24

24.0

2

50

25.0

3

74

24.7

4

94

23.5

5

110

22.0

6

122

20.3

7

130

18.6

8

134

16.8

9

134

14.9

Marginal product 24

The completed table is above.

Topic: Total Product and Costs Skill: Analytical

26 24 20 16 12 8 4 0

The above table shows the total product schedule for Hair Today, a hair styling salon. a) What is the first worker’s marginal product? The second worker? The third worker? The fourth worker? The fifth worker? b) Over what range of workers is there increasing marginal returns? Over what range is there decreasing marginal returns? Answer:

Ajax Manufacturing has a fixed scale of plant with the levels of total product given in the table for different levels of labor. Complete the table by calculating the average product and marginal product. Quantity of labor (workers) 0

Total product (hair stylings per day) 0 10 25 45 60 70

4)

Labor Total product (workers) (shirts per day) 0 0 1 20 2 36 3 48 4 56 5 60 The table above shows Randy’s Shirts’ short-run production function. Randy hires workers at a wage rate of $50 a day and his total fixed cost is $100. a) What is the marginal product of the 3rd worker?

OUTPUT AND COSTS

b) What is Randy’s average fixed cost if 48 shirts are produced? c) What is Randy’s average variable cost if 56 shirts are produced? d) What is Randy’s marginal cost of producing the 52nd sweater e) What is Randy’s average total cost if 48 sweaters are produced?

209

Topic: Short Run Costs Skill: Analytical

Quantity of labor (workers) 0 1 2 3 4 5 6

Answer:

a) When the 3rd worker is hired, output increases from 36 to 48, an increase of 12 shirts per day. So the marginal product of the 3rd worker is 12 shirts per day. b) Average fixed cost is total fixed cost divided by the output produced. Total fixed cost is $100, so Randy’s average fixed cost is $100/48, which is $2.08 per shirt. c) Average variable cost is total variable cost divided by the output produced. To produce 56 shirts, Randy needs 4 workers. So Randy’s total variable cost is $50 × 4, which is $200. Then, his average variable cost is $200/56, which is $3.57 per shirt. d) The 52nd sweater is produced when the 4th worker is hired and the output increases from 48 to 56 shirts. The marginal cost of producing these additional 8 shirts is the cost of employing the additional worker, $50. So the marginal cost of producing the 52nd shirt is $50/8 = $6.25. Alternatively, the total cost of producing 48 shirts is $100, the fixed cost, plus 3 × $50, the variable cost, which is $250. And the total cost of producing 56 shirts is $100, the fixed cost, plus 4 × $50, the variable cost, which is $300. Then the marginal cost equals the change in the total cost divided by the change in output, or $50/8, which is $6.25. e) Average total cost is total cost divided by the output produced. Total cost is fixed cost plus variable cost. Randy’s fixed cost is $100. To produce 48 shirts, he needs 3 workers, so his total variable cost is $50 × 3 = $150. Randy’s total cost is $100 + $150 = $250. Then his average total cost is $250/48, which is $5.21.

Total product (widgets per day) 0 5 15 35 50 60 65 Total product (widgets per day) 0 5 15 35 50 60 65 5)

Total product (widgets per day) 0 5 15 35 50 60 65

TFC

AFC

TVC

AVC

TC

ATC

MC

The first table above has the total product schedule for an imaginary good called a widget. Each unit of labor costs $25 and the total cost of capital is $100. a) Use this information to complete the remaining two tables. In the tables, TFC is the total fixed cost, TVC is the total variable cost, TC is the total cost, AFC is the average fixed cost, AVC is the average variable cost, ATC is the average total cost, and MC is the marginal cost.

210

CHAPTER 10

Total product (widgets per day) 0 5 15 35 50 60 65 Total product (widgets per day) 0 5 15 35 50 60 65

TFC

AFC

AVC

TVC

ATC

TC

Answer:

TFC 100 100 100 100 100 100 100

TVC 0 25 50 75 100 125 150

TC 100 125 150 175 200 225 250

AFC

AVC

ATC

MC

20.00 6.67 2.86 2.00 1.67 1.54

5.00 3.33 2.14 2.00 2.08 2.31

25.00 10.00 5.00 4.00 3.75 3.85

5.00 2.50 1.25 1.67 2.50 5.00

a) The completed tables are above. All the costs in dollars.

MC

b) Suppose that labor becomes twice as expensive (so that one unit of labor now costs $50) but nothing else changes. Complete the above tables with the new cost schedules. If you plotted the cost curves, how would the increased wage rate affect the cost curves? Total product (widgets per day) 0 5 15 35 50 60 65

Total product (widgets per day) 0 5 15 35 50 60 65

Total product (widgets per day) 0 5 15 35 50 60 65 Total product (widgets per day) 0 5 15 35 50 60 65

TFC 100 100 100 100 100 100 100

TVC 0 50 100 150 200 250 300

TC 100 150 200 250 300 350 400

AFC

AVC

ATC

MC

20.00 6.67 2.86 2.00 1.67 1.54

10.00 6.67 4.29 4.00 4.17 4.62

30.00 13.33 7.14 6.00 5.83 6.15

10.00 5.00 2.50 3.33 5.00 10.00

b) The completed tables are above. All the costs are in dollars. There would be no change in the total fixed and average fixed cost curves because the fixed cost did not change. The total cost, total variable cost, average total cost, average variable cost, and marginal cost curves would all shift upward. Topic: Short Run Costs Skill: Analytical

6)

This month, the local widget factory produced 100 widgets. The total variable cost of production was $500 and the average total cost of production was $8. a) What is the total cost? b) What is the total fixed cost? c) What is the average fixed cost?

OUTPUT AND COSTS

211

ing that the total variable cost, TVC, for 4 oil changes is $70. Then, the average variable cost, AVC, equals the total variable cost divided by output, or TVC ÷ Q. In this case, the AVC is equal to $70 ÷ 4 = $17.50. d) The average fixed cost of 2 oil changes is $5. The average fixed cost, AFC, equals the total fixed cost divided by output, or TFC ÷ Q. From part (a), the total fixed cost is $10, so the average fixed cost equals $10 ÷ 2 = $5. e) The marginal cost is $15. Marginal cost is defined as the change in the total cost that results from a one-unit increase in output. The total cost of 2 oil changes is $35 and the total cost of 3 oil changes is $50. Therefore the change in the total cost is $15, which is the marginal cost.

d) What is the average variable cost? Answer:

a) Total cost can be calculated as average total cost times output, which is $8 × 100 = $800. b) Total fixed cost equals total cost minus total variable cost. The total cost from part (a) is $800. The total variable cost is $500. Therefore the total fixed cost is $800 − $500 = $300. c) Average fixed cost equals total fixed cost divided by output, which is $300/100 = $3. d) Average variable cost equals total variable cost divided by output. The total variable cost from part (b) is $500, so the average variable cost is $500/100 = $5. Topic: Short Run Costs Skill: Analytical

Output (oil changes per hour) 0 1 2 3 4 7)

Total cost (dollars per hour) 10 20 35 50 80

The above table gives the total cost schedule for oil changes at the local Jiffy Lube. a) What is Jiffy Lube’s total fixed cost? b) What is the total variable cost of 2 oil changes? c) What is the average variable cost of 4 oil changes? d) What is the average fixed cost of 2 oil changes? e) What is the marginal cost of the 3rd oil change?

Topic: Short Run Cost Curves Skill: Analytical

8)

Draw an average total cost curve, an average variable cost, and a marginal cost curve all on the same graph. Make sure to correctly label the axes. What relationship must exist between the marginal cost curve and the average total cost and average variable cost curves? Answer:

Answer:

a) The total fixed cost is $10. At zero output, the total variable cost is $0, so the entire cost, $10, must be the fixed cost. b) The total variable cost of 2 oil changes is $25. The total cost, TC, is $35 and from part (a) the total fixed cost, TFC, is $10. The total variable cost, TVC, is equal to TC − TFC or $35 − $10 = $25. c) The average variable cost for 4 oil changes is $17.50. Start similarly to part (b) by calculat-

Typical cost curves are above. The average variable cost curve, AVC, and average total cost curve, ATC, are U-shaped. The marginal cost curve, MC, intersects the AVC curve and ATC curve at their minimum points.

212

CHAPTER 10

Topic: Short Run Cost Curves Skill: Analytical

e) What is the marginal cost of one of the 50 newspapers folded and bagged by the fourth student? Answer:

9)

Are the curves in the figure above drawn correctly? If not, what’s wrong? Answer:

The curves are not drawn correctly. First, the marginal cost curve must intersect the average variable cost curve and the average total cost curve where they are at their minimums, which is not the case in the figure. Second, the vertical distance between the ATC curve and the AVC curve is the AFC, which decreases as output increases. So the vertical distance between the ATC curve and the AVC curve should shrink in size, which is not the case in the figure. Topic: Cost Curves and Product Curves Skill: Analytical

10) Suppose the local newspaper hires students to fold and bag newspapers for delivery and pays them $20 per shift. Five students can fold and bag 300 newspapers per shift. The fourth student added 50 newspapers to total output. The capital cost is fixed at $50 per shift. a) Is the newspaper operating in the long run or short run? Why? b) What is the average product of 5 students? c) Calculate the total fixed, total variable, and total costs of folding and bagging 300 newspapers. d) Calculate the average fixed, average variable, and average total costs of folding and bagging 300 newspapers.

a) The firm is operating in the short run because its capital cost is fixed. b) Average product equals 300 ÷ 5 = 60 newspapers per shift. c) Total fixed cost = $50, total variable cost = 5 × $20 = $100, and so total cost = $50 + $100 = $150. d) Average fixed cost = $50 ÷ 300 = $0.17, average variable cost = $100 ÷ 300 = $0.33, and average total cost = $150 ÷ 300 = $0.50 (alternatively, average total cost = average fixed cost + average variable cost = $0.17 + $0.33 = $0.50). e) Marginal cost = change in total cost divided by change in output, or $20 ÷ 50 = $0.40. Topic: Long-Run Average Cost Skill: Analytical

Year 1 2 3 4 5 6

Total product 100 240 500 1,000 1,800 2,500

Total cost Average cost (dollars) (dollars) 1,000 ___ 2,000 ___ 4,000 ___ 8,000 ___ 16,000 ___ 32,000 ___

11) Jones Production started business with a small scale plant. Fortunately for Smith, the owner of Jones Production, the business grew rapidly. It doubled its plant scale and its labor force every year for the next six years. The table above gives the total costs and the associated total products for each year. a) Complete the table by finding the average cost for each scale. b) Over what range of total product (output) did Jones Production experience economies of scale, constant returns to scale, and diseconomies of scale?

OUTPUT AND COSTS

213

Answer:

Year 1 2 3 4 5 6

Total product 100 240 500 1,000 1,800 2,500

Total cost Average cost (dollars) (dollars) 1,000 10.00 2,000 8.33 4,000 8.00 8,000 8.00 16,000 8.88 32,000 12.80

Topic: Long-Run Average Cost Curve Skill: Analytical

a) The completed table is above. b) There are economies of scale from 100 to 500 units of total product, constant returns to scale from 500 to 1,000 units of total product, and diseconomies of scale from 1,000 to 2,500 units of total product. Topic: Long-Run Average Cost Curve Skill: Analytical

12) Ayanna grows herbs. Last year she grew 2,000 pounds of herbs in a year while using 250 square feet of land and 1 worker. This year she doubled her land to 500 square feet, doubled her workers to 2, and grew 4,500 pounds of herbs. She sells her rare, organic herbs for $50 a pound. She pays her workers $25,000 a year and rents her land for $100 per square foot for a year. These are her only costs. a) What was Ayanna’s total cost last year and this year? b) What was Ayanna’s average total cost last year and this year? c) Did Ayanna experience economies or diseconomies of scale? Answer:

a) Ayanna’s total cost last year was $50,000 and her total cost this year is $100,000. b) Ayanna’s average total cost last year was $25 a pound and her average total cost this year is $22.22 a pound. c) Ayanna’s average total cost decreased so she experienced economies of scale.

13) The above figure represents the average total cost curves of a wheat farmer. a) Which average total cost curve has the lowest average total cost of producing 30,000 bushels of wheat? b) Over what range of output is the farmer experiencing economies of scale? c) Over what range of output is the farmer experiencing diseconomies of scale? d) Which average total cost curve has the lowest possible average cost of production? e) Which average total cost curve represents the largest plant? Answer:

a) ATC3 has the lowest average total cost to produce 30,000 bushels of wheat. b) The farmer is experiencing economies of scale between 0 to 30,000 bushels of wheat. c) The farmer is experiencing diseconomies of scale for more than 30,000 bushels of wheat. d) ATC3 has the lowest possible average total cost, which occurs at 30,000 bushels of wheat. e) ATC4 represents the largest plant.

CHAPTER 11

PERFECT COMPETITION

Topic: Total Revenue and Marginal Revenue Skill: Analytical

Quantity (dozens of roses a week) 1 2 3 4 1)

Topic: Total Revenue and Marginal Revenue Skill: Analytical

Price (dollars per dozen roses) 12 12 12 12

Pete is a perfectly competitive rose grower. The above table gives quantities and the price for which Pete can sell his roses. a) What is Pete’s total revenue if he sells 1 dozen roses? 2 dozen roses? 3 dozen roses? 4 dozen roses? b) What is the marginal revenue of the 2nd dozen roses sold? Of the 3rd dozen? Of the 4th dozen? Answer:

a) The total revenue when 1 dozen roses is sold is $12. When Pete sells 2 dozen roses, the total revenue is $24. When 3 dozen roses are sold, the total revenue is $36. And the total revenue when 4 dozen roses are sold is $48. b) The marginal revenue is always $12 per dozen roses.

2)

Farmer Brown produces corn in a perfectly competitive market. Farmer Brown produces and sells 500 bushels of corn. The market supply and demand curves are illustrated in the above figure. a) What is Farmer Brown’s total revenue? b) What is Farmer Brown’s marginal revenue? Answer:

a) Total revenue = price × quantity. The price is determined by the intersection of the demand and supply curves, $6 a bushel. As a result, Farmer Brown’s total revenue is $6 × 500 = $3,000. b) For a perfectly competitive firm, the marginal revenue equals the price, so Farmer Brown’s marginal revenue is $6.

216

CHAPTER 11

Topic: Total Revenue, Total Cost, and Economic Profit Skill: Analytical

Quantity (wreaths) 0 1 2 3 4 5 6 7 8 9 3)

Total cost (dollars) 1 10 18 24 28 33 40 49 60 73

Total revenue (dollars)

Profit or loss (dollars)

The above table gives Amy’s total cost schedule for producing holiday wreaths. Amy is a perfect competitor and can sell each wreath for $9. a) Complete the table by calculating Amy’s total revenue and her profit or loss schedule. b) When Amy is producing 4 wreaths, what is her total cost? What is her total revenue? What is her economic profit or economic loss? c) What number of wreaths maximizes Amy’s profit?

Topic: Profit-Maximizing Output Skill: Analytical

Quantity (thousands of bushels of corn a week)) 0 10 20 30 40 50 60 70 80 4)

Total cost (dollars) 1 10 18 24 28 33 40 49 60 73

Total revenue (dollars) 0 9 18 27 36 45 54 63 72 81

Profit or loss (dollars) −1 −1 0 3 8 12 14 14 12 8

a) The completed table is above. b) When Amy is producing 4 wreaths, her total cost is $28, her total revenue is $36, and her economic profit is $8. c) Amy can produce 6 or 7 wreaths, with a maximum economic profit of $14.

Total cost (thousands of dollars per week)

0 30 60 90 120 150 180 210 240

50 70 80 90 110 140 200 280 400

Jimmy grows corn. His total revenue and total cost are in the above table. What quantity of corn maximizes his profit and what is his profit? What is the marginal revenue and marginal cost at this quantity? Answer:

Jimmy’s profit is greatest if he grows either 40,000 or 50,000 bushels of corn. His (economic) profit at either amount is $10,000 a week. Between 40,000 and 50,000 bushels of corn, Jimmy’s marginal revenue is $30,000, or $3 per bushel and his marginal cost is $30,000, or $3 per bushel.

Answer:

Quantity (wreaths) 0 1 2 3 4 5 6 7 8 9

Total revenue (thousands of dollars per week)

Topic: Profit-Maximizing Output Skill: Analytical

Quantity (units) 0 1 2 3 4 5 5)

Total cost (dollars) 3 6 12 21 33 49

The above table gives the quantity of output and the total cost for a perfectly competitive firm that can sell all of its output at $9 per unit. a) Find the profit maximizing level of output for this firm. b) How much economic profit is the firm making?

PERFECT COMPETITION

217

Answer:

Quantity (units) 0 1 2 3 4 5

Answer:

Total cost (dollars) 3 6 12 21 33 49

Total revenue (dollars) 0 9 18 27 36 45

Profit or loss (dollars) −3 3 6 6 3 −4

a) Total revenue equals price times quantity sold. We can find the total revenue for this firm because the market price is a constant $9. The total revenue schedule is given in the table above. The total profit (or loss) equals total revenue minus total cost. The last column shows that the total profit is largest if either 2 or 3 units are produced. b) When the firm produces either 2 or 3 units of output, the total economic profit is $6.

Quantity (units) 0 1 2 3 4 5 6 7 8 9

6)

Total cost (dollars) 400 550 600 650 740 850 990 1,140 1,400 1,690

Total revenue (dollars) ___ ___ ___ ___ ___ ___ ___ ___ ___ ___

Profit or loss (dollars) ___ ___ ___ ___ ___ ___ ___ ___ ___ ___

The above table shows the total cost schedule for a perfectly competitive firm. The market price is $250 per unit. Complete the table.

Total revenue (dollars) 0 250 500 750 1,000 1,250 1,500 1,750 2,000 2,250

Profit or loss (dollars) −400 −300 −100 100 260 400 510 610 600 560

The completed table is above. Topic: Profit-Maximizing Output Skill: Analytical

Topic: Profit-Maximizing Output Skill: Analytical

Quantity (units) 0 1 2 3 4 5 6 7 8 9

Total cost (dollars) 400 550 600 650 740 850 990 1,140 1,400 1,690

7)

Quantity Total cost (units) (dollars) 0 12.00 1 20.00 2 26.00 3 30.00 4 32.00 5 36.00 6 42.00 7 49.80 8 64.00 9 87.60 10 124.00 Acme is a perfectly competitive firm. It has the total cost schedule given in the above table. Acme’s product sells for $8.00 per unit. What amount of output is the most profitable and what is Acme’s economic profit or economic loss? Answer:

Acme’s profit-maximizing level of output is 7 units. Acme’s total economic profit equals its revenue, $56.00 ($8.00 per unit × 7 units), minus its total cost, $49.80 at this level of output, or $56.00 − $49.80 = $6.20.

218

CHAPTER 11

300 cones, so 300 cones maximizes Cory’s profit. b) Cory’s profit is maximized when his marginal cost is equal to the market price. Marginal cost equals the price of $2.25 when Cory sells 300 cones, so 400 cones maximizes Cory’s profit. c) Cory’s profit is maximized when his marginal cost is equal to the market price. Marginal cost equals the price of $2.75 when Cory sells 300 cones, so 500 cones maximizes Cory’s profit. d) One point on Cory’s supply curve is a price of $1.75, quantity of 300 cones; another point is a price of $2.25, quantity of 400 cones; and a third point is price of $2.75, quantity of 500 cones.

Topic: Profit-Maximizing Output Skill: Analytical

Quantity (cones per day) 0 100 200 300 400 500 600 8)

Total cost (dollars per day) 100 250 350 500 700 950 1,2500

Cory’s is one of many ice cream stands in town. Cory’s costs are shown in the table above. a) If the market price of ice cream is $1.75 per cone, what is Cory’s profit-maximizing quantity of ice cream sold? b) If the market price of ice cream is $2.25 per cone, what is Cory’s profit-maximizing quantity of ice cream sold? c) If the market price of ice cream is $2.75 per cone, what is Cory’s profit-maximizing quantity of ice cream sold? d) Can you determine any points on Cory’s supply curve?

Topic: Profit-Maximizing Output Skill: Analytical

Quantity (units) 1

Average variable cost (dollars) 8.00

2

7.00

3

6.00

4

5.00

5

4.80

1.50

6

5.00

1.00

7

5.40

8

6.50

9

8.40

Answer:

Quantity (cones per day) 0 100

Total cost (dollars per day) 100

Marginal cost (dollars per cone)

250

200

350

300

500

1.50 2.00 400

700

500

950

2.50 3.00 600

1,2500

a) Cory’s profit is maximized when his marginal cost is equal to the market price. Marginal cost is the change in the total cost divided by the change in the quantity. Cory’s marginal costs are shown in the table above. Marginal cost equals the price of $1.75 when Cory sells

9)

Marginal cost (dollars) 6.00 4.00 2.00 4.00 6.00 7.80 14.20 23.60 36.40

Acme is a perfectly competitive firm. It has the cost schedules given in the above table and has a fixed cost of $12.00. The price of Acme’s product is $14.20. What is Acme’s most profitable amount of output? What is Acme’s total economic profit or loss? Answer:

The profit maximizing level of output is either 7 or 8 units. Acme’s total economic profit is the economic profit per unit times the number of units produced. The economic profit per unit equals the price minus the average total cost. To calculate av-

PERFECT COMPETITION

219

erage total cost, note that when Acme produces 8 units, the average variable cost per unit is $6.50 and the average fixed cost is $1.50, so Acme’s average total cost equals $8.00. Thus Acme makes an economic profit of $14.20 − $8.00 = $6.20 per unit. Hence Acme’s total economic profit is ($6.20) × (8 units) = $49.60. Acme’s total economic profit when it makes 7 units is (except for rounding) identical.

Topic: Profit-Maximizing Output Skill: Analytical

Topic: Profit-Maximizing Output Skill: Analytical

Quantity (units) 1

Average variable cost (dollars) 8.00

2

7.00

3

6.00

4

5.00

5

4.80

6

5.00

7

5.40

8

6.50

9

8.40

Marginal cost (dollars) 6.00 4.00 2.00 4.00 6.00 7.80 14.20 23.60 36.40

10) Acme is a perfectly competitive firm. It has the cost schedules given in the above table and has a fixed cost of $12.00. The price of Acme’s product is $4.00. What is Acme’s most profitable amount of output? What is Acme’s total economic profit or loss? Answer:

Acme’s most profitable (which means, in this case, minimum loss) is output of 0 units. The price is below Acme’s minimum average variable cost, so Acme shuts down. Acme’s economic loss equals its fixed costs, $12.00.

11) The above figure illustrates a perfectly competitive wheat farmer. a) What will be the firm’s profit-maximizing price and output? b) When the farmer produces 25,000 bushels of wheat, the difference between the firm’s average total cost and the price is at its maximum. Explain why this amount of wheat either is or is not the profit-maximizing quantity. Answer:

a) The firm will maximize its profits by producing where its marginal revenue equals its marginal cost, or 30,000 bushels of wheat. The price will equal the marginal revenue, $3.00 a bushel. b) At 25,000 bushels, the difference between average cost and price (which is the average revenue) is indeed at the maximum, but choosing 25,000 will maximize the profit per bushel of wheat, not the total profit. The firm is interested in maximizing the total profit not the profit per bushel. At 30,000 bushels of wheat, the total profit is maximized because this is the amount of output where the difference between total revenue and total cost is at a maximum.

220

CHAPTER 11

Topic: Shutdown Point Skill: Analytical

Economic profit equals total revenue minus total cost = $1,000 − $800 = $200. Topic: Economic Profits and Economic Losses in the Short Run Skill: Analytical

12) The above diagram shows the cost curves for a perfectly competitive wheat farmer. At what price does the wheat farmer shut down? Answer:

The wheat farmer shuts down if the price is less than the minimum average variable cost. So in the figure, the wheat farmer shuts down if the price is less than $2 per bushel. Topic: Economic Profits and Economic Losses in the Short Run Skill: Analytical

13) John keeps beehives and sells 100 quarts of honey per month. The honey market is perfectly competitive, and the price of a quart of honey is $10. John has an average variable cost of $5 and an average fixed cost of $3. At 100 quarts per month, John’s marginal cost is $10. a) Is John maximizing his profit? If not, what should John do? b) Calculate John’s total revenue, total cost, and total economic profit or economic loss when he produces 100 quarts of honey. Answer:

a) Yes, John is maximizing his profit because marginal revenue (price) equals marginal cost. b) Total revenue equals price times quantity = $10 × 100 = $1,000. Total cost equals average total cost times quantity = ($5 + $3) × 100 = $800.

14) The above diagram shows the cost curves for a perfectly competitive wheat farmer. At what price(s) does the wheat farmer earn an economic profit? Earn a normal profit? Incur an economic loss? How many bushels of wheat does the farmer produce if the price is $3 per bushel? If the price is $0.50 per bushel? Answer:

At any price that exceeds the minimum of the average total cost the farmer earns an economic profit. So the farmer earns an economic profit if the price is greater than $2 per bushel. The farmer earns a normal profit if the price equals the minimum total cost. So the farmer earns a normal profit if the price is $2 per bushel. Finally, the farmer incurs an economic loss if the price is less than the minimum average total cost. So the farmer incurs an economic loss if the price is less than $2 per bushel. If the price is $3 per bushel, the farmer produces 30,000 bushels of wheat per year. If the price is $0.50 per bushel, the farmer has shut down because the price is less than the minimum average variable cost and so the farmer produces 0 bushels per year.

PERFECT COMPETITION

Topic: Long-Run Adjustments; Entry Skill: Analytical

221

Topic: Long-Run Adjustments; Change in Price Skill: Analytical

15) The above figure shows the cost curves of a profitmaximizing perfectly competitive firm. If the price equals $7, a) how much will the firm produce? b) how much is the firm’s average total, average variable, and marginal costs? c) how much is the firm’s total, total variable, and total fixed costs? d) how much is the firm’s total revenue and economic profit? e) what will happen in this market in the long run? Answer:

a) The firm will produce 40 units of output because that is where the marginal revenue equals the marginal cost. b) The firm’s average total cost equals $4, its average variable cost equals $3, and its marginal cost equals $7. c) The firm’s total cost is $160 (= $4 × 40), its total variable cost is $120 (= $3 × 40), and its total fixed cost is $40 (= $160 − $120). d) The firm’s total revenue is $280 (= $7 × 40) and its economic profit is $120 (= $280 − $160). e) In the long run, firms will enter the market in response to the economic profit. The market supply curve will shift rightward, the price will fall, and the economic profit will be eliminated.

16) American restaurants receive their supply of baby back-ribs from American farms and from farms in Denmark. In the figures above, the first diagram shows the perfectly competitive market for baby back ribs in the United States. The second figure shows the situation at Premium Standard Farm in Kansas, one of the many U.S. farms supplying these ribs. Now assume that the United States imposes a ban on European meat in response to the foot-andmouth disease that has infected livestock in Europe. (Which the United States did in April 2001.) In particular, suppose that the U.S. ban decreases the supply by 40 tons a year. Using the

222

CHAPTER 11

first figure, show the impact of this ban on the baby back rib market. Using the second figure, show the impact on Premium Standard Farm in Kansas. Answer:

dard Farm will shift upward, from MR0 to MR1. As a result, the farm increases its production to 40,000 pounds of ribs. Because the price exceeds the average total cost, the Premium Standard Farm makes an economic profit. Topic: Long-Run Equilibrium Skill: Analytical

17) Suppose the bobby pin industry is perfectly competitive. The price of a packet of bobby pins is $2.00. Pins and Needles, Inc. is a firm in this industry and is producing 1,000 packets of bobby pins per day at the point where the MC = MR. The average cost of production at this output level is $1.50 per packet. a) What is the marginal cost of the 1,000th packet? b) Is this firm making an economic profit, a normal profit, or an economic loss? How much? c) Is the firm in long-run equilibrium? Why or why not? Answer:

a) The price per packet is $2, which is also the Pins and Needle’s marginal revenue. The marginal cost of the 1,000th packet is equal to marginal revenue, so for Pins and Needles the marginal cost is $2 per packet. b) The firm is making a $0.50 economic profit per unit (which equals the price minus the average total cost). Because Pins and Needles produces 1,000 packets, its total economic profit is $500. c) The firm is making an economic profit, so it is not in long-run equilibrium. In the long run, a perfectly competitive firm cannot earn an economic profit. The only outcome possible in the long run is a normal profit. The ban on European meat decreases the supply of baby back ribs and shifts the supply curve leftward, as shown by the shift from S0 to S1. The price rises to $4 a pound. As the figure on the right shows, the MR curve for the Premium Stan-

CHAPTER 12

MONOPOLY

Topic: Price and Marginal Revenue Skill: Analytical

Quantity (units) 15

Price (dollars) 0

Total revenue (dollars) ____

13

1

____

11

2

____

9

3

____

7

4

____

5

5

____

3

6

____

1

7

____

0

8

____

1)

Topic: Price and Marginal Revenue Skill: Analytical

Marginal revenue (dollars) ____

Price (dollars) 6

Total revenue (dollars) ____

2

5

____

3

4

____

4

3

____

5

2

____

6

1

____

____ ____ ____ ____ ____ ____ ____

2)

Marginal revenue (dollars) ____ ____ ____ ____ ____

The above table gives a monopolist’s demand schedule. Complete the table by calculating the total revenue and the marginal revenue. Answer:

The above table gives a monopolist’s demand schedule. Complete the table by calculating the total revenue and the marginal revenue. Answer:

Quantity (units) 15

Price (dollars) 0

Total revenue (dollars) 0

13

1

13

11

2

22

9

3

27

7

4

28

−3

5

5

25

−7

3

6

18

−11

1

7

7

−7

0

8

0

The completed table is above.

Quantity (units) 1

Marginal revenue (dollars) 13 9 5 1

Quantity (units) 1

Price (dollars) 6

Total revenue (dollars) 6

2

5

10

3

4

12

4

3

12

−2

5

2

10

−4

6

1

6

The completed table is above.

Marginal revenue (dollars) 4 2 0

224

CHAPTER 12

Topic: Price and Marginal Revenue Skill: Analytical

3)

Quantity (units) 0

Price (dollars) 24

1

22

2

20

3

18

4

16

5

14

6

12

7

10

8

8

Topic: Marginal Revenue and Elasticity Skill: Analytical

Marginal revenue (dollars) ____ ____ ____ ____ ____ ____ ____ ____ 4)

The demand schedule for a monopolist is given in the above table. Calculate the marginal revenue. Answer:

Quantity (units) 0

Price (dollars) 24

Marginal revenue (dollars)

1

22

2

20

3

18

4

16

5

14

6

12

−2

7

10

−6

8

8

The completed table is above.

22 18 14 10 6 2

The above figure represents the demand and marginal revenue curves for Sue’s Seafood, a seller of fresh fish. a) Over what range of output is demand elastic? b) Over what range of output is demand inelastic? c) What price maximizes total revenue? d) What is the demand elasticity at the revenue maximizing price? Answer:

a) Demand is elastic from 0 to 20 pounds of fish. b) Demand is inelastic from 20 to 40 pounds of fish. c) Total revenue is maximized when the price is $8 a pound. d) Total revenue is maximized when demand is unit elastic.

MONOPOLY

225

Topic: Single-Price Monopoly’s Output and Price Decisions Skill: Analytical

5)

Quantity (units)

Price (dollars)

Marginal revenue (dollars)

Marginal cost (dollars)

1

22

20

6

2

20

16

8

3

18

12

12

4

16

8

18

5

14

4

28

6

12

0

40

7

10

−4

54

8

8

−8

70

A single-price monopolist has the demand and marginal cost schedules given in the above table. What is the profit-maximizing level of output and price? Answer:

The profit-maximizing output is 3 units, because that is the quantity for which the marginal revenue equals the marginal cost. The price is $18 a unit.

Quantity (rutabagas per day) 0 20 40 60 80 100

Total cost (dollars per day) 80 100 120 180 280 420

b) Bob has the cost schedule shown in the table above. Draw the demand curve faced by Bob and his marginal revenue curve. Draw Bob’s marginal cost curve. If Bob wants to maximize his profit, how many fried rutabagas should he sell? c) What price will Bob charge? d) What is Bob’s economic profit? e) Bob currently charges $3 per fried rutabaga. Is he maximizing his profit? Why or why not Answer:

a) When Bob sells 40 rutabagas, his total revenue is $160. When he sells 60 rutabagas, his total revenue is $180. Bob’s total revenue increases by $20 when 20 additional rutabagas are sold, or by $1 per additional rutabaga. So the marginal revenue from the 50th fried rutabaga is $1.

Topic: Single-Price Monopoly’s Output and Price Decisions Skill: Analytical

6)

Price Quantity demanded (dollars per rutabaga) (rutabagas per day) 1 100 2 80 3 60 4 40 5 20 6 0 Bob’s Country Bunker is the only restaurant in town that serves fried rutabaga. Bob faces the demand schedule shown in the table above. a) What is Bob’s marginal revenue from the 50th rutabaga?

b) The figure above has Bob’s demand curve, marginal revenue curve, and marginal cost curve. Bob maximizes his profit if he sells 40 fried rutabagas per day. At this quantity, MC = MR = $2.

226

CHAPTER 12

c) As the figure shows, Bob will sell 40 fried rutabagas per day if he charges $4 per rutabaga d) Bob sells 40 fried rutabagas at a price of $4, so his total revenue is $160. The total cost of producing 40 fried rutabagas is $120. So Bob earns a $40 economic profit. e) If Bob charges $3 per fried rutabaga, his sells 60 rutabagas and his marginal cost, $4, is greater than his marginal revenue, $0. In this case he is not maximizing his profit. Bob maximizes his profit if he charges $4 per fried rutabaga and sells 40 rutabagas per day. At this quantity, MC = MR = $2.

Answer:

Topic: Single-Price Monopoly’s Output and Price Decisions Skill: Analytical

The completed figure is above. Topic: Single-Price Monopoly’s Output and Price Decisions Skill: Analytical

7)

In the above figure, draw and label the demand and cost curves of a monopoly. Identify the quantity a single-price monopoly will produce by labeling it Qm and identify the price by labeling it Pm. 8)

Ron’s Hamburger Joint is the only restaurant in town. The above figure represents Ron’s cost, demand, and marginal revenue curves. Ron operates as a single-price monopoly. a) How many hamburgers does Ron produce? b) What price does Ron charge for a hamburger? c) What is Ron’s total revenue? d) What is his total cost? e) What is Ron’s economic profit? Answer:

a) Ron produces 20 hamburgers per hour. b) The price is $6 for a hamburger.

MONOPOLY

c) Ron’s total revenue is $120 an hour. d) Ron’s total cost is $80 an hour. e) Ron’s economic profit is $40 an hour. Topic: Comparing Output and Price Skill: Analytical

227

Topic: Perfect Price Discrimination Skill: Analytical

Quantity (units) 1 2 3 4 5 6 7 8

Price (dollars) 22 20 18 16 14 12 10 8

Marginal cost (dollars) 9 10 12 16 22 30 40 52

10) A monopolist has the demand and marginal cost schedules given in the above table. If the monopoly can perfectly price discriminate, what is the profit-maximizing level of output and price? Answer:

As a perfectly price discriminating monopoly, the profit-maximizing level of output is 4 units and the price ranges from $22 to $16 for each unit. 9)

The above figure represents a perfectly competitive industry that is taken over by a single firm and operated as a monopoly. a) What was the competitive price and quantity? b) What is the monopoly price and quantity? c) What area represents consumer surplus under perfect competition? d) What area represents consumer surplus under monopoly? e) What area represents the deadweight loss of monopoly?

Topic: Perfect Price Discrimination Skill: Analytical

Answer:

a) The competitive price was P2 and the competitive quantity was Q2. b) The monopoly price is P3 and the monopoly quantity is Q1. c) The consumer surplus with perfect competition is the area P4P2c. d) The consumer surplus with monopoly is the area P4P3a. e) The deadweight loss from the monopoly is the area abc.

11) The above figure represents the cost, demand, and marginal revenue curves for a monopolist. a) Indicate the price and quantity a single-price monopolist selects by labeling the price Pm and the quantity Qm. b) In the figure, lightly shade in the area that represents the single-price monopoly’s economic profit.

228

CHAPTER 12

c) Indicate the quantity a perfectly pricediscriminating monopolist selects by labeling it Qppd. d) In the figure, more darkly shade in the area that represents the additional economic profit the monopoly earns as a result of the perfect price discrimination.

Topic: Regulating Natural Monopoly Skill: Analytical

Answer:

a) The price is labeled Pm and the quantity is labeled Qm. b) The single-price monopoly’s economic profit is the lightly shaded rectangle. c) The quantity is labeled Qppd. d) The additional profit is the two darker areas in the figure.

12) The above figure illustrates the market for electric power that is served by the one utility in Alberta, Canada. a) If the government did not regulate this utility, what would be the price of a kilowatt hour in this region and how much power would be generated? b) If the government regulates the utility and chooses an average cost pricing rule, what would be the price of a kilowatt hour and how much power would be generated? c) If the government regulates the utility and chooses a marginal cost pricing rule, what would be the price of a kilowatt hour and how much power would be generated? Answer:

a) The price would be 12¢ a kilowatt hour and 20 megawatts per hour would be generated. b) The price would be 8¢ a kilowatt hour and 30 megawatts per hour would be generated. c) The price would be 4¢ a kilowatt hour and 40 megawatts per hour would be generated.

MONOPOLY

Topic: Regulating Natural Monopoly Skill: Analytical

13) The above figure shows the demand for cable and the cable company’s cost of providing cable. a) What price and quantity will be produced if the company is unregulated and profit maximizes? b) What price and quantity will be produced if the company is regulated using the marginal cost pricing rule? c) What is the advantage of the marginal cost pricing rule? d) What price and quantity will be produced if the company is regulated using the average cost pricing rule? e) What is the advantage of the average cost pricing rule?

229

Answer:

a) The price will be $90 per month and the quantity will be 20,000 households. b) The price will be $30 per month and the quantity will be 40,000 households c) This rule results in the efficient level of production. d) The price will be $60 per month and the quantity will be 30,000 households e) This rule results in the firm earning a normal profit.

CHAPTER 13

MONOPOLISTIC COMPETITION AND OLIGOPOLY Therefore it is necessary to determine the marginal revenue. The marginal revenue going from 3 to 4 units is $12 and the marginal revenue going from 4 to 5 units is $8. Thus the marginal revenue at 4 units is $10, which equals the marginal cost. Therefore the firm produces 4 units. The demand schedule shows that for 4 units, the price will be $16 per unit. The firm’s economic profit per unit equals the price, $16, minus its average total cost, $10.50, or an economic profit of $5.50 per unit. The firm produces 4 units, so its total economic profit is $22.00. The firm is in a short-run equilibrium because it is able to earn an economic profit. In the long run, entry will decrease its demand so that it no longer can earn an economic profit.

Topic: Monopolistic Competition; Profit Maximization Skill: Analytical

Price (dollars per unit) 26 24 22 20 18 16 14 12 10 Quantity produced (units) 0 1 2 3 4 5 6 7 8 1)

Quantity demanded (units) 0 1 2 3 4 5 6 7 8

Average total cost (dollars)

Marginal cost (dollars)

18.00 12.00 10.66 10.50 11.20 12.66 15.14 23.25

8.00 6.00 8.00 10.00 14.00 20.00 30.00 80.00

Topic: Monopolistic Competition; Profit Maximization Skill: Analytical

The demand and cost schedules for a firm in monopolistic competition are in the above tables. What is the profit-maximizing level of output and price? What amount of profit is the firm earning? Is this firm in a short-run or long-run equilibrium? Why? Answer:

To determine the quantity produced, the firm will set marginal cost equal to marginal revenue.

2)

The above figure represents Tony’s Pizza Parlor, a firm in monopolistic competition.

232

CHAPTER 13

a) b) c) d) e) f)

What quantity will be produced? What price will be charged? What is Tony’s total cost? What is Tony’s total revenue? What is Tony’s economic profit or loss? Is this a long-run equilibrium? Why or why not?

Topic: Monopolistic Competition; Long-Run Equilibrium Skill: Analytical

Answer:

a) b) c) d) e) f)

40 pizzas per day $12 per pizza $320 $480 Tony has an economic profit of $160. This is not a long-run equilibrium because Tony is earning an economic profit. In the long run, new firms will enter and Tony’s economic profit will be eliminated.

Topic: Monopolistic Competition; Short-Run Economic Profit Skill: Analytical

3)

4)

Draw an example of a firm in monopolistic competition that is earning an economic profit. Be sure to label all the curves. Indicate the area that equals the firm’s economic profit. Answer:

The above figure represents a restaurant operating in monopolistic competition. a) What is the profit-maximizing level of output? b) What price will the firm charge? c) What is the firm’s profit (or loss)? d) Is this a long-run equilibrium? Why or why not? e) Is this firm producing its capacity output? Answer:

a) The quantity is 20 meals a day. b) The price is $12 per meal. c) The firm is making zero economic profit, that is, the firm is earning a normal profit. d) This is a long-run equilibrium because the firm is making zero economic profits so there is no incentive for either entry or exit. e) No, the firm is not producing its capacity output. It is producing less than the capacity. The capacity, where the average total cost is at is minimum, is 50 meals a day. Hence the firm has excess capacity. The completed figure is above.

Topic: Monopolistic Competition; Long-Run Equilibrium Skill: Analytical

5)

Sharp Vision, Inc., a firm in monopolistic competition, produces TV sets. The company can sell no TV sets at $400, and for each $50 cut in price, the quantity of TV sets it can sell increases by 50 a day. This relationship continues to hold until

MONOPOLISTIC COMPETITION AND OLIGOPOLY

the price falls to $100. The firm’s total fixed cost is $15,000 a day. Its marginal cost is constant at $100 per unit. a) What is the firm’s profit maximizing level of output? b) What is the firm’s profit maximizing price? c) What is the firm’s economic profit?

Topic: Cartel Skill: Analytical

Price (dollars per unit) 30 25 20 15 10 5 0

Answer:

6)

a) As the figure above shows, the profitmaximizing level of output is 150 TV sets per day. At this level of output MR = MC = $100. b) Given the demand for the firm’s product, the figure above shows that the price at which 150 TV sets will be sold is $250. c) The average total cost at the profit maximizing level of output is $200 per television set. So the economic profit per television set is $250 − $200, which is $50 per set. Because 150 sets are produced, the total economic profit is $50 per set × 150 sets = $7,500 per day.

233

Quantity demanded (units) 0 10 20 30 40 50 60

The table above has the market demand schedule in an industry that has two firms in it. The marginal cost of this product is zero because these two firms have exclusive ownership of the resource and it does not cost any additional amount to produce additional units. a) If the firms cooperate with each other so that they operate as a monopoly, what price will they charge and what (total) output will they produce? b) If the firms cannot cooperate but instead behave as perfect competitors, what will be the price and the (total) output they produce? Answer:

a) As a monopoly, the price will be $15 and the total output will be 30 units. This price and output combination is where they maximize their total profit because it is here that the marginal revenue equals zero. (The marginal revenue equals zero because this is the price and output combination for which total revenue is maximized and marginal revenue equals zero when total revenue is maximized.) b) The perfectly competitive price is equal to marginal cost. Because marginal cost is equal to zero, the price will be $0 and the output will be 60 units.

CHAPTER 14

REGULATION AND ANTITRUST LAW

Topic: Natural Monopoly, Marginal Cost and Average Cost Pricing Rule Skill: Analytical

1)

In the figure above, complete the graph of the electric utility company by adding the marginal revenue and marginal cost curves. Assume the marginal cost is constant at 4¢ per kilowatt-hour. Now discuss the two options regulators have in trying to regulate the firm. Be sure to state the price and quantity that are selected for each option. Also, what price and quantity does the firm select if it is not regulated?

Answer:

The completed figure is above. The two methods regulators might use are a marginal cost pricing rule and an average cost pricing rule. If the regulators use a marginal cost pricing rule, the firm must set its price equal to its marginal cost. In the figure, the firm sets a price of 4¢ a kilowatt-hour and produces 400 megawatts per hour. The firm produces the efficient quantity of output, but it incurs an economic loss because ATC > P. If the regulators use an average cost pricing rule, the firm must set its price equal to its average total cost. In the figure, the firm sets a price of 8¢ a kilowatthour and produces 300 megawatts per hour. In this case the firm covers its costs (so that it earns a normal profit) but it produces an inefficient amount of output. Finally, if the firm is allowed to maximize its profit, it produces where its MR = MC. So the firm produces 200 megawatts of power and, setting its price from its demand curve, charges a price of 12¢ per kilowatt-hour.

236

CHAPTER 14

Topic: Natural Monopoly, Marginal Cost Pricing Rule Skill: Analytical

Topic: Natural Monopoly, Capture Theory Skill: Analytical

3) 2)

The above figure represents the cost and demand curves for a natural monopoly that is regulated using a marginal cost pricing rule. a) What is the quantity? b) What price is charged? c) What area represents the consumer surplus when the firm is regulated using a marginal cost pricing rule? d) What distance represents the firm’s loss per unit when the firm is regulated using a marginal cost pricing rule? Answer:

a) The quantity is the efficient quantity, Q3. b) The price is P2. c) When the firm is regulated using a marginal cost pricing rule, the consumer surplus is equal to the area of the triangle P1dP2. d) The loss per unit is the amount equal to the distance cd.

The above figure shows the demand, marginal revenue, and cost curves for a natural monopoly. a) Which price and quantity is set if the capture theory is correct? b) If production is at the price and quantity specified in part (a), what area represents the economic profit? c) If production is at the price and quantity specified in part (a), what area represents the deadweight loss? d) If production is at the price and quantity specified in part (a), what area represents the consumer surplus? Answer:

a) The profit-maximizing price and quantity are P2 and Q1. b) The economic profit equals the area of the rectangle P2abP3. c) The deadweight loss is the area of the triangle acd. d) The consumer surplus is the area of the triangle P1aP2.

REGULATION AND ANTITRUST LAW

237

Topic: Regulating Cartels Skill: Analytical

Topic: Regulating Cartels Skill: Analytical

4)

5)

The above figure shows the market for moving companies, who provide moving services. If the market is perfectly competitive, what is the price per mile and the number of miles per year? If the market is regulated, and the regulators follow the social interest regulation, what is the price per mile and the number of miles per year? If the market is regulated and the regulators are captured by the movers, what is the price per mile and the number of lines per year? Answer:

If the market is perfectly competitive, production will be where the demand and supply curves intersect, 300 million miles of moving per year. The price will be 8¢ per mile. If the market is regulated in the social interest, the outcome is the same as it were perfectly competitive, 300 million miles of moving and a price of 8¢ a mile. If the market is regulated and the producers capture the regulators, the outcome is the same as the monopoly result. Production is at the point where MR = MC, only 200 million miles of moving per year, and the price is 12¢ per mile.

Two trucking companies transport cotton from Sunsboro to Fabricville. The figure above shows the demand curve for trips and the marginal cost curve of each firm. The companies are regulated and cannot price discriminate. a) What is the number of trips per week if the regulation is in the social interest? b) What is the price of a trip if the regulation is in the social interest? c) What is the total surplus if the regulation is in the social interest? d) What is the number of trips per week if the regulation is in the self-interest of producers? e) What is the price of a trip if the regulation is in the self-interest of producers? f) What is the total surplus if the regulation is in the self-interest of producers? Answer:

a) The number of trips per week is 125 for each company. The regulation is in the social interest if total surplus is maximized and deadweight loss is eliminated. This happens at the output level at which marginal benefit equals marginal cost. The demand curve, which is also the marginal benefit curve, intersects the marginal cost curve where the number of trips is 125. b) The price is $20. c) The consumer surplus is the triangular area under the demand curve and above the price, $20. So the consumer surplus is equal to

238

CHAPTER 14

($45 − $20) × 125/2 = $1,562.50. The producer surplus is the triangular area under the price and above the marginal cost curve. So the producer surplus is ($20 − $7.50) × 125/2 = $781.25. The total surplus is the sum of the consumer surplus plus the producer surplus, so the total surplus is $2,343.75 for each company. d) The number of trips per week is 75 for each company. The regulation is in the self-interest of producers if the outcome is the same as that of an unregulated profit-maximizing monopoly. An unregulated monopoly maximizes its profit by producing the quantity at which marginal cost equals marginal revenue. In the graph, MR = MC = $15 if each company produces 75 trips per week. e) The price is $30. f) The consumer surplus is the area under the demand curve and above the price, so the consumer surplus is ($45 − $30) × 75/2, which is $562.50. The producer surplus is the area under the price and above the marginal cost curve. This area can be divided into a triangle shape (the area below $15 and above the marginal cost curve) and a rectangle (the area between $30 and $15 and running from 0 to 75 trips). So the producer surplus is equal to [($15 − $7.50) × 75/2] + [($30 − $15) × 75] = $1,406.25. The total surplus is the sum of the consumer surplus plus producer surplus and so is $1,968.75 for each company.

crease the HHI by 800 points. The Department of Justice’s guidelines are to challenge a merger if the initial HHI exceeds 1,800 and the merger raises the HHI by more than 50 points. The merger considered in the problem easily falls under these guidelines. Topic: Current Merger Rules Skill: Analytical

Firm A B C D E F G H I J 7)

Market share (percent) 15 15 15 10 10 10 10 5 5 5

A market has ten firms, whose market shares are given in the table above. a) If firms I and J wanted to merge, according to the Department of Justice guidelines, would the Department of Justice challenge the merger? b) If firms A and B wanted to merge, according to the Department of Justice guidelines, would the Department of Justice challenge the merger? Answer:

Topic: Current Merger Rules Skill: Analytical

6)

a) The decision whether to challenge the merger depends, in part, on the market’s HerfindahlHirschman Index (HHI). The HHI for the market initially is 1,150. Thus the Department of Justice guidelines say it will challenge a merger if the merger raises the HHI by 100 or more points. If firms I and J merge, the HHI becomes 1,200. The Department of Justice will not challenge this merger. b) If firms A and B merge, the HHI becomes 1,600. The Department of Justice will challenge this merger.

Suppose the industry for washing machines has only four firms. The market shares are: Firm A, 40 percent; Firm B, 20 percent; Firm C 20, percent; and Firm D, 20 percent. a) What is the Herfindahl-Hirschman Index (HHI)? b) If Firms C and D were to announce a merger, would the Department of Justice oppose the merger? Answer:

a) The HHI is 2,800. b) Yes, the Department of Justice would oppose the merger. If the merger occurred, the new HHI would be 3,600. The merger would in-

Topic: Current Merger Rules Skill: Analytical

8)

The Herfindahl-Hirschman Index is used as a guideline to determine if a market is competitive

REGULATION AND ANTITRUST LAW

or concentrated. Calculate the index value for each market described below. a) 100 firms, each of which produces 1 per cent of market output b) 50 firms, each of which produces 2 per cent of market output c) 25 firms, each of which produces 4 per cent of market output d) 20 firms, each of which produces 5 per cent of market output e) 10 firms, each of which produces 10 per cent of market output f) 5 firms, each of which produces 20 per cent of market output

239

g) 2 firms, each of which produces 50 per cent of market output Answer:

a) b) c) d) e) f) g)

100 × 1 = 100 50 × 4 = 200 25 × 16 = 400 20 × 25 = 500 10 × 100 = 1,000 5 × 400 = 2,000 2 × 2,500 = 5,000

CHAPTER 15

EXTERNALITIES

Topic: Marginal Social Cost Skill: Analytical

1)

If the marginal social cost of a good is $70 and the marginal external cost is $20, what does the marginal private cost equal? Answer:

The marginal social cost equals the marginal private cost plus the marginal external cost. Hence, substituting in the numbers from the problem gives $70 = marginal private cost + $20, so that the marginal private cost = $50. Topic: Marginal Social Cost Skill: Analytical

2)

The marginal private cost of a chemical is $100 per ton and its marginal external cost is $20 per ton. What is the marginal social cost of the chemical? Answer:

Marginal social cost equals marginal private cost plus marginal external cost. So the marginal social cost of the chemical is $100 + $20 = $120 per ton. Topic: Marginal External Cost Skill: Analytical

3)

The marginal social cost of a chemical is $100 per ton and its marginal private cost is $85 per ton. What is the marginal external cost of the chemical? Answer:

Marginal external cost equals marginal social cost minus marginal private cost. So the marginal external cost of the chemical is $100 – $85 = $15 per ton.

Topic: Marginal Social Cost Skill: Analytical

Quantity (tons of paper) 100 200 300 400 500 4)

Marginal private cost (dollars) 10 20 30 40 50

Marginal external cost (dollars) 5 10 15 20 25

Marginal social cost (dollars) ____ ____ ____ ____ ____

The table above gives the private costs and external costs of producing paper. a) Complete the table by finding the marginal social cost at each level of production. b) If the market is competitive and is left unregulated and 400 tons of paper are produced, what is the price of a ton of paper? c) If the government imposes a tax equal to the external cost at each level of production, what price would be charged if 400 tons are produced? Answer:

Quantity (tons of paper) 100 200 300 400 500

Marginal private cost (dollars) 10 20 30 40 50

Marginal external cost (dollars) 5 10 15 20 25

Marginal social cost (dollars) 15 30 50 60 75

a) The table above completes the marginal social cost column. At any level of output, the marginal social cost equals the sum of the marginal private cost plus the marginal external cost. b) The price will equal the marginal private cost of $40 a ton. c) If a tax is imposed equal to the marginal external cost, when 400 tons of paper are pro-

242

CHAPTER 15

duced this tax will be $20 a ton. Hence the price will rise by $20, to $60 a ton, which is the same as the marginal social cost.

efficient amount of output? Illustrate the deadweight loss. Answer:

Topic: Government Policies Skill: Analytical

5)

Suppose unregulated production of pesticides results in an equilibrium price and quantity of $400 and 1,000 tons per day, respectively, and a marginal external cost of $10 a ton. a) If the government were to eliminate the external cost by using emission charges, what should the emission charge be set at? b) If the government were to eliminate the external cost by using taxes, what should the tax equal? c) Would the government actions described above affect the quantity of pesticides produced? If yes, how? If no, why not? Answer:

The marginal social cost curve shows the sum of marginal private cost and marginal external cost. Marginal external cost is the cost of producing an extra unit of a good that falls on people other than the firm. The (inefficient) market equilibrium is 8 units with a price of $150 per unit. The efficient quantity is 6 units. The deadweight loss is illustrated in the figure.

a) $10 a ton b) $10 a ton c) Yes, for both the emission charge and the tax, the output of pesticides would be decreased to the efficient level. Topic: Marketable Permits Skill: Analytical

Topic: Government Policies Skill: Analytical

6)

Use the figure above to answer this question. Explain what the marginal social cost curve and the marginal external cost mean. In the figure, if the market is competitive and unregulated, what is the equilibrium price and quantity? What is the

7)

The above figure shows the market for fertilizer. When fertilizer is applied to lawns, it runs off into

EXTERNALITIES

neighboring streams and ponds, killing fish and creating an external cost. a) What is the equilibrium price and quantity of fertilizer in an unregulated, competitive market? b) What is the efficient quantity of fertilizer? c) Suppose government imposes a tax equal to the marginal external cost. What is the equilibrium price paid by consumers and the equilibrium quantity after implementation of the tax? d) At the output level in part (c), how much is the tax? e) How much tax revenue does government collect? f) What is the deadweight loss borne by society if the externality is left uncorrected? Answer:

a) The equilibrium price is $1,000 a ton and the equilibrium quantity is 6 tons per day. b) The efficient quantity is 4 tons per day. c) The price paid by consumers after the tax is $1,200 a ton and the equilibrium quantity is the efficient quantity, 4 tons per day. d) The tax is $400 a ton. e) The government collects $1,600 a day in taxes. f) The deadweight loss with no government action is $600 a day.

243

Topic: Marketable Permits Skill: Analytical

8)

The figure above shows the market for steel, the production of which creates pollution. a) What point represents the equilibrium price and what point represents the equilibrium quantity in an unregulated, competitive market? b) What area represents the deadweight loss of the unregulated, competitive market outcome? c) What point represents the efficient quantity? d) If the output level in part (c) was achieved through the use of a government imposed tax, what price would consumers pay? What price would the producers receive? What distance represents the amount of the tax? e) If government successfully uses marketable permits to eliminate the external cost, what point represents how much output would be produced? Answer:

a) The price is given by point c and the quantity is given by point f. b) The deadweight loss is area gij. c) The efficient quantity is given by point e. d) Consumers would pay a price represented by point b. Producers would receive a price represented by point d. The amount of the tax is the distance bd (which is the same as gh). e) The amount of output would be the efficient quantity, represented by point e.

244

CHAPTER 15

Topic: An Unregulated Market Skill: Analytical

Marginal social cost (dollars per course) 100 80 60 40 20 9)

Quantity of students 4,500 4,000 3,500 3,000 2,500

Marginal private benefit (dollars per course) 20 40 60 80 100

Topic: An Unregulated Market Skill: Analytical

Marginal social benefit (dollars per course) 60 80 100 120 140

The table above gives the marginal social cost (which equals the price), marginal private benefit, and marginal social benefit of students attending Diablo Valley Junior College (DVC) in Concord, California. a) When 4,500 students attend DVC, what does the marginal external benefit equal? b) If the market is competitive and left without government intervention, what is the quantity of students that will attend DVC and what will be the price of a course? c) What is the efficient quantity of students attending DVC? d) If the government can set the price per course, in order to have the efficient quantity of students attending DVC, what should the government set as the price? Answer:

a) The marginal external benefit equals $40, the difference between the marginal social benefit ($60) and the marginal private benefit ($20). b) 3,500 students will attend and the price of a course will be $60. c) The efficient quantity is 4,000 students because at this quantity the marginal social benefit equals the marginal social cost. d) In order to have 4,000 students attend DVC, the government must set the price at $40 per course.

10) The figure above shows the market for polio vaccination in Africa. Polio vaccination confers an external benefit because one person’s vaccination makes it less likely that other people will catch polio. a) If the market is competitive and left unregulated, how many doses of vaccine will be administered? b) If the Melinda and Bill Gates Foundation underwrites the cost of the vaccine by paying for a large fraction of the preparation and delivery cost, what will happen to the number of doses administered? Why? Answer:

a) If the market is competitive and left unregulated, 4 million doses will be administered. b) If the Foundation underwrites the cost of the vaccine, the marginal cost of the vaccine will drop. The result will be similar to a government subsidy: the marginal cost curve will shift rightward, thereby increasing the number of doses administered. The market’s efficiency will be increased.

EXTERNALITIES

245

Topic: An Unregulated Market Skill: Analytical

Topic: Subsidies Skill: Analytical

11) Attractive landscaping increases the property values of surrounding homes, creating a marginal benefit. The figure above represents the market for monthly landscaping contracts. a) What is the marginal social benefit of the 40th contract? Of the 60th contract? b) What is the marginal private benefit of the 40th contract? c) What is the marginal external benefit of the 40th contract? d) What is the unregulated competitive equilibrium price and quantity? e) What is the efficient quantity? f) What is the amount of the deadweight loss?

12) The figure above shows the market for education. Education has an external benefit. a) If the market is competitive and left unregulated, how many students will be enrolled per year? b) What is the efficient number of students? c) In the figure, show the effect of a government subsidy that moves the market to the efficient number of students. What is the amount of the subsidy and what tuition must the students pay?

Answer:

a) The marginal social benefit of the 40th contract is $100. The marginal social benefit of the 60th contract is $80. b) The marginal private benefit of the 40th contract is $60. c) The marginal external benefit of the 40th contract is $40. d) The equilibrium price is $60 per contract and the equilibrium quantity is 40 contracts a month. e) The efficient quantity is 60 contracts a month. f) The deadweight loss equals $400 per month.

Answer:

a) If the market is competitive and left unregulated, 4 million students will be enrolled. b) The efficient number of students enrolled is 6 million students.

246

CHAPTER 15

c) The above figure shows the effect of the government subsidy and how the supply curve shifts. The subsidy is $6,000 and students pay tuition of $10,000.

CHAPTER 16

PUBLIC GOODS AND COMMON RESOURCES

Topic: Marginal Benefit Skill: Analytical

Quantity (cars on duty) 1 2 3 4 5 1)

Topic: Efficient Quantity of a Public Good Skill: Analytical

Paul’s marginal benefit (dollars per car) 50 40 30 20 10

Art’s marginal benefit (dollars per car) 90 75 60 45 30

The table above shows the marginal benefit that arises from providing police protection in an economy of two people, Paul and Art. a) Construct a table of the economy’s marginal benefit from providing police protection. b) What is the economy’s marginal benefit from having 3 police cars on duty? Answer:

Quantity (cars on duty) 1 2 3 4 5

Economy’s marginal benefit (dollars per car) 140 115 90 65 40

a) The table showing the economy’s marginal benefit from providing police protection is above. At each quantity, the economy’s marginal benefit is the sum of Paul’s marginal benefit plus Art’s marginal benefit. b) The economy’s marginal benefit from having three cars on duty is $90.

Number of police cars on duty 1 2 3 4 5 2)

Jake’s marginal benefit (dollars) 45 35 25 15 5

Elwood’s marginal benefit (dollars) 20 16 12 8 4

The table above shows the marginal benefit from providing police protection in a community of two people, Jake and Elwood. a) What is the marginal benefit to community from the 4th police car on duty? b) If the marginal cost of a police car on duty is $37, what is the efficient number of cars on duty? c) If the marginal cost of a police car on duty is $70, what is the efficient number of cars on duty? Answer:

a) The community’s marginal benefit is the sum of Jake’s marginal benefit and Elwood’s marginal benefit. So the marginal benefit to community from the 4th police car is $15 + $8 = $23. b) The number of cars on duty is efficient if the community’s marginal benefit equals the marginal cost. The community’s marginal benefit is the sum of Jake’s marginal benefit and Elwood’s marginal benefit. When three cars are on duty, the marginal benefit to the community is $25 + $12 = $37, which is the same as the marginal cost. So the efficient number of cars is 3. c) As before, the number of cars on duty is efficient if the community’s marginal benefit equals the marginal cost. And also as before,

248

CHAPTER 16

the community’s marginal benefit is the sum of Jake’s marginal benefit and Elwood’s marginal benefit. But in this case, because for any number of police cars on duty marginal cost exceeds marginal benefit, the efficient number

Topic: Efficient Quantity of a Public Good Skill: Analytical

Quantity (streetlights per block) 0 1 2 3 4 5 6

Topic: Efficient Quantity of a Public Good Skill: Analytical

Level of contaminates (parts per million) 20 15 10 5 0 3)

Marginal benefit (dollars per year) 2 7 12 16 20

Marginal cost (dollars per year) 0 4 7 16 24

Ten homeowners live around Lake Alice. Unfortunately, the lake is contaminated with various chemicals and the level of contamination is currently 20 parts per million. All the homeowners benefit equally if the level of contamination is reduced. The table above gives the marginal cost and marginal benefit for removing the contaminants from the lake. a) What will be the parts per million if the homeowners alone are in charge of reducing the contaminants? b) What is the efficient amount of contamination? Answer:

a) Homeowners will not reduce the contaminants at all because a free-rider problem prevails. The level of contamination will be 20 parts per million. b) The efficient level of contamination is 10 parts per million, because that is the level for which the marginal cost is equal to the marginal benefit.

4)

Marginal benefit (dollars per year) 0 14 12 10 8 6 4

Marginal cost (dollars per year) 0 4 7 10 13 16 19

The table above provides information about the marginal cost and marginal benefit of streetlights, which are a public good. a) What quantity would a private company provide? Why? b) What is the efficient quantity? Answer:

a) Street lights are a public good, so a private company would provide zero street lights. The company would provide zero because everyone would free ride, that is, no one would voluntarily pay for the street lighting. Because the company knows that no one will pay, it will not provide any street lights. b) The efficient quantity is 3 street lights per block because that is the quantity that equates the marginal benefit to the marginal cost.

CHAPTER 17

DEMAND AND SUPPLY IN FACTOR MARKETS perfectly competitive and car washes sell for $5 each. The labor market is competitive and the wage rate is $50 per day. What is the marginal revenue product for each worker? How many workers does the firm hire to maximize profit?

Topic: Marginal Revenue Product Skill: Analytical

Quantity of labor (workers) 1 2 3 4 1)

Marginal product (units per hour) 10 8 6 4

Marginal revenue product (dollars) ____ ____ ____ ____

The above table has the marginal product schedule for a firm. If the firm is a perfect competitor and the price of the product is constant at $2 a unit, complete the table. If the wage rate is $8 an hour, how many workers does the firm hire?

Answer:

Quantity of labor (workers) 1 2 3 4 5

Answer:

Quantity of labor (workers) 1 2 3 4

Marginal product (units per hour) 10 8 6 4

Marginal revenue product (dollars) 20 16 12 8

If the wage rate is $8 an hour, the firm hires 4 workers because that is the quantity that sets the marginal revenue product equal to the wage rate. Topic: Marginal Revenue Product Skill: Analytical

Quantity of labor (workers) 1 2 3 4 5 2)

Cars washed (cars per day) 8 23 33 40 45

The table above shows the total product schedule for Shines Car Wash. The market for car washes is

Cars washed (cars per day) 8 23 33 40 45

Marginal revenue product (dollars per day) 75 50 35 25

The marginal revenue product is the marginal product multiplied by the price of a car wash. The marginal revenue product schedule is given in the table above. To maximize its profit, when the wage rate is $50 per day, Shines Car Wash hires 2 workers because at this point the marginal revenue product equals the wage rate. Topic: Marginal Revenue Product Skill: Analytical

Quantity of labor (workers) 1 2 3 4 5 3)

Marginal product (blouses cleaned per day) 4 4 3 2 1

Marginal revenue product (dollars per day) ____ ____ ____ ____ ____

The above table has the marginal product schedule for Nick’s Dry Cleaners, a perfectly competitive dry cleaning firm. a) If the price to dry clean a blouse is $8 each, complete the last column of the table.

250

CHAPTER 17

b) If Nick can hire workers at the going wage rate of $16 an hour, how many workers does Nick hire? Answer:

Quantity of labor (workers) 1 2 3 4 5

Marginal product (blouses cleaned per day) 4 4 3 2 1

Marginal revenue product (dollars per day) 32 32 24 16 8

Answer:

Quantity of labor (workers) 1 2 3 4 5

Topic: Marginal Revenue Product Skill: Analytical

Quantity of labor (workers) 1 2 3 4 5 4)

Marginal revenue product, $2 per pound (dollars) ____ ____ ____ ____ ____

Marginal revenue product, $1 per pound (dollars) ____ ____ ____ ____ ____

Tom and Mary own a perfectly competitive tomato farm. They can hire different numbers of college students to help plant, cultivate, and harvest the tomatoes. The above table gives their marginal product schedule. a) If the price of a pound of tomatoes is $2 a pound, complete the first marginal revenue product column in the table. If Tom and Mary must pay their workers $10 an hour, how many workers do they hire? b) If the price of a pound of tomatoes falls to $1 a pound, complete the second marginal revenue product column in the table. If Tom and Mary still must pay their workers $10 an hour, how many workers do they hire? c) When the price of a pound of tomatoes falls, what happens to Tom and Mary’s demand for labor curve?

Marginal revenue product, $2 per pound (dollars) 16 20 14 10 8

Marginal revenue product, $1 per pound (dollars) 8 10 7 5 4

a) The completed table is above. Tom and Mary hire 4 workers because that is the number of workers that sets the marginal revenue product equal to the wage rate. b) The completed table is above. Tom and Mary hire 2 workers because that is the number of workers that sets the marginal revenue product equal to the wage rate. c) When the price of a pound of tomatoes falls, Tom and Mary’s demand for labor decreases and their demand for labor curve shifts leftward.

a) The completed table is above. b) Nick hires 4 workers because that is the quantity of workers that sets the marginal revenue product equal to the wage rate.

Marginal product (pounds per hour) 8 10 7 5 4

Marginal product (pounds per hour) 8 10 7 5 4

Topic: Profit Maximization Skill: Analytical

5)

A worker has a marginal product of 15 units a day, each of which can be sold for $10. Is it profitable to hire this worker if the wage rate is $100 a day? Briefly explain your answer. Answer:

It is profitable to hire this worker because the worker’s marginal revenue product is $150 a day (the price, $10, times the marginal product, 15 units), which exceeds the wage rate of $100 a day. The worker creates $150 more revenue for the firm at a cost of only $100. Hence it is profitable to hire this worker.

DEMAND AND SUPPLY IN FACTOR MARKETS

251

Topic: Labor Market Equilibrium Skill: Analytical

Topic: Labor Market Equilibrium Skill: Analytical

6)

7)

The above figure represents the market for professional minor-league baseball umpires. a) If umpires are offered $90 a game, what is the quantity of umpires supplied? b) If umpires are offered $90 a game, is there a surplus or shortage of games umpired? What does the shortage or surplus equal? c) What is the equilibrium wage rate and quantity of umpires? Answer:

a) The quantity of umpires supplied is 20 umpires. b) At $90 a game, there is a shortage of umpires. The shortage equals the quantity demanded, 50 umpires, minus the quantity supplied, 20 umpires, for a shortage of 30 umpires. c) The equilibrium wage rate is $120 a game and the equilibrium quantity is 40 umpires.

The above figure represents the market for teenage workers at fast-food restaurants in Kansas City. a) What is the equilibrium wage rate and employment? b) Describe the market at a wage rate of $6 per hour. c) Describe the market at a wage rate of $12 an hour. d) How would an increase in the number of young, married college graduates, who tend to eat at fast-food restaurants, affect the figure, the equilibrium wage rate, and employment? Answer:

a) The equilibrium wage rate is $8 an hour and the equilibrium quantity of employment is 4,000 workers. b) If the wage rate is $6 an hour, the quantity of labor supplied is less than the quantity of labor demanded. There is a shortage in the market. c) If the wage rate is $12 an hour, the quantity of labor supplied is greater than the quantity of labor demanded. There is a surplus in the market. d) An increase in the number of people who eat in fast-food restaurants will increase the demand for workers at fast-food restaurants. As a result the demand for labor curve will shift rightward and the equilibrium wage rate and

252

CHAPTER 17

equilibrium quantity of employment will both increase. Topic: Net Present Value Skill: Analytical

8)

Simon’s Sites is a firm that designs Web sites. Simon is considering buying a new computer that costs $2,500. He expects that the computer will generate an additional $2,000 at the end of the first year and $1,000 at the end of the second year, after which it will be worthless. If the interest rate in the next two years is 6 percent per year, what is the net present value of the computer? Answer:

The present value of the computer is $2000 $1,000 + = $2,777. And its net pre1 + 0.06 (1 + 0.06 ) 2 sent value is $2,777 − $2,500 = $277. Topic: Net Present Value Skill: Analytical

9)

Simon’s Sites is a firm that designs Web sites. Simon is considering buying a new computer. He expects that the computer will generate an additional $2,000 at the end of the first year and $1,000 at the end of the second year, after which it will be worthless. If the interest rate in the next two years is 6 percent per year, what is the highest price at which Simon will buy the computer? Answer:

Simon will buy the computer if it’s net present value is positive. The present value of the com$2000 $1,000 + = $2,777. And the puter is 1 + 0.06 (1 + 0.06 ) 2 net present value of capital is the difference between its present value and its price. So Simon will buy the computer if its price is lower than $2,777. Topic: Demand for Capital Skill: Conceptual

10) Kevin plans to open a used boat engine store. To do so, he needs $100,000 to buy engines and out-

fit the store. Suppose Kevin has $20,000 in his saving account, which pays an interest rate of 5 percent per year. Further suppose that the current interest rate for loans also is 5 percent per year. What is Kevin’s opportunity cost for the financial capital necessary to open the store? If the demand for financial capital decreases just when Kevin plans to enter the financial capital market, what happens to his opportunity cost? Answer:

Kevin’s opportunity cost is the interest he forgoes on his savings, 5 percent multiplied by $20,000, plus the interest he pays on his loan, 5 percent of $80,000. So the opportunity cost of the financial capital necessary to open the store is $5,000. If the demand for capital decreases, the interest rate falls. In this case, Kevin’s opportunity cost decreases. Topic: Economic Rent Skill: Analytical

11) Phil is an instructor who is paid $50,000 per academic year to teach four classes of principles of microeconomics. One day, Phil tells his fellow instructors that he likes teaching so much, that he would be willing to teach as long as the college paid him at least $23,500. How much, if any, economic rent does Phil earns? Answer:

Economic rent is income that is over and above the opportunity cost of supplying a factor. The opportunity cost of obtaining Phil to teach is $23,500 because at any lower income, he will do something other than teach. Therefore Phil receives $50,000 − $23,500 = $26,500 of economic rent.

CHAPTER 18

ECONOMIC INEQUALITY

Topic: Lorenz Curve Skill: Analytical

Household percentage Lowest 20 percent Second 20 percent Third 20 percent Fourth 20 percent Highest 20 percent 1)

Topic: Lorenz Curve Skill: Analytical

Percent of income Alpha 5 10 20 30 35

Percent of income Beta 3 6 12 24 55

Household percentage Lowest 20 percent Second 20 percent Third 20 percent Fourth 20 percent Highest 20 percent

Percent of income Alpha 5 10 20 30 35

Percent of income Beta 3 6 12 24 55

The above table shows the distribution of income in two imaginary countries, Alpha and Beta. a) What does the table tell you about the second 20 percent group in each country? b) Calculate the cumulative percentage for both countries. c) Interpret the cumulative percentage for the third 20 percent group in both countries. Answer:

a) The second 20 percent group earns 10 percent of the income in country Alpha and 6 percent of the income in country Beta.

Household percentage Lowest 20 percent Second 20 percent Third 20 percent Fourth 20 percent Highest 20 percent

Cumulative percentage of income Alpha 5 15 35 65 100

Cumulative percentage of income Beta 3 9 21 45 100

b) The table above has the cumulative percentages. c) In country Alpha, the poorest 60 percent of households earns 35 percent of income. In country Beta, the poorest 60 percent of households earns 21 percent of income.

2)

The above table shows the distribution of income in two imaginary countries, Alpha and Beta. a) In the figure above, draw the Lorenz curves for Alpha and Beta. b) Compare the distribution of income in Alpha with that in Beta. Which distribution is more unequal? c) What would be the distribution of income in a country with perfect income equality?

254

CHAPTER 18

Answer:

3) a) The Lorenz curves are above. b) The distribution of income in Beta is more unequal than the distribution of income in Alpha because the Beta Lorenz curve lies further from the line of equality. c) Each 20 percent group would earn 20 percent of the income, so the Lorenz curve would lie on the line of equality. Topic: Lorenz Curve Skill: Analytical

Household percentage Lowest 20 percent Second 20 percent Third 20 percent Fourth 20 percent Highest 20 percent

Household percentage Lowest 20 percent Second 20 percent Third 20 percent Fourth 20 percent Highest 20 percent

Average income (dollars) 10,000 26,000 38,000 55,000 140,000

Percentage of income ___ ___ ___ ___ ___

The first table above gives the average income received by each 20 percent group of households. Complete the second table. Label the axes above and then plot the Lorenz curve. Answer:

Household percentage Lowest 20 percent Second 20 percent Third 20 percent Fourth 20 percent Highest 20 percent

Percentage of income 3.72 9.67 14.13 20.45 52.04

Cumulative percentage of income 3.72 13.39 27.52 47.97 100.00

Cumulative percentage of income ____ ___ ___ ___ ___

The completed table and the resulting Lorenz curve are above.

ECONOMIC INEQUALITY

255

Topic: Lorenz Curve Skill: Analytical

Household percentage Lowest 20 percent Second 20 percent Third 20 percent Fourth 20 percent Highest 20 percent

Answer:

Cumulative percentage of income, Country A 10 25 45 70 100

Cumulative percentage of income, Country B 4 9 18 40 100

The Lorenz curves are above. Income is distributed more equally in Country A. Topic: Gini Ratio Skill: Analytical

5)

On a Lorenz curve graph, the area between the line of equality and the Lorenz curve is 2,000, and the entire area beneath the line of equality is 5,000. What does the Gini ratio equal? Answer:

4)

The table above gives cumulative percent of income received by each 20 percent group of households. Label the axes in the figure above and then plot the Lorenz curves. In which nation is income more equally distributed?

The Gini ratio is the ratio of the area between the line of equality and the Lorenz curve to the entire area beneath the line of equality. So the Gini ratio is 2,000/5,0000 = 0.4. Topic: Gini Ratio Skill: Analytical

6)

On the Lorenz curve graph, the area between the line of equality and the Lorenz curve is 2,500, and the entire area beneath the line of equality is 5,000. What does the Gini ratio equal? Answer:

The Gini ratio is the ratio of the area between the line of equality and the Lorenz curve to the entire area beneath the line of equality. So the Gini ratio is 2,500/5,0000 = 0.5. Topic: Comparing Like with Like Skill: Analytical

7)

If Bianca earns 5 percent on her wealth and that income amounts to $40,000, what is Bianca’s wealth?

256

CHAPTER 18

Answer:

Bianca’s wealth is $800,000. Topic: Measured Wealth Skill: Analytical

Human capital Nonhuman capital Total 8)

Sam (dollars) 100,000 200,000 300,000

Janet (dollars) 400,000 10,000 410,000

The table shows the distribution of human and non-human capital for two people, Sam and Janet. a) Looking just at tangible assets (non-human capital), by how many times does Janet’s wealth exceed Sam’s? b) Assume that both human and non-human capital earn a 10 percent annual interest rate. Calculate Sam’s and Janet’s total income.

c) By how many times does Janet’s total income exceed Sam’s? d) Which comparison results in a more equal distribution? e) Which comparison gives a better indication of each person’s economic condition? Answer:

a) Janet has 20 times as much wealth as Sam. b) Sam’s income is $30,000 and Janet’s is $41,000. c) Janet’s income is 1.37 times Sam’s. d) The comparison of income results in a more equal distribution. e) The comparison of income gives a better indication of each person’s economic condition because it includes both human and nonhuman capital, while the comparison of wealth only includes non-human capital.

CHAPTER 19

UNCERTAINTY AND INFORMATION

Topic: Expected Wealth Skill: Analytical

1)

Mike owns a car worth $20,000, and that is his only wealth. There is a 10 percent chance that Mike will have an accident within a year. If he does have an accident, his car is worthless. What is Mike’s expected wealth? Answer:

The probability of an accident is 0.1, and the probability of not having an accident is 0.9. So Mike’s expected wealth is $20,000 × 0.9 + $0 × 0.1 = $18,000. Topic: Gains From Insurance Skill: Analytical

2)

Mike has the utility of wealth curve shown in the figure above. He owns a car worth $20,000, and that is his only wealth. There is a 10 percent chance that Mike will have an accident within a year. If he does have an accident, his car is worthless. a) What is Mike’s expected utility? b) What is the maximum amount that Mike is willing to pay for auto insurance?

c) Suppose all car owners are like Mike insofar as they have a 10 percent chance of having an accident. An insurance company agrees to pay each person who has an accident the full value of his or her car. The company’s operating expenses are $1,000. What is the minimum insurance premium that the company is willing to accept? d) Will Mike buy the company’s policy? Why or why not? Answer:

a) If Mike has an accident, his utility is zero. If he does not have an accident, his utility is 100. The probability of an accident is 0.1, and the probability of not having an accident is 0.9. So Mike’s expected utility is 100 × 0.9 + 0 × 0.1 = 90.

b) The probability of an accident is 0.1, and the probability of not having an accident is 0.9. So Mike’s expected wealth is $20,000 × 0.9 + $0 × 0.1 = $18,000. If Mike has an accident, his utility is zero. If he does not have an accident, his utility is 100. So Mike’s expected

258

CHAPTER 19

utility is 100 × 0.9 + 0 × 0.1 = 90. Given his utility of wealth curve, the figure above shows that Mike gets the same utility if his wealth is $14,000 with certainty. That is, Mike’s utility of a guaranteed wealth of $14,000 is the same as his utility of an expected wealth of $18,000 with the degree of risk he faces. This result means that Mike is willing to pay up to $18,000 − $14,000 = $4,000 for auto insurance. c) The probability of an accident is 0.1. So the company will pay out $20,000 to 1/10 of car owners, or an average of $2,000 per person. The company covers all its costs if it offers in-

surance for $2,000 + $1,000 = $3,000. So $3,000 is the minimum insurance premium that the company is willing to accept. d) Mike is willing to pay up to $4,000 for auto insurance. The minimum insurance premium that the company is willing to accept is $3,000. So Mike will buy the policy because the maximum amount he is willing to pay is greater than the minimum amount that the insurance company is willing to accept.

CHAPTER 20

TRADING WITH THE WORLD**

Topic: Balance of Trade Skill: Analytical

Country France Korea Mexico Peru 1)

Exports (billions of dollars) 100 50 110 30

Topic: Gains From Trade Skill: Conceptual

Imports (billions of dollars) 80 44 115 35

A country trades with only France, Korea, Mexico, and Peru. The table above has information about the nation’s trade. What is the balance of trade with each country and the overall balance of trade? Answer:

The country has a trade surplus with France of $20 billion, a trade surplus with Korea of $6 billion, a trade deficit with Mexico of $5 billion, and a trade deficit with Peru of $5 billion. Its overall trade balance is a trade surplus of $16 billion. Topic: Comparative Advantage Skill: Analytical

2)

Japan can use all of its resources to produce 100 videos or 400 shoes. China can use all of its resources to produce 25 videos or 200 shoes. Which nation has the comparative advantage in shoes and which nation has the comparative advantage in videos? Answer:

In Japan, the opportunity cost of producing a video is 4 shoes and in China it is 8 shoes. Therefore Japan has the comparative advantage in producing videos because its opportunity cost is lower. In Japan, the opportunity cost of producing a shoe is 1/4 of a video and in China the opportunity cost of producing a shoe is 1/8 of a video. China has the comparative advantage in producing shoes because its opportunity cost is lower.

*

* This is Chapter 33 in Economics.

3)

The United States can use all of its resources to produce 50 computers or 4,000 shoes. Suppose that at world market prices, one computer exchanges for 100 shoes. Explain how the United States can gain from trade. Answer:

In the United States, the opportunity cost to produce 1 computer is 80 pairs of shoes. The United States can then sell the computers on the world market for 100 shoes each and thereby be ahead by 20 shoes per computer. Topic: Effects of a Tariff Skill: Analytical

Price (dollars per pound) 21 18 15 12 9 4)

Quantity supplied (thousands of pounds per year) 120 100 80 60 40

Quantity demanded (thousands of pounds per year) 60 100 140 180 220

The United States imports cheese from a variety of countries. The table above gives the domestic supply of, and demand for, cheese in the United States. The world price of cheese is $12 per pound, and trade is unrestricted. a) How many pounds of cheese are consumed in the United States? b) How many pounds of cheese are produced in the United States? c) How many pounds of cheese are imported into the United States? If a $3 per pound tariff is imposed, d) How many pounds of cheese are consumed in the United States?

260

CHAPTER 20

d) With the $5 per unit tariff, what is the quantity imported? e) How much revenue does the government collect with a tariff of $5 per unit?

e) How many pounds of cheese are produced in the United States? f) How many pounds of cheese are imported into the United States? g) How much will the U.S. government collect in tariff revenue? h) Who benefits from the tariff? Who loses?

Answer:

a) Domestic consumption is 8 million units per year and domestic production is 0. b) The quantity imported is 8 million units per year. c) Domestic consumption is 6 million units per year and domestic production is 2 million units per year. d) The quantity imported is 4 million units per year. e) The government collects $5 per unit imported and 4 million units are imported, so the government’s revenue from the tariff is $5 × 4 million = $20 million per year.

Answer:

a) b) c) d) e) f) g)

180,000 pounds of cheese are consumed. 60,000 pounds of cheese are produced. 120,000 pounds of cheese are imported. 140,000 pounds of cheese are consumed. 80,000 pounds of cheese are produced. 60,000 pounds of cheese are imported. The government collects $3 per pound × 60,000 pounds = $180,000. h) U.S. producers and the U.S. government gain while U.S. consumers and foreign producers lose. Topic: Effects of a Tariff Skill: Analytical

5)

The above figure shows the domestic supply of and domestic demand for an imported good. The world price is $15 per unit. a) At the world price of $15 per unit, what is the domestic consumption and domestic production? b) At the world price of $15 per unit, what is the quantity imported? c) If the government imposes a tariff of $5 per unit, what is the domestic consumption and domestic production?

Topic: Rent Seeking Skill: Analytical

6)

Suppose that elimination of tariffs on agricultural products means that 1,000 farm workers lose jobs that pay an average of $20,000 per year. At the same time, because of the importation of relatively cheaper foreign vegetables, 150 million consumers save $2 per year on their grocery bills. a) What is the total income lost by farm workers because of the free trade? b) What is the total dollar amount saved by all consumers combined? c) Which is greater, the lost income or the consumer savings? Do the benefits of free trade outweigh the costs in this simple example? d) Which group is most likely to become politically involved over the issue of removing the tariffs, the farm workers or the consumers? Why? Answer:

a) The total income lost by farm workers is 1,000 × $20,000 = $20,000,000. b) The total saving by all consumers is 150,000,000 × $2 = $300,000,000. c) The consumer savings is much larger than the lost income, so the benefits of free trade outweigh the costs. d) The farm workers are more likely to become involved, because their individual loss is much greater than an individual consumer’s gain.

TRUE OR FALSE QUESTIONS

CHAPTER 1

WHAT IS ECONOMICS?

Topic: Production Trends Skill: Conceptual

True Or False Questions

9)

Topic: Scarcity Skill: Conceptual

1)

Answer: TRUE

Scarcity applies to both the rich and the poor.

Answer: TRUE

Topic: How Question Skill: Conceptual

Topic: Scarcity Skill: Conceptual

2)

10) A country using mainly labor to build a dam instead of using mainly machines is answering the “how” question.

Rich people don’t have to deal with scarcity.

Answer: FALSE

Answer: TRUE

Topic: Scarcity Skill: Conceptual

3)

The United States produces more services than goods.

Topic: What Question Skill: Conceptual

Scarcity affects only those who are in need.

11) When a business increases its workers’ wages, it is answering the “what” question.

Answer: FALSE Topic: Scarcity Skill: Conceptual

Answer: FALSE

4)

Topic: The New Economy Skill: Conceptual*

Scarcity arises because of opportunity costs.

Answer: FALSE

12) The new products and processes introduced during the Information Revolution of the last twenty years were a result of a successful implementation of a strategic economic plan designed by the U.S. government.

Topic: Microeconomics Skill: Conceptual

5)

Microeconomics is the study of topics such as national production and unemployment.

Answer: FALSE

Answer: FALSE

Topic: Macroeconomics Skill: Conceptual

Topic: Opportunity Cost Skill: Conceptual

6)

13) When I buy an $8.00 movie ticket rather than two paperback books, the opportunity cost of going to the movie is the two paperback books I did not buy.

Macroeconomics is the study of aggregate variables such as national production and unemployment.

Answer: TRUE

Answer: TRUE

Topic: Human Capital Skill: Recognition*

7)

Topic: Economic Models Skill: Conceptual

The tools, instruments, machines, and buildings that people use to produce goods and services are called human capital.

14) Economic models make some assumptions in order to simplify the real world.

Answer: FALSE

Answer: TRUE

Topic: Labor Skill: Conceptual*

8)

Most income in the United States is earned by business owners as profit.

Answer: FALSE 263

264

CHAPTER 1

Topic: Post Hoc Fallacy Skill: Conceptual

Topic: Post Hoc Fallacy Skill: Conceptual

15) Keeping constant all but the two economic variables you are interested in is an example of the post hoc fallacy.

17) If the stock market booms and some months later the economy expands, you know that the stock market caused the economy to expand.

Answer: FALSE

Answer: FALSE

Topic: Post Hoc Fallacy Skill: Conceptual

Topic: Agreement and Disagreement Skill: Recognition

16) “Every time I buy stock, the stock market falls. So my buying decisions must be a good forecast of the market's behavior.” This statement is an example of the fallacy of composition.

18) Most economists agree that tariffs and import restrictions make most people worse off.

Answer: FALSE

Answer: TRUE

APPENDIX 1 GRAPHS IN ECONOMICS Topic: Graphing Data Skill: Recognition

Topic: Graphs Used in Economic Models Skill: Conceptual

1)

7)

The vertical axis of a graph shows only positive values.

A graph cannot be used to show that two variables are unrelated.

Answer: FALSE

Answer: FALSE

Topic: Graphing Data Skill: Recognition

Topic: Graphs Used in Economic Models Skill: Recognition

2)

8)

A scatter diagram plots the value of one economic variable against time.

When graphed, variables that are unrelated are shown by either a horizontal or a vertical line.

Answer: FALSE

Answer: TRUE

Topic: Graphing Data Skill: Conceptual

Topic: The Slope of a Relationship Skill: Recognition

3)

9)

A time series graph can show both the level of a variable and the speed with which the variable changes over time.

The slope of a line is the change in the y-axis variable divided by the change in the x-axis variable.

Answer: TRUE

Answer: TRUE Topic: Graphing Data Skill: Recognition

4)

A trend is a general tendency for a variable to increase or decrease over time.

Topic: The Slope of a Relationship Skill: Conceptual

10) The slope of a straight line increases as values get higher. Answer: FALSE

Answer: TRUE Topic: Graphing Data Skill: Recognition

5)

A cross-section graph can show how economic variables for different groups of people vary over time.

Topic: The Slope of a Relationship Skill: Conceptual

11) To calculate the slope of a curved line, you can calculate the slope at a point on the curve or across an arc of the curve. Answer: TRUE

Answer: FALSE Topic: Graphs Used in Economic Models Skill: Conceptual

6)

If the x-axis variable increases while the y-axis variable decreases, the variables x and y are negatively related.

Answer: TRUE

Topic: The Slope of a Relationship Skill: Analytical

12) If the change in the y-axis variable is 4 and the change in the x-axis variable is 2, the slope of this line is 1/2. Answer: FALSE

266

Topic: The Slope of a Relationship Skill: Analytical

13) If the change in the y-axis variable is 6 and the change in the x-axis variable is 5, the slope of this line is 6/5. Answer: TRUE

APPENDIX 1

Topic: Graphing Relationships Among More Than Two Variables Skill: Conceptual

15) “Ceteris paribus” refers to the idea that if more than two variables are graphed, only one variable must be held constant. Answer: FALSE

Topic: Graphing Relationships Among More Than Two Variables Skill: Conceptual

14) To graph a relationship that involves more than two variables, we use the “ceteris paribus” assumption. Answer: TRUE

CHAPTER 2

THE ECONOMIC PROBLEM

Topic: Production Possibilities Frontier Skill: Recognition*

Topic: Using Resources Efficiently Skill: Conceptual*

1)

7)

Points outside the production possibilities frontier illustrate production points that cannot be attained.

We are using resources efficiently if we can produce more of one good without producing less of some other good that we value more highly.

Answer: TRUE

Answer: FALSE

Topic: Production Possibilities and Opportunity Costs Skill: Recognition

Topic: Using Resources Efficiently Skill: Conceptual

2)

8)

An economy is producing efficiently if it is producing on its PPF.

The more we have of a good or service, the smaller is its marginal benefit and the less we are willing to pay for an additional unit of it.

Answer: TRUE

Answer: TRUE

Topic: Production Possibilities and Opportunity Costs Skill: Recognition

Topic: Using Resources Efficiently Skill: Conceptual

3)

If a country operates on its PPF, it achieves production efficiency.

9)

As long as the marginal benefit from a good is greater than its marginal cost, an economy is operating efficiently.

Answer: TRUE

Answer: FALSE

Topic: Production Possibilities and Opportunity Costs Skill: Conceptual

Topic: Using Resources Efficiently Skill: Conceptual

4)

As you move downward along a PPF, the opportunity cost of another unit of the good measured along the horizontal axis decreases.

10) If marginal benefit is greater than marginal cost for the production of cars, an economy should increase the production of cars. Answer: TRUE

Answer: FALSE Topic: Production Efficiency Skill: Conceptual*

5)

We have achieved production efficiency if we can produce more of one good without producing less of some other good.

Topic: Using Resources Efficiently Skill: Conceptual

11) When a nation is producing the allocatively efficient quantity of a product, the marginal benefit of producing the good equals the marginal cost of producing that good.

Answer: FALSE

Answer: TRUE

Topic: Allocative Efficiency Skill: Conceptual*

Topic: Economic Growth Skill: Conceptual

6)

Each point on the production possibilities frontier achieves allocative efficiency.

Answer: FALSE

12) As long as technology increases occur, economic growth is considered “free”. Answer: FALSE

268

CHAPTER 2

Topic: Economic Growth Skill: Conceptual

Topic: Gains from Trade Skill: Conceptual

13) To expand its PPF, a country may devote resources to accumulating capital.

18) If two countries specialize in the production of goods in which they have a comparative advantage, they can experience gains from trade.

Answer: TRUE Topic: Economic Growth Skill: Recognition*

14) Over the past several decades, the United States has devoted a greater fraction of its resources to consumption than Hong Kong, which is why the U.S. economy grew faster than Hong Kong’s economy.

Answer: TRUE Topic: Gains from Trade Skill: Recognition

19) Learning-by-doing is a process that can help a person gain a comparative advantage. Answer: TRUE

Answer: FALSE

Topic: The Market Economy Skill: Conceptual

Topic: Comparative Advantage Skill: Recognition

20) Property rights facilitate the development of trade.

15) A country has a comparative advantage in the production of a good if its opportunity cost is lower compared to another country.

Answer: TRUE

Answer: TRUE Topic: Comparative Advantage Skill: Recognition*

16) A person has a comparative advantage in producing a good if that person can produce more of it with a given amount of resources than can another person. Answer: FALSE Topic: Gains from Trade Skill: Recognition

17) Specialization and trade allow countries to consume beyond their PPFs. Answer: TRUE

Topic: The Market Economy Skill: Recognition

21) A circular flow diagram shows the flows from the goods and resources markets. Answer: TRUE Topic: The Market Economy Skill: Recognition

22) Households are buyers in factor markets and sellers in goods markets. Answer: FALSE Topic: The Market Economy Skill: Recognition*

23) In the United States, the government coordinates most of the economic activity. Answer: FALSE

CHAPTER 3

DEMAND AND SUPPLY

Topic: Markets and Prices Skill: Recognition

Topic: Demand Curve Skill: Conceptual

1)

7)

A relative price is the product of two money prices.

Answer: FALSE Topic: Markets and Prices Skill: Recognition

2)

The relative price of a good is greater than the money price of a good.

Answer: FALSE Topic: Demand Curve Skill: Recognition*

3)

Answer: FALSE Topic: Demand Curve Skill: Conceptual

8)

A demand curve is also a willingness-and-abilityto-pay curve.

Answer: TRUE Topic: Demand Curve Skill: Recognition*

4)

A movement along the demand curve shows a change in demand.

Answer: FALSE Topic: Demand Curve Skill: Conceptual

5)

An increase in the incomes of baseball fans in New York leads to a rightward movement along the demand curve but does not shift the demand curve for Yankee’s tickets.

Answer: FALSE Topic: Demand Curve Skill: Conceptual

6)

For consumers, chocolate chip cookies and doughnuts are substitutes. So, an increase in the price of chocolate chip cookies will lead to a rightward shift in the demand for doughnuts.

Answer: TRUE

Young drivers often buy used cars. An increase in the legal driving age to twenty-one would result in a leftward movement along the demand curve for used cars, whereas lowering the age to fifteen would result in a rightward movement along the demand curve.

Young drivers often buy used cars. An increase in the legal driving age to twenty-one shifts the demand curve for used cars leftward, whereas lowering the age to fifteen shifts the demand curve rightward.

Answer: TRUE Topic: Supply Skill: Conceptual

9)

The supply curve indicates the minimum quantity that a producer would be willing to supply at alternative prices.

Answer: FALSE Topic: Supply Skill: Conceptual*

10) A supply curve is also a maximum-supply-price curve. Answer: FALSE Topic: Supply Skill: Conceptual*

11) An increase in price results in increase in supply but not an increase in the quantity supplied. Answer: FALSE

270

CHAPTER 3

Topic: Supply Curve Skill: Conceptual

Topic: Predicting Changes in Price and Quantity Skill: Analytical

12) An increase in the hourly wage of hot dog vendors at Yankee Stadium shifts the supply curve of hot dogs leftward.

16) When both the demand for a good increases and the supply of the good increases, the equilibrium quantity definitely increases.

Answer: TRUE

Answer: TRUE

Topic: Supply Curve Skill: Conceptual

Topic: Predicting Changes in Price and Quantity Skill: Analytical

13) An increase in technology will shift the good’s supply curve rightward.

17) In the market for chocolate chip cookies, if the demand decreases while the supply increases, the price definitely falls but the quantity might increase, decrease, or remain the same.

Answer: TRUE Topic: Predicting Changes in Price and Quantity Skill: Analytical

14) If house purchases and apartment rentals are substitutes, then an increase in the price of a new house results in an increase in apartment rents. Answer: TRUE Topic: Predicting Changes in Price and Quantity Skill: Analytical

15) During the 1990s, the average price of a used car fell by nearly $500 and the quantity sold nationwide decreased by several thousands each year. This set of results is a contradiction of the law of demand. Answer: FALSE

Answer: TRUE

CHAPTER 4

ELASTICITY

Topic: The Price Elasticity of Demand Skill: Recognition*

Topic: Total Revenue and Elasticity Skill: Conceptual

1)

7)

The price elasticity of demand equals the slope of the demand curve.

If the demand is unit elastic, a price cut will leave the quantity demanded unchanged.

Answer: FALSE

Answer: FALSE

Topic: Calculating Elasticity Skill: Analytical

Topic: Total Revenue and Elasticity Skill: Conceptual*

2)

8)

If the quantity demanded of a good decreases by 10 percent when the price of the good increases by 5 percent, the elasticity of demand is –2.00.

If the demand for KFC chicken is price inelastic, a fall in the price of KFC chicken will raise the total revenue.

Answer: FALSE

Answer: FALSE

Topic: Inelastic and Elastic Demand Skill: Analytical

Topic: Total Revenue and Elasticity Skill: Conceptual*

3)

9)

If the demand for cigarettes decreases after the U.S. Surgeon General publicizes five new diseases associated with smoking, this is conclusive evidence that the demand for cigarettes is elastic.

Answer: FALSE Topic: Inelastic and Elastic Demand Skill: Analytical

4)

If the demand for a good is perfectly elastic, then the demand curve is horizontal.

Answer: TRUE

If Taco Bell determines that the demand for its food is elastic, Taco Bell should raise its price to increase its total revenue.

Answer: FALSE Topic: Factors That Influence the Price Elasticity of Demand Skill: Analytical

10) The larger the portion of a person’s total budget spent on a good, the more inelastic the demand for the good. Answer: FALSE

Topic: Elasticity Along a Straight-Line Demand Curve Skill: Conceptual

5)

The elasticity of demand is constant along a downward sloping straight-line demand curve.

Answer: FALSE Topic: Total Revenue and Elasticity Skill: Conceptual*

6)

If the demand for KFC chicken is price elastic, a fall in the price of KFC chicken will raise the total revenue.

Answer: TRUE

Topic: Total Revenue and Elasticity Skill: Analytical

11) If the price of a pumpkin rises and consumers’ total expenditure on pumpkins increases, then the demand for pumpkins is inelastic. Answer: TRUE Topic: Total Revenue and Elasticity Skill: Analytical

12) If a hot dog vendor on a street corner experiences an increase in total revenue after lowering the price of a hot dog, then the demand for the hot dogs must be elastic. Answer: TRUE

272

CHAPTER 4

Topic: Cross Elasticity of Demand Skill: Conceptual

Topic: Elasticity of Supply Skill: Analytical

13) The cross elasticity of demand for substitutes is always positive.

19) Damage from floods and hurricanes which destroy a large portion of this year’s crop of oranges will definitely make oranges more price inelastic in supply.

Answer: TRUE Topic: Cross Elasticity of Demand Skill: Analytical

14) If the cross elasticity of demand between Jeep Cherokees and Chevy Lumina Vans is 1.55, then the two vehicles are not substitutes in the eyes of car buyers. Answer: FALSE Topic: Income Elasticity of Demand Skill: Recognition*

15) For inferior goods, the income elasticity of demand is negative. Answer: TRUE Topic: Income Elasticity of Demand Skill: Conceptual

16) An inferior good is a good whose income elasticity of demand is less than 0. Answer: TRUE Topic: Income Elasticity of Demand Skill: Analytical

17) Studies show that smoking cigars is inversely related to income. Thus cigars are an inferior good. Answer: TRUE Topic: Income Elasticity of Demand Skill: Analytical

18) If the demand for farm products were income elastic, that would mean that farm products were a necessity. Answer: FALSE

Answer: FALSE Topic: Inelastic and Elastic Supply Skill: Recognition

20) For baseball memorabilia fans, the baseball with which Hank Aaron hit his 735th career home run is perfectly inelastic in supply and in demand. Answer: FALSE Topic: Inelastic and Elastic Supply Skill: Analytical

21) Fantastic growing conditions that produce a bumper crop of oranges on each tree this year will definitely make the short run supply of oranges more price elastic. Answer: FALSE Topic: Inelastic and Elastic Supply Skill: Conceptual

22) Goods or services that can be produced only by using unique or rare productive resources tend to have a low elasticity of supply. Answer: TRUE

CHAPTER 5

EFFICIENCY AND EQUITY

Topic: Marginal Benefit Skill: Recognition

Topic: Efficiency And Inefficiency Skill: Analytical*

1)

7)

The value of one more unit of a good or service is its marginal benefit.

Answer: TRUE Topic: Marginal Benefit Skill: Conceptual

2)

The opportunity cost to the consumer of purchasing and consuming one more unit of some good is called the marginal benefit.

Answer: TRUE Topic: Efficiency And Inefficiency Skill: Conceptual*

8)

Answer: FALSE Topic: Marginal Cost Skill: Conceptual

3)

The opportunity cost to the firm of producing one more unit of output is also called marginal cost.

Answer: TRUE Topic: Efficiency And Inefficiency Skill: Analytical*

4)

If the marginal benefit of pizza exceeds the marginal cost of pizza, we are producing the efficient quantity of pizza.

If marginal cost exceeds marginal benefit, it would increase society’s total net surplus by decreasing production.

As long as production is such that marginal benefit is greater than marginal cost, an efficient outcome occurs.

Answer: FALSE Topic: Consumer Surplus Skill: Recognition*

9)

Consumer surplus is the value of a good minus the cost of producing it, summed over the quantity bought.

Answer: FALSE Topic: Consumer Surplus Skill: Recognition

10) Buyers receive a consumer surplus when the price exceeds the marginal benefit.

Answer: FALSE

Answer: FALSE

Topic: Efficiency And Inefficiency Skill: Analytical

Topic: Consumer Surplus Skill: Conceptual*

5)

11) Consumers don’t always have to pay the maximum price they are willing to pay.

If the marginal cost exceeds the marginal benefit for the last pair of shoes produced, then the economy is producing more than the efficient amount.

Answer: TRUE Topic: Efficiency And Inefficiency Skill: Analytical

6)

If the marginal benefit exceeds the marginal cost of producing the next kilowatt hour of electricity, then it is efficient to produce as many kilowatt hours as possible.

Answer: FALSE

Answer: TRUE Topic: Marginal Benefit Curve Skill: Conceptual

12) A consumer’s demand curve for popcorn is downward sloping as is the consumer’s marginal benefit curve for popcorn. Answer: TRUE

274

CHAPTER 5

Topic: Marginal Cost Skill: Recognition

Topic: An Efficient Market Skill: Conceptual

13) Marginal cost is the minimum price that producers must receive to induce them to produce another unit of a good or service.

18) In a competitive equilibrium, the total consumer surplus must equal the total producer surplus.

Answer: TRUE Topic: Producer Surplus Skill: Recognition

14) The producer surplus on a unit of output is the difference between the market price and the opportunity cost of producing it. Answer: TRUE Topic: Producer Surplus Skill: Recognition*

15) Producer surplus is the price of a good minus the opportunity cost of producing it, summed over the quantity produced. Answer: TRUE Topic: Producer Surplus Skill: Analytical

16) If the hot dog vendors at Yankee Stadium are earning a producer surplus on each hot dog they sell, then baseball fans cannot be gaining any consumer surplus on they hot dogs they buy. Answer: FALSE Topic: An Efficient Market Skill: Conceptual

17) Although the efficient quantity to produce of any good is located where marginal benefit and marginal cost are equal, there will usually be other quantities where the sum of consumer and producer surplus are greater. Answer: FALSE

Answer: FALSE Topic: Deadweight Loss Skill: Conceptual

19) When the efficient quantity of output is being produced and sold, the deadweight loss is maximized. Answer: FALSE Topic: Is the Competitive Market Efficient? Skill: Recognition*

20) A competitive market is always efficient. Answer: FALSE Topic: Is the Competitive Market Efficient? Skill: Analytical

21) If the marginal benefit of ice cream curve shifts rightward because people now like ice cream better than before, the marginal cost curve of ice cream will shift leftward. Answer: FALSE Topic: Obstacles to Efficiency, Externalities Skill: Recognition*

22) One of the obstacles to efficiency is monopoly. Answer: TRUE Topic: Obstacles to Efficiency, Externalities Skill: Conceptual

23) When there are external costs of production, such as when electric utilities burn coal, a competitive market will produce an inefficient level of output. Answer: TRUE

CHAPTER 6

MARKETS IN ACTION

Topic: A Regulated Housing Market Skill: Conceptual*

Topic: Price Ceilings Skill: Conceptual

1)

7)

A rent ceiling set above the equilibrium rent has no effect.

Answer: TRUE

Topic: Inefficiency of Rent Ceilings Skill: Conceptual

2)

Answer: FALSE Topic: The Labor Market Skill: Conceptual

8)

An effective rent ceiling efficiently allocates resources in the housing market.

Answer: FALSE Topic: Search Activity Skill: Conceptual

3)

One good aspect of rent ceilings is that they help reduce the time and expense associated with apartment hunting.

Rent controls and the minimum wage are both examples of price ceilings.

The supply curve of low-skilled labor is upward sloping because those workers know that government will protect them with minimum wage legislation.

Answer: FALSE Topic: The Minimum Wage Skill: Recognition

9)

The minimum wage is a example of a price floor.

Answer: TRUE

Answer: FALSE Topic: Discrimination Skill: Conceptual

4)

Discrimination based on age, race, gender, or family status very likely increases as a result of rent ceilings.

Topic: The Labor Market and the Minimum Wage Skill: Conceptual*

10) A minimum wage set above the equilibrium wage rate has no effect. Answer: FALSE

Answer: TRUE

Topic: The Labor Market and the Minimum Wage Skill: Conceptual

Topic: Black Markets Skill: Conceptual

11) Most economists believe that raising the minimum wage decreases the employment opportunities for low-skilled workers.

5)

One good aspect of rent ceilings is that such price controls never result in black markets because apartment buildings are too large to conceal from the police.

Answer: TRUE

Answer: FALSE

Topic: The Labor Market and the Minimum Wage Skill: Conceptual

Topic: Housing Markets and Rent Ceilings Skill: Conceptual

Answer: FALSE

6)

One way to assure that poor people will have a plentiful supply of affordable housing available to them is by imposing rent ceilings.

Answer: FALSE

12) Economists generally agree that increases in the minimum wage increase employment. Topic: The Minimum Wage in Practice Skill: Conceptual

13) A policy of raising the minimum wage is beneficial to all low-skilled workers. Answer: FALSE

276

CHAPTER 6

Topic: Who Pays the Sales Tax? Skill: Conceptual

Topic: Tax Incidence and the Elasticity of Demand Skill: Conceptual

14) A sales tax on sellers of a good shifts the demand curve leftward because the tax causes consumers to like the product less.

19) The incidence of the tax between buyers and sellers depends only on the elasticity of demand.

Answer: FALSE Topic: Who Pays the Sales Tax? Skill: Conceptual

15) A sales tax on sellers of a good causes the supply curve to shift leftward because the tax is just like a cost of production. Answer: TRUE Topic: Who Pays the Sales Tax? Skill: Conceptual*

16) A tax on buyers raises the price of the good more than would the same tax if it was imposed on sellers.

Answer: FALSE Topic: Subsidy Skill: Analytical*

20) Because a subsidy raises marginal benefit, it can be used to eliminate the deadweight loss from overproduction. Answer: FALSE Topic: Production Quota Skill: Conceptual*

21) A production quota on tobacco lowers the price of tobacco and the marginal cost of producing it. Answer: FALSE

Answer: FALSE

Topic: Markets for Illegal Goods Skill: Analytical

Topic: Taxes and Deadweight Loss Skill: Conceptual

22) The black market price for a legally prohibited good will be the same whether a $1,000 fine is placed on the buyer or the seller.

17) A sales tax on sellers a good leads to a loss of consumer surplus, but a price ceiling or a price floor on that same product will not. Answer: FALSE Topic: Tax Incidence and the Elasticity of Demand Skill: Conceptual*

18) If the demand is perfectly elastic, buyers pay the entire tax. Answer: FALSE

Answer: FALSE

CHAPTER 7

UTILITY AND DEMAND

Topic: Marginal Utility Skill: Recognition

Topic: Maximizing Utility Skill: Conceptual

1)

7)

Total utility is the benefit received from consuming an extra unit of a good.

Answer: FALSE Topic: Total Utility and Marginal Utility Skill: Recognition

2)

According to the principle of diminishing marginal utility, as an individual consumes more and more of a good or service, the total utility increases while the marginal utility decreases.

A consumer will maximize utility when all income is spent and the marginal utility is equal for all goods.

Answer: FALSE Topic: Maximizing Utility Skill: Conceptual

8)

When a person equates marginal utility per dollar across all goods and spends all of her budget, total utility reaches its maximum.

Answer: TRUE

Answer: TRUE

Topic: Total Utility and Marginal Utility Skill: Analytical

Topic: Maximizing Utility Skill: Analytical

3)

9)

Tom’s marginal utility of Coca Cola exceeds his marginal utility of crackers. Therefore, his total utility of Coca Cola must exceed his total utility of crackers.

Answer: FALSE Topic: Diminishing Marginal Utility Skill: Recognition*

4)

Marginal utility diminishes as consumption of a good decreases.

Answer: FALSE Topic: Diminishing Marginal Utility Skill: Recognition

5)

If Tom’s total utility from watching one more minute of television increases but the increase for each additional minute is smaller than the previous minute, he has diminishing marginal utility.

Answer: TRUE Topic: Diminishing Marginal Utility Skill: Conceptual

6)

As a person consumes more and more slices of pizza, the marginal utility of each extra slice diminishes.

Answer: TRUE

If it costs $6.00 to go to the movies and $25.00 to go to a hockey game, Tom is maximizing his utility between movies and hockey if his marginal utility of movies is 12 units and his marginal utility from hockey is 25.

Answer: FALSE Topic: Maximizing Utility Skill: Analytical

10) If it costs $6.00 to go to the movies and $25.00 to go to a hockey game, Tom is maximizing his utility between movies and hockey if his marginal utility of movies is 12 units and his marginal utility from hockey is 50. Answer: TRUE Topic: Maximizing Utility Skill: Conceptual*

11) A consumer’s total utility is maximized by following the rule: allocate all the available budget and equalize the marginal utility from the last unit of all goods purchased. Answer: FALSE

278

Topic: Equalizing Marginal Utility per Dollar Spent Skill: Conceptual*

12) A consumer’s total utility is maximized when the total utility per dollar spent on all goods is equalized. Answer: FALSE

CHAPTER 7

Topic: Predictions of Marginal Utility Theory, Change in Income Skill: Conceptual

17) When Tom’s income increases, his demand curve for Coca Cola shifts rightward because the higher income increases his marginal utility of Coca Cola.

Topic: Marginal Analytical Skill: Analytical

Answer: FALSE

13) It costs $6.00 to go to the movies and $25.00 to go to a hockey game. If Tom’s marginal utility of movies is 12 units and his marginal utility from hockey is 48, to maximize his utility, Tom should see more movies and fewer hockey games.

Topic: Market Demand Curve Skill: Recognition

Answer: TRUE Topic: Marginal Analytical Skill: Analytical

14) It costs $6.00 to go to the movies and $25.00 to go to a hockey game. If Tom’s marginal utility of movies is 12 units and his marginal utility from hockey is 48, to maximize his utility, Tom should see more hockey games and fewer movies. Answer: FALSE Topic: Marginal Analytical Skill: Conceptual*

15) If Dana spends all her income on ice cream and Coke and her marginal utility per dollar spent on ice cream is greater than her marginal utility per dollar spent on Coke, she should buy more ice cream and less Coke. Answer: TRUE Topic: Predictions of Marginal Utility Theory, Change in Price Skill: Conceptual

16) If the price of a good increases, a consumer will substitute away from the relatively more expensive good, which will increase the marginal utility for that good and bring the consumer back to equilibrium. Answer: TRUE

18) The market demand curve is the horizontal summation of individual demand curves. Answer: TRUE Topic: Paradox of Value Skill: Recognition

19) The Paradox of Value is resolved by the willingness for an individual to pay a high price for a good or service that has a high marginal utility per dollar. Answer: FALSE Topic: Paradox of Value Skill: Analytical

20) Tom’s marginal utility of Coca Cola exceeds his marginal utility of crackers at his consumer equilibrium. Therefore, his consumer surplus from Coca Cola must exceed his consumer surplus from crackers. Answer: FALSE

CHAPTER 8

POSSIBILITIES, PREFERENCES, AND CHOICES

Topic: Consumption Possibilities, Budget Line Skill: Conceptual

1)

To draw your budget line between steak and lobster, all you need to know is the price of a steak and the price of a lobster.

Topic: Consumption Possibilities, Change in Income Skill: Conceptual*

7)

A change in money income changes the relative price but does not affect real income.

Answer: FALSE

Answer: FALSE

Topic: Consumption Possibilities, Budget Line Skill: Conceptual

Topic: Consumption Possibilities, Change in Income Skill: Conceptual

2)

To draw your budget line between steak and lobster, all you need to know is your income.

8)

Answer: FALSE

A change in money income changes real income and therefore changes the slope of the budget line.

Answer: FALSE Topic: Consumption Possibilities, Change in Price Skill: Conceptual*

3)

The lower the price of the good measured on the vertical axis, other thing remaining the same, the flatter the budget line.

Answer: FALSE

Topic: Consumption Possibilities, Change in Income Skill: Conceptual

9)

An increase in income rotates the budget line for steak and lobster inward.

Answer: FALSE Topic: Consumption Possibilities, Change in Price Skill: Conceptual*

4)

The higher the price of the good measured on the vertical axis, other thing remaining the same, the flatter the budget line.

Topic: Marginal Rate of Substitution Skill: Recognition

10) The slope of the budget line between steak and lobster is called the marginal rate of substitution.

Answer: TRUE

Answer: FALSE

Topic: Consumption Possibilities, Change in Price Skill: Conceptual

Topic: Marginal Rate of Substitution Skill: Recognition

5)

A fall in either the price of a steak or the price of a lobster will shift the budget line for the two leftward and not change its slope.

11) The marginal rate of substitution is the rate at which a person is willing to substitute one good for another good.

Answer: FALSE

Answer: TRUE

Topic: Consumption Possibilities, Change in Income Skill: Conceptual

Topic: Marginal Rate of Substitution Skill: Recognition

6)

An increase in income will shift the budget line rightward and not change its slope.

Answer: TRUE

12) The magnitude of the slope of the indifference curve between steak and lobster is called the marginal rate of substitution. Answer: TRUE

280

Topic: Marginal Rate of Substitution Skill: Recognition*

13) The steeper the indifference curve the higher the marginal rate of substitution.

CHAPTER 8

Topic: Predicting Consumer Behavior, Best Affordable Point Skill: Conceptual

Answer: TRUE

20) When your budget line is just tangent to your indifference curve, you are at your best affordable point.

Topic: Marginal Rate of Substitution Skill: Conceptual

Answer: TRUE

14) The slope of the indifference curve between steak and lobster is always equal to the ratio of their prices.

Topic: Predicting Consumer Behavior, Best Affordable Point Skill: Conceptual

Answer: FALSE

21) A consumer will maximize utility, given income and prices, when the marginal rate of substitution is equal to the ratio of the prices of the two goods.

Topic: Marginal Rate of Substitution Skill: Conceptual

15) The larger the marginal rate of substitution, the larger is the amount of one good that the consumer is willing to give up in exchange for another good and still remain at the same level of satisfaction. Answer: TRUE

Answer: TRUE Topic: Predicting Consumer Behavior, Best Affordable Point Skill: Analytical

22) A decrease in price allows a consumer to attain a higher indifference curve. Answer: TRUE

Topic: Degree of Substitutability Skill: Conceptual

16) If you considered steak and lobster to be perfect substitutes for each other, your indifference curves between them would be L-shaped. Answer: FALSE Topic: Degree of Substitutability Skill: Conceptual*

Topic: Predicting Consumer Behavior, Demand Curve Skill: Analytical

23) Holding your income and the price of lobster constant, you can derive your demand curve for steak from an indifference curve/budget line diagram by determining how your consumption of steak changes when the price of a steak changes.

17) When two goods are perfect substitutes, their indifference curves are straight lines.

Answer: TRUE

Answer: TRUE

Topic: Predicting Consumer Behavior, Substitution Effect Skill: Analytical

Topic: Degree of Substitutability Skill: Conceptual

18) In order for you to consider steak and lobster to be perfect substitutes, their prices per pound must be identical.

24) You consume only steak and lobster. Your substitution effect from a drop in the price of lobster is measured by a movement along your indifference curve between steak and lobster.

Answer: FALSE

Answer: TRUE

Topic: Predicting Consumer Behavior, Best Affordable Point Skill: Conceptual

Topic: Predicting Consumer Behavior, Income Effect Skill: Analytical

19) When your budget line is just tangent to your indifference curve, you are at the point on the budget line that you least prefer.

25) You consume only steak and lobster. Your income effect from a drop in the price of lobster is measured by a movement along your indifference curve between steak and lobster.

Answer: FALSE

Answer: FALSE

CHAPTER 9

ORGANIZING PRODUCTION

Topic: The Firm's Goal Skill: Recognition*

Topic: Economic Efficiency Skill: Conceptual

1)

8)

A firm’s goal is to maximize accounting profit.

Answer: FALSE

If a firm produces a given amount of output using the least amount of inputs, it definitely achieves economic efficiency.

Topic: Opportunity Cost Skill: Recognition

Answer: FALSE

2)

Topic: Information and Organization Skill: Recognition

A firm’s opportunity cost of production can be divided into two components, depreciation and interest.

Answer: FALSE

9)

Firms organize production by using a mixture of two systems: (1) command systems and (2) incentive systems.

Topic: Explicit and Implicit Costs Skill: Recognition

Answer: TRUE

3)

Topic: Principal-Agent Problem Skill: Conceptual

A firm’s opportunity cost of production can be divided into two components, explicit costs and implicit costs.

Answer: TRUE Topic: Economic Profit Skill: Recognition

4)

Economic profit equals total revenue less explicit cost.

Answer: FALSE Topic: Technological Efficiency Skill: Recognition

5)

Technological efficiency occurs when it is not possible for a firm to get more output from the inputs it is currently using.

Answer: TRUE Topic: Technological Efficiency Skill: Recognition*

6)

Technological efficiency occurs when the firm produces a given output by using the least amount of inputs.

Answer: TRUE Topic: Economic Efficiency Skill: Conceptual

7)

If a firm is achieving economic efficiency it must also be achieving technological efficiency.

Answer: TRUE

10) The possibility that the managers of a corporation might not always act in the best interest of its owners is an example of the principal-agent problem. Answer: TRUE Topic: The Principal-Agent Problem, Solutions Skill: Conceptual

11) The only two known ways of dealing with the principal agent problem are the use of long-term contracts and monitoring. Answer: FALSE Topic: Types of Business Organization Skill: Conceptual

12) The decision about whether a firm in each particular industry must operate as a proprietorship, partnership, or corporation is made by the Internal Revenue Service. Answer: FALSE Topic: Market Structure, Monopolistic Competition Skill: Recognition*

13) A monopolistically competitive firm produces a good or service that has no close substitutes. Answer: FALSE

282

CHAPTER 9

Topic: Topic: Types of Business Organization Skill: Recognition

Topic: Market Structure in the U.S. Economy Skill: Recognition

14) Proprietorships generally have unlimited liability, whereas partnerships and corporations have limited liability.

18) The most prevalent market structures of the U.S. economy are perfect competition and monopolistic competition.

Answer: FALSE

Answer: TRUE

Topic: Topic: Types of Business Organization Skill: Recognition*

Topic: Market Structure in the U.S. Economy Skill: Recognition

15) Since the tax on dividends has been repealed, corporate profits are no longer taxed twice.

19) Although a corporation is the most common type of firm, corporations generate only a small share of business revenue in the U.S. economy.

Answer: FALSE Topic: Measures of Concentration Skill: Recognition

16) Concentration ratios take barriers to entry into a market into consideration. Answer: FALSE Topic: Measures of Concentration Skill: Recognition

17) Concentration ratios are computed for industries, not markets. Answer: TRUE

Answer: FALSE Topic: Why Firms?; Economies of Scale and Scope Skill: Conceptual

20) Economies of scope has to do with lowering production costs by increasing the quantity of a product produced, whereas economies of scale has to do with lowering production costs by producing several products within the same firm. Answer: FALSE

CHAPTER 10

OUTPUT AND COSTS

Topic: Total Product Curve Skill: Conceptual*

Topic: Fixed Cost Skill: Conceptual

1)

7)

All the production points that lie above the total product curve are inefficient.

The term “fixed cost” refers to the cost a firm incurs to produce a specific quantity of output.

Answer: FALSE

Answer: FALSE

Topic: Marginal Product of Labor Skill: Conceptual

Topic: Fixed Cost Skill: Conceptual

2)

8)

The marginal product of labor is defined as the increase in output attributable to the last worker hired divided by the total number of workers employed.

In the short run, average fixed cost is constant as output increases.

Answer: FALSE

Answer: FALSE

Topic: Marginal Cost Skill: Conceptual

Topic: Diminishing Marginal Returns Skill: Conceptual

9)

3)

When there are diminishing marginal returns to labor, the marginal product of the last worker hired must be negative.

Marginal cost refers to the increase in cost attributable to hiring one more unit of labor, capital, or some other input.

Answer: FALSE

Answer: FALSE

Topic: Average Cost Curves Skill: Conceptual*

Topic: Relationship Between the Marginal Product and Average Product of Labor Skill: Conceptual

10) The vertical distance between the average variable cost curve and the average total cost curve is average fixed cost.

4)

Answer: TRUE

When the marginal product of labor exceeds the average product of labor, the average product must be increasing.

Answer: TRUE Topic: Relationship Between the Marginal Product and Average Product of Labor Skill: Conceptual*

5)

When the marginal product of labor is below the average product of labor, the average product is increasing.

Answer: FALSE Topic: Total Cost Skill: Conceptual

6)

A firm’s total cost in the short run is the sum of its fixed cost plus variable cost plus marginal cost.

Answer: FALSE

Topic: Relationship Between Marginal Cost Curve and Average Variable and Average Total Cost Curves Skill: Conceptual

11) The marginal cost curve intersects the average fixed, average variable, and average total cost curves all at their minimum points. Answer: FALSE Topic: Relationship Between Marginal Cost Curve and Average Variable and Average Total Cost Curves Skill: Conceptual

12) When marginal cost is greater than average cost, average cost decreases as output increases. Answer: FALSE

284

Topic: Relationship Between Cost Curves and Product Curves Skill: Conceptual

13) Over the range of output for which the average product of labor curve is negatively sloped, the average variable cost curve is positively sloped.

CHAPTER 10

Topic: Long Run Costs Skill: Recognition

18) When a firm’s long-run average cost is constant as output increases, the firm is experiencing constant returns to scale. Answer: TRUE

Answer: TRUE Topic: Relationship Between Cost Curves and Product Curves Skill: Conceptual

14) Over the range of output for which the marginal product of labor curve is negatively sloped, the marginal cost curve is negatively sloped.

Topic: Diseconomies of Scale Skill: Conceptual

19) Diminishing marginal returns and diseconomies of scale are two different names for the same thing. Answer: FALSE

Answer: FALSE

Topic: Long-Run Average Cost Curve Skill: Conceptual*

Topic: Long Run Costs Skill: Conceptual

20) The long-run average cost curve is the relationship between the highest attainable average total cost and output when both the plant size and labor are varied.

15) In the long run, total fixed cost equals zero. Answer: TRUE Topic: Long Run Costs Skill: Conceptual*

16) In the long run, total variable cost is zero. Answer: FALSE Topic: Long Run Costs Skill: Recognition

17) A firm’s minimum efficient scale is the largest quantity of output at which long-run average cost reaches its highest level. Answer: FALSE

Answer: FALSE Topic: Long-Run Average Cost Curve Skill: Conceptual

21) A firm’s long-run average cost curve is derived from a series of short-run average total cost curves. Answer: TRUE Topic: Long-Run Average Cost Curve Skill: Conceptual

22) A firm’s long-run average cost curve is derived by adding together its short-run average total cost curves. Answer: FALSE

CHAPTER 11

PERFECT COMPETITION

Topic: Perfect Competition Skill: Recognition

Topic: Shutdown Point Skill: Recognition

1)

8)

In a perfectly competitive market, many firms sell an identical product.

Answer: TRUE Topic: Price Takers Skill: Recognition

2)

Perfectly competitive firms are price takers.

Answer: TRUE

Answer: TRUE Topic: Economic Profits and Economic Losses in the Short Run Skill: Conceptual

9) Topic: Price Takers Skill: Recognition

3)

A perfectly competitive firm sets its price equal to its marginal cost.

A firm’s shutdown point is the output and price at which the firm’s total revenue just equals its total variable cost.

Perfectly competitive firms will sometimes operate even though they incur an economic loss in the short run.

Answer: TRUE

Answer: TRUE

Topic: The Firm’s Short-Run Supply Curve Skill: Recognition

Topic: Profit-Maximizing Output Skill: Recognition*

10) The supply curve for a perfectly competitive firm is the portion of its marginal cost curve that lies above its marginal revenue curve.

4)

A perfectly competitive firm maximizes its profit by producing the level of output so that its average total cost equals the market price.

Answer: FALSE

Answer: FALSE

Topic: The Firm’s Short-Run Supply Curve Skill: Recognition

Topic: Marginal Analytical Skill: Conceptual

11) The supply curve for a perfectly competitive firm is the portion of its marginal cost curve that lies above the average variable cost curve.

5)

If a firm is maximizing profits, the extra revenue it receives from selling its last unit of output exceeds the extra cost of producing that unit.

Answer: TRUE

Answer: FALSE

Topic: Long Run Equilibrium Skill: Conceptual

Topic: Entry and Exit Skill: Recognition*

12) In the long run, a perfectly competitive firm can earn an economic profit because its marginal cost equals its average total cost.

6)

In perfect competition, firms enter the industry whenever the market price exceeds the minimum average variable cost.

Answer: FALSE

Answer: FALSE

Topic: Long Run Equilibrium Skill: Recognition

Topic: Shutdown Point Skill: Conceptual

13) In the long run, perfectly competitive firms cannot earn an economic profit.

7)

A perfectly competitive firm definitely will shut down in the short run if its price is below its average total cost.

Answer: FALSE

Answer: TRUE

286

CHAPTER 11

Topic: Long-Run Adjustments; Entry Skill: Conceptual

Topic: Long Run Supply Curve Skill: Recognition

14) Entry of new firms into a perfectly competitive market raises the product’s price.

19) A long-run industry supply curve shows how the quantity supplied by an industry varies as the market price varies after all the possible adjustments have been made, including changes in plant size and the number of firms in the industry.

Answer: FALSE Topic: Long-Run Adjustments; Entry Skill: Conceptual

15) Entry of new firms into a perfectly competitive market lowers the profits of the existing firms. Answer: TRUE Topic: Long-Run Adjustments; Exit Skill: Conceptual*

16) In the long run, a perfectly competitive firm leaves the industry if the market price is less than the firm’s average total cost. Answer: TRUE Topic: Long-Run Equilibrium Skill: Conceptual

17) Easy entry and exit ensures that perfectly competitive firms cannot earn a long-run economic profit. Answer: TRUE Topic: Long-Run Equilibrium Skill: Recognition*

18) In the long run, perfectly competitive firms earn a normal profit. Answer: TRUE

Answer: TRUE Topic: A Permanent Change in Demand Skill: Conceptual

20) In a perfectly competitive industry, in the long run a permanent decrease in the market demand results in a smaller number of firms. Answer: TRUE Topic: A Permanent Change in Demand Skill: Conceptual

21) When the market demand increases in a perfect competition, the long-run result is a larger number of firms, a higher price, and a permanent economic profit for the firms. Answer: FALSE Topic: Competition and Efficiency Skill: Conceptual

22) In the absence of external benefits and costs, perfectly competitive markets are inefficient. Answer: FALSE

CHAPTER 12

MONOPOLY

Topic: How Monopoly Arises Skill: Recognition*

Topic: Marginal Revenue and Elasticity Skill: Conceptual

1)

8)

A monopoly is a firm that produces a good or service for which no close substitute exists.

A monopoly always operates on the elastic portion of its demand curve.

Answer: TRUE

Answer: TRUE

Topic: How Monopoly Arises Skill: Recognition

Topic: Single-Price Monopoly’s Output and Price Decisions Skill: Conceptual

2)

A monopolist is the sole supplier of a good or service for which there are no close substitutes.

Answer: TRUE

9)

The monopolist always maximizes its profits by producing the amount of output that sets the marginal revenue equal to zero.

Topic: How Monopoly Arises Skill: Conceptual

Answer: FALSE

3)

Topic: A Single Price Monopoly’s Output and Price Decisions Skill: Recognition

For a monopoly, the demand for its product is perfectly elastic at the market price.

Answer: FALSE Topic: Natural Monopoly Skill: Recognition*

4)

A natural monopoly is a firm that owns a key natural resource.

Answer: FALSE Topic: Natural Monopoly Skill: Recognition

5)

A natural monopoly is any market in which competition and entry are restricted by the granting of a public franchise, government license, patent, or copyright.

10) A profit maximizing single-price monopolist sets price equal to marginal cost. Answer: FALSE Topic: A Single Price Monopoly’s Output and Price Decisions Skill: Conceptual

11) In order to maximize its profit, a single-price monopoly always produces output in the inelastic range of the demand for its product. Answer: FALSE

Answer: FALSE

Topic: A Monopoly’s Economic Profit Skill: Recognition

Topic: Natural Monopoly Skill: Recognition*

Answer: TRUE

6)

A natural monopoly is likely to experience diseconomies of scale.

12) Monopolists can enjoy an economic profit in the long-run because of barriers to entry.

Answer: FALSE

Topic: Single-Price Monopoly, Deadweight Loss Skill: Conceptual*

Topic: Marginal Revenue* Skill: Conceptual

Answer: FALSE

7)

A monopoly’s marginal revenue is equal to the market price of its product.

Answer: FALSE

13) A monopoly creates no deadweight when it maximizes its profit.

288

CHAPTER 12

Topic: Single-Price Monopoly, Deadweight Loss Skill: Recognition

Topic: Perfect Price Discrimination Skill: Conceptual

14) The deadweight from a monopoly loss measures the inefficiency created by monopoly.

19) A monopoly that can perfectly price discriminate creates no deadweight loss.

Answer: TRUE

Answer: TRUE

Topic: Single-Price Monopoly, Deadweight Loss Skill: Conceptual

Topic: Economies of Scope Skill: Recognition

15) A monopoly creates a deadweight loss because it reduces output and increases the price of the good.

20) A decrease in average total cost as a firm produces more of one type of output is a sign of economies of scope.

Answer: TRUE

Answer: FALSE

Topic: Rent Seeking Skill: Recognition

Topic: Economies of Scope Skill: Recognition

16) Attempts to achieve a monopoly are examples of rent seeking.

21) A decrease in average total cost because of the production of another type of product is a sign of economies of scope.

Answer: TRUE Topic: Price Discrimination Skill: Conceptual

17) Differences in the average willingness-to-pay between at least two distinct groups are needed to allow a monopoly to price discriminate. Answer: TRUE Topic: Perfect Price Discrimination Skill: Recognition

18) For a monopoly able to perfectly price discriminate, the marginal revenue curve coincides with the demand curve. Answer: TRUE

Answer: TRUE Topic: Economies of Scope Skill: Conceptual

22) If economies of scope are present, a monopolist might charge a price below that of a competitive market. Answer: TRUE

CHAPTER 13

MONOPOLISTIC COMPETITION AND OLIGOPOLY

Topic: Product Differentiation Skill: Conceptual

1)

In monopolistic competition, it is barriers to entry that give the firms the power to set their price.

Topic: Monopolistic Competition; Long-Run Profit Maximization Skill: Conceptual

7)

Answer: FALSE

In monopolistic competition, free entry and free exit mean that in the long run firms in the industry make zero economic profit.

Topic: Product Differentiation Skill: Conceptual

Answer: TRUE

2)

Topic: Monopolistic Competition; Long-Run Profit Maximization Skill: Conceptual*

In monopolistic competition, it is product differentiation that gives the firms the power to set their prices.

Answer: TRUE

8)

Topic: Product Differentiation Skill: Conceptual

Answer: FALSE

3)

Topic: Monopolistic Competition; Long-Run Profit Maximization Skill: Conceptual

In monopolistic competition, firms compete on product quality, price and marketing.

Answer: TRUE Topic: Monopolistic Competition; Profit Maximization Skill: Conceptual

4)

Firms in monopolistic competition maximize their profit by setting their price equal to their marginal revenue.

Answer: FALSE Topic: Monopolistic Competition; Profit Maximization Skill: Conceptual*

5)

In the short run, a firm in monopolistic competition produces where P = MC

Answer: FALSE Topic: Monopolistic Competition; Profit Maximization Skill: Conceptual*

6)

In the short run, a firm in monopolistic competition produces where MR = MC

Answer: TRUE

9)

Firms in monopolistic competition can earn an economic profit in the long run.

In the long run, firms in monopolistic competition become price takers.

Answer: FALSE Topic: Monopolistic Competition; Long-Run Profit Maximization Skill: Conceptual*

10) In long-run equilibrium in monopolistic competition, price equals marginal cost. Answer: FALSE Topic: Monopolistic Competition; Long-Run Profit Maximization Skill: Conceptual

11) Under monopolistic competition, firms earn zero economic profit in the long run and produce at the minimum ATC. Answer: FALSE

290

Topic: Monopolistic Competition; Long-Run Profit Maximization Skill: Conceptual

CHAPTER 13

Topic: Game Theory Skill: Recognition

12) In the long run, monopolistically competitive firms earn zero economic profits because of government regulations.

19) Game theory is a tool for studying competitive behavior between firms in monopolistic competition because of the mutual interdependence among the firms.

Answer: FALSE

Answer: FALSE

Topic: Monopolistic Competition; Long-Run Profit Maximization Skill: Conceptual

Topic: Prisoners’ Dilemma Skill: Analytical

13) In the long run, monopolistically competitive firms can earn an economic profit because of product differentiation.

Answer: FALSE

20) In the prisoners’ dilemma game, each player has only one possible strategy.

Answer: FALSE

Topic: Game Theory, Nash Equilibrium Skill: Recognition

Topic: Monopolistic Competition; Long-Run Excess Capacity Skill: Recognition

21) In a Nash equilibrium, each player takes the best possible action given the actions of the other players.

14) Excess capacity refers to any unsold output due to insufficient demand.

Answer: TRUE

Answer: FALSE

Topic: Game Theory, Nash Equilibrium Skill: Recognition

Topic: Monopolistic Competition; Long-Run Excess Capacity Skill: Conceptual

22) In game theory, a Nash equilibrium is the equilibrium that always yields the best result.

15) In the long run, firms in monopolistic competition have excess capacity. Answer: TRUE Topic: Innovation and Product Development Skill: Conceptual

16) In monopolistic competition, firms do not have to produce innovative products because they have downward-sloping demand curves. Answer: FALSE Topic: Oligopoly Skill: Conceptual

17) Oligopoly differs from perfect competition because a single competitive firm’s behavior does not affect the behavior of its competitors while the behavior of a single oligopolistic firm does affect the behavior of its rivals. Answer: TRUE Topic: Oligopoly Skill: Conceptual*

18) Economies of scale and limited demand can form a natural barrier to entry that can create a natural oligopoly. Answer: TRUE

Answer: FALSE Topic: Cartel; Incentive To Cheat Skill: Conceptual

23) Collusive agreements tend to break apart because the incentive to cheat is so great. Answer: TRUE Topic: Contestable Market Skill: Recognition*

24) A contestable market is one in which there are one or a few firms and entry into the market is not costly. Answer: TRUE Topic: Limit Pricing Skill: Recognition

25) Limit pricing is a strategy which is intended to deter entry into an industry. Answer: TRUE

CHAPTER 14

REGULATION AND ANTITRUST LAW

Topic: Public Choice and the Political Marketplace Skill: Recognition*

Topic: Social interest Theory of Regulation Skill: Recognition

1)

7)

In the political marketplace voters are consumers.

Answer: TRUE

According to the social interest theory of regulation, regulation promotes the monopoly practice of restricting output to achieve higher profits.

Topic: Public Choice and the Political Marketplace Skill: Recognition*

Answer: FALSE

2)

Topic: Social interest Theory of Regulation Skill: Conceptual*

In the political marketplace firms are producers.

Answer: FALSE Topic: Political Equilibrium Skill: Recognition*

3)

A political equilibrium is a situation in which all groups are in agreement.

Answer: FALSE Topic: Regulation Skill: Recognition

4)

Antitrust law generally consists of rules, administered by government agencies, which determine prices and control conditions under which new firms may enter an industry.

Answer: FALSE Topic: Economic Theory of Regulation Skill: Recognition

5)

The economic theory of regulation is based on the demand for and supply of regulation.

Answer: TRUE Topic: Demand for Regulation Skill: Conceptual

6)

The demand by consumers for regulation of the cable TV industry depends partly on consumer surplus per subscriber and partly on the number of subscribers in the market.

Answer: TRUE

8)

According to the social interest theory, political process supplies the regulation that increases the benefits of small, easily identified social groups.

Answer: FALSE Topic: Capture Theory of Regulation Skill: Recognition

9)

The capture theory is that the regulations are supplied to satisfy the demand of producers to maximize producer surplus and so maximize economic profit.

Answer: TRUE Topic: Equilibrium Regulation Skill: Recognition*

10) Political liberals tend to the view that most regulation is in the self-interest of producers. Answer: FALSE Topic: Equilibrium Regulation Skill: Recognition*

11) Political conservatives tend to the view that most regulation is in the interest of consumers. Answer: FALSE Topic: The Scope of Regulation Skill: Recognition*

12) Since the late 1970s, the U.S. economy has been on a trend of moving from more regulation to less regulation. Answer: TRUE

292

CHAPTER 14

Topic: Natural Monopoly Skill: Recognition

Topic: Cartel Regulation Skill: Recognition*

13) The airline and trucking industries are two examples of industries that were regulated because they were natural monopolies.

18) According to the capture theory, oligopoly is regulated to ensure a competitive outcome.

Answer: FALSE Topic: Natural Monopoly, Average Cost Pricing Rule Skill: Conceptual

Answer: FALSE Topic: Sherman Act Skill: Recognition*

19) The first federal antitrust law ever passed was the Sherman Act.

14) Average cost pricing of a natural monopoly creates a deadweight loss.

Answer: TRUE

Answer: TRUE

Topic: Clayton Act Skill: Recognition*

Topic: Natural Monopoly, Marginal Cost Pricing Rule Skill: Recognition*

20) The Clayton Act of 1914 was passed to prohibit, in part, price discrimination if the effect is to substantially lessen competition or create monopoly.

15) For a natural monopoly, if price is equal to marginal cost then the firm incurs an economic loss.

Answer: TRUE

Answer: TRUE

Topic: Tying Contracts Skill: Recognition*

Topic: Natural Monopoly, Marginal Cost Pricing Rule Skill: Conceptual

21) Tying arrangements are always held to be illegal under U.S. antitrust law.

16) If the local cable TV company is a natural monopoly and required by regulators to set its price equal to marginal cost, it would earn a normal profit and produce the efficient level of output. Answer: FALSE Topic: Natural Monopoly, Marginal Cost Pricing Rule Skill: Conceptual

17) If the local cable TV company is a natural monopoly and required by regulators to set its price equal to marginal cost, there would be a deadweight loss in the market and the firm might need a government subsidy to survive. Answer: FALSE

Answer: FALSE Topic: Current Merger Policies Skill: Conceptual

22) The sum of the squares of the market share for the fifty largest firms in a market is the basis of the government’s current merger guidelines. Answer: TRUE Topic: Antitrust Law and Efficiency Skill: Conceptual

23) The manner in which antitrust laws have been enforced has generally increased efficiency. Answer: TRUE

CHAPTER 15

EXTERNALITIES

Topic: Positive Externalities: Education Skill: Conceptual

Topic: Coase Theorem Skill: Conceptual

1)

7)

Education possesses a positive consumption externality.

If property rights for air and water exist, then polluters bear the costs of pollution.

Answer: TRUE

Answer: TRUE

Topic: Marginal External Cost Skill: Recognition

Topic: Coase Theorem Skill: Conceptual

2)

8)

A marginal external cost is the cost of producing an additional unit of a good that falls on the producer.

The Coase Theorem points that for an efficient outcome to result, it is irrelevant which party receives the property rights.

Answer: FALSE

Answer: TRUE

Topic: Marginal External Cost Skill: Recognition

Topic: Coase Theorem Skill: Conceptual

3)

9)

The marginal social cost is the cost of producing an additional unit of a good or service that falls on people other than the producer of the good or service.

The Coase Theorem states that externalities can always be eliminated by the market as long as property rights exist, the number of parties involved is small, and transactions costs are low.

Answer: FALSE

Answer: TRUE

Topic: Marginal Social Cost Skill: Conceptual*

Topic: Government Policies Toward Pollution Skill: Recognition

4)

10) Emission charges, marketable permits, and taxes are tools that government can use to limit pollution.

If a product has zero external costs, then marginal social costs equal marginal private costs.

Answer: TRUE Topic: An Unregulated Market Skill: Conceptual*

5)

In an unregulated competitive market, the presence of marginal external cost of a good or service results in overproduction.

Answer: TRUE Topic: Efficiency Skill: Conceptual*

6)

When marginal social cost equals marginal social benefit resources are utilized efficiently.

Answer: TRUE

Answer: TRUE Topic: Taxes and External Costs Skill: Conceptual*

11) When external costs are present and government imposes a tax equal to the external marginal cost, then efficiency can be achieved. Answer: TRUE Topic: Marketable Permits Skill: Recognition*

12) The government can aid in reducing pollution by using marketable permits, which means that polluters are taxed on the amount of pollution they discharge. Answer: FALSE

294

CHAPTER 15

Topic: Marketable Permits Skill: Conceptual

Topic: An Unregulated Market Skill: Conceptual*

13) In the case of marketable permits, firms that have a greater marginal benefit from pollution will pollute less than firms which have a lower marginal benefit from pollution.

19) In an unregulated competitive market, the presence of marginal external benefit from a good or service results in overproduction.

Answer: FALSE Topic: Marginal Social Benefit Skill: Recognition

14) The marginal social benefit is the sum of the benefit enjoyed by the consumer of an additional unit of a good or service plus the marginal benefit enjoyed by others.

Answer: FALSE Topic: Positive Externalities: Education Skill: Recognition

20) In the case of an external benefit generated by an activity, such as education, a subsidy for that activity can achieve an efficient outcome. Answer: TRUE

Answer: TRUE

Topic: Education Skill: Conceptual

Topic: Subsidies Skill: Recognition*

21) Subsidizing education can lead an efficient level of production because education has external benefits.

15) One way that government can encourage activities that have external benefits is to subsidize the activity.

Answer: TRUE

Answer: TRUE

Topic: Education Skill: Conceptual

Topic: Government Policies Toward Education Skill: Recognition

22) The tuitions charged by state colleges are set below cost so that the state colleges can drive private colleges out of business.

16) In the case of an external benefit generated by an activity, such as education, an excise tax on that activity can lead to the efficient level of output being produced. Answer: FALSE Topic: An Unregulated Market Skill: Conceptual

17) When people make decisions about the quantity of education to undertake, they balance the marginal social benefit against the marginal cost. Answer: FALSE Topic: An Unregulated Market Skill: Conceptual

18) In an unregulated competitive market, the presence of marginal external benefit from a good or service results in less than the efficient quantity being produced. Answer: TRUE

Answer: FALSE Topic: Patents Skill: Conceptual

23) Patents stimulate research by allowing subsequent inventors to utilize patented inventions. Answer: FALSE Topic: Patents Skill: Conceptual*

24) Patents involve creating a temporary monopoly. Answer: TRUE

CHAPTER 16

PUBLIC GOODS AND COMMON RESOURCES

Topic: Rival Skill: Recognition*

Topic: The Free-Rider Problem Skill: Recognition*

1)

8)

If the consumption of a good decreases the consumption of the good by another person, the good is rival.

A free rider problem is created by private goods.

Answer: FALSE

Answer: TRUE

Topic: The Free-Rider Problem Skill: Conceptual

Topic: Nonexcludable Skill: Recognition*

9)

2)

If it is impossible or very costly to prevent someone from benefiting from a good even if the person does not pay for it, the good is nonrival.

Because of free riders, a private, unregulated market would not produce the efficient quantity of a public good.

Answer: TRUE

Answer: FALSE

Topic: Benefit of a Public Good Skill: Recognition

Topic: Public Goods Skill: Recognition

10) Society’s marginal benefit curve from a public good is found by horizontally summing the marginal benefit curves of all individuals.

3)

Cable television is an example of a public good.

Answer: FALSE

Answer: FALSE

Topic: Public Goods Skill: Recognition*

Topic: Efficient Quantity of a Public Good Skill: Conceptual

4)

11) The efficient quantity of a public good is the quantity that has the highest possible marginal benefit.

An example of a public good is national defense services.

Answer: TRUE Topic: Common Resource Skill: Recognition*

5)

A common resource is nonrival but excludable.

Answer: FALSE Topic: Free-Rider Problem Skill: Recognition*

6)

Free riding is possible if the good is nonexcludable.

Answer: TRUE Topic: The Free-Rider Problem Skill: Recognition

7)

A free rider is a person who consumes a good without paying for it.

Answer: TRUE

Answer: FALSE Topic: Efficient Quantity of a Public Good Skill: Conceptual

12) The efficient quantity of a public good is the quantity that sets the marginal benefit from the good equal to the good’s marginal cost. Answer: TRUE Topic: Rational Ignorance Skill: Conceptual

13) The concept of rational ignorance can explain why the government might provide more than the efficient quantity of a public good. Answer: TRUE

296

Topic: Rational Ignorance Skill: Conceptual

14) If a voter is ignorant of some of the issues that affect public policy, then the voter is definitely behaving irrationally. Answer: FALSE

CHAPTER 16

Topic: Marginal Private Benefit Versus Marginal Social Benefit Skill: Analytical

20) For a common resource, the marginal private benefit of the resource is greater than the marginal social benefit. Answer: TRUE

Topic: Rational Ignorance Skill: Conceptual

15) Because of voter rational ignorance, the government might not produce the efficient quantity of a public good.

Topic: Marginal Private Benefit Versus Marginal Social Benefit Skill: Analytical*

Answer: TRUE

21) For a common resource, the marginal private benefit of the resource is equal to the marginal social benefit.

Topic: Rational Ignorance Skill: Conceptual

Answer: False

16) If voters are rationally ignorant, then the interest of the bureaucracy to maximize its budget will result in an oversupply of a public good.

Topic: Efficient Use of the Commons Skill: Recognition*

Answer: TRUE

22) A common resource is used efficiently if marginal social benefit equals marginal cost. Answer: TRUE

Topic: Social interest Theory Skill: Recognition

17) Social interest theory suggests that the size and growth of government is inconsistent with voter preferences. Answer: FALSE Topic: Public Choice Theory Skill: Recognition*

18) Public choice theory predicts that governments make choices that result in inefficiency. Answer: TRUE Topic: Problem of the Commons Skill: Recognition*

19) The problem of the commons is the absence of incentives to prevent overuse and depletion of the common resource. Answer: TRUE

Topic: Efficient Use of the Commons Skill: Recognition*

23) One way of overcoming the problem of the commons is to make it private property. Answer: TRUE

CHAPTER 17

DEMAND AND SUPPLY IN FACTOR MARKETS

Topic: Overview of a Competitive Factor Market; Income Skill: Analytical

Topic: Equivalence of Two Conditions for Profit Maximization Skill: Conceptual

1)

7)

If the demand for labor in a particular industry is inelastic, then a decrease in the supply of labor increases incomes for workers in that industry.

Answer: TRUE Topic: Derived Demand Skill: Recognition*

2)

The demand for a good or service is called a derived demand because it is derived from the demand for the factors of production that produce this good or service.

If a firm maximizes profits by hiring enough labor so that MRP = W, it might produce a different level of output than if it had produced the amount of output so that MR = MC.

Answer: FALSE Topic: Changes in the Demand for Labor Skill: Conceptual

8)

An increase in the price of the output shifts the demand curve for labor leftward.

Answer: FALSE

Answer: FALSE Topic: Labor Market Skill: Recognition*

3)

For most people, capital markets are the major source of income.

Topic: Substitution Effect and Income Effect Skill: Conceptual

9)

As a person’s wage rate increases, the substitution effect motivates an increase in work and the income effect motivates a decrease in work.

Answer: FALSE

Answer: TRUE

Topic: Marginal Product Skill: Recognition

Topic: Substitution Effect and Income Effect Skill: Conceptual

4)

10) The labor supply curve is backward bending because at higher wages the income effect eventually dominates the substitution effect.

The marginal product of labor equals the marginal revenue product of labor times the price of the output produced.

Answer: FALSE

Answer: TRUE

Topic: Marginal Revenue Product Skill: Conceptual

Topic: Substitution Effect and Income Effect Skill: Conceptual

5)

11) If the income effect is larger than the substitution effect, then a wage hike will be accompanied by an increase in the quantity of labor supplied.

As the quantity of labor employed decreases, the marginal revenue product diminishes.

Answer: FALSE Topic: Labor Demand Curve Skill: Conceptual

6)

The demand curve for labor is negatively sloped only because the firm must lower its price if it wishes to sell more output.

Answer: FALSE

Answer: FALSE Topic: Income Effect Skill: Conceptual

12) With respect to labor supply, the income effect leads a person to want to work more in order to raise his or her income. Answer: FALSE

298

CHAPTER 17

Topic: Present Value Skill: Recognition

Topic: Supply of Renewable Natural Resources Skill: Conceptual

13) Discounting is converting a future amount of money to a present value.

19) The supply of a renewable resource is perfectly elastic.

Answer: TRUE

Answer: FALSE

Topic: Present Value Skill: Analytical

Topic: Supply of Nonrenewable Natural Resources Skill: Conceptual

14) If the interest rate is 7.5 percent, the present value of $500 to be received three years from today is $477.50.

20) The flow supply of a nonrenewable natural resource is perfectly inelastic at a quantity that equals the expected flow in the next period.

Answer: FALSE

Answer: FALSE

Topic: Demand Curve for Capital Skill: Conceptual

Topic: Resource Prices Skill: Recognition

15) An increase in the interest rate shifts the demand curve for capital leftward.

21) Over the past twenty years, nonrenewable resource prices have typically risen at a rate equal to the interest rate as predicted.

Answer: FALSE Topic: Supply of Capital Skill: Conceptual

16) If a fifty year old person is told that he will receive two million dollars as a gift when he retires, he will probably save less now than he would have otherwise. Answer: TRUE Topic: Renewable Resources Skill: Recognition

17) Land is an example of a renewable resource. Answer: TRUE Topic: Nonrenewable Resources Skill: Recognition

18) Land is an example of an nonrenewable resource. Answer: FALSE

Answer: FALSE Topic: Opportunity Cost Skill: Conceptual

22) A consultant who bills clients at $150 because that is what she can receive from other jobs and would not do the work for anything less is earning some economic rent. Answer: FALSE Topic: Economic Rent Skill: Conceptual

23) When a resource’s supply curve is upward sloping, the resource’s income is entirely economic rent. Answer: FALSE

APPENDIX 17

MARKET POWER IN THE LABOR MARKET

Topic: Craft Union Skill: Recognition

1)

A craft union is a group of workers with a similar range of skills.

Topic: How Unions Try to Change the Demand for Labor Skill: Conceptual

6)

Answer: TRUE Topic: Union Shops Skill: Recognition

2)

Union shops are outlawed in states that have right-to-work laws.

Answer: TRUE

Answer: FALSE Topic: Monopsony Skill: Conceptual

7) Topic: Open Shops Skill: Recognition

3)

In states with right-to-work laws, only open shop labor arrangements are legal.

Answer: TRUE Topic: Union Negotiations Skill: Recognition

4)

Collective bargaining is a process whereby the union and the employer allow a third party to determine wages and benefits.

Answer: TRUE Topic: Unions in a Competitive Labor Market Skill: Conceptual

5)

If a union cannot increase the demand for union labor, then an increase in union wages will result in a decrease in union employment.

Answer: TRUE

One reason unions might support an increase in the minimum wage is because such a policy would cause a leftward shift in the demand curve for union labor.

For a monopsony, the marginal cost of labor exceeds the wage rate because the firm must increase the wage it pays to all of its workers if it wishes to increase its employment.

Answer: TRUE Topic: Monopsony Labor Market Skill: Conceptual

8)

If a monopsony paid its workers a wage equal to their marginal revenue product, it would be maximizing its profit.

Answer: FALSE

CHAPTER 18

ECONOMIC INEQUALITY

Topic: Lorenz Curve Skill: Recognition

Topic: Distribution of Wealth Skill: Conceptual

1)

7)

A Lorenz curve graphs the difference between money income and market income.

Answer: FALSE Topic: Lorenz Curve Skill: Analytical

2)

An increase in income equality causes the Lorenz curve to become more bowed away from the line of equality.

The distribution of wealth is skewed because more families are below the average level of wealth than are above.

Answer: TRUE Topic: Trends in the Distribution of Income Skill: Recognition*

8)

If income is equally distributed, the Gini ratio is 1.

Answer: FALSE

Answer: FALSE

Topic: Measured Wealth Skill: Recognition

Topic: Trends in the Distribution of Income Skill: Recognition*

3)

9)

In the United States, income is more equally distributed than measured wealth.

If income is equally distributed, the Gini ratio is zero.

Answer: TRUE

Answer: TRUE

Topic: Measured Wealth Skill: Recognition

Topic: Trends in the Distribution of Income Skill: Recognition*

4)

10) If one person has all the income and everyone else has none, the Gini ratio is 1.

In the United States, measured wealth is distributed more unequally than income.

Answer: TRUE

Answer: TRUE

Topic: Measured Wealth Skill: Conceptual

Topic: Trends in the Distribution of Income Skill: Recognition*

5)

11) If one person has all the income and everyone else has none, the Gini ratio is zero.

The actual income distribution might be more equal than current official figures suggest because current official figures ignore the return on human capital.

Answer: FALSE

Answer: FALSE

Topic: Annual or Lifetime Wealth? Skill: Conceptual

Topic: Measured Wealth Skill: Conceptual

12) The data show that an individual’s wealth tends to first increase and then to decrease over the person’s lifetime.

6)

The income distribution is a more accurate measure of economic inequality than the wealth distribution because income excludes human capital.

Answer: FALSE

Answer: TRUE Topic: Who Are the Rich and the Poor? Skill: Recognition

13) The biggest single factor affecting household income is the householder’s age. Answer: FALSE

302

Topic: Demand for High-Skilled and Low-Skilled Labor Skill: Conceptual

14) The demand curve for high-skilled workers lies to the right of the demand curve for low-skilled workers because of the costs associated with obtaining skills. Answer: FALSE Topic: Demand for High-Skilled and Low-Skilled Labor Skill: Conceptual

CHAPTER 18

Topic: Assortative Mating Skill: Conceptual

18) Wealth inequality persists across generations due, in part, to assortative mating. Answer: TRUE Topic: Progressive Tax Skill: Recognition

19) A progressive tax is a tax that taxes income so that the average tax rate increases with income. Answer: TRUE

15) The demand curve for high-skilled workers lies to the right of the demand curve for low-skilled workers because of the higher productivity of high-skilled workers.

Topic: Proportional Tax Skill: Recognition

Answer: TRUE

Answer: TRUE

Topic: Demand for High-Skilled and Low-Skilled Labor Skill: Conceptual

Topic: Proportional Tax Skill: Recognition

16) Differences in human capital and discrimination can help explain economic inequality. Answer: TRUE Topic: Wage Rates of High-Skilled and Low-Skilled Labor Skill: Conceptual

17) The higher the marginal revenue product of a skill and the more costly it is to acquire the skill, the larger is the wage differential between high-skilled and low-skilled labor. Answer: TRUE

20) A proportional income tax is a tax that taxes income at a constant rate.

21) A proportional income tax is a tax for which the total amount paid does not increase with income. Answer: FALSE Topic: Market Income Skill: Recognition

22) Market income is defined as wages, interest, rent, and profit earned in factor markets plus cash payments to households by the government. Answer: FALSE Topic: Big Tradeoff Skill: Conceptual

23) “The Big Tradeoff” refers to the effect of redistribution on efficiency. Answer: TRUE

CHAPTER 19

UNCERTAINTY AND INFORMATION

Topic: Utility of Wealth Skill: Recognition

Topic: Risk Aversion Skill: Conceptual

1)

8)

A risk averse person has diminishing marginal utility of wealth.

Answer: TRUE Topic: Utility of Wealth Curve Skill: Recognition*

2)

The utility of wealth curve of a risk-neutral person is a straight line.

All people are risk averse.

Answer: FALSE Topic: Optimal-Search Rule Skill: Recognition*

9)

The optimal-search rule is: search for a lower price until the expected marginal benefit of additional search is greater than the marginal cost of search.

Answer: TRUE

Answer: FALSE

Topic: Utility of Wealth Curve Skill: Recognition*

Topic: Optimal-Search Rule Skill: Recognition*

3)

10) A buyer’s reservation price is the lowest price that the buyer is willing to pay for a good.

The utility of wealth curve of a risk-neutral person is a horizontal line.

Answer: FALSE

Answer: FALSE

Topic: Utility of Wealth Curve Skill: Recognition*

Topic: Minimum Cost of Insurance Skill: Conceptual

4)

11) A risk averse person is willing to pay more than the minimum cost of insurance.

The utility of wealth curve of a risk-averse person is a horizontal line.

Answer: FALSE

Answer: TRUE

Topic: Expected Utility Skill: Conceptual*

Topic: Gains From Insurance Skill: Conceptual

5)

12) A risk averse person will always buy insurance against risk.

Expected utility is the utility that arises from expected wealth.

Answer: FALSE

Answer: FALSE

Topic: Cost of Risk Skill: Conceptual*

Topic: Moral Hazard Skill: Recognition

6)

13) Moral hazard occurs when an agreement encourages undesirable behavior.

The cost of risk is the same for everyone.

Answer: FALSE Topic: Risk Aversion Skill: Conceptual*

7)

Most people are risk neutral.

Answer: FALSE

Answer: TRUE Topic: Moral Hazard Skill: Conceptual

14) If reckless drivers are more likely than safe drivers to buy automobile insurance, then a moral hazard problem has occurred. Answer: FALSE

304

CHAPTER 19

Topic: Adverse Selection Skill: Recognition

Topic: Diversification Skill: Recognition

15) Adverse selection occurs when a sales offer attracts the kinds of customers that the seller does not want.

18) Diversification means buying a variety of assets. Answer: TRUE

Answer: TRUE

Topic: Efficient Market Skill: Recognition

Topic: Adverse Selection Skill: Conceptual

19) A market in which the actual price embodies all currently available relevant information is called an efficient market.

16) If insured drivers decide to drive more recklessly than uninsured drivers do, then an adverse selection has occurred.

Answer: TRUE

Answer: FALSE

Topic: Efficient Market Skill: Conceptual

Topic: Diversification Skill: Recognition

20) In an efficient market, changes in price are easily predicted.

17) Diversification benefits investors by reducing the probability of extreme outcomes.

Answer: FALSE

Answer: TRUE

CHAPTER 20

TRADING WITH THE WORLD**

Topic: Patterns and Trends in International Trade Skill: Recognition

Topic: Comparative Advantage Skill: Conceptual*

1)

7)

Most of U.S. exports are services.

Answer: FALSE

While a nation can have a comparative advantage producing some goods it cannot have an absolute advantage at producing all goods.

Topic: Patterns and Trends in International Trade Skill: Recognition

Answer: FALSE

2)

Topic: Gains from Trade Skill: Conceptual

The region in which the United States trade is largest is Latin America.

Answer: FALSE Topic: Patterns and Trends in International Trade; Balance of Trade Skill: Recognition

3)

The value of exports minus the value of imports is the balance of trade.

Answer: TRUE Topic: Patterns and Trends in International Trade; Balance of Trade Skill: Recognition*

4)

Over the past 70 years, the United States has reduced its average tariff rate so today it is less than 5 percent.

Answer: TRUE Topic: Comparative Advantage Skill: Recognition

5)

A country has a comparative advantage in producing a good if it can produce that good at a lower opportunity cost than any other country.

Answer: TRUE Topic: Comparative Advantage Skill: Conceptual

6)

Suppose there are only two countries and each produces only the same two goods. If one country has a comparative advantage in one good, then the other country must have a comparative advantage in the other good.

Answer: TRUE *

* This is Chapter 33 in Economics.

8)

Suppose two countries produce two goods with different opportunity costs. If these two countries trade, both can gain from the trade.

Answer: TRUE Topic: Gains from Trade Skill: Conceptual

9)

Suppose two countries produce two goods. If one country has absolute advantages in both goods, free trade between the two countries means that only one of the countries can gain from the trade.

Answer: FALSE Topic: Gains from Trade Skill: Conceptual

10) Free trade between two countries will shift each country’s production possibilities frontier outward and this is the gain from trade. Answer: FALSE Topic: Gains from Trade Skill: Conceptual

11) With international trade, the production possibility frontier of the exporting country shifts outward. Answer: FALSE

306

Topic: Gains from Trade; Consumption Possibilities Skill: Conceptual

12) By trading, countries are able to consume at a point beyond their production possibilities frontier.

CHAPTER 20

Topic: Trade Restrictions; Quota Skill: Conceptual

18) Quotas are less damaging to an economy than are tariffs. Answer: FALSE

Answer: TRUE

Topic: The Case Against Protection; Dumping Skill: Recognition

Topic: Gains from Trade; Consumption Possibilities Skill: Conceptual

19) Dumping occurs when a foreign firm sells its exports at a lower price than it costs to produce them.

13) Typically, when two countries trade, the country with the lower opportunity cost is the only country to obtain any of the gains from trade.

Answer: TRUE

Answer: FALSE

Topic: The Case Against Protection; Saving Jobs Skill: Conceptual

Topic: Terms of Trade Skill: Conceptual

Answer: FALSE

14) The terms of trade are the rate at which one good is traded for another. Answer: TRUE Topic: Gains from Trade in Reality; Economies of Scale Skill: Conceptual

15) One reason countries may both export and import some of the same goods is because of economies of scale. Answer: TRUE Topic: Trade Restrictions; Effects of a Tariff Skill: Conceptual

16) As a result of an increase in tariffs, imports decrease and government revenue increases. Answer: TRUE Topic: Trade Restrictions; Effects of a Tariff and a Quota Skill: Conceptual

17) Tariffs and quotas both decrease the amount of a good consumed and raise the price paid by domestic residents for the good. Answer: TRUE

20) Most economists would agree that “saving jobs” is a valid reason for restricting trade. Topic: The Case Against Protection; Infant Industry Skill: Conceptual

21) The infant-industry argument is the only perfectly valid argument for protection. Answer: FALSE Topic: Why Is International Trade Restricted?; Tariff Revenue Skill: Conceptual

22) Less developed countries, compared to industrialized ones, are more likely to have higher tariff rates. Answer: TRUE

EXTENDED PROBLEMS

CHAPTER 1

WHAT IS ECONOMICS?

Extended Problems 1)

Suppose you plan to go to school this summer. The cost of tuition and textbooks is $1,400 and housing, board, and entertainment will cost you $500. If you didn’t go to school, you’d live in your parents’ house for free, but your other living expenses would be about the same. Also, if you didn’t go to school you’d work full time and could earn $8,000. You can still work part time while attending the summer school, but you will earn only $3,000. a) What will the summer school cost you in terms of money explicitly paid? b) What are the opportunity costs of going to summer school that you don’t pay explicitly? Explain. c) What is your total opportunity costs of going to school this summer? Explain your answer.

2)

a) b) c) d) e)

Jane is deciding whether to go to school for 8 weeks this summer. The cost of tuition and textbooks is $1,700 and housing and other expenses will cost her $600. If she does not go to school, she will live in her parents’ house for free and they will cover her food and other expenses for her. Also, if Jane does not go to summer school she could work fulltime. But the best job she can get pays only $600 per week, and Jane would only agree to give up her free time for no less than $750 per week. However, if she goes to summer school, she’ll have to spend 40 hours a week attending classes and studying. What will the summer school cost Jane in terms of money spent? What are the opportunity costs of going to summer school that Jane does not pay explicitly? Explain. What is Jane’s total opportunity cost of going to school this summer? Explain your answer. Suppose that if Jane does not go to summer school, she will eventually take the classes anyway. What is Jane’s marginal benefit of going to summer school? Suppose Jane decides to go to school in the summer. Explain her decision using the concepts of marginal cost and marginal benefit.

3)

Suppose Canon Inc. decided to invest 45 billion yen in developing and launching a new model of its digital camera, expecting that it will bring additional sales of 60 billion yen. The company has already invested 38 billion yen when the marketing department suddenly finds out that the introduction of a similar camera by Sony will reduce Canon’s expected additional sales to 30 billion yen. The company’s management is trying to decide whether to continue investing in the new product or close the project. Canon hires you as an economic consultant. So, think like an economist to help the company’s management make their decision: a) At this point in time, what is Canon’s marginal cost of introducing the new product? b) What is Canon’s marginal benefit from introducing the new product? c) Will you advise Canon to finish the project and introduce the new product? Why or why not? What principles of economic thinking will help you analyze the situation and make the right choice?

309

310

CHAPTER 1

4)

Your student association is looking for an auditorium to rent for an all-day conference. The university’s Performing Arts Center is vacant on that day, so the association wants to rent it. The physical plant manager tells you that the daily rent is $660, which includes $400 to cover part of the cost paid to build the Center, $40 to cover part of its regular maintenance cost, $50 to help pay for the building’s insurance, $100 to cover the extra cost of electricity that the university would incur because of the conference, and $70 to pay for additional janitorial services for the conference. You know that no one else wants to rent the Center on that day and you think that the price that the manager charges is too high. But how much should you pay? Use the economic way of thinking to answer this question and to convince the manager to accept your offer: a) If you rent the Center, what will be the university’s marginal cost of renting the center to you? b) If you rent the Center, what will be the university’s marginal benefit of renting the center to you? c) What amount of rent should you offer? Convince the manager to accept your offer.

Answers: 1a) You explicitly pay the cost of tuition and textbooks ($1,400) and the cost of housing ($500), so your total explicit costs are $1,900. 1b) Your opportunity cost is what your give up to go to summer school. You forego a fulltime job, at which you would earn $8,000, in exchange for a part-time job, where you earn $3,000, which means you give up $5,000. Although you don’t pay this money explicitly, you lose the opportunity to earn it and so it’s an opportunity cost of attending summer school. 1c) First, your opportunity cost includes the cost that you pay explicitly ($1,900), which you have to pay only if you go to school. If you decide not to go to school, you can use this money to buy something else—an opportunity you are giving up. Second, as explained in the previous part, you are also giving up $5,000, although not paying this money explicitly. So your full opportunity cost of going to school is $1,900 + $5,000 = $6,900. 2a) Jane explicitly pays the cost of tuition and textbooks ($1,700) and the cost of housing ($600), so her total explicit costs are $2,300. 2b) Jane’s opportunity cost is what she gives up (her best alternative forgone) to go to summer school. In this case she foregoes 40 hours per week of her free time, which she values more highly then the income from the best job she could find. To place a dollar value on this time, notice that the value that she places on this time is the amount of money she is willing to accept to give it up: $750 per week. So for the eight weeks, her free time has a value of $750×8 = $6,000. Although she does not pay $6,000 explicitly, she gives up this value of her free time and hence it’s an opportunity cost to her. 2c) Jane’s total opportunity cost includes the cost that she pays explicitly, $2,300, which she has to pay only if she goes to school. Also, as explained in the previous part, Jane is giving up the value of her free time, $6,000. Thus, her full opportunity cost of going to school is $2,300 + $6,000 = $8,300. 2d) Jane’s marginal benefit of going to summer school is the possibility of getting her degree faster. For instance, if the summer school allows her to graduate one semester earlier, she can start to work and earn income earlier. The additional income and work experience that she gets because of her earlier graduation is what she gains if she decides to go to school in the summer. 2e) Jane decides to go to school in the summer if her marginal benefit from this decision, the value of extra income and work experience that she gets if she graduates earlier, is greater than the marginal cost of her summer school, $8,300. 3a) Canon’s marginal cost is the additional investment needed to finish the project, which is 7 billion yen. 3b) Canon’s marginal benefit is the benefit that arises from the new product, the additional revenue from sales, which in the changed situation is expected to be 30 billion yen.

WHAT IS ECONOMICS?

311

3c) The principle of choosing at the margin will help. According to this principle, the amount of money already spent is irrelevant to the decision you are making now. That is, you should only consider the marginal costs and marginal benefits that will result from the decision in question. Now, if Canon goes ahead, finishes the project and introduces the new camera, it will cost them additional 7 billion yen, but they will gain additional sales of 30 billion yen. The marginal benefit of introducing the new product exceeds the marginal cost, which means the company should invest 7 billion yen to finish development and introduce the new product. Notice also that if Canon abandons the project, there will be no additional money costs, but the opportunity cost will be the additional sales (30 million yen) that the company is potentially losing. Thus, the concept of opportunity cost also helps to clarify the situation. 4a) The university’s marginal cost is $170. These are the extra cost of electricity ($100) and janitorial services ($70) that the university will only pay if you rent the Center. The costs of building the Center, insurance, and regular maintenance costs are not extra costs incurred because you rent the Center. The university has already paid for building it and pays the cost of insurance and regular maintenance no matter whether you rent the Center or not. Therefore these costs are not marginal costs of renting the center to you. 4b) The university’s marginal benefit is the amount of rent that you pay. 4c) You should start negotiating from $171. Because the university’s marginal cost is $170 and the amount you pay is its marginal benefit, the university will be better off if it accepts any amount greater than $170. If the manager is still not convinced, tell the manager that, since no one else wants to rent the Center on that day, declining your offer is not cost free. The opportunity cost of not accepting it will be the difference between the offered rent and $170. In practice, of course, there are transaction costs, such as the time spent by both parties to negotiate and sign the agreement, and accepting your offer will cost the manager some extra time and organizational effort. Also, as you learnt in this chapter, people are guided by self interest when they make their decisions and the manager’s self interest is not necessarily the same as the university’s interest. On the other hand, you might want to support your university. Therefore the amount of rent you will agree upon is likely to be higher than $171.

CHAPTER 2

Extra study time (hours per week) 10 8 6 4 2 0 1)

a) b) c) d) e) f)

Quidditch (hours per week) 0 2 4 6 8 10

THE ECONOMIC PROBLEM

Harry’s grade (points) 90 87 82 75 66 55

Quidditch (hours per week) 1 3 5 7 9

Marginal benefit of an extra hour (points) 3.0 2.5 2.0 1.5 1.0

Harry usually spends 10 hours a week playing Quidditch. However, his Defense Against the Dark Arts exam is coming and he needs more time to study for the exam. If he does not study at all, the lowest grade he will receive is 55. But Harry realizes that this grade is unacceptable and so he has to give up some of his Quidditch time. But how much? He asks his friend Hermione, who is very good at Magical Economics, to help him figure this out. Together, they come up with the schedules of Harry’s possibilities and values in the tables above. Then Hermione quickly finds the solution. But Harry still looks puzzled. Help Hermione to explain her solution to Harry. Draw Harry’s production possibilities frontier with the Quidditch hours on the horizontal axis and Harry’s grade, starting at 55, on the vertical axis. What is the opportunity cost of the first two hours of Quidditch? What is the marginal cost of the 5th hour? What is Harry’s opportunity cost of raising his grade from 82 points to 87 points? Draw Harry’s marginal cost of playing Quidditch curve. What happens to the marginal cost if Harry spends more time playing Quidditch? Draw Harry’s marginal benefit from playing Quidditch curve. Describe the relationship between Harry’s time spent playing Quidditch and the marginal benefit from playing Quidditch. For how many extra hours did Hermione recommend Harry to study? Why? If Harry follows her advice, what grade will he expect? What do you think Hermione said when Harry asked why he should not spend more time playing Quidditch?

314

CHAPTER 2

Combination A B C D E F G 2)

a) b) c) d) e) 3)

a) b) c) d)

Wine (thousands of bottles per year) 0 2 4 6 8 10 12

Bread (thousands of loaves per year) 21 20 18 15 11 6 0

Wine (thousands of bottles per year) 1 3 5 7 9

Willingness to pay (loaves of bread per bottle of wine) 2.5 2.0 1.5 1.0 0.5

The Hobbits of the Shire are trying to decide how much bread and how much wine to produce. They ask you to be their economic consultant and give you the information in the first table above about different combinations of wine and bread that they can produce if they are fully employed and doing their best. The Hobbits also give you the information in the second table above about their willingness to pay for wine depending on how much wine they already have. To help the Hobbits solve their problem: Draw the Shire’s production possibilities frontier. Put wine on the horizontal axis. What is the opportunity cost of the first two thousand bottles of wine? What is the marginal cost of the 3000th bottle of wine? What is the marginal cost of the 3,000th loaf of bread? Draw the marginal cost of wine curve. What happens to the marginal cost if production of wine increases? Why? Draw the marginal benefit from wine curve on the same figure on which you put the marginal cost curve. Describe the relationship between the quantity of wine produced and the marginal benefit from wine. What combination of bread and wine will you recommend the Hobbits to produce? Why? Explain to the Hobbits why they would be worse off by producing a different combination of bread and wine. Sarah and her boyfriend Mike want to save some money to pay for their wedding. So they decided to help people in the neighborhood by cleaning their garages and mowing lawns. Sarah takes 60 minutes to clean a garage and 80 minutes to mow a lawn. Mike takes 80 minutes to clean a garage and 80 minutes to mow a lawn. Sarah and Mike devote 10 hours per week each to these activities and get paid $25 for each garage they clean and $25 for each lawn they mow. Sarah says to Mike: “I have an absolute advantage in cleaning and we are equally productive in mowing. Therefore I should do both cleaning and mowing but you should only mow lawns.” Mike disagrees. He thinks Sarah should specialize in cleaning garages and he should specialize in mowing lawns. Help Sarah and Mike to resolve their dispute. Who has an absolute advantage in cleaning garages? In mowing lawns? Explain. Draw Sarah’s and Mike’s production possibilities frontiers. What are each individual’s opportunity costs? Who has a comparative advantage in cleaning garages? In mowing lawns? Explain. Is Sarah right when she says that she should do both cleaning and mowing while Mike should only mow lawns? Or may be Mike is right when he suggests that Sarah specializes in cleaning and he specializes in mowing? Illustrate and substantiate your answer with a numerical example.

THE ECONOMIC PROBLEM

315

Answers:

1a) The figure above shows Harry’s production possibilities frontier. 1b) Harry’s opportunity cost of the first two hours of Quidditch is three points off his grade. According to the PPF, if Harry does not play Quidditch at all, the best grade he can get is 90 points, but if he plays Quidditch two hours a week, his grade falls to 87 points. So Harry gives up three points of his grade to get his first two hours of Quidditch. When Harry increases his Quidditch time from 4 hours to 6 hours, he gives up 7 points of his grade, which means each additional hour costs him 3.5 points. So the marginal cost of the 5th hour is 3.5 points. 1c) If Harry wants to raise his grade from 82 points to 87 points, he must give up 2 hours of Quidditch by playing 2 hours per week instead of 4 hours. So the opportunity cost of these 5 extra points is 2 hours of Quidditch.

1d) To draw Harry’s marginal cost curve, we calculate the marginal costs of the 1st, 3rd, 7th, and 9th hours of Quidditch the same way we calculated the marginal cost of the 5th hour. The marginal cost curve is shown in the figure above as MC. Harry’s marginal cost of playing Quidditch increases as he spends more time playing.

316

CHAPTER 2

1e) Harry’s marginal benefit from playing Quidditch is measured by the value of an extra hour of Quidditch in terms of Harry’s grade. The marginal benefit curve is graphed in the figure above using the data in the second table. The relationship between Harry’s time spent to play Quidditch and the marginal benefit from playing is an inverse relationship. 1f) Hermione recommended to Harry that he study 7 extra hours a week, which leaves him only 3 hours per week to play Quidditch. The figure above shows that if Harry plays Quidditch 3 hours a week, the marginal cost of playing Quidditch equals the marginal benefit (Hermione calls this allocative efficiency). If Harry follows Hermione’s advice, his grade will be 84.5 points. When Harry asked why he should not spend more time playing Quidditch, Hermione (who is a magical economist) probably said: “If you play for one more hour, your marginal cost will be 3 points off your grade, whereas your marginal benefit from this hour is only 2.25 points. I mean, the value that you get from your fourth hour of Quidditch is lower then the cost you pay for it, so you are worse off by spending more then three hours a week on Quidditch. This isn’t magic, it’s pure economics. Only the greatest wizards truly understand economics…”

2a) The figure above shows the Shire’s production possibilities frontier. 2b) As the PPF shows, if the Hobbits produce no wine, they can produce 21,000 loaves of bread, but if they want to produce 2,000 bottles of wine, they can only produce 20,000 loaves of bread. The 1,000 loaves of bread that they have to give up to produce 2,000 bottles of wine is their opportunity cost of producing the first 2,000 bottles of wine, so the opportunity cost of the first 2,000 bottles of win is 1,000 loaves of bread. When the Hobbits increase their production of wine from 2,000 to 4,000, they give up 2,000 loaves of bread, which means each additional bottle of wine costs them 1 loaf of bread. The 3,000th bottle of wine is midway between 2,000 and 4,000, so the marginal cost of the 3,000th bottle is 1 loaf of bread. Similarly, when the Hobbits increase their production of bread from zero to 6,000, they give up 2,000 bottles of wine, or 1/3 of a bottle per loaf. The 3,000th loaf is in the middle of 0 and 6,000 loaves, so the marginal cost of the 3,000th loaf of bread is 1/3 of a bottle of wine.

THE ECONOMIC PROBLEM

317

2c) To draw the marginal cost curve, we calculate the marginal costs of the 1,000th, 5,000th, 7,000th, 9,000th, and 11,000th bottles of wine the same way we calculated the marginal cost of the 3,000th bottle. The marginal cost curve is shown in the figure above as the curve labeled MC. The marginal cost of wine is increasing because all Hobbits are not equally productive in both activities. Some Hobbits are good at producing bread but not very good at producing wine. So if they stop producing bread and start to produce wine instead, the Shire will lose a large quantity of bread for a small quantity of additional wine, which means the cost of an additional bottle of wine in terms of bread, or the marginal cost of wine, will increase. 2d) The marginal benefit of wine is measured by what the Hobbits are willing to pay for an additional bottle. So, the marginal benefit curve can be graphed using the willingness to pay data in the table and is the curve in the figure labeled MB. The relationship between the quantity of wine produced and the marginal benefit from wine is an inverse relationship. If wine is relatively scarce, the Hobbits place a high value on it. But if they already have plenty of wine, they are not likely to be willing to pay a high price to get an extra bottle. 2e) You recommend that the Hobbits produce 5,000 bottles of wine and 16,500 loaves of bread. The figure shows that if the Shire produces 5,000 bottles, the marginal cost of wine equals the marginal benefit, and the Hobbits will achieve allocative efficiency. If the Hobbits produce 5,000 bottles of wine, they will have just enough resources (be on their PPF) to produce 16,500 loaves of bread. If the Hobbits produce a different combination of these two goods, they will be worse off. For example, if they produce 4,000 bottles of wine and 18,000 loaves of bread, the marginal benefit from wine is 1.75 loaves of bread while the marginal cost of wine is only 1.25 loaves. In this case, the Hobbits value an additional bottle of wine more than it costs them to produce it. So they can get more value from their resources by producing additional wine. On the other hand, if the Hobbits produce 6,000 bottles of wine, each additional bottle after the 5000th will cost them more than the value they place on it. The Hobbits will be better off if they decrease their production of wine and increase their production of bread. 3a) Sarah has an absolute advantage in cleaning as she is more productive in this activity. She can clean 10 garages per week while Mike can only clean 7.5 garages. Sarah and Mike are equally productive in mowing lawns (each can mow 7.5 lawns per week) and therefore none of them has an absolute advantage in this activity.

318

CHAPTER 2

3b) Sarah’s and Mike’s PPFs are shown in the figure above. If Sarah works 10 hours a week, she can either mow 7.5 lawns or clean 10 garages. If she mows 7.5 lawns, she gives up 10 garages, which means her opportunity cost of one lawn is 1.33 garages (10 ÷ 7.5). If she cleans 10 garages, she gives up 7.5 lawns and her opportunity cost is 0.75 lawns per garage (7.5 ÷ 10). Mike can either mow 7.5 lawns or clean 7.5 garages, so his opportunity costs are 1 lawn per garage and 1 garage per lawn. 3c) Sarah has a comparative advantage in cleaning garages. Her opportunity cost of cleaning a garage (0.75 lawns) is lower than Mike’s (1 lawn). Mike has a comparative advantage in mowing lawns. His opportunity coast of mowing a lawn (1 garage) is lower than Sarah’s (1.33 garages). 3d) Sarah is wrong and Mike is right. Individuals can gain if they specialize in the activities where they have a comparative advantage. Therefore Sarah should specialize in cleaning while Mike should specialize in mowing. A simple numerical example can show how Mike and Sarah are better off if they specialize this way. Suppose first that Sarah and Mike specialize as Sarah suggests. For example, Sarah spends 8 hours per week cleaning garages and 2 hours per week mowing lawns. Then, she will clean 8 garages and move 1.5 lawns per week. Mike spends all his time mowing lawns and mows 7.5 lawns per week. Together they will clean 8 garages and move 9 lawns, earning $425 ($25 × 8 garages + $25 × 9 lawns). Now, suppose Sarah and Mike specialize as Mike suggests. Then, Sarah will clean 10 garages a week while Mike will mow 7.5 lawns a week, so they will earn $437.5 ($25 × 10 garages + $25 × 7.5 lawns).

CHAPTER 3 1)

a) b) c) d) e)

Using supply-and-demand diagrams, show and explain the effects of the following events on the price of CDRs and the quantity of CD-Rs sold. For each event, identify which of the determinants of demand or supply is affected, how it influences demand or supply, and what happens to the equilibrium price and quantity. The price of a CD burner falls. Workers who make CD-Rs get a pay raise. Producers introduce new cost-saving technologies in their CD-R production plants. Consumers’ incomes increase and CD-Rs are a normal good. Free peer-to-peer music exchange through the Internet becomes legal.

Price (cents per gallon) 90 100 110 120 130 140 150 2)

DEMAND AND SUPPLY

Quantity demanded (thousands of gallons per week) 80 70 60 50 40 30 20

Quantity supplied (thousands of gallons per week) 20 30 40 50 60 70 80

A market research team has come up with the demand and supply schedules for gasoline in Motorville in the table above. Use these data to analyze the situation in the market for gas in Motorville. a) Draw a figure showing the demand curve for gasoline and the supply curve of gasoline. What are the equilibrium price and quantity? b) Suppose the price is $1.30. Describe the situation in the market and explain how the market adjusts. Now suppose the price is $1.00. Describe the situation in the market and explain how the market adjusts. c) The market research report also predicts that a rise in the price of crude oil will decrease the quantity of gas supplied by 20,000 gallons a week at each price. Suppose the price of crude oil does rise. Use your figure to show how this will affect the market for gas. How will the market adjust? What will be the new equilibrium price and quantity?

320

CHAPTER 3

Price (dollars per pizza) 5 6 7 8 9 10 11 12

Quantity demanded (pizzas per week) 750 700 650 600 550 500 450 400

Quantity supplied (pizzas per week) 300 400 500 600 700 800 900 1,000

3)

A market research team has come up with the demand and supply schedules for pizza in Cheeseboro. These schedules are given in the table above. Use these data to analyze the situation in the market for pizza. a) Draw a figure showing the demand curve for pizza and the supply curve of pizza. What are the equilibrium price and quantity? b) Suppose the price is $10. Describe the situation in the market and explain how the price of pizza adjusts. Now suppose the price is $6. Describe the situation in the market and explain how the price of pizza adjusts. c) The market research report also includes a prediction about the effect on the market for pizza in Cheeseboro of a recent news published in Cheeseboro Herald. The Herald reported that pizza has been discovered to help prevent heart diseases. Unfortunately, your dog chewed up the report and all you can read about the prediction is “quantity… by 150 at each price.” What does the prediction say? Use your graph to show the predicted effects on the market for pizza. What are the predicted equilibrium price and quantity? How will the market adjust?

Answers:

1a) The figure above shows the effect of fall in the price of CD burner. CD burners and CD-Rs are complementary goods in consumption. So when the price of a CD burner falls, the demand for CD-Rs increases and the demand curve shifts rightward. As a result, the equilibrium price rises, from P0 to P1 and the equilibrium quantity increases from Q0 to Q1.

DEMAND AND SUPPLY

321

1b) The figure above shows the effect of rise in the wage page workers who make CD-Rs. In this case, the price of a resource (labor) used to produce CD-Rs rises. So the supply of CD-Rs decreases and the supply curve shifts leftward. As a result, the equilibrium price rises from P0 to P1 and the equilibrium quantity decreases from Q0 to Q1.

1c) The figure above shows the effect of a technological advancement in the production of CD-Rs. The new technology increases the supply of CD-Rs, shifting the supply curve rightward. As a result, the equilibrium price falls from P0 to P1 and the equilibrium quantity increases from Q0 to Q1.

322

CHAPTER 3

1d) The figure above shows the effect of an increase in income. Because CD-Rs are a normal good, an increase in income increases the demand for CD-Rs and the demand curve shifts rightward. As a result, the equilibrium price rises, from P0 to P1 and the equilibrium quantity increases from Q0 to Q1.

1e) The figure above shows the effect of making free peer-to-peer downloading of music legal. Making free peerto-peer music downloading legal leads to more downloads and more people copying music files to CDs. Therefore, the demand for CD-Rs increases and the demand curve shifts rightward. As a result, the equilibrium price rises, from P0 to P1 and the equilibrium quantity increases from Q0 to Q1.

DEMAND AND SUPPLY

323

2a) See the figure above. The equilibrium price is $1.20 and the equilibrium quantity is 50,000 gallons per week. 2b) If the price is $1.30, there is a surplus of 20,000 gallons. The surplus leads to a fall in the price. The lower price increases the quantity demanded and decreases the quantity supplied bringing the market to equilibrium at $1.20. If the price is $1.00, there is a shortage of 40,000 gallons. The shortage leads to a rise in the price. The higher price decreases the quantity demanded and increases the quantity supplied bringing the market to equilibrium at $1.20.

2c) The figure above shows the effect of the rise in crude oil prices. This rise decreases the supply and the supply curve shifts leftward by 20,000 gallons at each price from S0 to S1. As a result, at the old price, $1.20, there is a shortage of 20,000 gallons of gasoline. Therefore the price rises, bringing the market to equilibrium at $1.30 with 40,000 gallons of gas sold.

324

CHAPTER 3

3a) See the figure above. The equilibrium price is $8 and the equilibrium quantity is 600 pizzas. 3b) If the price is $10, there is a surplus of 300 pizzas. The surplus leads to a fall in the price. The lower price increases the quantity demanded and decreases the quantity supplied bringing the market to equilibrium at $8, where the quantity demanded equals the quantity supplied. If the price is $6, there is a shortage of 300 pizzas. The shortage leads to a rise in the price. The higher price decreases the quantity demanded and increases the quantity supplied bringing the market to equilibrium at $8.

3c) The report predicts that the news will change consumer preferences in favor of pizza, which will “increase the quantity of pizza demanded by 150 at each price.” The increase in demand means the demand curve shifts rightward by 150 pizza at every price, as illustrated in the above figure by the shift from D0 to D1. The equilibrium price is $9 and quantity is 700. When the demand curve shifts, at the old price, $8, there is a shortage of 150 pizzas and therefore the price rises bringing the market to equilibrium at $9.

CHAPTER 4

ELASTICITY

Price (dollars per ride) 3 4 5 6 7 1)

a) b) c) d) e) 2)

Quantity demanded (rides per month) 160 120 80 40 0

The number of taxicabs in Motorville and the taxicab fares are regulated. The fare currently charged is $5 a ride. Motorville taxicab drivers want to obtain government’s permission to raise the fare to increase their revenues and ask you to be their economic adviser. After studying the market, you come up with the following demand schedule for taxicab rides: Calculate the price elasticity of demand for taxicab rides as the fare rises from $5 to $6. (Use the midpoint method in your calculations.) Is the demand price elastic or inelastic for this fare rise? What happens to the taxicab drivers’ total revenue if the fare rises from $5 to $6? How can you use your answers in part a to answer this question? Should the drivers try to obtain permission to raise the fare? Calculate the price elasticity of demand for taxicab rides as the fare falls from $5 to $4. (Use the midpoint method in your calculations.) Is the demand price elastic or inelastic for this fare decrease? What happens to the taxicab drivers total revenue if the fare falls from $5 to $4? How can you use your answers in part c to answer this question? Should the drivers try to obtain permission to lower the fare? What fare will maximize the taxicab drivers total revenue? Explain.

The cab fare in Horseville is regulated. Recently, the government decided to raise it from $2.00 to $2.50 per ride. After this rise in fare, cab ridership decreased by 10 percent. a) What is the price elasticity of demand for cab rides in Horseville? (Use the midpoint formula to calculate the percentage change in the price.) Is the demand for rides elastic or inelastic? b) According to your estimate, what happened to the cab drivers’ revenue after the fare rose? Explain. c) Why might your estimate of elasticity be inaccurate?

326

3)

a) b) c) d)

CHAPTER 4

Price Quantity demanded, 2003 Quantity demanded, 2004 (dollars per CD) (CDs per year) (CDs per year) 4 40 70 8 30 50 12 20 30 16 10 10 18 5 0 20 0 0 Cory enjoys listening to all types of music. His demand schedules for CDs in 2003 and 2004 are shown in the table above. Cory’s income was $20,000 in 2003 and $25,000 in 2004. In 2004, he signed up for a new paid music service, which allowed him to download music files from the Internet. Draw Cory’s demand curves for CDs in 2003 and 2004. How did Cory’s demand for CDs change? Why? For each year, calculate Cory’s price elasticity of demand as the price of CDs decreases from $16 to $12. (Use the midpoint method in your calculations.) Why might the price elasticity of Cory’s demand for CDs in 2004 be different from that in 2003? If all CD buyers have the same demand as does Corry, what price should the record companies charge in 2004 to maximize their total revenue from CD sales?

Price (dollars per ticket) 0 100 200 300 400 500 4) a) b) c) d)

Quantity demanded, business travelers (tickets per week) 1,800 1,600 1,400 1,200 1,000 800

Quantity demanded, leisure travelers (tickets per week) 1,600 1,200 800 400 0 0

Suppose that business travelers and leisure travelers have the demand schedules for airline tickets from Atlanta to Philadelphia given in the table above. Draw both demand curves. As the price of tickets rises from $200 to $300, what is the price elasticity of business travelers’ demand? What is the price elasticity of leisure travelers’ demand? (Use the midpoint method in your calculations.) Why do business travelers and leisure travelers have different price elasticities of demand for airline tickets? At what price is the price elasticity of demand equal to 1 for business travelers? For leisure travelers? Explain.

Answers: 1a) When the fare rises from $5 to $6, the number of rides decreases from 80 to 40. The percentage change in the quantity demanded is the change in the number of rides, 40, divided by the average number of rides, 60, multiplied by 100 or (40 ÷ 60) × 100, which is 66.7 percent. The percentage change in the price is the change in the price, $1, divided by the average price, $5.50, multiplied by 100, or ($1 ÷ $5.50) × 100, which is 18.2 percent. So the price elasticity of demand is 66.7% ÷ 18.2%, which equals 3.67. The demand is elastic because the elasticity, 3.67, exceeds 1. 1b) A direct calculation shows that the total revenue decreases from $400 ($5 × 80) to $240 ($6 × 40). In part a, we calculated that the demand is elastic. When the demand is elastic, the decrease in the number of rides decreases the total revenue by more than the rise in fare increases it and so, on net, total revenue falls. Therefore the drivers should not try to raise the fare.

ELASTICITY

327

1c) When the fare falls from $5 to $4, the number of rides increases from 80 to 120. The percentage change in the quantity demanded is the change in the number of rides, 40, divided by the average number of rides, 100, multiplied by 100 or (40 ÷ 100) × 100, which is 40.0 percent. The percentage change in the price is the change in the price, $1, divided by the average price, $4.50, multiplied by 100, or ($1 ÷ $4.50) × 100, which is 22.2 percent. So the price elasticity of demand is 40.0% ÷ 22.2%, which equals 1.80. The demand is elastic because the elasticity, 1.80, exceeds 1. 1d) Once again a direct calculation show that in this case the total revenue increases from $400 ($5 × 80) to $480 ($4 × 120). In part c, we calculated that the demand is elastic. When the demand is elastic, the increase in the number of rides increases the total revenue by more than the fall in fare decreases it and so, on net, total revenue rises. Therefore the drivers should try to lower the fare. 1e) Total revenue is maximized at the point where the price elasticity of demand is 1. On a straight-line demand curve, demand is unit elastic at the midpoint. The demand curve intersects the price axis at $7, so the midpoint on the demand curve is where the price is $3.50. This fare will maximize the taxicab drivers total revenue. 2a) The percentage change in the price is the change in the price, $0.50, divided by the average price, $2.25, multiplied by 100, or ($0.50 ÷ $2.25) × 100, which is 22.2 percent. The percentage change in quantity is 10 percent. So the price elasticity of demand is 10% ÷ 22.2%, which equals 0.45. The demand for rides is inelastic because the elasticity, 0.45, is less than 1. 2b) The drivers’ revenue increased. If the demand for rides is inelastic, the percentage change in price, which raises total revenue, is greater than the percentage change in quantity, which lowers total revenue. So, on net, total revenue increases. 2c) The estimate may be inaccurate because we are not sure that the decline in ridership resulted only from the rise in fare and not from other factors that could influence the quantity of rides demanded.

3a) The figure above shows Cory’s two demand curves. Cory’s demand for CDs increased for the prices below $16 and decreased for the prices above $16. There are two factors that affected Cory’s demand for CDs. First, Cory’s income changed. The increase in income increased Cory’s demand and shifted the demand curve rightward. Second, a substitute for CDs, the new music service, arrived. The availability of the new music service decreased Cory’s demand and shifted the demand curve leftward. Apparently, the effect of the increased income was greater at lower prices and the effect of the new music service was greater at higher prices.

328

CHAPTER 4

3b) The percentage change in price is the change in price, $4, divided by the average price, $14, multiplied by 100 or ($4 ÷ $14) × 100, which is 28.6 percent. In 2003, the percentage change in the quantity demanded is the change in quantity, 10, divided by the average quantity, 15, multiplied by 100 or (10 ÷ 15) × 100, which is 66.7 percent. So in 2003 the price elasticity of demand is 66.7% ÷ 28.6% or 2.33. In 2004, the percentage change in the quantity demanded in the change in quantity, 20, divided by the average quantity, 20, multiplied by 100 or (20 ÷ 20) × 100, which is 100 percent. So in 2004 the price elasticity of demand is 100% ÷ 28.6% or 3.50. These calculations show that Cory’s demand for CDs became more elastic in 2004. 3c) The availability of substitutes is a factor that influences the price elasticity of demand. The new file download service is a substitute for pre-recorded CDs and when Cory started to use it, between the prices of $16 to $12, his demand for CDs became more price elastic. 3d) Total revenue is maximized at the point where the price elasticity of demand is 1. And on a straight-line demand curve, demand is unit elastic at the midpoint. If the demand curve intersects the price axis at $18, the midpoint on the demand curve is where the price is $9. So to maximize their revenue, the record companies must charge $9 per CD.

4a) The figure above shows the demand curves. 4b) The percentage change in price is the change in price, $100, divided by the average price, $250, multiplied by 100 or ($100/$250) × 100, which is 40.0 percent. For business travelers, the percentage change in the quantity of tickets demanded is the change in the quantity demanded, 200, divided by the average quantity, 1,300, multiplied by 100, or (200/1,300) × 100, which is 15.4 percent. So for business travelers the price elasticity of demand is 15.4 percent/40.0 percent, which equals 0.39. For leisure travelers, the percentage change in the quantity of tickets demanded is the change in the quantity demanded, 400, divided by the average quantity, 600, multiplied by 100, or (400/600) × 100, which is 66.7 percent. So for leisure travelers, the percentage change the price elasticity of demand is 66.7 percent/40.0 percent, which equals 1.67. 4c) Business travelers schedules are usually less flexible than those of leisure travelers. Therefore, leisure travelers are more likely to choose driving or taking the train if the price of airline tickets rises, which means their demand for airline tickets is more elastic. Also, leisure travelers’ demand is likely to be more elastic because the price of business travelers’ tickets is often paid by the company. 4d) For business travelers the demand is unit elastic at $450. For leisure travelers it’s unit elastic at $200. Both demand curves are straight lines. And on a straight-line demand curve, demand is unit elastic at the midpoint. The business travelers’ demand curve intersects the quantity axis at 1,800, so the midpoint on the demand curve is 900 tickets, which are sold if the price is $450. The leisure travelers’ demand curve intersects the quantity axis at 1,600, so the midpoint on the demand curve is where 800 tickets are sold at $200.

CHAPTER 5

EFFICIENCY AND EQUITY

1) The figure above shows the market for ink-jet printers. a) What are the equilibrium price and equilibrium quantity of printers? Is this equilibrium efficient? Explain. b) Calculate the total amount consumers paid for printers bought. c) Calculate the consumer surplus. d) Calculate the producer surplus. Price (cents per bottle) 40 80 120 160 200 240 280 2)

Quantity demanded (bottles per day) 1,200 1,000 800 600 400 200 0

Quantity supplied (bottles per day) 0 200 400 600 800 1,000 1,200

The table above gives the demand and supply schedules for bottled spring water in Springsboro. Assume that the only people who benefit from spring water are the people who drink it and the only people who bear the cost of bottled spring water are the people who produce it. a) Draw the demand and supply curves. What are the equilibrium price and equilibrium quantity of spring water? Is this equilibrium efficient? Explain. b) What is the maximum price that consumers are willing to pay for the 400th bottle? What is the minimum price that producers are willing to accept for the 400th bottle? Explain. c) Are 400 bottles a day less than or greater than the efficient quantity? Explain your answer.

330

d) e)

3)

a) b) c) d) e)

CHAPTER 5

If the market for spring water is efficient, what is the consumer surplus? Show it on your graph. What is the producer surplus? Show it on your graph. If spring water bottlers produce 400 bottles a day, is there a deadweight loss? If yes, what is it? Explain your answer using your graph. Price Quantity demanded Quantity supplied (dollars per gallon) (gallons per day) (gallons per day) 1.00 600 0 1.50 500 200 2.00 400 400 2.50 300 600 3.00 200 800 3.50 100 1,000 4.00 0 1,200 The table gives the demand and supply schedules for milk in Cowburg. Assume that the only people who benefit from milk are the people who consume it and the only people who bear the cost of milk are the people who produce it. Draw the demand and supply curves. What are the equilibrium price and equilibrium quantity of milk? Is this equilibrium efficient? Explain. What is the maximum price that consumers are willing to pay for the 500th gallon? What is the minimum price that producers are willing to accept for the 500th gallon? Explain. Are 500 gallons a day less than or greater than the efficient quantity? Explain your answer. If the market for milk is efficient, what is the consumer surplus? Show it on your graph. What is the producer surplus? Show it on your graph. If farmers produce 500 gallons a day, is there a deadweight loss? If yes, what is it? Explain your answer using your graph.

Price (dollars per gallon) 2.00 2.50 3.00 3.50 4.00 4.50 5.00 4)

Quantity demanded in July (gallons per day) 300 250 200 150 100 50 0

Quantity demanded in November (gallons per day) 150 100 50 0 0 0 0

Quantity supplied (gallons per day) 0 100 200 300 400 500 600

The table gives the demand and supply schedules for ice cream in Sweetsville in July and November. Assume that the only people who benefit from ice cream are the people who consume it and the only people who bear the cost of ice cream are the people who produce it. a) Draw the demand and supply curves. What are the equilibrium price and equilibrium quantity of ice cream in July and November? Is the allocation of resources efficient in July? Is it efficient in November? Explain. b) What is the maximum price that consumers are willing to pay for the 100th gallon of ice cream in July? In November? What is the minimum price that producers are willing to accept for the 100th gallon in July and November? Explain. c) What happens to consumer surplus and producer surplus in November compared to July? Why?

EFFICIENCY AND EQUITY

331

Answers: 1a) The equilibrium price is $100 and the equilibrium quantity is 80 printers per week. The equilibrium is efficient if the only people who benefit from printers are the people who buy them, so that the demand curve is the marginal social benefit (MSB) curve, and if the only people who bear the cost of printers are the people who produce them, so that the supply curve is the marginal social cost (MSC) curve. In this case, when the market is in equilibrium, MSB = MSC, which means the equilibrium is efficient. 1b) Because consumers buy 80 printers and pay the market price, $100 per printer, the total amount they pay is $100 × 80 = $8,000.

1c) The consumer surplus is the value of the printers buy minus the price paid summed over the quantity bought. The consumer surplus is the area labeled in the figure above. Recalling that the area of a triangle is 1/2 multiplied by the base multiplied by the height, the consumer surplus equals 1/2 × (80 × $20), or $800. 1d) The producer surplus is the price of a printer minus the marginal cost of producing it, summed over the quantity sold. The producer surplus is the area labeled in the figure above. The area of a triangle is 1/2 multiplied by the base multiplied by the height, so the producer surplus equals 1/2 × (80 × $10) = $400.

2a) The figure above shows the supply and demand curves. The equilibrium price is $1.60 and the equilibrium quantity is 600 bottles a day. The equilibrium is efficient. Because the demand curve is the same as the mar-

332

CHAPTER 5

ginal social benefit (MSB) curve and the supply curve is the same as the marginal social cost (MSC) curve, when the market is in equilibrium, MSB = MSC, which is the efficiency condition. 2b) The demand curve shows consumers’ willingness to pay. If 400 bottles are produced, the maximum price that consumers are willing to pay for the last bottle is $2.00. The supply curve shows the minimum price that producers are willing to accept to produce a certain quantity. The minimum price they are willing to accept to produce the 400th bottle is $1.20. 2c) The efficient quantity is 600, so 400 is less than the efficient quantity. There is underproduction compared to the efficient quantity. When 400 bottles of water are produced, the marginal benefit from an additional bottle is $2.00 while the marginal cost of this bottle is only $1.20. So society can get more value from its resources if more water is produced.

2d) If the market for spring water is efficient, 600 bottles are produced. The consumer surplus is the value of water that consumers buy minus the amount they pay summed over the quantity purchased, so the consumer surplus is the area of the triangle labeled in the figure above. The area of a triangle is 1/2 multiplied by the base multiplied by the height, so the consumer surplus equals 1/2 × (600 × $1.20), which is $360. The producer surplus is the price of water minus the marginal cost of producing it, summed over the quantity sold, so the producer surplus is the area of the triangle labeled in the figure above. This area is equal to 1/2 × (600 × $1.20), which is $360.

EFFICIENCY AND EQUITY

333

2e) If 400 bottles are produced, the economy produces 200 bottles less than the efficient quantity. For each of the bottles between 400 and 600, society loses the value equal to the difference between the marginal benefit and the marginal cost, which is the same as the distance between the demand curve and the supply curve. This loss is the deadweight loss, illustrated in the figure above. The deadweight loss is the area of the triangle, which equals 1/2 × (200 × $0.80) = $80.

3a) The figure above shows the demand and supply curves. The equilibrium price is $2.00 and the equilibrium quantity is 400 gallons a day. The equilibrium is efficient. The demand curve is the same as the marginal social benefit (MSB) curve and the supply curve is the same as the marginal social cost (MSC) curve, so when the market is in equilibrium, MSB = MSC, which is the efficiency condition. 3b) The demand curve shows consumers willingness to pay. If 500 gallons are produced, the maximum price that consumers are willing to pay for the last gallon is $1.50. The supply curve shows the minimum price that producers are willing to accept to produce a certain quantity. The minimum price they are willing to accept to produce the 500th gallon of milk is $2.25. 3c) The efficient quantity is 400 gallons per day, so 500 gallons is greater than the efficient quantity. There is overproduction compared to the efficient quantity. When 500 gallons of milk are produced, the marginal

334

CHAPTER 5

benefit from an additional gallon is $1.50 while the marginal cost of this gallon is $2.25. In this case, MSB < MSC, which means that society loses value if 500 gallons of milk are produced.

3d) If the market for milk is efficient, 400 gallons of milk are sold and bought. The consumer surplus is the value of water that consumers buy minus the amount they pay summed over the quantity purchased, so the consumer surplus is the area of the triangle labeled in the figure above. The area of a triangle is 1/2 multiplied by the base multiplied by the height, so the consumer surplus equals 1/2 × (200 × $2.00), which is $400. The producer surplus is the price of water minus the marginal cost of producing it, summed over the quantity sold, so the producer surplus is the area of the triangle labeled in the figure above. This area is equal to 1/2 × (400 × $1.00), which is $200.

3e) If 500 gallons of milk are produced, the economy produces 100 gallons more than the efficient quantity. Each of the additional 100 gallons of milk has a marginal cost that exceeds the marginal benefit. So each imposes a loss on society equal to the marginal cost minus the marginal benefit, which is the distance between the supply curve and the demand curve. This loss is the deadweight loss, illustrated in the figure above. The deadweight loss is the area of the triangle, which equals 1/2 × (100 × $0.75) = $37.50.

EFFICIENCY AND EQUITY

335

4a) The figure is shown above. In July, the equilibrium price is $3.00 and the equilibrium quantity is 200 gallons of ice cream a day. In November, the equilibrium price is $2.50 and the equilibrium quantity is 100 gallons a day. The allocation of resources is efficient in both July and November because the demand curve is the marginal social benefit (MSB) curve and the supply curve is the marginal social cost (MSC) curve, so at equilibrium MSB = MSC, which is the efficiency condition. 4b) The demand curve shows consumers willingness to pay. If 100 gallons of ice cream are produced, the maximum price that consumers are willing to pay for the last gallon is $4.00 in July and $2.50 in November. The supply curve shows the minimum price that producers are willing to accept to produce a certain quantity. Because the supply curve remains the same, the minimum price that producers are willing to accept to produce the 100th gallon of ice cream is $2.50 in both July and November. 4c) Consumer surplus is the value that consumers receive minus the price they pay summed over the quantity bought. Both the surplus received from each gallon of ice cream bought and the quantity bought decreased in November compared to July. Therefore, consumer surplus decreased from $200 in July to $50 in November. Producer surplus is the price of ice cream minus the marginal cost of producing it, summed over the quantity sold. Because the market price of ice cream fell while the marginal costs remained the same, both the surplus received from each gallon of ice cream sold and the quantity sold decreased in November compared to July. Therefore, producer surplus decreased from $100 in July to $25 in November.

CHAPTER 6 Rent (dollars per month) 500 600 700 800 900 1,000 1,100 1)

a) b) c) d)

MARKETS IN ACTION

Quantity demanded (units per month) 1,200 1,000 800 600 400 200 0

Quantity supplied (units per month) 0 100 200 300 400 500 600

The table above shows the demand for and supply of rental housing in Crainsboro. The city government is considering imposing a rent ceiling of $700 a month. Help the government to analyze the effects of the proposed rent ceiling. Draw the demand and supply curves. With no rent ceiling, what is the rent and how many apartments are rented? With the rent ceiling, what is the rent and how many apartments are rented? What is the shortage of housing? Explain. If the rent ceiling is strictly enforced, what is the maximum price that someone is willing to pay for the last unit of housing available? Is the housing market efficient? Explain. If a black market develops, how high could the black market rent be? Explain your answer. Wage rate (dollars per hour) 3 4 5 6 7

Quantity demanded (hours per month) 800 700 600 500 400

Quantity supplied (hours per month) 400 500 600 700 800

2) The table above shows the demand for and supply of labor of students in University Heights. a) Draw the demand and supply curves. What are the equilibrium wage and the level of employment? b) Currently, the minimum wage is set at $4.50 per hour. How many hours do students work? How many hours of their labor are unemployed? c) If the minimum wage is raised to $6 per hour, what are employment and unemployment?

338

CHAPTER 6

Price (dollars per pound) 1 2 3 4 5 3) a) b) c) d) e)

a) b) c) d)

Quantity supplied (pounds per day) 0 0 240 480 720

The table above shows the demand and supply schedules for the market for coffee in Roastville. A tax on coffee of 75 cents per pound is proposed and the local government asks you to examine the effects of the tax. Draw the demand and supply curves. If there is no tax on coffee, what is the price and how many pounds are sold? With the tax, what is the price that consumers pay? What is the price that sellers receive? How many pounds of coffee are sold? What is the government’s total tax revenue? How much of the 75¢ per pound tax is paid by buyers? How much is paid by sellers? If there are no external costs and benefits, what is the efficient level of coffee production? If the tax is imposed, will the level of production be efficient? Why or why not?

Price (dollars per gallon) 1 2 3 4 5 4)

Quantity demanded (pounds per day) 480 360 240 120 0

Quantity demanded (millions of gallons per year) 320 240 160 80 0

Quantity supplied (millions of gallons per year) 0 0 80 160 240

The table above shows the demand and supply schedules for milk in Cowland. A subsidy on milk of $1 per gallon is proposed and the government asks you to examine the effects of the subsidy. Draw the demand and supply curves. If there is no subsidy, what is the price of milk and how many gallons are sold? With the subsidy, what is the price that consumers pay? What is the price that sellers receive? How many gallons of milk are sold? Assuming no external costs and benefits, what is the efficient level of milk production? Explain. With the subsidy, what is the marginal social benefit from milk? What is the marginal social cost of milk? If the subsidy is introduced, will the level of production be efficient? Why or why not?

MARKETS IN ACTION

339

Answers:

1a) The figure above shows the housing market. With no rent ceiling, the equilibrium rent is $900 per month and 400 units are rented. 1b) Because the rent ceiling is below the equilibrium rent, the rent equals the rent ceiling, $700. At this rent, landlords are willing to supply only 200 apartment units while consumers want to rent 800. So the quantity of housing rented is 200 units per month and there is a shortage of 600 units. 1c) The demand curve shows consumers’ willingness to pay. The maximum price that someone is willing to pay for the last unit of housing available is $1,000. The housing market is not efficient. With 200 apartments rented, the marginal benefit of the last apartment ($1,000) exceeds the marginal cost ($700), so there is a deadweight loss. 1d) The black market rent could be as high as $1,000. With strict enforcement of the rent ceiling, the black market rent is close to the maximum price that renters are willing to pay.

2a) The figure above shows the labor market. The equilibrium wage rate is $5 and the equilibrium level of employment is 600 hours a month. 2b) The minimum wage of $4.50 an hour is below the equilibrium wage rate, so the actual wage is the equilibrium wage, at which 600 hours of students’ labor are employed and none are unemployed.

340

CHAPTER 6

2c) The minimum wage of $6 an hour is above the equilibrium wage rate, so the actual wage is the minimum wage, at which 500 hours of labor per month are employed. At this wage rate, the quantity of labor supplied (700 hours) is greater than the quantity demanded (500 hours), so the surplus of labor, or unemployment, is 200 hours.

3a) The figure is above. With no tax, the equilibrium price is $3 and 240 pounds of coffee per day are sold. 3b) The tax shifts the supply curve upward by $0.75. The new equilibrium price that consumers pay is $3.50. Sellers receive the price that consumers pay minus the tax, or $3.50 – $0.75 = $2.75. With the tax, 180 pounds of coffee per day are sold. 3c) The government’s tax revenue is the tax per pound, 75¢, multiplied by the number of pounds sold, 180. So the government’s total tax revenue is 75¢ × 180, which is $135 per day. With the tax, the price that buyers pay is 50¢ higher than with no tax, so the buyers pay 50¢ of the tax. Sellers pay the rest of the tax, 25¢. 3d) The efficient level of production is 240 pounds of coffee per day. At this level, the marginal social benefit ($3.00) equals the marginal social cost. 3e) If the tax is imposed, the level of production is not efficient. The marginal benefit from the last pound of coffee ($3.50) is greater than the marginal cost ($2.75). The underproduction is 60 pounds of coffee per day.

MARKETS IN ACTION

341

4a) The figure above shows that with no subsidy, the equilibrium price is $3.50 and 120 million gallons of milk per year are sold. 4b) The subsidy increases the supply and shifts the supply curve downward by $1. The new equilibrium price paid by consumers is $3.00. Sellers receive the price that consumers pay plus the subsidy, or $3 + $1 = $4. With the subsidy, 160 million gallons of milk per year are sold. 4c) The efficient level of production is 120 million gallons per year. At this level, the marginal social benefit, $3.50, equals the marginal social cost. 4d) With the subsidy, 160 million gallons per year are produced. At this level of output, the marginal social benefit of the last gallon of milk is $3.00 while the marginal social cost of this gallon is $4.00, so the level of output is not efficient. The overproduction is 40 million gallons of milk and a deadweight loss arises.

CHAPTER 7 Quantity (per year) 0 1 2 3 4 5

UTILITY AND DEMAND

Utility from CDs (units) 0 50 92 126 152 170

Utility from DVDs (units) 0 70 130 180 220 250

1) Margo buys music CDs and movies on DVDs. The table above shows her total utility from each good. a) Draw graphs showing Margo’s utility from music CDs and from movies on DVDs. b) Compare the two utility graphs. Describe Margo’s preferences. c) Draw graphs that show Margo’s marginal utility from music CDs and from movies on DVDs. d) What do the two marginal utility graphs tell you about Margo’s preferences? 2)

Margo spends $108 a year on music CDs and movies on DVDs. Initially the price of a music CD is $18 and the price of a movie on DVD is $18. Then record companies, faced with the growing competitions from digital music download services, lower the price of a music CD to $13.50. a) Draw graphs showing Margo’s consumption possibilities before and after the price of a CD fell. b) How has the relative price of a CD changed? If Margo bought 4 CDs per year, how many DVDs could she buy before and after the price of a CD fell?

Quantity (movies per year) 0 4 8 12 16 20 3)

Utility from movies (units) 0 80 152 216 272 320

Quantity (gallons of ice cream per year) 0 4 8 12 16 20

Utility from ice cream (units) 0 120 224 312 384 440

Sarah spends $100 a year on movie tickets and ice cream. The table above shows her utility from each good. If the price of a movie ticket is $5 and the price of ice cream is $5 per gallon, how does Sarah spend the $100?

344

CHAPTER 7

Answers:

1a) The figure with the total utilities is above. 1b) Margo gets more utility from any number of movies on DVDs than she does from the same number of music CDs.

1c) The figure with the marginal utilities is above. 1d) Margo’s marginal utility from an additional DVD is greater than that from an additional music CD when she has the same number of each.

UTILITY AND DEMAND

345

2a) The figure with the budget lines is above. 2b) The relative price of a CD fell from 1 DVD to 0.75 DVDs. If Margo bought 4 CDs per year, she could buy 2 DVDs per year before the price of a CD fell and 3 DVDs after the price of a CD fell. 3)

Sarah buys 6 movie tickets and 14 gallons of ice cream. She spends $100 and her marginal utility per dollar spent on movie tickets and ice cream is the same, 3.6 units per dollar.

346

CHAPTER 8 1) a) b) c) d)

POSSIBILITIES, PREFERENCES, AND CHOICES

Olivia’s income is $216 a year and she spends all of it on music CDs and movies on DVDs. The price of a music CD is $18 and the price of a DVD is $18. What is Olivia’s real income in terms of CDs? What is her real income in terms of DVDs? What is the relative price of a CD in terms of DVDs? What is the opportunity cost of a DVD? Calculate the equation for Olivia’s budget line. Place the quantity of DVDs on the left side of the equation. Draw a graph of Olivia’s budget line (with CDs on the horizontal axis). What is the slope of Olivia’s budget line? What determines its magnitude?

2)

Olivia’s income is $216 a year and she spends all of it on music CDs and movies on DVDs. The price of a music CD is $18 and the price of a DVD is $18. The figure above illustrates Olivia’s preferences. a) What quantities of CDs and DVDs does Olivia buy? Explain your solution. b) What is Olivia’s marginal rate of substitution at the point at which she consumes? Explain.

3)

a) b) c) d)

Record companies, faced with the growing competitions from digital music download services, lower the price of a music CD from $18.00 to $13.50. The price of a DVD is $18. Olivia’s income is $216 a year and she spends all of it on music CDs and movies on DVDs. What is Olivia’s real income in terms of CDs? What is her real income in terms of DVDs? What is the relative price of a CD in terms of DVDs? What is the opportunity cost of a DVD? Calculate the equation for Olivia’s budget line. Place the quantity of DVDs on the left side of the equation. Draw a graph of Olivia’s budget line (with CDs on the horizontal axis). What is the slope of Olivia’s budget line? What determines its magnitude?

348

CHAPTER 8

The figure above illustrates Olivia’s preferences. Given the price of a CD, the price of a DVD, and Olivia’s income, what quantities of CDs and DVDs does Olivia buy? Explain your solution. f) What is Olivia’s marginal rate of substitution at the point at which she consumes? Explain. g) Determine how many CDs Olivia buys if the price of a CD is $18 and there is no change in the price of DVD nor in Oliva’s income. Derive Olivia’s demand curve for CDs for the price change from $18 to $13.50 per CD. e)

Answers: 1a) Olivia’s real income in terms of CDs is the maximum number of CDs that she can buy, which is the quantity her money income can buy if she spends it all on CDs. So her real income in terms of CDs is her money income divided by the price of a CD. So, in terms of CDs, Olivia’s real income is $216/$18, which is 12 CDs. Similarly, her real income in term of DVDs is her money income divided by the price of a DVD, which is $216/$18 or 12 DVDs. 1b) The relative price of a CD in terms of DVDs is the money price of a CD divided by the money price of a DVD. So the relative price of a CD is $18/$18, which is 1 DVD per CD. The opportunity cost of a DVD is the quantity of CDs that must be given up to get a DVD. The price of a DVD is $18. To get a DVD, Olivia must give up $18, which is what she pays to get a CD. So the opportunity cost of a DVD is 1 CD. 1c) The amount that Olivia spends on CDs is $18 × QCD, where QCD is the quantity of CDs she buys. The amount that she spends on DVDs is $18 × QDVD, where QDVD is the quantity of DVDs she buys. Since Olivia spends all her income of $216 on CDs and DVDs, we can write $18 × QCD + $18 × QDVD = $216. Solving for QDVD gives the budget equation as QDVD = 12 – QCD.

POSSIBILITIES, PREFERENCS, AND CHOICES

349

1d) The budget line is in the figure above. The slope of the budget line over is the change in DVDs over a range divided by the resulting change in CDs. Because the budget line is a straight line, the slope can be calculated over any range. When the quantity of CDs increases by 1 CD, the quantity of DVDs decreases by 1 DVD, so the slope is −1 DVD/1 CD, which equals −1 DVD per CD.

2a) Olivia chooses the quantities of CDs and DVDs on her budget line that are on the highest indifference curve. At this point the budget line is tangent to the indifference curve. The highest indifference curve that Oliva can reach on the budget line shows that so Olivia buys 5 CDs and 7 DVDs. 2b) The marginal rate of substitution is the rate at which Olivia will give up DVDs to get an additional CD. The marginal rate of substitution is measured by the magnitude of the slope of an indifference curve. At the point where Olivia consumes, the slope of the indifference curve is the same as that of the budget line. The slope of the budget line is −1, so the marginal rate of substitution is 1. 3a) Olivia’s real income in terms of CDs is the maximum number of CDs that she can buy, which is the quantity her money income can buy if she spends it all on CDs. So her real income in terms of CDs is her money income divided by the price of a CD. So, in terms of CDs, Olivia’s real income is $216/$13.50, which is 16 CDs. Similarly, her real income in term of DVDs is her money income divided by the price of a DVD, which is $216/$18 or 12 DVDs.

350

CHAPTER 8

3b) The relative price of a CD in terms of DVDs is the money price of a CD divided by the money price of a DVD. So the relative price of a CD is $13.50/$18.00, which is 0.75 of a DVD per CD. The opportunity cost of a DVD is the quantity of CDs that must be given up to get a DVD. The price of a DVD is $18. To get a DVD, Olivia must give up $18, which is what she pays to get 1.33 CDs. So the opportunity cost of a DVD is 1.33 CDs. 3c) The amount that Olivia spends on CDs is $13.50 × QCD, where QCD is the quantity of CDs she buys. The amount that she spends on DVDs is $18.00 × QDVD, where QDVD is the quantity of DVDs she buys. Since Olivia spends all her income of $216 on CDs and DVDs, we can write $13.50 × QCD + $18.00 × QDVD = $216. Solving for QDVD gives the budget equation as QDVD = 12 – 0.75QCD.

3d) The budget line is in the figure above. The slope of the budget line over is the change in DVDs over a range divided by the resulting change in CDs. Because the budget line is a straight line, the slope can be calculated over any range. When the quantity of CDs increases by 1 CD, the quantity of DVDs decreases by 0.75 of a DVD, so the slope is −0.75 DVD/1 CD, which equals −0.75 of a DVD per CD.

POSSIBILITIES, PREFERENCS, AND CHOICES

351

3e) Olivia chooses the quantities of CDs and DVDs on her budget line that are on the highest indifference curve. At this point the budget line is tangent to the indifference curve. The highest indifference curve that Oliva can reach on the budget line shows that so Olivia buys 8 CDs and 6 DVDs. 3f) The marginal rate of substitution is the rate at which Olivia will give up DVDs to get an additional CD. The marginal rate of substitution is measured by the magnitude of the slope of an indifference curve. At the point where Olivia consumes, the slope of the indifference curve is the same as that of the budget line. The slope of the budget line is −0.75, so the marginal rate of substitution is 0.75.

3g) The demand curve is illustrated in the figure above. When the price of a CD is $18, Olivia buys 5 CDs. When the price of a CD is $13.50, Olivia buys 8 CDs.

CHAPTER 9 1)

a) b) c) d) 2)

a) b) c) d)

ORGANIZING PRODUCTION

Two university graduates, Bill and Steve, worked for an advertising agency at an annual salary of $40,000 each for 3 years after they graduated. Then, they decided to quit their jobs and start a partnership that designs and builds Web sites. They rented an office for $12,000 a year and bought capital for $30,000. To pay for the equipment, Bill and Steve borrowed money from a bank at an annual interest rate of 6 percent. During their first year of operation, the partners’ total revenue was $100,000. The market value of their capital at the end of the year was $20,000. If Bill and Steve do not design Web pages, their best alternatives are to return to their previous job. What are the partnership’s explicit costs? What is the firm’s economic depreciation? What are the firm’s implicit costs? What is the firm’s economic profit in the first year of operation? One year ago, Ms. Case and Mr. Bond opened a jewelry store called T & J. They invested $1,000,000 of their savings into the partnership to buy equipment and initial inventory. They rented a building for $90,000 a year, and hired two employees for an annual wage of $40,000 each. Case and Bond believed that the best alternative investment of their money would be government bonds, which could yield an annual return of 8 percent. To run the store, Case quit her previous job, at which she earned $100,000, but her former boss told her that she was welcome to return anytime. Bond kept his job with the government, but gave up 6 hours of leisure each week (for 50 weeks), the time he used to spend playing golf. Bond used to say: “I’d only give up my golf time if someone paid me $100 an hour.” During the first year of operation, T & J paid $20,000 for utilities and the firm’s total revenue was $350,000. The market value of T & J’s equipment was $200,000 at the beginning of the year and $170,000 at the end of the year. What are T & J’s explicit costs? What is the economic depreciation of their capital? What are T & J’s implicit costs? What is the firm’s economic profit in the first year of operation?

Technique A B C D E 3)

Labor (hours) 120 80 60 30 20

Capital (units) 10 21 20 22 30

FasterChip, Inc. is considering five alternative techniques for assembling personal computers. The table shows the amounts of labor and capital required by each of these techniques to make 10 computers a day. Labor costs $15 an hour and capital costs $100 a unit. a) Which techniques are technologically efficient and which are not? Explain your answer. b) Which technique is economically efficient? Explain.

354

c)

CHAPTER 9

If FasterChip uses its plant in Mexico, it can lower the labor cost to $10 an hour. Which technique will the company use in Mexico? Explain.

Firm Nimbus, Inc Cleansweep, Inc Bogey, Inc Muggle Handles, Inc Salazar’s Sticks Draco Dormiens, Inc Avada’s Brrom

Sales (thousands of galleons) 57.5 33.3 12.6 9.9 11.1 8.6 7.8

4) The table above shows sales of the firms in the broomstick industry: a) Calculate the four-firm concentration ratio. b) Calculate the Herfindahl-Hirschman Index. c) Based on your calculations, what is the structure of the industry?

Answers: 1a) The partnership’s only explicit cost is the rent, which is $12,000, and the interest expense, which is $1,800. So their total explicit cost is $13,800. 1b) The economic depreciation of the firm’s capital is the market value of its equipment at the beginning of the year, $30,000, minus its market value at the end of the year, $20,000, so the economic depreciation is $10,000. 1c) The firm’s implicit costs are the economic depreciation and the opportunity cost of the owners’ resources. The best alternative use of their resources is working at their previous job for $40,000 each, for a total of $80,000. Thus, the firm’s total implicit cost are $10,000 + $80,000 = $90,000. 1d) A firm’s economic profit is its total revenue minus its total cost. Bill and Steve’s total cost is the sum of their explicit and implicit costs: $13,800 + $90,000 = $103,800. Thus, the firm’s economic profit is $100,000 – $103,800 = −$3,800, which means that the firm has an economic loss. 2a) T & J’s explicit costs are the rent, wages, and utilities: $90,000 + $80,000 + $20,000 = $190,000. 2b) The economic depreciation of the firm’s capital is the market value of its equipment at the beginning of the year, $200,000, minus its market value at the end of the year, $170,000, so the economic depreciation is $30,000. 2c) The firm’s implicit costs are the economic depreciation, the interest forgone, and the opportunity cost of the owners’ resources. The economic depreciation is $30,000. The interest foregone is the lost interest on the $1,000,000 invested in the business. This interest is equal to $1,000,000 × 0.08, which is $80,000. Case’s opportunity cost is her wage at the previous job, $100,000. Bond’s opportunity cost is the 300 hours of his leisure time , which he values at $100 per hour, so his total opportunity cost is $100 × 300 = $30,000. Thus, T & J’s total implicit costs are $30,000 + $80,000 + $100,000 + $30,000 = $240,000. 2d) A firm’s economic profit is its total revenue minus its total opportunity cost. T & J’s total opportunity cost is the sum of the firm’s explicit and implicit costs: $190,000 + $240,000 = $330,800. So the firm’s economic profit is $350,000 – $330,000 = $20,000. 3a) Technique B is technologically inefficient. It requires more labor and more capital than technique C to produce a given output. All other techniques are technologically efficient. 3b) Economic efficiency occurs when the firm produces a given output at the least cost. Technique D has the lowest cost and therefore is economically efficient.

ORGANIZING PRODUCTION

355

3c) With the change in costs, the economically efficient method of production changes. The company will now use a more labor intensive technique A, which, with the lower labor cost, becomes the economically efficient method. 4a) The four-firm concentration ratio is the percentage of the value of sales accounted for by the four largest firms in an industry. The percentage of the value of sales accounted for by the four largest firms in the broomstick industry (Nimbus, Inc., Cleansweep, Inc., Bogey, Inc., and Salazar’s Sticks) is (114.5/140.8) × 100 = 81.3 percent. So the four-firm concentration ratio in the broomstick industry is 81.3 percent. Firm Nimbus, Inc Cleansweep, Inc Bogey, Inc Muggle Handles, Inc Salazar’s Sticks Draco Dormiens, Inc Avada’s Brrom Total

Sales (thousands of galleons) 57.5 33.3 12.6 9.9 11.1 8.6 7.8 140.8

Market share (percent) 40.8 23.7 8.9 7.0 7.9 6.1 5.5 100.0

Market share squared 1667.7 559.3 80.1 49.4 62.1 37.3 30.7 2486.8

4b) The Herfindahl-Hirschman Index (HHI) is the square of the percentage market share of each firm summed over the largest 50 firms or over all firms if there are fewer than 50. The table above shows the HHI calculations. Summing the squared market shares gives the HHI as 2,487. 4c) A four-firm concentration ratio that exceed 60 percent indicates that the market is highly concentrated and dominated by a few firms in an oligopoly. A market in which the HHI exceeds 1,800 is regarded as being uncompetitive. So both the four-firm concentration ratio (81.3) and the HHI (2,487) indicate that the broomstick industry is an oligopoly.

CHAPTER 10 Labor (workers) 1 2 3 4 5 6 7 Ovens

OUTPUT AND COSTS

Plant 1 8 18 26 31 34 35 33 1

Output (pizzas per day) Plant 2 Plant 3 11 13 22 24 30 33 36 40 40 45 42 48 40 47 2 3

Plant 4 14 25 35 43 50 54 54 4

1) Tony’s Pizza’s production function is shown in the table above. a) Suppose Tony operates Plant 2. He hires 2 workers and produces 20 pizzas a day. Is the pizzeria technologically efficient? Why or why not? b) Suppose Tony operates Plant 1. He hires 2 workers and produces 18 pizzas per day. Is Tony’s Pizza technologically efficient? Why or why not? Can Tony increase production to 22 pizzas a day in the short run? If yes, how? c) Suppose Tony operates Plant 3. What is the marginal product of labor when the fourth worker is hired? When operating Plant 3, does Tony experience diminishing marginal returns? Explain. d) Suppose Tony currently uses Plant 3. Can he increase production from 40 to 50 pizzas per day in the short run? In the long run? If yes, how? 2) a) b) c) d) 3) a) b) c) d) 4)

Tony’s Pizza’s production function is shown in the table above. Tony currently operates Plant 1. He hires workers at a wage rate of $50 a day and his total fixed cost is $100. Calculate Tony’s Pizza’s total variable cost and total cost for each output level. Calculate Tony’s marginal costs. Calculate the average fixed costs, average variable costs, and average total costs. Draw Tony’s marginal, average variable, and average total cost curves in one figure. What is the relationship between marginal cost and average cost? Tony’s Pizza’s production function is shown in the table above. Tony currently operates Plant 2. He hires workers at a wage rate of $50 a day and his total fixed cost is $150. Calculate Tony’s Pizza’s total variable cost and total cost for each output level. Calculate the marginal costs. Calculate the average fixed costs, average variable costs, and average total costs. Suppose that total fixed cost increases to $200. Explain what changes occur in the costs.

Tony’s Pizza’s production function is shown in the table above. Tony hires workers at a wage rate of $50 a day and his total fixed cost is $100 for Plant 1 and $150 for Plant 2. a) Calculate Tony’s Pizza’s average total costs for Plant 1. Draw the short-run average total cost curve.

358

b) c) d)

CHAPTER 10

Calculate average total costs for Plant 2. Draw the short-run average total cost curve on the same figure that you used for part a. In the figure, illustrate Tony’s long-run average cost curve. If Tony wants to produce 34 pizzas per day, what plant size should he choose? Over what output range is it efficient to operate Plant 1?

Answers: 1a) The firm is not technologically efficient. The production function shows that with 2 ovens and 2 workers it is possible to produce 22 pizzas a day. 1b) Tony’s Pizza is technologically efficient. The production function shows that 18 pizzas is the maximum output that 2 workers can produce with 1 oven. Tony can increase production to 22 pizzas per day in the short run by hiring more labor. 1c) When the 4th worker is hired, the marginal product of labor is the change in output, 40 pizzas – 33 pizzas = 7 pizzas, divided by the change in the labor input, 1 worker. So the marginal product of labor for the 4th worker is 7 pizzas per day. Tony experiences diminishing marginal returns because the marginal product of labor diminishes as more workers are hired. 1d) If Tony uses Plant 3, he cannot increase production from 40 to 50 pizzas per day in the short run. The short-run production function for Plant 3 shows that the maximum output that the firm can produce with this plant is 48 pizzas per day. In the long run, however, Tony can increase his output to 50 pizzas per day by increasing his plant size to 4 ovens (Plant 4) and hiring 5 workers. Labor (workers) 0

Output (pizzas per day) 0

TVC

TC

0

100

MC AFC (dollars per day)

AVC

ATC

12.50

6.25

18.75

5.56

5.56

11.11

3.85

5.77

9.62

3.23

6.45

9.68

2.94

7.35

10.29

2.86

8.57

11.43

6.25 1

8

50

150 5.00

2

18

100

200 6.25

3

26

150

250 10.00

4

31

200

300 16.67

5

34

250

350 50.00

6

35

300

400

2a) The total variable cost and total cost for each level of output are in the table above. 2b) The firm’s marginal costs are shown in the table above. 2c) The firm’s average fixed costs, average variable costs, and average total costs are shown in the table above.

OUTPUT AND COSTS

359

2d) The figure above has the cost curves. The marginal cost curve intersects both the average total cost and the average variable cost curves at the minimum point on each curve. When the marginal cost is below the average cost, the average cost is decreasing. When the marginal cost is above the average cost, the average cost is increasing. Because ATC = AFC + AVC, it is the case that AFC = ATC − AVC, so that the average fixed cost is the vertical distance between the ATC and AVC curves. Labor (workers) 0

Output (pizzas per day) 0

TVC

TC

0

150

1

8

50

200

2

18

100

250

MC AFC (dollars per day)

AVC

ATC

13.64

4.45

18.18

6.82

4.55

11.36

5.00

5.00

10.00

4.17

5.56

9.72

3.75

6.25

10.00

3.57

7.14

10.71

4.55 4.55 6.25 3

26

150

300 8.33

4

31

200

350 12.50

5

34

250

400 25.00

6 3a) 3b) 3c) 3d)

35

300

450

The total variable cost and total cost for each level of output are in the table above. The firm’s marginal costs are shown in the table above. The firm’s average fixed costs, average variable costs, and average total costs are shown in the table above. The increase in total fixed cost raises the average fixed costs and average total costs. However, the increase in the fixed costs has no effect on the average variable cost or the marginal cost. So, the ATC and AFC curves shift upward, but the AVC and MC curves do not change.

360

CHAPTER 10

Output (pizzas per day) 8 18 26 31 34 35

ATC for Plant 1 (dollars per day) 18.75 11.11 9.62 9.68 10.29 11.43

4a) The table above shows Tony’s short-run average total costs for Plant 1. The figure shows this short-run average total cost curve, labeled ATC1. (The ATC curve is shaded both black and gray.) Output (pizzas per day) 11 22 30 36 40 42

ATC for Plant 2 (dollars per day) 18.18 11.36 10.00 9.72 10.00 10.71

4b) The table above shows Tony’s short-run average total costs for Plant 2. The figure shows this short-run average total cost curve, labeled ATC2. (The ATC curve is shaded both black and gray.) 4c) The figure above shows Tony’s long-run average total cost curve, the curve shaded black and made up of part of the ATC1 curve and the ATC2 curve. For each level of output, the long-run average cost curve shows the lowest attainable average cost of production. 4d) If Tony produces 34 pizzas per day operating Plant 1, his average total cost is $10.29. If he operates Plant 2, the ATC is below $10 (see the figure). Therefore Tony should choose Plant 2. It is efficient to operate the plant that has the lowest average total cost of a pizza. Plant 1 is efficient as long as the output does not exceed 33 pizzas per day.

CHAPTER 11

PERFECT COMPETITION

Quantity (gallons per day) 0 100 200 300 400 500 600 1)

a) b) c) d)

Brennan’s Farm produces and sells milk. The market for milk is perfectly competitive. The market price of milk is $2.50 per gallon. The relationship between the farm’s output and total costs is shown in the table above. Draw Brennan’s average variable, average total, and marginal cost curves. Use your graphs to find Brennan’s profit-maximizing output. If Brennan maximizes his profit, how much profit does he make? Should Brennan stay in business? Will other farms with costs the same as Brennan’s enter the milk market? Explain. Quantity (gallons per day) 0 100 200 300 400 500 600

2)

a) b) c) d) e)

Total cost (dollars per day) 500 713 800 838 900 1,063 1,400

Total cost (dollars per day) 400 703 880 1,008 1,160 1,410 1,840

Petunia’s Farm produces and sells milk. The market for milk is perfectly competitive. The market price of milk is $2.50 per gallon. The relationship between the farm’s output and total costs is shown in the table above. Draw Petunia’s average variable, average total, and marginal cost curves. Use your graphs to find Petunia’s profit-maximizing output. If Petunia maximizes her profit, how much profit does she make? What is Petunia’s shutdown point? What is her economic profit at the shut-down point? Should Petunia shut down? Will farms with costs the same as Petunia’s enter or exit the milk market? Explain.

362

CHAPTER 11

Price (dollars per gallon) 2.50 3.50 4.50 3)

a) b) c) d)

Quantity demanded (thousands of gallons per day) 600 500 400

The market for milk is perfectly competitive. There are 1,000 farms in the industry and the relationship between a typical farm’s output and total costs is the same as in Problem 2. The market demand schedule for milk is shown in the table above. What is a typical farm’s supply schedule and what is the market supply schedule? What is the market price? What quantity of milk is sold? What is the output produced by each farm? What type of profit or loss is made by each farm? Do farms enter or exit the industry?

PERFECT COMPETITION

363

Answers: Quantity (gallons per day) 0

TC

MC

TVC AVC (dollars per day)

ATC

500 2.13

100

713

213

2.13

7.13

300

1.50

4.00

338

1.13

2.79

400

1.00

2.25

563

1.13

2.13

900

1.50

2.33

0.87 200

800 0.38

300

838 0.62

400

900 1.63

500

1,063 3.37

600

1,400

1a) Brennan’s cost schedules are shown in the table above and the cost curves are shown in the figure above. 1b) As the figure indicates, marginal cost equals the price when the farm produces 510 gallons of milk per day. So Brennan’s profit-maximizing output is 510 gallons per day. 1c) At the profit maximizing level of output, the average total cost is (about) $2.13 per gallon. Because the price is $2.50 per gallon, the economic profit per gallon is $2.50 – $2.13 = $0.37 and so the total economic profit is $0.37 × 510 = $188.70 per day. 1d) Brennan should stay in business because his economic profit is positive. Other firms, attracted by economic profit, will enter the market.

364

CHAPTER 11

Quantity (gallons per day) 0

TC

MC

TVC AVC (dollars per day)

ATC

400 3.03

100

703

303

3.03

7.03

480

2.40

4.40

608

2.03

3.36

760

1.90

2.90

1,010

2.02

2.82

1,440

2.40

3.07

1.77 200

880 1.28

300

1,008 1.52

400

1,160 2.50

500

1,410

600

1,840

4.30

2a) Petunia’s cost schedules are shown in the table above and the cost curves are shown in the figure above. 2b) As shown in the graph, marginal cost equals the price when the farm produces 450 gallons of milk per day. So Petunia’s profit-maximizing level of output is 450 gallons per day. 2c) At the profit maximizing level of output, the average total cost is $2.86 per gallon. Because the price is $2.50 per gallon, the economic profit per gallon is $2.50 − $2.86 = −$0.36 per gallon, that is, an economic loss of $0.36 per gallon. The total economic loss is −$0.36 × 450 = −$162 per day. 2d) A firm’s shut-down point is the output and price at which the firm just covers its variable costs. As shown in the graph, Petunia can just cover her variable costs when the price falls to $1.90 per gallon and the farm produces 400 gallons of milk per day. At the shut-down point, the farm suffers an economic loss equal to the fixed costs. So Petunia’s economic profit is negative $400. 2e) As soon as the market price is above the shut-down point, Petunia should not shut down. If the farm continues to operate, it can cover part of the fixed costs and only loses $162 per day. If the firm shuts down, the economic loss equals total fixed cost, i.e. $400 per day. But since Petunia’s profit is below normal, her farm and other farms with the same costs will exit the market in the long run.

PERFECT COMPETITION

Price (dollars per gallon) 2.50 4.50

365

Firm quantity supplied (gallons per day) 450 550

Market quantity supplied (thousands of gallons per day) 450 550

3a) A typical farm’s supply schedule is the same as its marginal cost schedule for prices above the minimum average variable cost, $1.90. The table above gives this schedule. Then the market supply schedule is the sum of what 1,000 firms will supply, so the market supply schedule is 1,000 times the quantity supplied by an individual firm. 3b) The equilibrium market price is $3.50, at which 500,000 gallons of milk per day are sold. 3c) At the market price, each farm maximizes its profit by producing 500 gallons of milk per day. At this level of output the price exceeds the average total cost, so the farms are earning an economic profit. 3d) Because the firms are earning an economic profit, new firms enter the industry.

CHAPTER 12 Price (dollars per ounce) 100 200 300 400 500 600 1) a) b) c) d)

MONOPOLY

Quantity demanded (ounces per day) 100 80 60 40 20 0

Quantity produced (ounces per day) 9 20 40 60 80 100

Total cost (dollars per day) 6,000 7,200 8,800 10,800 13,200 16,000

Anastasia’s Gold Mines, a single price monopoly, faces the demand schedule shown in the first table above and has the cost schedule shown in the second table above. Calculate Anastasia’s marginal revenue schedule. In a figure, draw the demand curve and the marginal revenue curve. Calculate Anastasia’s marginal cost and average total cost schedules. In the same figure that you drew the demand and marginal revenue curves, draw the marginal and the average total cost curves. What are Anastasia’s profit-maximizing output and price? What is Anastasia’s economic profit? Explain your answer. Does Anastasia’s Gold Mines use resources efficiently? Explain your answer.

Price Quantity demanded Quantity produced Total cost (dollars per bottle) (bottles per week) (bottles per week) (dollars per week) 1.50 250 0 120.00 2.00 200 50 145.00 2.50 150 100 157.50 3.00 100 150 195.00 3.50 50 200 257.50 4.00 0 250 345.00 2) Adele’s Springs produces a unique mineral water. The firm faces the demand schedule shown in the first table above and has the cost schedule shown in the second table above. a) Calculate Adele’s marginal revenue schedule. In a figure, draw the demand curve and the marginal revenue curve. b) Calculate Adele’s marginal cost and average total cost schedules. In the same figure that you drew the demand and marginal revenue curves, draw the marginal and the average total cost curves. c) What are Adele’s profit-maximizing output and price? What is Adele’s economic profit? Explain your answer. d) Does Adele’s Springs use resources efficiently? Explain your answer.

368

CHAPTER 12

Price (dollars per copy) 0.40 0.50 0.60 0.70 0.80 0.90 3) a) b) c) d) e) f)

Quantity demanded (copies per day) 500 400 300 200 100 0

Quantity produced (copies per day) 0 100 200 300 400 500

Total cost (dollars per day) 100 105 120 145 180 225

Pinesboro Herald is the only local newspaper in the city of Pinesboro. The publisher faces the demand schedule shown in the first table above and has the cost schedule shown in the second table above. Calculate the marginal revenue schedule. In a figure, draw the demand curve and the marginal revenue curve. Calculate the publisher’s marginal cost and average total cost schedules. In the same figure that you drew the demand and marginal revenue curves, draw the marginal and the average total cost curves. What are the publisher’s profit-maximizing output and price? What is the publisher’s economic profit per day? At the price charged, is the demand for newspapers elastic or inelastic? Explain your answer. Does the publisher use resources efficiently? What is the deadweight loss? Explain your answer. Will the publisher try to price discriminate? Why or why not?

Answers: TR

Quantity (ounces per day) 0

Price 600

0

20

500

10,000

40

400

16,000

60

300

18,000

80

200

16,000

100

100

10,000

MR TC (dollars) 6,000 500 7,200 300 8,800 100 10,800 −100 13,200 −300 16,000

MC

ATC

60 360 80 220 100 180 120 165 140 160

1a) Anastasia’s marginal revenue schedule is shown in the table above. The figure below shows the demand curve faced by the firm and its marginal revenue curve.

MONOPOLY

369

1b) Anastasia’s marginal cost and average total cost schedules are shown in the table above. The figure above shows the firm’s marginal and average total cost curves. 1c) The firm maximizes its profit by producing 50 ounces of gold per day. At this level of output MC = MR = $100. The price is $350 per ounce. If the firm produces 50 ounces per day, the figure shows that its average total cost is $200 per ounce. So the economic profit per ounce is $350 − $200, which is $150 per ounce and the total economic profit $150 per ounce × 50 ounces, or $7,500 per day. 1d) The monopoly is the source of inefficiency because at the output level that it produces the marginal benefit, $350 from the demand curve, exceeds the marginal cost, $100 from the marginal cost curve. Quantity (bottles per week)

Price

TR

0

4.00

0

50

3.50

175

100

3.00

300

150

2.50

375

200

2.00

400

250

1.50

375

MR

TC

(dollars) 120.00 3.50 145.00 2.50 157.50 1.50 195.00 0.50 257.50 −0.50 345.00

MC

ATC

0.50 2.90 0.25 1.58 0.75 1.30 1.25 1.29 1.75 1.38

2a) Adele’s marginal revenue schedule is shown in the table above. The figure below shows the demand curve faced by the firm and its marginal revenue curve.

370

CHAPTER 12

2b) Adele’s marginal cost and average total cost schedules are shown in the table above. The figure above shows the firm’s marginal and average total cost curves. 2c) The firm maximizes its profit by producing 150 bottles of mineral water per week. At this level of output MC = MR = $1.00. The price is $2.50 per bottle. If the firm produces 150 bottles of water per week, its average total cost is $1.30 per bottle. So the economic profit per bottle is $2.50 − $1.30, or $1.20 per bottle. So the total economic profit is $1.20 per bottle × 150 bottles per week = $180 per week. 2d) The firm does not use resources efficiently. At the output level that it produces, the marginal benefit, $2.50 from the demand curve, exceeds the marginal cost, $1.00 from the marginal cost curve. Quantity (copies per day) 0

Price

TR

0.90

0

100

0.80

80

200

0.70

140

300

0.60

180

400

0.50

200

500

0.40

200

MR TC (dollars) 100 0.80 105 0.60 120 0.40 145 0.20 180 0.00 225

MC

ATC

0.05 1.05 0.15 0.60 0.25 0.48 0.35 0.45 0.45 0.45

3a) The publisher’s marginal revenue schedule is shown in the table above. The figure below shows the demand curve faced by the firm and its marginal revenue curve.

MONOPOLY

371

3b) The publisher’s marginal cost and average total cost schedules are shown in the table above. The figure above shows the firm’s marginal and average total cost curves. 3c) The publisher maximizes its profit by producing 300 copies per day. At this level of output MC = MR = $0.30. The market price is $0.60 per copy. If the publisher prints 300 copies per day, its average total cost is $0.48 per copy. So the economic profit per copy is $0.60 − $0.48 = $0.12 per copy. And the total economic profit is $0.12 profit per copy× 300 copies per day, or $36 per day. 3d) The demand for newspapers is elastic. At the price charged, the marginal revenue is positive, which means that means the total revenue increases if the quantity sold increases and decreases if the quantity sold decreases. 3e) The publisher does not use resources efficiently. At the profit-maximizing output level, the marginal benefit is $0.60, from the demand curve, which exceeds the marginal cost of $0.48. The efficient level of output is 450 copies per day. The deadweight loss is equal to the area of the darkened triangle in the figure above. This area equals ($0.60 − $0.30) × (450 − 300)/2 = $22.5 per day. 3f) The publisher will try to price discriminate if it can identify and separate different types of buyers and find a way to prevent resale of the newspaper. For example, the publisher can give a discount to its low-income subscribers such as students and senior citizens if the paper is delivered to their residences.

CHAPTER 13

1)

a) b) c) d) 2)

a) b) c)

d) 3)

MONOPOLISTIC COMPETITION AND OLIGOPOLY

Shower Power, Inc., a firm in monopolistic competition, produces shower radios. The company’s economists know that it can sell no radios at $80, and for each $10 cut in price, the quantity of radios it can sell increases by 50 a day. This relationship continues to hold until the price falls to $20. The firm’s total fixed cost is $3,000 a day. Its marginal cost is constant at $20 per radio. Draw the demand curve faced by the firm and its marginal revenue curve. Also draw Shower Power’s marginal cost and average total cost curves. What quantity of radios should Shower Power produce to maximize its profit? What price should it charge? What is the firm’s short-run economic profit or loss? In the long run, what will happen to the demand for Shower Power’s radios, the quantity of radios sold, the price charged, and the firm’s economic profit? Shower Power, Inc., a firm in monopolistic competition, produces shower radios. The company’s economists know that it if it does not advertise, it can sell no radios at $80, and for each $10 cut in price, the quantity of radios it can sell increases by 50 a day. This relationship continues to hold until the price falls to $20. The firm’s total fixed cost is $3,000 a day. Its marginal cost is constant at $20 per radio. Shower Power’s economists have determined that if the firm spends $3,500 a day on advertising, it can double the quantity of radios sold at each price. Draw the new demand curve faced by the firm and its new marginal revenue curve. Also draw Shower Power’s marginal cost and average total cost curves. If Shower Power does not advertise, what quantity of radios sold maximizes its profit? What is the profitmaximizing price? What is the firm’s economic profit? If Shower Power does advertise, what quantity of radios sold maximizes its profit? What is the profitmaximizing price? What is the maximum economic profit that the firm can make on its shower radios? Draw a diagram to show the effects of advertising. Will Shower Power advertise? Why or why not?

Shower Power, Inc., a firm in monopolistic competition, produces shower radios. The company’s economists know that it can sell no radios at $80, and for each $10 cut in price, the quantity of radios it can sell increases by 50 a day. This relationship continues to hold until the price falls to $20. The firm’s total fixed cost is $3,000 a day. Its marginal cost is constant at $20 per radio. Shower Power’s managers are considering some quality improvements in their shower radios, which would raise the firm’s total fixed cost by $200 per day and double its marginal cost. Shower Power’s economists report that if the firm undertakes these quality improvements, from the demand relationship given above Shower Power will be able to increase the quantity of radios sold by 20 percent at each price. a) Draw the new demand curve faced by the firm and its new marginal revenue curve. Draw Shower Power’s new marginal cost and new average total cost curves. b) With no quality improvements, what quantity of radios maximizes Shower Power’s profit? What is the profit-maximizing price? What is the firm’s economic profit?

374

CHAPTER 13

c)

If Shower Power undertakes the quality improvements, what quantity of radios should it sell to maximize its profit? What is the profit-maximizing price? What is its economic profit? Should Shower Power implement the quality improvements? Why or why not?

d) 4)

Nimbus, Inc., and Cleansweep, Inc., are the only producers of flying brooms. Each firm has two strategies: Spend 30,000 galleons a year on research and development (R&D) or spend nothing on R&D. If neither firm spends on R&D, Nimbus’ economic profit is 80, 000 galleons and Cleansweep’s economic profit is 40,000 galleons. If each firm conducts R&D, market shares are maintained, but each firm’s profit is lower by the amount spent on R&D. If Nimbus conducts R&D and Cleansweep does not, Nimbus makes an economic profit of 120,000 galleons, while Cleansweep incurs an economic loss of 20,000 galleons. If Cleansweep conducts R&D and Nimbus does not, Cleansweep makes a profit of 60,000 galleons while Nimbus loses 10,000 galleons. a) Construct a payoff matrix for the game that Nimbus and Cleansweep must play. b) Find the Nash equilibrium. In the Nash equilibrium, what is each firm’s equilibrium profit? c) What is the cooperative outcome? Would the firms make more economic profit if they collude to achieve the cooperative outcome?

Answers:

1a) The figure above shows Shower Power’s demand curve, marginal revenue curve, average total cost curve, and marginal cost curve. 1b) Shower Power will produce 150 radios per day. At this level of output MC = MR. Given the demand for the firm’s radios, the price at which 150 radios will be sold is $50. 1c) When the firm produces 150 radios per day, its average total cost is $40 per radio. So its economic profit per radio is $50 − $40 = $10 per radio. Because it produces 150 radios, its total economic profit is $10 × 150, which is $1,500 per day. 1d) Because Shower Power is making an economic profit, other firms have an incentive to enter the industry. As other firms start to make shower radios similar to Shower Power radios, the demand for Shower Power radios decreases. The demand curve for Shower Power radios and the marginal revenue curve shift leftward. The profit maximizing quantity decreases and the price falls. Firms continue to enter the industry until the economic profit disappears. So in the long run, Shower Power is making zero economic profit on its shower radios.

MONOPOLISTIC COMPETITION AND OLIGOPOLY

375

2a) The figure above shows Shower Power’s new demand curve, new marginal revenue curve, average total cost curve, and marginal cost curve. The old (pre-advertising) demand and marginal cost curves are shown in the answer to problem 1. Notice that the quantity scale along the horizontal axis differs between the two figures. 2b) As problem 1 demonstrated, Shower Power firm maximizes its profit by producing 150 radios per day and selling the radios at $50 per radio. The total economic profit is $1,500 per day. 2c) As the figure above shows, if Shower Power advertises, its fixed cost increases by $3,500 per day, shifting the average total cost curve upward. The demand curve for Shower Power radios and the marginal revenue curve shift rightward (or rather rotate outward). So the firm maximizes its profit by producing 300 radios per day. At this level of output, MC = MR = $20. The price at which the firm will be able to sell 300 radios per day is $50 per radio. If the firm produces 300 radios a day, its average total cost is $41.67 per radio. So the economic profit per radio is $50 – $41.67, or $8.33 per radio and the total economic profit is $8.33 × 300 = $2,500 per day. 2d) Shower Power will advertise because advertising increases its economic profit from $1,500 to $2,500 per day.

376

CHAPTER 13

3a) The figure above shows Shower Power’s new demand curve, new marginal revenue curve, new average total cost curve, and new marginal cost curve. Once again, notice that the quantity scale along the horizontal axis differs from the other figures. 3b) As problem 1 demonstrated, Shower Power firm maximizes its profit by producing 150 radios per day and selling the radios at $50 per radio. The total economic profit is $1,500 per day. 3c) If Shower Power undertakes the quality improvements, its marginal cost and average total cost curves shift upward and are as illustrated above. The demand curve for Shower Power radios and the marginal revenue curve shift rightward (or rather rotate outward). The firm maximizes its profit by producing 120 radios per day. At this level of output MC = MR = $40. The price at which the firm will be able to sell 120 radios per day is $60 per radio. If the firm produces 120 radios a day, its average total cost is $66.67 per radio. So the economic profit per radio is $60.00 − $66.67, which is −$6.67 per radio. In other words, Shower Power has an economic loss of $6.67 per radio. So Shower Power’s total economics loss is −$6.67 × 120 = −$800 per day. The firm incurs an economic loss of $800 a day. 3d) Shower Power will not implement the quality improvements because they will result in an economic loss. Cleansweep’s strategies R&D

–20

10

R&D Nimbus’ strategies

No R&D

120

50 60

40

No R&D

–10

80

4a) The payoff matrix is above. The profits are in thousands of galleons. 4b) The Nash equilibrium is for both firms to conduct R&D. Nimbus makes 50,000 galleons and Cleansweep makes 10,000 galleons.

MONOPOLISTIC COMPETITION AND OLIGOPOLY

377

4c) The cooperative outcome is for both firms not to conduct R&D. If Nimbus and Cleansweep collude and neither of them cheats, Nimbus makes 80,000 galleons and Cleansweep makes 40,000 galleons. So each firm makes more economic profit compared to the Nash equilibrium.

CHAPTER 14

REGULATION AND ANTITRUST LAW

Price (dollars per thousand cubic feet) 10 30 50 70 90 1)

a) b) c) d) 2)

Quantity demanded (thousands of cubic feet per day) 80 60 40 20 0

West Coast Gas, Inc., is a natural gas supplier. The firm faces the demand schedule shown in the table above and cannot price discriminate. The company’s fixed cost is $1,000 per month and its marginal cost is constant at $10 per thousand of cubic feet. Draw the demand curve faced by West Coast Gas and the marginal revenue curve. Draw the company’s marginal cost and average total cost curves. Is West Coast Gas a natural monopoly? Why or why not? What are the firm’s profit-maximizing output and price? What is the firm’s economic profit per month? If West Coast Gas maximizes its profit, does it also maximize total surplus? Why or why not? What is the deadweight loss (if any)?

West Coast Gas, Inc., is a natural gas supplier. The firm faces the demand schedule shown in the table above and cannot price discriminate. The company’s fixed cost is $1,000 per month and its marginal cost is constant at $10 per thousand of cubic feet. The government imposes a marginal cost pricing rule on the company. a) What is the price of natural gas supplied by West Coast Gas? How many cubic feet does the company sell? What is the firm’s economic profit per month? b) How does the regulation affect total surplus? c) Is the regulation in the social interest? Explain.

380

3)

a) b) c) d) e)

CHAPTER 14

Two phone companies offer local calls in an area. The figure shows the demand curve for calls and the marginal cost curve of each firm. The companies are regulated and cannot price discriminate or use a two-part price. What are the price of a call and the number of calls per day if the regulation is in the social interest? Explain your answer. What are the price of a call and the number of calls per day if the regulation is in the self-interest of producers? Explain your answer. What is the total surplus gain resulting from the regulation if it is in the social interest? In each case, what is the deadweight loss? How would the companies be regulated according to the social interest theory?

Answers:

1a) The figure above shows the demand curve faced by the company and the firm’s marginal revenue, marginal cost, and average total cost curves. Because the marginal cost is constant, total variable cost equals marginal cost multiplied by the number of thousands of cubic feet of gas produced, and then TC = FC + VC. 1b) West Coast Gas is a natural monopoly. Because the marginal cost is constant, the industry experiences economies of scale at all levels of output. As shown in the figure, the average total cost curve slopes down-

REGULATION AND ANTITRUST LAW

381

ward where it intersects the demand curve, which means one firm can supply the entire market at a lower average cost and price than could two or more firms. 1c) The firm maximizes its profit by producing 40,000 cubic feet of natural gas per month. At this level of output MR = MC = $10. The profit-maximizing price is $50 per thousand cubic feet. If the firm supplies 40,000 cubic feet per month, its average total cost is $35 per thousand cubic feet. So its economic profit per thousand cubic feet of gas is $50 − $35 = $15 per thousand cubic feet. Thus its total economic profit is $15 × 40,000, which is $600,000 per month. 1d) At the profit-maximizing output level, the marginal benefit of $50, measured using the demand curve, exceeds the marginal cost of $10, measured using the marginal cost curve. Because the marginal benefit exceeds the marginal cost, total surplus is not maximized The deadweight loss is the area of the darkened triangle in the figure, which equals ($50 − $10) × (80,000 – 40,000) ÷ 2 = $800,000 per month. 2a) Marginal cost pricing regulation sets the price equal to marginal cost, which is determined where the marginal cost curve intersects the demand curve. So the price is $10 per thousand cubic feet. At this price, the company sells 80,000 cubic feet of gas. The average total cost at this level of output is $22.50 per thousand cubic feet, so the company’s economic profit per thousand cubic feet of natural gas is $10 − $22.50 = −$12.50 per thousand cubic feet, that is, the company incurs an economic loss of $12.50 per thousand cubic feet. The company produces 80,000 cubic feet, so its total economic loss is −$12.50 × 80,000 = −$1,000,000 per month. West Coast Gas incurs an economic loss of $1 million per month. 2b) If West Coast Gas is not regulated, the company maximizes its profit by producing 40,000 cubic feet of gas per month and charging $50 per thousand cubic feet. In this case, the consumer surplus is ($90 − $50) × 40,000 ÷ 2 = $800,000 and the producer surplus is ($50 − $10) × 40,000 = $1,600,000. The total surplus is $800,000 + $1,600,000 = $2,400,000. With the marginal cost pricing, the total surplus equals the consumer surplus, which is ($90 − $10) × 80,000 ÷ 2 = $3,200,000. So the regulation increases total surplus by $800,000. 2c) With no regulation, the marginal benefit ($50) exceeds the marginal cost ($10), the output produced is below the efficient level and the deadweight loss is $800,000 per month. With the regulation, MB = MC, so the output level is efficient and total surplus is maximized. The regulation is in the social interest.

3a) If the company is regulated in the social interest, a marginal cost price rule is used. The figure above shows that the price per call is 4 cents and each company provides 800 calls per day. The regulation is in the social interest if the total surplus is maximized and deadweight loss is eliminated. The total surplus is maximized at the level of output for which marginal benefit equals marginal cost. In the figure, the demand curve, which

382

3b)

3c)

3d)

3e)

CHAPTER 14

is also the marginal benefit curve, intersects the marginal cost curve where the price is 4 cents per call and the number of calls per day is 800. If the regulation is in the self-interest of the producers, the price is 20 cents per call and each company provides 400 calls per day. The regulation is in the self-interest of producers if the outcome is the same as that of an unregulated profit-maximizing monopoly. An unregulated monopoly maximizes its profit by producing the quantity at which marginal cost equals marginal revenue. As shown in the figure above, MR = MC = 4 cents if the companies produce 400 calls per day each and charge 20 cents per call. With no regulation, the consumer surplus is the area below the demand curve and above the price, 20 cents per call. So the consumer surplus is ($0.36 − $0.20) × 400 ÷ 2 = $32. The producer surplus is the area below the price and above the marginal cost curve, so the producer surplus is ($0.20 − $0.04) × 400 = $64. The total surplus is the sum of the consumer surplus plus producer surplus, or $32 + $64 = $96 for each company. If the regulation is in the social interest, the consumer surplus is ($0.36 − $0.04) × 800 ÷ 2 = $128 and the producer surplus is zero. The total surplus is $128 for each company. So if the regulation is in the social interest, the total surplus gain is $128 − $96 = $32 per day for each company. If the regulation is in the social interest, the outcome is efficient and there is no deadweight loss. If the regulation is in the self-interest of producers, the deadweight loss the area between the demand curve (which is the marginal benefit curve) and the marginal cost curve for the output “not produced” between 800 and 400 calls per day. So the deadweight loss is ($0.20 − $0.04) × 400 ÷ 2 = $32 for each company. The price would be set so that marginal benefit equals marginal cost. This price would be 4 cents a call, and each company will provide 800 calls per day. A price cap regulation at 4 cents a call could achieve this outcome.

CHAPTER 15 Quantity of paper produced (tons per week) 10 20 30 40 50 60 1) a) b) c) d) e) f)

a) b) c) d)

Paper mill’s marginal cost (dollars per ton) 10 15 20 25 30 35

Catfish farmer’s marginal cost from pollution (dollars per ton) 5 10 15 20 25 30

Marginal benefit of paper (dollars per ton) 55 45 35 25 15 5

A paper mill dumps waste into a lake used by a catfish farmer. The table above shows costs and benefits arising from the production of paper. Draw the paper mill’s marginal private cost curve, the marginal social cost curve, and the marginal benefit of paper curve. What is the efficient level of paper production? If no one owns the lake and there is no regulation of pollution, what is the quantity of paper produced per week? Illustrate the deadweight loss in your figure. If the government introduces a Pigovian tax, what is the tax per ton of paper produced that achieves an efficient outcome? What is the amount of the tax? If the catfish farmer owns the lake, how much paper is produced per week and what does the paper mill pay the farmer per ton? Explain you answer. If the paper mill owns the lake, how much paper is produced per week? Explain. Compare the quantities of paper produced in (c) and (d) and explain the relationship between these quantities. Enrollment (millions of students per year) 4 8 12 16 20 24

2)

EXTERNALITIES

Marginal cost (thousands of dollars per student per year) 4 6 8 10 12 14

Marginal private benefit (thousands of dollars per student per year) 16 14 12 10 8 6

Marginal external benefit (thousands of dollars per student per year) 8 7 6 5 4 3

The table above shows costs and benefits arising from college education. There is no government involvement and the schools are competitive. Draw the marginal cost, marginal private benefit, and marginal social benefit curves. How many students are enrolled and what is the tuition? What is the efficient level of enrollment? What is the deadweight loss? Why does it arise?

384

CHAPTER 15

e)

If the government provides the efficient amount of education, how many school places does it offer and what is the tuition? Explain your answer. If the government provides the efficient amount of education, of the tuition paid by a student what does the government pay and what does what a student pay?

f)

Answers:

1a) The figure above shows the marginal social cost curve, MSC, the marginal private cost curve, MC, and the marginal benefit curve, MB. 1b) The efficient level of paper production is the one at which marginal social cost equals marginal benefit, which is 30 tons of paper per week. If no one owns the lake, the market equilibrium occurs at the quantity of paper for which the marginal private cost equals the marginal benefit, which is 40 tons per week. The deadweight loss is illustrated as the darkened triangle. 1c) A Pigovian tax equals the marginal external cost. If a Pigovian tax is imposed, the paper mill’s marginal private cost curve, which is its supply curve, becomes the same as the marginal social cost curve. In this case the market equilibrium quantity of paper becomes the same as the efficient, quantity $30 tons of paper. At the 30th tom of paper the marginal external cost is $15 per ton, so the tax per ton of paper produced is $15. 1d) If the catfish farmer owns the lake, the transacting parties take all the costs into account because the catfish farmer charges the paper mill for the mill’s pollution. With this charge the marginal social cost curve becomes the paper mill’s marginal private cost curve and therefore its supply curve. The equilibrium now occurs at the quantity of paper for which marginal social cost equals marginal benefit, which is at 30 tons per week. At this level of production the catfish farmer’s marginal cost from pollution is $15 per ton, the farmer charges the paper mill $15 per ton of paper produced for using the lake. 1e) If the paper mill owns the lake, again the transacting parties take all the costs into account because the catfish farmer pays the paper mill not to produce paper. With this payment the marginal social cost curve becomes the paper mill’s marginal private cost curve and therefore its supply curve. For example, for the 60th ton of paper, the catfish farmer is willing to pay $30, the marginal external cost of this ton, for the mill not to produce the 60th ton. So if the paper mill produces this ton, the mill’s marginal cost is $35, the marginal cost of production, plus $30 of forgone revenue from the farmer, for a total marginal cost of $65, which is the same as the marginal social cost. The equilibrium occurs at the quantity of paper for which marginal social cost equals marginal benefit, which is again at 30 tons per week. 1f) The quantities of paper produced in the answers to parts (d) and (e) are equal because the Coarse theorem applies.

EXTERNALITIES

385

2a) The figure above shows the marginal cost curve, MC, the marginal private benefit curve, MB, and the marginal social benefit curve, MSB. 2b) If the market for college education is competitive, the marginal cost curve is the supply curve and the private marginal benefit curve is the demand curve. The market equilibrium occurs at a tuition of $10,000 per student per year and 16 million students per year. 2c) The efficient level of enrollment is the one at which marginal social benefit of college education equals marginal social cost. So the efficient level of enrollment is 20 million students per year. 2d) The deadweight loss is illustrated in the figure above as the darkened triangular area. This area equals ($5,000 × 4,000,000)/2, which is $10 billion. The deadweight loss arises because too few students are enrolled. At the equilibrium level of enrollment, marginal social benefit exceeds marginal cost and society loses the value that it could gain if more students enrolled. 2e) The efficient quantity occurs where marginal social benefit equals marginal cost. This quantity is 20 million students. Tuition is set at the level that equals the marginal private benefit at the efficient level of enrollment, so tuition is $8,000 a year. 2f) The government pays the cost of college education not covered by tuition. At the efficient level of enrollment, 20 million, the marginal private benefit is $8,000, so this is the amount of tuition paid by a student. The remaining $4,000, which is the marginal external benefit, is paid by the government.

CHAPTER 16

PUBLIC GOODS AND COMMON RESOURCES

1) The graph shows costs and benefits of a mosquito control program: a) What is the quantity of spraying that achieves maximum net benefit? b) What is the political equilibrium if voters are well informed? c) What is the political equilibrium if voters are rationally ignorant and bureaucrats achieve the highest attainable budget? Capacity (millions of gallons per month) 1.5 3.0 4.5 6.0 7.5 9.0 2)

Total cost (thousand of dollars per month) 550 1,060 1,630 2,260 2,950 3,700

Marginal benefit to a resident (cents per 1,000 gallons) 1.2 1.0 0.8 0.6 0.4 0.2

A city of 50,000 people is considering installing a sewage disposal system. The total fixed cost of the system is $100,000 per month. The marginal benefit of an average city resident from the first thousand gallons of capacity is 1.4 cents. The table shows the relationship between the system’s capacity, its total cost, and the marginal benefit received by an average city resident: a) Draw the marginal cost curve of the sewage system and the city residents’ marginal benefit curve. What is the capacity that achieves maximum net benefit? b) How much will each person have to pay in taxes to support the efficient capacity level? c) What is the political equilibrium if voters are well informed?

388

CHAPTER 16

Number of goats 80 160 240 320 400 3) a) b) c) d) e)

Milk output (gallons per day) 640 960 960 640 0

In Sweetgrass village, goats graze on common pasture. The amounts of milk they can produce are shown in the table above. The marginal cost of a goat is the equivalent of 2 gallons of milk. Draw the marginal cost, marginal private benefit, and marginal social benefit curves. What are the equilibrium number of goats and quantity of milk produced? What are the efficient number of goats and quantity of milk to produce? If the common pasture was privatized, how many goats would the landowner keep? If the ITQs were issued for the efficient quantity of milk production, what would be the market price of an ITQ?

Answers: 1a) Net benefit is maximized at the quantity where marginal benefit equals marginal cost, i.e. at 5 square miles per day. 1b) If voters are well informed, the political equilibrium will be the efficient quantity of spraying, 5 square miles per day. 1c) The bureaucrats will achieve the highest attainable budget if they increase the quantity of spraying until net benefit (total benefit minus total cost) is zero. The total benefit from the program at a given quantity of spraying is the area under the marginal benefit curve up to this quantity level. The total cost of the program at a given quantity is the area under the marginal cost curve up to this quantity. Total benefit equals total cost ($20,000), so that the net benefit is zero, when the quantity of spraying is 10 square miles per day. So if voters are rationally ignorant, the political equilibrium is 10 square miles sprayed per day.

2a) See the figure above. The marginal costs at each capacity are calculated using the average of the marginal costs of the sewage system at for two consecutive levels of capacity. For instance, the marginal cost between 1.5 million gallons per month to 3.0 million gallons per month is $0.36 per gallon and between 3.0 million

PUBLIC GOODS AND COMMON RESOURCES

389

gallons per month and 4.5 million gallons per month is $0.40. So the marginal cost at 3.0 million gallons per month is the average, $0.38 per gallon. The rest of the marginal costs are calculated similarly. The city’s marginal benefits are calculated by multiplying the marginal benefit of a resident by the city’s population. Net benefit is maximized at the capacity where the city residents’ marginal benefit equals the marginal cost of the system, which is $0.40. So that the efficient capacity is 4.5 million gallons per month. 2b) The efficient capacity is the one that maximizes the city residents’ net benefit, 4.5 million gallons per month. At this level of capacity the total cost of the sewage system is $1,630,000 per month. Because 50,000 people live in the city, each person must pay $1,630,000/50,000 = $32.60. 2c) If voters are well informed, the political equilibrium will be the efficient capacity, that is, 4.5 million gallons per month.

3a) See the figure above. The marginal social benefit, MSB, is equal to the change in milk output divided by the change in the number of goats. For instance, from 0 to 80 goats, the production of milk increases from 0 to 640 gallons, so the marginal social benefit is 640 gallons/80 goats, or 8 gallons per goat. This marginal social benefit is then plotted midway between 0 and 80 goats, so it is plotted at 40 goats. The remainder of the marginal social benefits are calculated similarly. The marginal private benefit, MB, is equal to the average quantity of milk per goat or the total output of milk divided by the total number of goats. So for 80 goats, the marginal private benefit is 640 gallons/80 goats, or 8 gallons per goat. This marginal private benefit is then plotted at 80 goats. The remainder of the marginal private benefits are calculated similarly. 3b) The equilibrium number of goats is the number of goats at which marginal private benefit equals marginal cost. As shown in the graph, MPB = MC when the number of goats is 320. With this number of goats, the quantity of milk produced is 640 gallons per day. 3c) The equilibrium number of goats is the one at which marginal social benefit equals marginal cost. As shown in the graph, MSB = MC when the number of goats is 160. With this number of goats, the quantity of milk produced is 960 gallons per day. 3d) If the common pasture was privatized, the landowner would face the same conditions as society faces. So the marginal social benefit curve would become the marginal private benefit curve and the efficient number of goats, 160, would be kept. 3e) If the ITQs were issued for the efficient quantity of milk production, the price of an ITQ would be equal to the marginal private benefit at the quota quantity minus the marginal cost: 6 – 2 = 4 gallons of milk.

CHAPTER 17

DEMAND AND SUPPLY IN FACTOR MARKETS

Quantity of labor (workers) 0 2 4 6 8 10 1) a) b) c) d)

Total product (car washes per day) 0 48 88 120 144 160

Sandy’s Shiny Car is a perfectly competitive car wash service. The table above shows Sandy’s total product schedule. Sandy can sell as many washes as she chooses at $5, and the wage rate of her workers is $60 a day. Draw Sandy’s workers’ marginal product of labor curve. Draw Sandy’s workers’ marginal revenue product curve. Find Sandy’s demand for labor curve. How many workers does Sandy employ?

2)

Sandy’s Shiny Car is a perfectly competitive car wash service. The table above shows Sandy’s total product schedule. Suppose the price of a car wash falls to $3 and the wage rate is $60 per day. a) What happens to the marginal product of labor? To the marginal revenue product? To Sandy’s demand for labor curve? b) When the price of a car wash falls to $3, how many workers does Sandy employ?

3)

Sandy’s Shiny Car is a perfectly competitive car wash service. The table above shows Sandy’s total product schedule. Suppose the price of car wash is $5 but the wage rate rises to $80 per day. a) What happens to the marginal product of labor? What happens to the marginal revenue product? What happens to Sandy’s demand for labor curve? b) What happens to the number of workers Sandy employs? How does this number compare with the number Sandy employs when the wage rate is $60 per day?

4)

a) b) c) d) e)

Margo’s CoolDesign, is a firm that designs personal Web sites. Margo is considering buying a new computer that costs $1,600. She expects that the computer will generate an additional $1,000 at the end of the first year, $600 at the end of the second year, and $200 at the end of the third year, after which it will be worthless. The interest rate in the next three years is 6 percent per year. What is the present value of the computer? What is the net present value of the computer? Will Margo buy the computer? Why or why not? What is the highest price that Margo is willing to pay for the computer? How will your answers in (a), (b), (c), and (d) change if the interest rate rises to 8 percent per year?

392

CHAPTER 17

Answers:

1a) See the figure above. The marginal product of labor, MP, is the increase in total product that results from hiring an additional worker. For example, if Sandy increases the number of workers hired from 2 to 4, her total product increases from 48 to 88 car washes. The marginal product of 2 additional workers is 88 – 48 = 40 washes, or 20 washes per worker. The marginal product of labor is plotted at the midpoint so that, for example, when the number of workers increases from 2 to 4, the point for 20 car washes is plotted at 3 workers.

1b) See the figure above. The marginal revenue product of labor, MRP, is the increase in total revenue that results from hiring one additional worker. MRP equals price multiplied by the MP. For example, when the number of workers increases from 2 to 4, MP = 20 washes and the price is $5 per wash, and so MRP = $5 × 20, which is $100. The marginal revenue product of labor is also plotted at the midpoint, so that the MRP of $100 is plotted at the midpoint between 2 and 4 workers, which is 3 workers.. 1c) The demand for labor curve is the same as the marginal revenue product curve. So the figure above that shows the marginal revenue product curve also shows Sandy’s demand for labor curve.

DEMAND AND SUPPLY IN FACTOR MARKETS

393

1d) Sandy hires the number of workers for which the marginal revenue product equals the wage rate. The wage rate is $60 per day. The figure above shows that this wage rate equals the marginal revenue product when Sandy hires 7 workers.

2a) The marginal product of labor depends on Sandy’s production function, not on the price of her product. So marginal product of labor does not change. Because MRP = P × MP, a fall in price decreases the MRP. The MRP curve shifts downward as illustrated in the figure above by the shift from MRP0 to MRP1. The demand for labor curve is the same as the marginal revenue product curve. So when Sandy’s MRP decreases, her demand for labor shifts leftward. 2b) Sandy hires the number of workers for which marginal revenue product equals wage rate. When the price of a car wash is $3, the figure above shows that she hires 3 workers.

3a) The marginal product of labor depends on Sandy’s production function, not on the wage rate. So marginal product of labor does not change. Because MRP = P × MP and neither the price nor the marginal product change, the MRP is unaffected. So as the figure above shows, Sandy’s “new” MRP curve is the same as her MRP curve in the first problem. The demand for labor curve is the same as the marginal revenue product curve. So Sandy’s demand for labor curve remains the same as in the first problem.

394

CHAPTER 17

3b) Sandy hires the number of workers for which marginal revenue product equals wage rate. When the wage rate is $80, the figure above indicates that she hires 5 workers. When the wage rate rises, the quantity of labor Sandy demands decreases. 4a) The present value of the computer is the present value of its marginal revenue product over the three years of $1,000 $600 $200 + + = $1,645. its life, which is 2 1 + 0.06 (1 + 0.06 ) (1 + 0.06 )3 4b) The net present value of the computer is the present value of the future flow of marginal revenue products generated by the computer minus the price of the computer, which is $1,645 − $1,600, or $45. 4c) Margo will buy the computer because its net present value is positive. 4d) Margo is willing to buy the computer as far as its net present value is positive. So she will buy the computer if its price is below $1,645. $1,000 $600 $200 + + 4e) If the interest rate is 8 percent, the present value of the computer is 2 1 + 0.08 (1 + 0.08) (1 + 0.08)3 which equals $1,599. The computer’s net present value is $1,599 − $1,600 = −$1. Because the net present value is negative, Margo will not buy the computer. The highest price that she is willing to pay is $1,599, the price at which she is indifferent between buying the computer or not..

CHAPTER 18

ECONOMIC INEQUALITY

Households Lowest 20 percent Second 20 percent Middle 20 percent Next highest 20 percent Highest 20 percent

Income (percent of total) United States Italy 3.6 8.7 8.9 14.0 14.8 18.1 23.1 22.9 49.6 36.3

1) The table above shows the income distribution in the United States and Italy in 2000 a) Draw a Lorenz curve for the United States. According to your graph, what is the percentage of income received by the poorest 30 percent of households? The richest 10 percent? b) On the same graph, draw a Lorenz curve for Italy. According to your graph, what is the percentage of income received by the poorest 30 percent of households? The richest 10 percent? c) Draw the line of equality. How is it related to the Lorenz curve? d) Is income distributed more equally in the United States or in Italy? Which country’s Gini ratio is closer to zero? Explain your answers.

Households Lowest 20 percent Second 20 percent Middle 20 percent Next highest 20 percent Highest 20 percent

Income (percent of total) United States Canada 3.6 7.4 8.9 13.2 14.8 18.1 23.1 24.9 49.6 36.4

2) The table shows income distribution in the United States and Canada: a) Draw a Lorenz curve for the United States. According to your graph, what is the percentage of income received by the poorest 30 percent of households? The richest 10 percent? b) On the same graph, draw a Lorenz curve for Canada. According to your graph, what is the percentage of income received by the poorest 30 percent of households? The richest 10 percent? c) Draw the line of equality. How is it related to the Lorenz curve? d) Is income distributed more equally in the United States or in Canada? Which country’s Gini ratio is closer to zero? Explain your answers.

396

3)

a) b) c) d)

CHAPTER 18

The figure above shows the demand for and supply of low-skilled labor. High-skilled workers have twice the marginal product of low-skilled workers. The cost of acquiring the skill adds $5 an hour to the wage that must be offered to attract high-skilled labor at each employment level. What is the equilibrium wage rate of low-skilled labor and the equilibrium quantity of low-skilled labor employed? Draw and explain the demand curve for high-skilled labor. Draw and explain the supply curve of high-skilled labor. What is the equilibrium wage rate of high-skilled labor and the equilibrium quantity of high-skilled labor employed?

Answers:

1a) See the figure above. The percentage of income received by the poorest 30 percent households is about 8 percent. The percentage of income received by the richest 10 percent households is about 27 percent. 1b) See the figure above. The percentage of income received by the poorest 30 percent households is about 16 percent. The percentage of income received by the richest 10 percent households is about 19 percent.

ECONOMIC INEQUALITY

397

1c) The line of equality is in the figure above. If income is equally distributed, the Lorenz curve is the same as the line of equality. 1d) The closer the Lorenz curve is to the line of equality, the more equal is the income distribution. Italy’s Lorenz curve is closer to the line of equality, so income distributed more equally in Italy than in the United States. The Gini ratio equals the ratio of the area between the line of equality and the Lorenz curve to the entire area beneath the line of equality. If income is equally distributed, the Gini ratio is zero. Because income in Italy is distributed more equally than in the United States, Italy’s Gini ratio is closer to zero than the U.S. Gini ratio.

2a) See the figure above. The percentage of income received by the poorest 30 percent households is about 8 percent. The percentage of income received by the richest 10 percent households is about 27 percent. 2b) See the figure above. The percentage of income received by the poorest 30 percent households is about 14 percent. The percentage of income received by the richest 10 percent households is about 19 percent. 2c) The line of equality is in the figure above. If income is equally distributed, the Lorenz curve is the same as the line of equality. 2d) The closer the Lorenz curve is to the line of equality, the more equal is the income distribution. Canada’s Lorenz curve is closer to the line of equality, so income distributed more equally in Canada than in the United States. The Gini ratio equals the ratio of the area between the line of equality and the Lorenz curve to the entire area beneath the line of equality. If income is equally distributed, the Gini ratio is zero. Because income in Canada is distributed more equally than in the United States, Canada’s Gini ratio is closer to zero than the U.S. Gini ratio.

398

CHAPTER 18

3a) The figure above shows the supply of labor curve for low-skilled workers as SL and the demand curve for low-skilled workers as DL. The equilibrium wage rate for low-skilled workers is $5 and the quantity of labor employed is 5,000 hours per day. 3b) The figure above shows the demand curve, DH, for high-skilled workers. The demand curve for high-skilled labor is the marginal revenue product, MRP, of high-skilled labor. Because the marginal product of highskilled labor, MP, is twice the marginal product of low-skilled labor and because the MRP is the MP multiplied by the price, because the MP of high-skilled workers is twice that of low-skilled workers, the MRP of high-skilled workers is twice the MRP of low-skilled labor. So at each quantity of labor the wage rate for high-skilled workers is twice that of low-skilled workers. 3c) The figure above shows the supply curve, SH, for high-skilled workers. The position of the supply curve of high-skilled labor reflects the cost of acquiring human capital. This cost is the vertical distance between the supply curve of low-skilled labor and the supply curve of high-skilled labor, $5 per hour. 3d) The figure above shows that the equilibrium wage rate for high-skilled workers is $10 and the equilibrium quantity of high-skilled labor employed is 5,000 hours per day.

CHAPTER 19

1)

a) b) c) d)

UNCERTAINTY AND INFORMATION

Al works as a sales clerk at a department store for a fixed salary of $2,500 per month. He is offered a job as a salesperson at a car dealership in which there is a 50 percent chance that he will make $5,000 a month and a 50 percent chance that he will make only $1,000 a month. The figure above Al’s utility of wealth curve: What is Al’s expected income from the offered job? What is Al’s expected utility from the offered job? Will Al accept the offer? Why or why not? What is the minimum fixed salary for which Al will continue to work for the department store and not accept the dealership’s offer?

400

2)

a) b) c) d)

3)

CHAPTER 19

Wendy works as a teller at a bank for a fixed salary of $1,800 per month. She is offered a job as a salesperson at which there is a 40 percent chance that she will make $5,000 a month and a 60 percent chance that she will make only $1,000 a month. The figure shows Wendy’s utility of wealth curve: What is Wendy’s expected income from the offered job? What is Wendy’s expected utility from the offered job? Will Wendy accept the offer? Why or why not? What is the minimum fixed salary for which Wendy will continue to work for the bank and not take the sales job?

Larry owns a car worth $20,000, and that is his only wealth. There is a 10 percent chance that Larry will have an accident within a year. If he does have an accident, his car is worthless. Larry’s utility of wealth curve is shown in the figure above. An insurance company agrees to pay a car owner like Larry the full value of his car in case of an accident if the car owner buys the company’s insurance policy. The company’s operating expenses are $2,500 per policy. a) What is Larry’s expected wealth? b) What is Larry’s expected utility? c) What is the maximum amount that Larry is willing to pay for car insurance?

UNCERTAINTY AND RISK

d) e)

4)

a) b) c) d) e)

401

What is the minimum premium that the insurance company is willing to accept? Will Larry buy the insurance policy? Why or why not?

Roy owns a sports car worth $40,000, and that is his only wealth. Roy is a reckless driver, and there is a 30 percent chance that he will have an accident within a year. If he does have an accident, his car is worthless. Roy’s utility of wealth curve is shown in the figure below. An insurance company agrees to pay a car owner like Roy the full value of his car in case of an accident if the car owner buys the company’s insurance policy. The company’s operating expenses are $2,000 per policy. What is Roy’s expected wealth? What is Roy’s expected utility? What is the maximum amount that Roy is willing to pay for car insurance? What is the minimum premium that the insurance company is willing to accept? Will Roy buy the insurance policy? Why or why not?

Answers: 1a) The probability that Al will make $5,000 a month is 0.5, and the probability that he’ll make $1,000 a month is also 0.5. Therefore Al’s expected income is $5,000 × 0.5 + $1,000 × 0.5 = $3,000 per month. 1b) Al’s utility of wealth curve shows that if he makes $5,000 a month, his utility is 100, and if he makes $1,000 a month, his utility is 40. Therefore Al’s expected utility is 100 × 0.5 + 40 × 0.5 = 70. 1c) Al chooses the job that maximizes his expected utility. Al’s utility of wealth curve shows that if he stays at the current job and makes $2,500 a month with certainty, his utility is 80. If Al accepts the car dealership’s offer, his expected utility (calculated in part (b)) is 70. So Al will not accept the dealership’s offer. 1d) Al will continue to work for the department store and not accept the dealership’s offer if his fixed salary gives him a greater utility than that expected from the offered job, 70. As Al’s utility of wealth curve shows, his utility is 70 if his risk free income is $2,000 a month. So Al will stay at the current job if his fixed salary is greater than $2,000 a month. 2a) The probability that Wendy will make $5,000 a month is 0.4, and the probability that she’ll make $1,000 a month is also 0.6. Therefore Wendy’s expected income is $5,000 × 0.4 + $1,000 × 0.6 = $2,600 per month. 2b) Wendy’s utility of wealth curve shows that if she makes $5,000 a month, her utility is 100, and if she makes $1,000 a month, her utility is 40. Therefore Wendy’s expected utility is 100 × 0.4 + 40 × 0.6 = 64.

402

CHAPTER 19

2c) Wendy chooses the job that maximizes her expected utility. Wendy’s utility of wealth curve shows that if she stays at the current job and makes $1,800 a month with certainty, her utility is 60. If Wendy takes the sales job, her expected utility (calculated in part (b)) is 64. So Wendy will take the sales job. 2d) Wendy will continue to work for the bank and not take the sales job if her fixed salary gives her a greater utility than that expected from the offered job, 64. As Wendy’s utility of wealth curve shows, her utility is 64 if her risk free income is $2,000 a month. So Wendy will stay at the current job if her fixed salary is greater than $2,000 a month. 3a) The probability that Larry will have an accident is 0.1, and the probability that he won’t have an accident is 0.9. Therefore Larry’s expected wealth is $20,000 × 0.9 + $0 × 0.1 = $18,000. 3b) If Larry has an accident, his utility is zero. If he does not have an accident, his utility is 100. Because the probability of an accident is 0.1, and the probability of not having an accident is 0.9, Larry’s expected utility is 100 × 0.9 + 0 × 0.1 = 90.

3c) With no insurance, Larry’s expected utility of wealth is 90. Given his utility of wealth curve, the figure above shows that Larry gets the same utility if his wealth is $12,000 with no risk. That is, Larry’s utility of a guaranteed wealth of $12,000 is the same as his utility of an expected wealth of $18,000 with the degree of risk he faces. This means that Larry is willing to pay up to $18,000 − $12,000 = $6,000 for car insurance. 3d) Because the probability of an accident is 0.1, the company will pay out $20,000 to 1/10 of car owners like Larry, or an average of $2,000 per person. The company covers all its costs if it offers insurance for $2,000 + $2,500 = $4,500. So $4,500 is the minimum insurance premium that the company is willing to accept. 3e) Larry will buy the policy because the maximum amount that he is willing to pay, $6,000, is greater than the minimum amount that the insurance company is willing to accept, $4,500. 4a) The probability that Roy will have an accident is 0.3, and the probability that he won’t have an accident is 0.7. Therefore Roy’s expected wealth is $40,000 × 0.7 + $0 × 0.3 = $28,000. 4b) If Roy has an accident, his utility is zero. If he does not have an accident, his utility is 100. Because the probability of an accident is 0.3, and the probability of not having an accident is 0.7, Roy’s expected utility is 100 × 0.7 + 0 × 0.3 = 70.

UNCERTAINTY AND RISK

403

4c) With no insurance, Roy’s expected utility of wealth is 70. Given his utility of wealth curve, the figure above shows that Roy gets the same utility if his wealth is $16,000 with no risk. That is, Roy’s utility of a guaranteed wealth of $16,000 is the same as his utility of an expected wealth of $28,000 with the degree of risk he faces. This means that Roy is willing to pay up to $28,000 − $16,000 = $12,000 for car insurance. 4d) Because the probability of an accident is 0.3, the company will pay out $40,000 to 3/10 of car owners like Roy, or an average of $12,000 per person. The company covers all its costs if it offers insurance for $12,000 + $2,000 = $14,000. So $14,000 is the minimum insurance premium that the company is willing to accept. 4e) Roy will not buy the policy because the maximum amount he is willing to pay, $12,000, is less than the minimum amount that the insurance company is willing to accept, $14,000.

CHAPTER 20

TRADING WITH THE WORLD**

Grainland Wheat Milk (billions of bushels (billions of gallons per year) per year) 0 288 8 280 16 264 24 240 32 208 40 168 48 120 56 64 64 0 1)

a) b) c) d) e) f) g) h) i)

*

Cowland Wheat Milk (billions of bushels (billions of gallons per year) per year) 0 336 8 320 16 288 24 240 32 176 40 96 48 0

The table above shows the production possibilities in two countries, Grainland and Cowland. Grainland produces 28 billion bushels of wheat and Cowland produces 36 billion bushels of wheat. The countries don’t trade. Draw Grainland’s production possibilities frontier. Draw Cowland’s production possibilities frontier. What combinations of wheat and milk do the countries consume? What is the opportunity cost of a bushel of wheat in each country? What is the relationship between the quantity of wheat produced and the opportunity cost of wheat in each economy? At the current levels of production, which country has a comparative advantage in wheat? In milk? If the two countries trade with each other, which country exports wheat? Explain. What can you say about the terms of trade (the price of wheat in terms of milk) under free trade? If the two countries trade with each other, what adjustments are made to the amount of each good produced by each country? If the two countries trade with each other, what adjustments are made to the amount of each good consumed by each country?

* This is Chapter 33 in Economics.

406

CHAPTER 20

Price (dollars per bushel of wheat) 4 5 6 7 8 9 10 2) a) b) c) d)

Quantity of wheat exported (billions of bushels) 0 8 16 24 28 30 32

Quantity of wheat imported (billions of bushels) 24 20 16 12 8 4 0

The table above shows Grainland’s export supply of wheat and Cowland’s import demand for wheat. The money price of milk is $1 per gallon. If the two countries did not engage in international trade, what would be the price of wheat in each country? If there is free trade between the two countries, what is the world price of wheat? What quantities of wheat are exported and imported? If Cowland imposes a tariff on wheat of $3 per bushel, what is the price of wheat in each of the two countries? How much wheat is traded internationally? What tariff revenue does Cowland’s government collect?

Answers:

1a) The figure above has Grainland’s production possibilities frontier.

TRADING WITH THE WORLD

407

1b) The figure above has Cowland’s production possibilities frontier. 1c) With no international trade, the countries consume the same quantities of wheat and milk as the quantities they produce. So Grainland consumes 28 billion bushels of wheat and 225 billion gallons of milk, and Cowland consumes 36 billion bushels of wheat and 138 billion gallons of milk. 1d) In Grainland, 28 billion bushels of wheat is the midpoint between 24 and 32 billion. To produce these 8 additional billion bushels, Grainland has to give up 240 − 208 = 32 billion gallons of milk, or 32/8 = 4 gallons of milk per bushel of wheat. So the opportunity cost of a bushel of wheat in Grainland is 4 gallons of milk. In Cowland, 36 billion bushels of wheat is the midpoint between 32 and 40 billion. To produce these 8 additional billion bushels, Cowland has to give up 176 − 96 = 80 billion gallons of milk, or 80/8 = 10 gallons of milk per bushel of wheat. So the opportunity cost of a bushel of wheat in Cowland is 10 gallons of milk. Both countries’ production possibilities frontiers bow outward, so in both countries the opportunity cost of wheat increases as more wheat is produced. 1e) The opportunity cost of wheat in Grainland (4 gallons of milk per bushel) is lower than in Cowland (10 gallons of milk per bushel), so Grainland has a comparative advantage in wheat. The opportunity cost of milk in Cowland (1/10 = 0.1 bushels of wheat per gallon) is lower than in Grainland (1/4 = 0.25 bushels of wheat per gallon), so Cowland has a comparative advantage in milk. 1f) A country will export wheat if it the price of the exported wheat is higher than the country’s opportunity cost of producing wheat. The opportunity cost of a bushel of wheat in Grainland is 4 gallons of milk, so Grainland will export wheat if Cowland offers a price higher than 4 gallons of milk per bushel. In Cowland the opportunity cost of a bushel of wheat in Cowland is 10 gallons of milk, so Cowland is better off if it imports wheat at a price lower than 10 gallons of milk per bushel. Cowland will offer a price higher than 4 gallons of milk per bushel and Grainland will export wheat to Cowland. 1g) The price of a bushel of wheat will be higher than 4 gallons of milk per bushel but lower than 10 gallons of milk per bushel. 1h) Because the price offered by Cowland for Grainland’s wheat is higher than Grainland’s opportunity cost of producing wheat, Grainland increases the production of wheat and decreases the production of milk, which it can now get from Cowland at a price lower than the opportunity cost of producing milk domestically. For the same reason, Cowland increases the production of milk and decreases the production of wheat. 1i) International trade allows each country to increase its consumption possibilities beyond the limits of the production possibilities frontier. The combinations of goods consumed by each country depend on the country’s preferences and the terms of trade. For example, if Grainland increases its production of wheat

408

CHAPTER 20

from 28 to 44 billion bushels, decreasing its production of milk from 225 to 145 billion gallons, and exports 16 billion bushels of wheat to Cowland in exchange for 96 billion gallons of milk (so that the price is 6 gallons of milk per bushel of wheat), Grainland will consume the same quantity of wheat as with no trade (28 billion bushels), but will increase its consumption of milk from 225 to 241 billion gallons. Cowland then will increase its production of milk from 138 to 266 billion gallons, decreasing its production of wheat from 36 to 20 billion bushels, import 16 billion bushels of wheat from Grainland in exchange for 96 billion gallons of milk, and consume the same quantity of wheat as with no trade (36 billion bushels), but increase its consumption of milk from 138 to 170 billion gallons. So both countries are better off with trade.

2a) The figure above shows Grainland’s export supply curve and Cowland’s import demand curve. Grainland exports no wheat if the price is $4 per bushel, so the no-trade price of wheat in Grainland is $4. Cowland imports no wheat if the price is $10 per bushel, so the no-trade price of wheat in Cowland is $10. 2b) As shown in the figure, when the price of wheat is $6 per bushel, the quantity of wheat that Grainland is willing to export, 16 billion bushels, equals the quantity of wheat that Cowland is willing to import. So with free trade, the world price of wheat is $6 per bushel and 16 billion bushels are exported and imported. 2c) The export supply curve shifts upward by the amount of the tariff, $3 per bushel. The new equilibrium occurs at a price of $8 per bushel, so the price of wheat in Cowland is $8 per bushel. Grainland receives the price that people in Cowland pay, $8 per bushels, minus the tariff collected by Cowland’s government. So the price of wheat in Grainland is $8 −$3 = $5 per bushel. At $5 per bushel, Grainland is willing to export 8 billion bushels of wheat, and at $8 per bushel, Cowland is willing to import 8 billion bushels of wheat, so $8 billion bushels of wheat is the equilibrium quantity that is traded internationally. 2d) Cowland’s government collects $3 per bushel and, with 8 billion bushels of wheat traded, $3 × 8 billion = $24 billion.

View more...

Comments

Copyright ©2017 KUPDF Inc.
SUPPORT KUPDF